Sei sulla pagina 1di 210

Continuum: Lifelong Learning in Neurology—Neurology of Pregnancy, Volume 20, Issue 1,

February 2014

Issue Overview

Neurology of Pregnancy, February 2014;20(1)

Continuum: Lifelong Learning in Neurology® is designed to help practicing neurologists stay

abreast of advances in the field while simultaneously developing lifelong self-directed learning

skills.

Learning Objectives

Upon completion of the Continuum: Lifelong Learning in Neurology Neurology of Pregnancy

issue, participants will be able to:

► Summarize safety concerns associated with diagnostic neuroimaging in women during

pregnancy and lactation

► Discuss diagnostic modalities and radiologic features of neurologic conditions encountered by

pregnant women

► Discuss the current understanding of the interactions between multiple sclerosis and

pregnancy and their implications for reproductive counseling, and discuss the issues related to

disease-modifying therapy and therapy of acute exacerbations of multiple sclerosis during

pregnancy and lactation

► Explain issues regarding the management of women with epilepsy and pregnancy, including

preconception planning, antiepileptic drug effects on the exposed offspring, and consequences of

seizures during pregnancy, with an emphasis on counseling and risk management

Copyright © American Academy of Neurology. Unauthorized reproduction of this article is prohibited.


► Identify the ways in which the physiologic changes of pregnancy affect risk of ischemic and

hemorrhagic stroke, and discuss an approach to the diagnosis and treatment of ischemic and

hemorrhagic stroke in pregnancy and the puerperium

► Outline the most common peripheral neuropathic disorders in pregnancy with a focus on

clinical recognition, diagnosis, and treatment

► Analyze available information regarding expectations and management for patients with

myasthenia gravis during childbearing years, pregnancy, and postpartum

► Diagnose and manage primary and secondary headaches that may occur during pregnancy

and postpartum

► Describe movement disorders that occur during pregnancy, the treatment of preexisting

movement disorders, and the influence the pregnant state has on movement disorder symptoms

► Evaluate and treat neuro-ophthalmic disorders in pregnant patients

► Summarize the available literature on reproductive issues in women with multiple sclerosis

and provide sound, objective counseling to facilitate well-informed, autonomous decision

making by patients

► Apply a practical framework for understanding, identifying, and managing legal risk when

treating pregnant women with epilepsy

► Demonstrate the use of Current Procedural Terminology and diagnosis codes for the

evaluation and management of neurologic disease in pregnant women, with a focus on those with

epilepsy

Core Competencies

The Continuum Neurology of Pregnancy issue covers the following core competencies:

Copyright © American Academy of Neurology. Unauthorized reproduction of this article is prohibited.


► Patient Care

► Medical Knowledge

► Practice-Based Learning and Improvement

► Interpersonal and Communication Skills

► Professionalism

► Systems-Based Practice

Disclosures

CONTRIBUTORS

Autumn Klein, MD, PhD, Guest Editor†


Chief of the Division of Women’s Neurology, Assistant Professor of Neurology and Obstetrics
and Gynecology, University of Pittsburgh Medical Center, Pittsburgh, Pennsylvania

Amaal AlDakheel, MD
Clinical Fellow, The Morton and Gloria Shulman Movement Disorders Centre in the Edmond J.
Safra Program for Parkinson’s Disease, Toronto Western Hospital, Krembil Neuroscience
Centre, University Health Network, University of Toronto, Toronto, Ontario, Canada
a,b
Dr AlDakheel reports no disclosures.

Riley M. Bove, MD
Instructor in Neurology, Harvard Medical School; Associate Neurologist, Brigham and Women’s
Hospital, Boston, Massachusetts
a,b
Dr Bove reports no disclosures.

Patricia K. Coyle, MD, FAAN


Professor of Neurology and Vice Chair (Clinical Affairs); Director, Stony Brook MS
Comprehensive Care Center, Stony Brook University, Stony Brook, New York
a
Dr Coyle has consulted for Acorda Therapeutics; Accordant Health Services; Bayer AG; Biogen Idec; Genentech,
Inc; Genzyme Corporation; Merck KGaA; Mylan Inc; Novartis Corporation; and Teva Pharmaceuticals. Dr Coyle
receives clinical trial support from Actelion Pharmaceuticals; Novartis Corporation; and Opexa Therapeutics, Inc.
b
Dr Coyle discusses the unlabeled use of IV immunoglobulin post partum.

Carolina De Jesus-Acosta, MD
Fellow, Neuromuscular Medicine, Duke University Medical Center, Durham, North Carolina
a
Dr De Jesus-Acosta reports no disclosure.
b
Dr De Jesus-Acosta discusses the use of drugs for the treatment of myasthenia gravis, none of which are labeled by
the US Food and Drug Administration for use in pregnancy.

Kathleen B. Digre, MD, FAAN

Copyright © American Academy of Neurology. Unauthorized reproduction of this article is prohibited.


Professor, Departments of Neurology of Ophthalmology; Adjunct Professor, Department of
Obstetrics and Gynecology, Moran Eye Center, University of Utah
a
Dr Digre has received a grant from the National Eye Institute and salary support from the Neuro-ophthalmology
Research Disease Investigator Consortium.
b
Dr Digre discusses the use of several drugs for the treatment of neuro-ophthalmic disorders, none of which are
labeled by the US Food and Drug Administration for use in pregnancy.

Steven K. Feske, MD
Associate Professor of Neurology, Harvard Medical School; Director, Stroke Division, Brigham
and Women’s Hospital, Boston, Massachusetts
a
Dr Feske has received royalties from Elsevier for his role as editor of Office Practice of Neurology, 2nd Edition,
and receives research support from the National Institute of Neurological Disorders and Stroke.
b
Dr Feske reports no disclosure.

Amanda C. Guidon, MD
Instructor of Neurology, Harvard Medical School; Assistant in Neurology, Massachusetts
General Hospital, Boston, Massachusetts
a,b
Dr Guidon reports no disclosures.

Cynthia L. Harden, MD
Professor of Neurology; Chief, Division of Epilepsy and Electroencephalography, Hofstra North
Shore–LIJ School of Medicine, Great Neck, New York
a
Dr Harden has received personal compensation for activities with GlaxoSmithKline; Lundbeck; UCB SA; and
Upsher-Smith Laboratories, Inc. Dr Harden has served in an editorial capacity for UpToDate and received
research support from the Epilepsy Therapy Project.
b
Dr Harden reports no disclosure.

Joseph S. Kass, MD, JD


Associate Professor of Neurology, Psychiatry, and Medical Ethics, Baylor College of Medicine,
Houston, Texas
a,b
Dr Kass reports no disclosures.

Krista Kinard, MD
Instructor, Department of Ophthalmology, Moran Eye Center, University of Utah
a
Dr Kinard reports no disclosure.
b
Dr Kinard discusses the use of several drugs for the treatment of neuro-ophthalmic disorders, none of which are
labeled by the US Food and Drug Administration for use in pregnancy.

Joshua P. Klein, MD, PhD


Chief, Division of Hospital Neurology, Brigham and Women’s Hospital; Assistant Professor of
Neurology and Radiology, Harvard Medical School, Boston, Massachusetts
a
Dr Klein receives financial compensation for serving on the editorial board of the Journal of Neuroimaging and
AccessMedicine Neurology, and royalties from McGraw-Hill for Adams and Victor’s Principles of Neurology.
b
Dr Klein reports no disclosure.

E. Anne MacGregor, MB BS, MD, FFSRH, MICR


Associate Specialist, Barts Sexual Health Centre, St Bartholomew’s Hospital; Honorary
Professor, Centre for Neuroscience and Trauma, Blizard Institute of Cell and Molecular Science,
Barts and the London School of Medicine and Dentistry, London, England, United Kingdom

Copyright © American Academy of Neurology. Unauthorized reproduction of this article is prohibited.


a
Professor MacGregor has acted as a paid consultant to and/or her department has received research funding from
Addex Therapeutics; Allergan, Inc; AstraZeneca; Berlin-Chemie AG; BTG International Ltd; Endo
Pharmaceuticals Inc; GlaxoSmithKline; the Menarini Group; Merck & Co, Inc; POZEN Inc; and UniPath.
b
Professor MacGregor discusses the use of several drugs for the treatment of headaches, none of which are labeled
by the US Food and Drug Administration for use in pregnancy.

E. Wayne Massey, MD, FAAN


Professor, Department of Neurology, Duke University Medical Center, Durham, North Carolina
a,b
Dr Massey reports no disclosures.

Janice M. Massey, MD, FAAN


Professor, Department of Neurology; Chief, Neuromuscular Division, Duke University Medical
Center, Durham, North Carolina
a
Dr Massey has received educational grants from Allergan, Inc; and Merz Pharma.
b
Dr Massey discusses the use of drugs for the treatment of myasthenia gravis, none of which are labeled by the US
Food and Drug Administration for use in pregnancy.

Janis M. Miyasaki, MD, MEd, FRCP, FAAN


Associate Professor, Department of Medicine, and Associate Clinical Director, The Movement
Disorders Centre, Toronto Western Hospital, Krembil Neuroscience Centre, University Health
Network, University of Toronto, Toronto, Ontario; Associate Professor, Neurology, University
of Alberta, Edmonton, Alberta, Canada
a
Dr Miyasaki has served as a speaker or on advisory boards for Novartis Corporation and Teva Pharmaceuticals.
Dr Miyasaki has received research support from the Canadian Agency for Drugs and Technologies in Health, the
Canadian Institute for Health Research, the Michael J. Fox Foundation for Parkinson’s Research, the National
Center for Complementary and Alternative Medicine, the National Parkinson Foundation, the NIH, the Ontario
Drug Benefits Program, and the Ontario Ministry of Health and Long-Term Care. The Movement Disorders Centre
at Toronto Western Hospital has received research support from Teva Pharmaceuticals Ltd.
b
Dr Miyasaki reports no disclosure.

Bethanie N. Morgan-Followell, MD
Assistant Professor of Pediatrics, Section of Child Neurology, Nationwide Children’s Hospital,
The Ohio State University, Columbus, Ohio
a
Dr Morgan-Followell reports no disclosure.
b
Dr Morgan-Followell discusses the unlabeled use of disease-modifying therapies during attempts at conception and
during pregnancy.

Jacqueline A. Nicholas, MD, MPH


Assistant Professor of Neurology, Division of Neuroimmunology, The Ohio State University
Medical Center, Columbus, Ohio
a
Dr Nicholas’ fellowship is funded through a Sylvia Lawry Physician Fellowship grant from the National Multiple
Sclerosis Society, and she receives additional funding for clinical research from the National Multiple Sclerosis
Society as an assistant professor.
b
Dr Nicholas discusses the unlabeled use of disease-modifying therapies during attempts at conception and during
pregnancy.

Laura B. Powers, MD, FAAN


Dr Powers is retired from private practice.
a
Dr Powers serves as ICD-9-CM Advisor for the Coding Subcommittee of the AAN Medical Economics and
Management Committee and serves in an editorial capacity for Neurology: Clinical Practice.
b
Dr Powers reports no disclosure.

Copyright © American Academy of Neurology. Unauthorized reproduction of this article is prohibited.


Aneesh B. Singhal, MD
Associate Professor of Neurology, Harvard Medical School; Director, Neurology Quality and
Safety, Massachusetts General Hospital, Boston, Massachusetts
a
Dr Singhal has served as a consultant for Biogen Idec and as a medical expert witness in cases of stroke. Dr
Singhal’s spouse holds stock or stock options greater than 5% of the company or greater than $10,000 in value in
Biogen Idec, and Vertex Pharmaceuticals Incorporated. Dr Singhal has received research support from the
National Institute of Neurological Disorders and Stroke, and his institution has received research support from
Pfizer Inc and PhotoThera, Inc, for clinical trial participation.
b
Dr Singhal reports no disclosure.

Pedro Weisleder, MD, PhD


Professor, Department of Pediatrics, Nationwide Children’s Hospital, The Ohio State University,
Columbus, Ohio
a
Dr Weisleder serves as a consultant for the Medical Review Institute of America and as associate editor of the
Journal of Child Neurology.
b
Dr Weisleder discusses the unlabeled use of disease-modifying therapies during attempts at conception and during
pregnancy.

Mark S. Yerby, MD, MPH, FAAN


Founder, North Pacific Epilepsy Research; Chair, Scientific Advisory Board, North American
Epilepsy and Pregnancy Registry; Associate Clinical Professor of Neurology and Public Health,
Oregon Health & Science University, Portland, Oregon
a
Dr Yerby serves on the speakers bureaus for Lundbeck and Supernus Pharmaceuticals, Inc.
b
Dr Yerby reports no disclosure.

QUESTION WRITERS

Eduardo E. Benarroch, MD, DSci, FAAN


Professor of Neurology, Mayo Clinic, Rochester, Minnesota
a
Dr Benarroch has received personal compensation in an editorial capacity for Neurology.
b
Dr Benarroch reports no disclosure.

Ronnie Bergen, MD
Assistant Professor of Clinical Neurology, University of Arizona College of Medicine, Tucson,
Arizona; Staff Neurologist, Southern Arizona VA Healthcare System, Tucson, Arizona
a,b
Dr Bergen reports no disclosures.

a
Relationship Disclosure
b
Unlabeled Use of Products/Investigational Use Disclosure

Died April 20, 2013

Methods of Participation and Instructions for Use

Copyright © American Academy of Neurology. Unauthorized reproduction of this article is prohibited.


Continuum: Lifelong Learning in Neurology® is designed to help practicing neurologists stay

abreast of advances in the field while simultaneously developing lifelong self-directed learning

skills. In Continuum, the process of absorbing, integrating, and applying the material presented is

as important as, if not more important than, the material itself.

The goals of Continuum include disseminating up-to-date information to the practicing

neurologist in a lively, interactive format; fostering self-assessment and lifelong study skills;

encouraging critical thinking; and, in the final analysis, strengthening and improving patient

care.

Each Continuum issue is prepared by distinguished faculty who are acknowledged leaders in

their respective fields. Six issues are published annually and are composed of review articles,

case-based discussions on ethical and practice issues related to the issue topic, coding

information, and comprehensive CME and self-assessment offerings, including a self-assessment

pretest, postreading CME test questions with preferred responses, and a patient management

problem. For detailed instructions regarding Continuum CME and self-assessment activities, visit

aan.com/continuum/cme.

The review articles emphasize clinical issues emerging in the field in recent years. Case reports

and vignettes are used liberally, as are tables and illustrations. Video material relating to the

issue topic accompanies issues when applicable.

The text can be reviewed and digested most effectively by establishing a regular schedule of

study in the office or at home, either alone or in an interactive group. If subscribers use such

regular and perhaps new study habits, Continuum’s goal of establishing lifelong learning patterns

can be met.

Copyright © American Academy of Neurology. Unauthorized reproduction of this article is prohibited.


LIFELONG LEARNING IN NEUROLOGY ®

Neurology of Pregnancy
Volume 20 Number 1 February 2014

CONTRIBUTORS
Autumn Klein, MD, PhD, Guest Editor†
Chief of the Division of Women’s Neurology, Assistant Professor of Neurology
and Obstetrics and Gynecology, University of Pittsburgh Medical Center,
Pittsburgh, Pennsylvania

Amaal AlDakheel, MD
Clinical Fellow, The Morton and Gloria Shulman Movement Disorders Centre in the
Edmond J. Safra Program for Parkinson’s Disease, Toronto Western Hospital, Krembil
Neuroscience Centre, University Health Network, University of Toronto, Toronto,
Ontario, Canada
a,bDr AlDakheel reports no disclosures.

Riley M. Bove, MD
Instructor in Neurology, Harvard Medical School; Associate Neurologist,
Brigham and Women’s Hospital, Boston, Massachusetts
a,bDr Bove reports no disclosures.

Patricia K. Coyle, MD, FAAN


Professor of Neurology and Vice Chair (Clinical Affairs); Director,
Stony Brook MS Comprehensive Care Center, Stony Brook University,
Stony Brook, New York
aDr Coyle has consulted for Acorda Therapeutics; Accordant Health Services; Bayer AG;
Biogen Idec; Genentech, Inc; Genzyme Corporation; Merck KGaA; Mylan Inc;
Novartis Corporation; and Teva Pharmaceuticals. Dr Coyle receives clinical trial support
from Actelion Pharmaceuticals; Novartis Corporation; and Opexa Therapeutics, Inc.
bDr Coyle discusses the unlabeled use of IV immunoglobulin postpartum.

aRelationship Disclosure
bUnlabeled Use of Products/Investigational Use Disclosure
†Died April 20, 2013

Continuum (Minneap Minn) 2014;20(1) www.ContinuumJournal.com

Copyright © American Academy of Neurology. Unauthorized reproduction of this article is prohibited.


LIFELONG LEARNING IN NEUROLOGY ®

CONTRIBUTORS continued
Carolina De Jesus-Acosta, MD
Fellow, Neuromuscular Medicine, Duke University Medical Center, Durham,
North Carolina
aDr De Jesus-Acosta reports no disclosure.
bDr De Jesus-Acosta discusses the use of drugs for the treatment of myasthenia gravis,
none of which are labeled by the US Food and Drug Administration for use in pregnancy.

Kathleen B. Digre, MD, FAAN


Professor, Departments of Neurology of Ophthalmology; Adjunct Professor,
Department of Obstetrics and Gynecology, Moran Eye Center, University of Utah
aDr Digre has received a grant from the National Eye Institute and salary support from the
Neuro-ophthalmology Research Disease Investigator Consortium.
bDr Digre discusses the use of several drugs for the treatment of neuro-ophthalmic disorders,
none of which are labeled by the US Food and Drug Administration for use in pregnancy.

Steven K. Feske, MD
Associate Professor of Neurology, Harvard Medical School; Director, Stroke Division,
Brigham and Women’s Hospital, Boston, Massachusetts
aDr Feske has received royalties from Elsevier for his role as editor of Office Practice of
Neurology, 2nd Edition, and receives research support from the National Institute of
Neurological Disorders and Stroke.
bDr Feske reports no disclosure.

Amanda C. Guidon, MD
Instructor of Neurology, Harvard Medical School; Assistant in Neurology,
Massachusetts General Hospital, Boston, Massachusetts
a,bDr Guidon reports no disclosures.

Cynthia L. Harden, MD
Professor of Neurology; Chief, Division of Epilepsy and Electroencephalography,
Hofstra North Shore–LIJ School of Medicine, Great Neck, New York
aDr Harden has received personal compensation for activities with GlaxoSmithKline; Lundbeck;
UCB SA; and Upsher-Smith Laboratories, Inc. Dr Harden has served in an editorial capacity for
UpToDate and received research support from the Epilepsy Therapy Project.
bDr Harden reports no disclosure.

aRelationship Disclosure
bUnlabeled Use of Products/Investigational Use Disclosure

www.ContinuumJournal.com February 2014

Copyright © American Academy of Neurology. Unauthorized reproduction of this article is prohibited.


LIFELONG LEARNING IN NEUROLOGY ®

CONTRIBUTORS continued
Joseph S. Kass, MD, JD
Associate Professor of Neurology, Psychiatry, and Medical Ethics, Baylor College
of Medicine, Houston, Texas
a,bDr Kass reports no disclosures.

Krista Kinard, MD
Instructor, Department of Ophthalmology, Moran Eye Center,
University of Utah
aDr Kinard reports no disclosure.
bDr Kinard discusses the use of several drugs for the treatment of neuro-ophthalmic
disorders, none of which are labeled by the US Food and Drug Administration for use
in pregnancy.

Joshua P. Klein, MD, PhD


Chief, Division of Hospital Neurology, Brigham and Women’s Hospital;
Assistant Professor of Neurology and Radiology, Harvard Medical School,
Boston, Massachusetts
aDr Klein receives financial compensation for serving on the editorial board of the Journal
of Neuroimaging and AccessMedicine Neurology, and royalties from McGraw-Hill for
Adams and Victor’s Principles of Neurology.
bDr Klein reports no disclosure.

E. Anne MacGregor, MB BS, MD, FFSRH, MICR


Associate Specialist, Barts Sexual Health Centre, St Bartholomew’s Hospital;
Honorary Professor, Centre for Neuroscience and Trauma, Blizard Institute of
Cell and Molecular Science, Barts and the London School of Medicine and
Dentistry, London, England, United Kingdom
aProfessor MacGregor has acted as a paid consultant to and/or her department has received
research funding from Addex Therapeutics; Allergan, Inc; AstraZeneca; Berlin-Chemie AG;
BTG International Ltd; Endo Pharmaceuticals Inc; GlaxoSmithKline; the Menarini Group;
Merck & Co, Inc; POZEN Inc; and UniPath.
bProfessor MacGregor discusses the use of several drugs for the treatment of headaches,
none of which are labeled by the US Food and Drug Administration for use in pregnancy.

E. Wayne Massey, MD, FAAN


Professor, Department of Neurology, Duke University Medical Center, Durham,
North Carolina
a,bDr Massey reports no disclosures.

aRelationship Disclosure
bUnlabeled Use of Products/Investigational Use Disclosure

Continuum (Minneap Minn) 2014;20(1) www.ContinuumJournal.com

Copyright © American Academy of Neurology. Unauthorized reproduction of this article is prohibited.


LIFELONG LEARNING IN NEUROLOGY ®

CONTRIBUTORS continued
Janice M. Massey, MD, FAAN
Professor, Department of Neurology; Chief, Neuromuscular Division, Duke
University Medical Center, Durham, North Carolina
aDr Massey has received educational grants from Allergan, Inc; and Merz Pharma.
bDr Massey discusses the use of drugs for the treatment of myasthenia gravis, none of
which are labeled by the US Food and Drug Administration for use in pregnancy.

Janis M. Miyasaki, MD, MEd, FRCP, FAAN


Associate Professor, Department of Medicine, and Associate Clinical
Director, The Movement Disorders Centre, Toronto Western Hospital, Krembil
Neuroscience Centre, University Health Network, University of Toronto,
Toronto, Ontario; Associate Professor, Neurology, University of Alberta,
Edmonton, Alberta, Canada
aDr Miyasaki has served as a speaker or on advisory boards for Novartis Corporation and
Teva Pharmaceuticals. Dr Miyasaki has received research support from the Canadian Agency
for Drugs and Technologies in Health, the Canadian Institute for Health Research, the
Michael J. Fox Foundation for Parkinson's Research, the National Center for Complementary
and Alternative Medicine, the National Parkinson Foundation, the NIH, the Ontario Drug
Benefits Program, and the Ontario Ministry of Health and Long-Term Care. The Movement
Disorders Centre at Toronto Western Hospital has received research support from Teva
Pharmaceuticals.
bDr Miyasaki reports no disclosure.

Bethanie N. Morgan-Followell, MD
Assistant Professor of Pediatrics, Section of Child Neurology, Nationwide
Children’s Hospital, The Ohio State University, Columbus, Ohio
aDr Morgan-Followell reports no disclosure.
bDr Morgan-Followell discusses the unlabeled use of disease-modifying therapies during
attempts at conception and during pregnancy.

Jacqueline A. Nicholas, MD, MPH


Assistant Professor of Neurology, Division of Neuroimmunology,
The Ohio State University Medical Center, Columbus, Ohio
aDr Nicholas’ fellowship is funded through a Sylvia Lawry Physician Fellowship grant from
the National Multiple Sclerosis Society, and she receives additional funding for clinical
research from the National Multiple Sclerosis Society as an assistant professor.
bDr Nicholas discusses the unlabeled use of disease-modifying therapies during attempts
at conception and during pregnancy.

Laura B. Powers, MD, FAAN


Dr Powers is retired from private practice.
aDr Powers serves as ICD-9-CM Advisor for the Coding Subcommittee of the AAN Medical
Economics and Management Committee and serves in an editorial capacity for Neurology:
Clinical Practice.
bDr Powers reports no disclosure.

aRelationship Disclosure
bUnlabeled Use of Products/Investigational Use Disclosure

www.ContinuumJournal.com February 2014

Copyright © American Academy of Neurology. Unauthorized reproduction of this article is prohibited.


LIFELONG LEARNING IN NEUROLOGY ®

CONTRIBUTORS continued
Aneesh B. Singhal, MD
Associate Professor of Neurology, Harvard Medical School; Director, Neurology
Quality and Safety, Massachusetts General Hospital, Boston, Massachusetts
aDr Singhal has served as a consultant for Biogen Idec and as a medical expert witness in
cases of stroke. Dr Singhal’s spouse holds stock or stock options greater than 5% of the
company or greater than $10,000 in value in Biogen Idec and Vertex Pharmaceuticals
Incorporated. Dr Singhal has received research support from the National Institute of
Neurological Disorders and Stroke, and his institution has received research support from
Pfizer Inc and PhotoThera, Inc, for clinical trial participation.
bDr Singhal reports no disclosure.

Pedro Weisleder, MD, PhD


Professor, Department of Pediatrics, Nationwide Children’s Hospital, The Ohio
State University, Columbus, Ohio
aDr Weisleder serves as a consultant for the Medical Review Institute of America and as
associate editor of the Journal of Child Neurology.
bDr Weisleder discusses the unlabeled use of disease-modifying therapies during attempts
at conception and during pregnancy.

Mark S. Yerby, MD, MPH, FAAN


Founder, North Pacific Epilepsy Research; Chair, Scientific Advisory Board,
North American Epilepsy and Pregnancy Registry; Associate Clinical Professor
of Neurology and Public Health, Oregon Health & Science University,
Portland, Oregon
aDr Yerby serves on the speakers bureaus for Lundbeck and Supernus Pharmaceuticals, Inc.
bDr Yerby reports no disclosure.

SELF-ASSESSMENT AND CME TEST WRITERS


Eduardo E. Benarroch, MD, DSci, FAAN
Professor of Neurology, Mayo Clinic, Rochester, Minnesota
aDr Benarroch has received personal compensation in an editorial capacity for Neurology.
bDr Benarroch reports no disclosure.

Ronnie Bergen, MD
Assistant Professor of Clinical Neurology, University of Arizona College of
Medicine, Tucson, Arizona; Staff Neurologist, Southern Arizona VA Healthcare
System, Tucson, Arizona
a,bDr Bergen reports no disclosures.

aRelationship Disclosure
bUnlabeled Use of Products/Investigational Use Disclosure

Continuum (Minneap Minn) 2014;20(1) www.ContinuumJournal.com

Copyright © American Academy of Neurology. Unauthorized reproduction of this article is prohibited.


Volume 20 n Number 1 n February 2014

LIFELONG LEARNING IN NEUROLOGY ®


www.ContinuumJournal.com

Neurology of Pregnancy
Guest Editor: Autumn Klein, MD, PhD

Dedication to Autumn Klein, MD, PhD (1971–2013). . . . . . . . . . . . . . . . . . . . . . . . 19


Steven K. Feske, MD; Riley M. Bove, MD

Editor’s Preface . . . . . . . . . . . . . . . . . . . . . . . . . . . . . . . . . . . . . . . . . . . . . . . . . . . . . . . . . 21

REVIEW ARTICLES
Neuroradiology in Women of Childbearing Age . . . . . . . . . . . . . . . . . . . . . . . . . . . 23
Riley M. Bove, MD; Joshua P. Klein, MD, PhD

Multiple Sclerosis in Pregnancy. . . . . . . . . . . . . . . . . . . . . . . . . . . . . . . . . . . . . . . . . . . 42


Patricia K. Coyle, MD, FAAN

Pregnancy and Epilepsy . . . . . . . . . . . . . . . . . . . . . . . . . . . . . . . . . . . . . . . . . . . . . . . . . 60


Cynthia L. Harden, MD

Cerebrovascular Disorders Complicating Pregnancy . . . . . . . . . . . . . . . . . . . . . . 80


Steven K. Feske, MD; Aneesh B. Singhal, MD

Peripheral Neuropathies in Pregnancy . . . . . . . . . . . . . . . . . . . . . . . . . . . . . . . . . . 100


E. Wayne Massey, MD, FAAN; Amanda C. Guidon, MD

Pregnancy and Myasthenia Gravis . . . . . . . . . . . . . . . . . . . . . . . . . . . . . . . . . . . . . . 115


Janice M. Massey, MD, FAAN; Carolina De Jesus-Acosta, MD

Headache in Pregnancy . . . . . . . . . . . . . . . . . . . . . . . . . . . . . . . . . . . . . . . . . . . . . . . . 128


E. Anne MacGregor, MB BS, MD, FFSRH, MICR

Movement Disorders in Pregnancy. . . . . . . . . . . . . . . . . . . . . . . . . . . . . . . . . . . . . . 148


Janis M. Miyasaki, MD, MEd, FRCP, FAAN; Amaal AlDakheel, MD

Neuro-ophthalmic Disorders in Pregnancy . . . . . . . . . . . . . . . . . . . . . . . . . . . . . . 162


Kathleen B. Digre, MD, FAAN; Krista Kinard, MD

Volume 20 n Number 1 www.ContinuumJournal.com 7

Copyright © American Academy of Neurology. Unauthorized reproduction of this article is prohibited.


LIFELONG LEARNING IN NEUROLOGY ®

ETHICAL PERSPECTIVES
Reproductive Issues in Women With Multiple Sclerosis:
Ethical Considerations . . . . . . . . . . . . . . . . . . . . . . . . . . . . . . . . . . . . . . . . . . . . . . . . . . 177
Bethanie N. Morgan-Followell, MD; Jacqueline A. Nicholas, MD, MPH;
Pedro Weisleder, MD, PhD

PRACTICE ISSUES
Epilepsy and Pregnancy: A Practical Approach
to Mitigating Legal Risk . . . . . . . . . . . . . . . . . . . . . . . . . . . . . . . . . . . . . . . . . . . . . . . . . 181
Joseph S. Kass, MD, JD

Coding in Pregnancy With a Focus on Epilepsy . . . . . . . . . . . . . . . . . . . . . . . . . . . . 186


Mark S. Yerby, MD, MPH, FAAN; Laura B. Powers, MD, FAAN

APPENDIX
Appendix A: US Food and Drug Administration–Assigned
Pregnancy Categories as Used in the Drug Formulary . . . . . . . . . . . . . . . . . . . . . 191

SELF-ASSESSMENT AND CME


Learning Objectives . . . . . . . . . . . . . . . . . . . . . . . . . . . . . . . . . . . . . . . . . . . . . . . . . . . . . . 9
Instructions for Completing the Self-Assessment Pretest and Tally Sheet . . . . . . . 11
Self-Assessment Pretest . . . . . . . . . . . . . . . . . . . . . . . . . . . . . . . . . . . . . . . . . . . . . . . . . 13
Instructions for Completing CME and Tally Sheet . . . . . . . . . . . . . . . . . . . . . . . . . 195
Postreading CME Test . . . . . . . . . . . . . . . . . . . . . . . . . . . . . . . . . . . . . . . . . . . . . . . . . . . 197
Postreading CME Test—Preferred Responses . . . . . . . . . . . . . . . . . . . . . . . . . . . . . 209
Patient Management Problem . . . . . . . . . . . . . . . . . . . . . . . . . . . . . . . . . . . . . . . . . . . 230
Patient Management Problem—Preferred Responses . . . . . . . . . . . . . . . . . . . . . 235
Riley M. Bove, MD

Index . . . . . . . . . . . . . . . . . . . . . . . . . . . . . . . . . . . . . . . . . . . . . . . . . . . . . . . . . . . . . . . . . . 245
List of Abbreviations . . . . . . . . . . . . . . . . . . . . . . . . . . . . . . . . . . . . . . . . . . . .Back Cover

8 www.ContinuumJournal.com February 2014

Copyright © American Academy of Neurology. Unauthorized reproduction of this article is prohibited.


Learning Objectives
Upon completion of this Continuum: Lifelong Learning in Neurology Neurology of
Pregnancy issue, participants will be able to:

Summarize safety concerns associated with diagnostic neuroimaging in women


s

during pregnancy and lactation


Discuss diagnostic modalities and radiologic features of neurologic conditions
s

encountered by pregnant women


Discuss the current understanding of the interactions between multiple sclerosis
s

and pregnancy and their implications for reproductive counseling, and discuss
the issues related to disease-modifying therapy and therapy of acute
exacerbations of multiple sclerosis during pregnancy and lactation
Explain issues regarding the management of women with epilepsy and
s

pregnancy, including preconception planning, antiepileptic drug effects on


the exposed offspring, and consequences of seizures during pregnancy, with
an emphasis on counseling and risk management
Identify the ways in which the physiologic changes of pregnancy affect risk of
s

ischemic and hemorrhagic stroke, and discuss an approach to the diagnosis and
treatment of ischemic and hemorrhagic stroke in pregnancy and the puerperium
Outline the most common peripheral neuropathic disorders in pregnancy with a
s

focus on clinical recognition, diagnosis, and treatment


Analyze available information regarding expectations and management for
s

patients with myasthenia gravis during childbearing years, pregnancy,


and postpartum
Diagnose and manage primary and secondary headaches that may occur during
s

pregnancy and postpartum


Describe movement disorders that occur during pregnancy, the treatment of
s

preexisting movement disorders, and the influence the pregnant state has on
movement disorder symptoms
Evaluate and treat neuro-ophthalmic disorders in pregnant patients
s

Summarize the available literature on reproductive issues in women with multiple


s

sclerosis and provide sound, objective counseling to facilitate well-informed,


autonomous decision making by patients
Apply a practical framework for understanding, identifying, and managing legal
s

risk when treating pregnant women with epilepsy


Demonstrate the use of Current Procedural Terminology and diagnosis codes for
s

the evaluation and management of neurologic disease in pregnant women, with a


focus on those with epilepsy

Core Competencies
This Continuum: Lifelong Learning in Neurology Neurology of Pregnancy issue
covers the following core competencies:

Patient Care
s

Medical Knowledge
s

Practice-Based Learning and Improvement


s

Interpersonal and Communication Skills


s

Professionalism
s

Systems-Based Practice
s

Continuum (Minneap Minn) 2014;20(1) www.ContinuumJournal.com

Copyright © American Academy of Neurology. Unauthorized reproduction of this article is prohibited.


Copyright © American Academy of Neurology. Unauthorized reproduction of this article is prohibited.
SELF-ASSESSMENT


INSTRUCTIONS FOR THE Neurology of Pregnancy
SELF-ASSESSMENT PRETEST
To earn American Board of Psychiatry and Volume 20 n Number 1 n February 2014
Neurology (ABPN) Maintenance of Certification
(MOC) self-assessment participation credit for this Tally Sheet
issue of , complete the Self-Assessment
Pretest in one of three ways: SELF-ASSESSMENT PRETEST RESPONSES
1. Go to www.aan.com/continuum/cme and complete After completing this tally sheet, please enter your
the test online (available to subscribers only), or
answers online at
2. Read through the questions in the issue and www.aan.com/continuum/cme.
mark your answers on the adjacent tally sheet
before entering them online at 1. a b c d e 21. a b c d e
www.aan.com/continuum/cme, or
3. Request a faxable paper scorecard if you do not 2. a b c d e 22. a b c d e
have computer access or you are a nonsubscriber
3. a b c d e 23. a b c d e
who has purchased a single issue (send an email to
ContinuumCME@aan.com); however, we encourage 4. a b c d e 24. a b c d e
all subscribers to use the online system.
Once you have completed the Self-Assessment 5. a b c d e 25. a b c d e
Pretest, use your results as a guide as you read the 6. a b c d e 26. a b c d e
issue. To obtain self-assessment CME credits, you
must complete the Postreading CME Test at the end 7. a b c d e 27. a b c d e
of the issue, along with an issue evaluation. No
self-assessment credit will be awarded for 8. a b c d e 28. a b c d e
completing the Self-Assessment Pretest alone.
9. a b c d e 29. a b c d e
Upon completion of the Self-Assessment Pretest and
Postreading CME Test, participants may earn up to 12 10. a b c d e 30. a b c d e
AMA PRA Category 1 CreditsTM toward ABPN MOC
self-assessment (part 2). Self-assessment credits 11. a b c d e 31. a b c d e
earned will appear on AAN member CME transcripts 12. a b c d e 32. a b c d e
within 2 business days and may be viewed at
www.aan.com/cme-and-moc/track-and-view-transcript/. 13. a b c d e 33. a b c d e
AAN nonmember subscribers may request a
transcript of credits earned by contacting AAN 14. a b c d e 34. a b c d e
Member Services at memberservices@aan.com
or (800) 879-1960. 15. a b c d e 35. a b c d e
AMA PRA Category 1 Credits not designated for 16. a b c d e 36. a b c d e
self-assessment may be obtained by completing
only the 40 Postreading CME Test (up to 10) and/or 17. a b c d e 37. a b c d e
the Patient Management Problem (up to 2) found at
18. a b c d e 38. a b c d e
the end of the issue.
Participants have up to 3 years from the date of 19. a b c d e 39. a b c d e
publication to earn CME credits. No CME will be
awarded for the Neurology of Pregnancy issue after 20. a b c d e 40. a b c d e
February 28, 2017.

The ABPN has reviewed Continuum: Lifelong


Learning in Neurology and has approved this
product as part of a comprehensive lifelong
learning and self-assessment program, which is
mandated by the American Board of Medical
Specialties (ABMS) as a necessary component of
maintenance of certification.


Continuum (Minneap Minn) 2014;20(1) www.ContinuumJournal.com 11

Copyright © American Academy of Neurology. Unauthorized reproduction of this article is prohibited.


Copyright © American Academy of Neurology. Unauthorized reproduction of this article is prohibited.
Self-Assessment

Self-Assessment Pretest 3. A 32-year-old woman is evaluated for abrupt-onset severe


headache that developed after 5 hours of labor. Examination
shows right eyelid ptosis; under dim light, her right pupil is
3 mm and the left pupil 5 mm in diameter. Pupil reactions are
The Self-Assessment Pretest is designed normal, and no papilledema is present. Her blood pressure is
to help neurologists meet the American Board of Psychiatry 160/90 mm Hg with a heart rate of 90 beats per minute.
and Neurology (ABPN) self-assessment and lifelong learning What is the most likely diagnosis?
component (part 2) for Maintenance of Certification (MOC). A. carotid artery dissection
To obtain self-assessment CME credits, complete the Self- B. cerebral venous sinus thrombosis
Assessment Pretest (40 questions) before reading this issue. C. pituitary apoplexy
Pretest results are intended to help you focus your learning D. posterior communicating artery aneurysm
E. reversible cerebral vasoconstriction syndrome
by identifying your current knowledge gaps with regard to
the topic. After submitting your pretest responses, study the 4. A 25-year-old primigravida is evaluated at the second
entire issue using your pretest results as a guide to direct trimester of pregnancy for paresthesia in both hands over
your learning. Then complete the Postreading CME Test at the past month. The symptoms worsen at night or when the
the end of the issue. Upon completion of both the patient is driving or typing. Examination shows reduced
Self-Assessment Pretest and the pinprick sensation in the median nerve distribution bilaterally.
Postreading CME Test, you may earn up to 12 AMA PRA She has bilateral Tinel signs over the median nerves at the
Category 1 Creditsi toward self-assessment. wrists, with no weakness in the intrinsic hand muscles. Which of
the following is a feature of this condition associated with
AMA PRA Category 1 Credits for
pregnancy?
may be obtained by completing only the Postreading CME
Test, but these credits will not qualify for self-assessment A. a significant proportion of patients may have ongoing
without completion of the Self-Assessment Pretest. symptoms postpartum
B. it is generally more severe than idiopathic forms
C. it is typically a harbinger of gestational diabetes
D. it typically develops early and improves during the third
trimester of pregnancy
E. surgical decompression is necessary in most cases

1. A patient with myasthenia gravis in her third trimester of 5. A 32-year-old woman with relapsing/remitting multiple
pregnancy develops acute dysuria and urgency. A urinalysis sclerosis (MS) with a baseline mild spastic ataxia develops
shows a urinary tract infection. She is allergic to penicillin. Which acute right arm and leg weakness in her second trimester of
of the following antibiotics would be most appropriate in this pregnancy. Before conception, her MS had been managed
patient, assuming the organism is susceptible to the agent? with oral fingolimod, and she had experienced about one
clinical relapse per year. Examination shows a new mild right
A. ampicillin hemiparesis and sensory level to pin at C4. Which of the
B. ciprofloxacin following management options is most appropriate in this
C. erythromycin patient?
D. gentamycin
E. trimethoprim-sulfamethoxazole A. initiation of glatiramer acetate
B. IV methylprednisolone therapy
2. A 28-year-old-woman with a 2-year history of depression is C. MRI of the cervical spine with gadolinium
evaluated in the first trimester of her first pregnancy for D. MRI of the cervical spine without contrast
abnormal facial movements and gait disorder. Examination E. treatment with IV immunoglobulin
shows dystonic facial grimacing, parkinsonism, and ataxia.
She has Kayser-Fleischer rings. Serum ceruloplasmin and 6. A 33-year-old woman is evaluated for right lower extremity
copper are undetectable. Genetic studies show a mutation in weakness noticed immediately after vaginal delivery by
the gene encoding P-type copper-transporting adenosine forceps under epidural anesthesia. The patient noticed
triphosphatase. Which of the following is the most appropriate difficulty standing due to buckling of the right leg and also
approach in this patient? noted numbness in her right anterior thigh and medial lower
leg. Examination shows weakness in the right quadriceps
A. hold therapy until the end of pregnancy muscles, with normal strength of the hip abductors and
B. start D-penicillamine after the second trimester and substitute adductors as well as lower leg muscles. Sensation to pinprick
trientine during lactation is diminished in the right anterior thigh and medial lower leg.
C. start trientine now and substitute zinc acetate during lactation The right patellar reflex is absent but the ankle jerk is
D. start zinc acetate after the second trimester and maintain preserved. What is the most likely cause of the patient’s
throughout lactation symptoms and findings?
E. start zinc acetate now and substitute D-penicillamine during
lactation A. cauda equina compression by epidural hematoma
B. femoral nerve compression at the inguinal ligament
C. L4 root compression by disk
D. obturator nerve compression at the pelvic brim
E. upper lumbosacral plexus compression by forceps

Continuum (Minneap Minn) 2014;20(1):13–18 www.ContinuumJournal.com 13

Copyright © American Academy of Neurology. Unauthorized reproduction of this article is prohibited.


Self-Assessment

7. A 32-year-old woman is evaluated at the 30th week of 10. A 33-year-old primigravida is evaluated for severe headache
gestation for left posterior auricular pain, facial weakness, and and blurred vision over the past 36 hours. She is in her third
hyperacusis that developed over 2 days. Examination shows trimester of pregnancy. Ophthalmic examination shows
weakness in her left upper and lower facial muscles and lack bilaterally reduced visual acuity, normal pupil light reflexes,
of taste in the anterior two thirds of the left side of her and papilledema. She is unable to describe the whole scene
tongue. Which of the following statements regarding treat- in the Cookie Theft Picture, reach under visual guidance, or
ment or prognosis most accurately characterizes this condi- redirect her gaze toward a peripheral stimulus. Her blood
tion in pregnancy? pressure is 165/100 mm Hg, with heart rate of 60 beats
A. acyclovir administration within 3 to 7 days of symptom per minute. Urinalysis shows proteinuria of 6 g in 24 hours.
onset is the treatment of choice Which of the following is the most likely cause of the patient’s
B. corticosteroid treatment should be considered after the visual symptoms?
first trimester of an uncomplicated pregnancy A. cerebral venous thrombosis
C. the course is typically less severe than in nonpregnant women B. intracerebral hemorrhage
D. recurrence in future pregnancies is common, particularly in C. posterior cerebral artery infarction
cases staring in the first trimester D. posterior reversible encephalopathy syndrome
E. valacyclovir administration within 3 to 7 days of symptom E. reversible cerebral vasoconstriction syndrome
onset is the treatment of choice
11. A patient with a history of migraine without aura whose
8. A previously healthy 23-year-old woman develops fatigable typical headache frequency is once per month delivers her
ptosis during her first trimester of pregnancy. Electrophysio- first child at full term after an uncomplicated pregnancy. She
logic studies are consistent with myasthenia gravis; acetyl- had undergone epidural anesthesia for delivery. Two days
choline receptor antibodies are positive. The patient’s after delivery, she develops a constant holocephalic headache
symptoms remain well controlled without specific treatment with nausea that is markedly worse with sitting or standing
throughout pregnancy but worsen 3 weeks after delivery, and relieved when supine. No change in vision is reported.
now involving the bulbar muscles with fatigable dysphagia She is given IV hydration and analgesics without resolution of
and dysarthria. She is breast-feeding her baby. Which of the her headaches over the next 3 days. Noncontrast head CT is
following immunomodulatory therapies is most appropriate normal. Which of the following treatments is most likely to be
in this setting? beneficial in this patient?
A. azathioprine A. antibiotic therapy
B. cyclosporine B. epidural blood patch
C. methotrexate C. intravenous heparin
D. mycophenolate mofetil D. oral propranolol
E. prednisone E. subcutaneous sumatriptan

9. A 33-year-old woman with seropositive ocular myasthenia 12. A 33-year-old primigravida in her third trimester of pregnancy
gravis (MG) controlled with pyridostigmine bromide becomes is evaluated for orbital pain and diplopia that developed over
pregnant and requests counseling about the effects of the past 2 weeks. Examination shows asymmetric proptosis,
pregnancy on her condition. Which of the following issues conjunctival injection, and bilateral impairment of eye
should be discussed with the patient? movements, particularly adduction. Visual acuity, pupil re-
A. her disease may prolong labor because of uterine weakness actions, and funduscopy are normal. Which of the following
B. her fetus is at high risk of developing MG later in life diagnoses is most likely in this patient?
C. her fetus is at less than 5% risk to develop transient neonatal A. carotid-cavernous fistula
weakness B. cavernous sinus thrombosis
D. her relatively mild disease predicts remission of her symptoms C. Graves disease
during pregnancy D. inflammatory orbital pseudotumor
E. worsening of symptoms may occur in about 20% to 30% of E. orbital varices
patients
13. Which of the following characteristics of a patient’s myasthe-
nia gravis is most predictive of exacerbation of disease during
pregnancy?
A. no reliable predictors exist
B. positive acetylcholine-receptor antibody status
C. prior bulbar weakness
D. prior poor response to pyridostigmine
E. worsening with previous pregnancy

14 www.ContinuumJournal.com February 2014

Copyright © American Academy of Neurology. Unauthorized reproduction of this article is prohibited.


Self-Assessment

14. Which of the following medications would be most appropriate 18. A previously healthy 33-year-old woman is evaluated during
if migraine prophylaxis is necessary during pregnancy? the first trimester of her first pregnancy for the subacute
A. gabapentin onset of rapid, irregular, random jerky movements of the face
B. propranolol and limbs. She takes no medications, over-the-counter
C. topiramate agents, or herbal substances. She has no family history of a
D. valproic acid similar disorder. Examination shows chorea, dysarthria, and
E. verapamil motor impersistence. Neurologic examination is otherwise
normal. She has no Kayser-Fleischer rings. Her general
15. A 22-year-old woman is evaluated for severe right shoulder physical examination and thyroid panel are normal. Drug
pain and difficulty lifting her right arm that developed after screen is negative. Which of the following studies is most
delivery of her second baby 6 days ago. The patient had appropriate at this point?
similar symptoms in the left upper limb during her previous A. antinuclear antibodies
pregnancy 2 years ago, resulting in residual weakness in the B. CSF for oligoclonal bands
left shoulder muscles. Family history is positive for similar C. Huntingtin gene testing
episodes. Examination shows short stature, hypotelorism, D. serum ferritin
epicanthal folds, and dysmorphic ears. She has weakness in E. urine organic acids
the right infraspinatus, deltoid, biceps and brachioradialis
muscles, absent right biceps and brachioradialis reflexes, and 19. Of the following factors in a multiple sclerosis patient’s
numbness in the right shoulder and lateral distal forearm. disease course, which has been shown most consistently to
There is also winging of the left scapula. This patient’s most be associated with increased risk of postpartum relapse?
likely disorder has been linked to mutations of which of the A. first full-term pregnancy
following genes? B. high lesion burden on MRI
A. connexin-32 (GJB1) C. high rate of prepregnancy disease activity
B. dynactin-1 (DCTN1) D. low baseline disability score
C. myelin protein zero (MPZ) E. quiescent disease during pregnancy
D. peripheral myelin protein-22 (PMP22)
E. septin-9 (SEPT9) 20. A 23-year-old woman in her second trimester of pregnancy is
evaluated for pain, numbness, and paresthesia in the left
16. Which of the following statements is most accurate regarding anterolateral thigh. The symptoms increase with standing and
fetal exposure to radiation during a noncontrast head CT scan walking. Examination shows sensory loss in the left
of the pregnant mother? anterolateral thigh with preserved strength and reflexes in
A. it is equivalent to the dose scattered throughout the mother’s the lower limbs. Which of the following approaches is most
body appropriate?
B. it is offset by use of a lead shield A. amitriptyline therapy
C. it may be up to 10 times higher than radiation from the B. avoidance of regional anesthesia during delivery
environment C. avoidance of tight-fitting clothes
D. the greatest risk is after week 20 D. gabapentin therapy
E. the radiation dose is impossible to estimate E. L2-L3 epidural block

17. A previously healthy 28-year-old woman develops chorea in 21. A 34-year-old primigravida is evaluated at 38 weeks’ gestation
the eighth week of her first pregnancy. The symptoms are for abrupt onset of severe headache, blurred vision, confu-
mild and do not interfere with her activities. She takes no sion, and a recent generalized tonic-clonic seizure. Her blood
medications or natural products. Her family history is pressure is 170/105 mm Hg, with heart rate of 80 beats per
negative for neurologic or psychiatric disorders. Her neuro- minute. Urinalysis shows proteinuria of 8 g in 24 hours. Fluid-
logic examination, apart from mild chorea, is normal. General attenuated inversion recovery (FLAIR) imaging shows in-
laboratory tests, including thyroid panel and serum cerulo- creased signal intensity in both parieto-occipital regions.
plasmin, are normal. Connective tissue disease markers and Magnetic resonance angiography shows areas of segmental
antiphospholipid antibodies are negative. Which of the narrowing and dilatation of large and medium-size cerebral
following should be considered in discussing the situation arteries. Which of the following is the single most appropriate
with the patient? treatment for this condition?
A. her symptoms may resolve spontaneously after delivery A. furosemide
B. she has an 80% risk of recurrence in future pregnancies B. gabapentin
C. she should be started on antidopaminergic treatment to C. magnesium sulfate
prevent further progression D. methylprednisolone
D. she should receive gene testing for Huntington disease E. verapamil
E. tetrabenazine would be the safest option if her symptoms
worsen within the first trimester

Continuum (Minneap Minn) 2014;20(1):13–18 www.ContinuumJournal.com 15

Copyright © American Academy of Neurology. Unauthorized reproduction of this article is prohibited.


Self-Assessment

22. A patient diagnosed 3 months ago with ocular myasthenia 26. A 43-year-old woman is evaluated in the third trimester of
gravis that is well controlled on pyridostigmine becomes pregnancy for progressively worsening headache and nausea
pregnant and requires advice regarding treatment of her over the past week. Neurologic examination is normal except
condition. Which of the following approaches is most for papilledema. MRI shows a hypertense signal in the area of
appropriate at this point? the left transverse sinus on fluid-attenuated inversion recov-
A. given the risk of worsening, she should be started on ery (FLAIR) images; magnetic resonance (MR) venogram
mycophenolate mofetil shows absence of flow-related signal in the left transverse
B. IV immunoglobulin may be used in case of myasthenic crises sinus and decreased flow-related signal within the left
C. prednisone would be contraindicated if her symptoms sigmoid sinus. Which of the following changes in the
worsen, due to teratogenic effects coagulation system is most likely to have contributed to
D. pyridostigmine bromide should be discontinued due to these findings?
teratogenic effects A. decreased plasminogen activator inhibitor level
E. regional anesthesia during labor may increase the severity of B. decreased protein C level
weakness C. decreased protein S level
D. increased antithrombin III level
23. Which of the following periods incurs the greatest risk of E. increased factor II prothrombin level
cerebral venous thrombosis?
A. delivery 27. A 25-year-old woman is evaluated for abrupt onset of
B. first trimester headache, vomiting, photophobia, and diplopia that devel-
C. postpartum oped within 1 hour postpartum. Examination shows reduced
D. second trimester left visual acuity, right temporal field defect, a left third and
E. third trimester right sixth nerve palsies, and a left dilated, unreactive pupil.
Her blood pressure is 80/50 mm Hg with a heart rate of 100
24. A 33-year-old primigravida in her late third trimester of pregnancy beats per minute. Which is the most likely diagnosis?
is evaluated for severe headache, transient episodes of visual loss, A. amniotic fluid embolism
diplopia, and pulsatile tinnitus over the past 2 weeks. Examination B. carotid-cavernous fistula
shows normal visual acuity and enlarged blind spots. Pupil C. cavernous sinus thrombosis
reactions are normal. Funduscopy shows bilateral papilledema. D. pituitary apoplexy
Neurologic examination is normal except for bilateral sixth nerve E. ruptured posterior communicating artery aneurysm
palsies. Her blood pressure is 150/90 mm Hg. Further testing
should aim to exclude which of the following conditions? 28. A 24-year-old primigravida is evaluated at the 30th week of
A. carotid-cavernous fistula pregnancy for sudden onset of right face and arm weakness
B. cerebral venous sinus thrombosis and aphasia that occurred 1 hour ago. Her blood pressure is
C. pituitary apoplexy 150/80 mm Hg with a heart rate of 90 beats per minute. A CT
D. posterior reversible encephalopathy syndrome scan of the head shows a hyperdense left middle cerebral
E. reversible cerebral vasoconstriction syndrome artery sign. Which of the following data should be considered
regarding use of tissue plasminogen activator (tPA) in this
25. A 28-year-old primigravida is evaluated at 38 weeks’ gestation patient?
for headache, nausea, somnolence, and right hemiparesis that A. IV tPA is contraindicated because of its teratogenic effects
developed over the course of 4 hours. Examination shows a B. IV tPA is only indicated in stroke associated with preeclampsia/
reduced level of arousal, right hemiparesis, and papilledema. eclampsia
Brain MRI shows a T1- and T2-hyperintense signal in the C. IV tPA would increase the risk of fetal hemorrhage
posterior portion of the superior sagittal sinus; magnetic D. the benefit of tPA likely outweighs the risks and should
resonance venography shows a filling defect in this area. be appropriate in this case
Small parasagittal hemorrhages are present, worse on the left. E. urokinase would be preferred over tPA because it is safer
Which of the following management options is most to the fetus
appropriate at this point?
A. hold anticoagulation because of the presence of intrace- 29. A 20-year-old college student is diagnosed with myasthenia
rebral hemorrhage and fetal risk of bleeding gravis 3 months after presenting with intermittent ptosis and
B. hold anticoagulation and consider low-molecular-weight diplopia. She is started on pyridostigmine, but 4 months later
heparin if the symptoms worsen her symptoms begin to worsen and she develops swallowing
C. start aspirin and consider rivaroxaban if the symptoms and voice problems. A decision is made to start cyclosporine
worsen therapy. What advice should be given to this patient
D. start full heparin anticoagulation followed by oral anticoagulation regarding contraception during her treatment?
after delivery A. contraception should be started a month before cyclosporine
E. start oral dabigatran given the risk of warfarin-induced B. cyclosporine can be continued if contraception is stopped
fetal hemorrhage C. cyclosporine may affect the reliability of oral contraceptives
D. only parenteral methods of contraception are reliable
E. oral contraceptives may worsen the myasthenia gravis

16 www.ContinuumJournal.com February 2014

Copyright © American Academy of Neurology. Unauthorized reproduction of this article is prohibited.


Self-Assessment

30. A 22-year-old primigravida is evaluated in the second 35. A 24-year-old primigravida is evaluated at 30 weeks of
trimester of pregnancy for acute visual loss in the right eye. pregnancy for severe low back pain and progressive numbness
Examination shows an altitudinal superior visual field defect and weakness in the lower extremities and hands that
of the right eye. Pupils react normally to light. Dilated fundus developed over the past 48 hours. Examination shows symmet-
examination of the right eye shows whitening of the lower ric weakness in intrinsic hand muscles and both proximal and
retina. Which of the following is the most likely cause of the distal lower extremity muscles; generalized areflexia; and absent
patient’s symptoms? joint position and vibration sense in the toes. CSF examination
A. amniotic fluid embolism shows elevated protein level with normal glucose and cell
B. antiphospholipid antibody syndrome count. Which of the following management options is most
C. carotid artery dissection appropriate in this clinical setting?
D. embolism from a cardiac source A. cesarean section instead of vaginal delivery is indicated
E. vasculitis because of impaired uterine contractility
B. IV immunoglobulin should be avoided because of
31. During the second trimester of her first pregnancy, a 23-year- adverse effects on the fetus
old woman presents because of an uncomfortable, creepy C. termination of pregnancy is recommended to improve outcome
crawling sensation associated with an urge to move her legs D. the fetus is at increased risk of developing a similar condition
that has been present for the past month. Neurologic E. the patient should be tested for cytomegalovirus infection
examination is normal. Serum ferritin and folate levels are
normal. Which of the following is the most appropriate 36. Which of the following antiepileptic drugs typically shows the
therapy in this setting? greatest decline in serum levels due to pregnancy?
A. levodopa-carbidopa A. carbamazepine
B. nortriptyline B. lamotrigine
C. pergolide C. levetiracetam
D. ropinirole D. phenobarbital
E. topiramate E. valproic acid

32. In terms of occurrence of seizures during pregnancy, which of 37. A 32-year-old woman with a history of restless legs syndrome
the following factors is most predictive of remaining seizure becomes pregnant and seeks counseling regarding the
free for that period? effects of pregnancy on her condition. Her symptoms are
A. low maternal IQ mild and require no treatment. Her serum ferritin and folate
B. multiparity levels are normal. Which of the following statements
C. polytherapy of antiepileptic drugs regarding counseling should be discussed with the patient?
D. prepregnancy seizure-free status A. further iron supplementation is required even with normal
E. use of oxcarbazepine ferritin levels
B. her symptoms are likely to improve or resolve during pregnancy
33. Which of the following fetal complications can occur in C. ropinirole is the treatment of choice if her symptoms worsen
offspring of mothers with myasthenia gravis, as a rare D. the majority of patients show worsening of symptoms during
consequence of transplacental passage of acetylcholine pregnancy
receptor antibodies? E. the symptoms typically improve early and worsen late during
A. arthrogryposis multiplex congenita pregnancy
B. cardiac hypoplasia
C. microcephaly 38. A 34-year-old primigravida is evaluated in the immediate
D. oligohydramnios postpartum period for severe headache and blurred vision
E. thymic hypoplasia over the past 36 hours. She had a generalized tonic-clonic
seizure 1 hour before evaluation. Her blood pressure is 165/
34. A 26-year-old woman with Charcot-Marie-Tooth disease 100 mm Hg, with heart rate of 60 beats per minute. Urinalysis
becomes pregnant. She has mild difficulty walking on her shows proteinuria of 6 g in 24 hours. Fluid-attenuated inversion
heels and toes but is otherwise asymptomatic. Which of the recovery (FLAIR) images show increased signal intensity in both
following should be discussed during counseling of this parieto-occipital regions. Which of the following is a feature of
patient? the underlying pathogenesis of this condition?

A. regional anesthesia during delivery should be avoided A. abnormal increase in renin and aldosterone levels
B. risk of miscarriage is increased B. hypersensitivity to angiotensin II
C. risk of preeclampsia is increased C. increased antithrombin III levels
D. risk of respiratory distress in the neonate is increased D. reduced levels of soluble vascular endothelial growth factor
E. transient worsening of neuropathy may occur in approximately receptor
32% of pregnancies E. reduced thromboxane A2/prostacyclin ratio

Continuum (Minneap Minn) 2014;20(1):13–18 www.ContinuumJournal.com 17

Copyright © American Academy of Neurology. Unauthorized reproduction of this article is prohibited.


Self-Assessment

39. Which of the following hemodynamic changes occurring 40. Which of the following migraine abortive or prophy-
during normal pregnancy increases susceptibility to cerebro- lactic medications should be strictly avoided during
vascular and cardiovascular disorders in this condition? breast-feeding?
A. hemoconcentration A. acetaminophen
B. increased peripheral vascular resistance B. dihydroergotamine
C. increased venous capacitance C. nortriptyline
D. increase in diastolic blood pressure D. propranolol
E. reduced stroke volume E. sumatriptan

18 www.ContinuumJournal.com February 2014

Copyright © American Academy of Neurology. Unauthorized reproduction of this article is prohibited.


Dedication

* 2014, American Academy

Dedication to Autumn of Neurology.

Klein, MD, PhD (1971–2013)


Steven K. Feske, MD; Riley M. Bove, MD

This edition of include all special


is dedicated to the memory of neurologic concerns
Dr Autumn Klein, who, as of women.
guest editor of this issue, Still at the begin-
assembled this particular ning of her career,
group of articles and authors Autumn had won an
on a subject for which she Epilepsy Foundation
held great passion. Research & Training
The course of Dr Klein’s Fellowship for Clini-
education foreshadowed her cians Award for her
future path: a BA magna cum work on women with
laude in both women’s and epilepsy. She was an
gender studies and neurosci- active author and
ence from Amherst College editor, and she
and an MD and PhD in neuroscience established herself on the national stage
from Boston University School of Medi- in education by founding a successful
cine. After an internship at Brown Uni- course on women’s health at the American
versity, she entered residency in the Academy of Neurology’s (AAN’s) Annual
Harvard Neurology Residency Program Meeting. As a tribute to her promise, in
at Brigham and Women’s Hospital and 2012 she was among a select group chosen
Massachusetts General Hospital and ulti- for the AAN’s Emerging Leaders Forum.
mately served as chief resident. She then But, ultimately, we think Autumn
focused her interest on women’s health would wish most to be remembered
issues in neurology. After completing a for her clinical work with patients. She
fellowship in clinical neurophysiology had a passion for her clinical mission to
and epilepsy, Autumn joined the Neurol- bring expertise to the care of women
ogy Department at Brigham and with neurologic problems and to help
Women’s Hospital to establish the first them achieve both their family and their
program in women’s neurology before medical goals. To do this, she made
departing to found a new Division of herself an expert, and she became a
Women’s Neurology at the University of valuable resource for her patients. She
Pittsburgh, where she was Assistant understood that patient education was
Professor of Neurology and Obstetrics essential for excellent care, and she
and Gynecology and an active practitioner dedicated herself to the role of an
at Magee-Womens Hospital of University available and supportive physician. She
of Pittsburgh Medical Center (UPMC). In gave patients time, and she listened as
her chosen field she was able to address she sought to help them understand
the specific needs of women balancing their illnesses so they might become
the complex issues of epilepsy their own advocates. Many of her
and antiepileptic drugs and their in- patients have expressed their deep
teractions with pregnancy. It was a appreciation for her caring approach
natural extension of her practice to and have sung her praises.

Continuum (Minneap Minn) 2014;20(1):19–20 www.ContinuumJournal.com 19

Copyright © American Academy of Neurology. Unauthorized reproduction of this article is prohibited.


Dedication

This edition of arose In her honor and memory, we are happy


out of her intense interest in the neurol- to bring it to completion with the hope
ogy of pregnancy. She was editing this that it helps the members of our neurol-
volume at the time of her death, and she ogy community provide the best care for
would have been proud to see it in print. their pregnant patients.

20 www.ContinuumJournal.com February 2014

Copyright © American Academy of Neurology. Unauthorized reproduction of this article is prohibited.


Editor’s Preface

* 2014, American Academy

The Neurology of of Neurology.

Pregnancy: A Tribute to
Dr Autumn Klein
This issue of cians who care for wom-
is dedicated to the memory of This issue of en with seizure disorders
Dr Autumn Klein, whose too- is who are of childbearing
short career was devoted to the dedicated to the age will benefit from the
care of pregnant women with very practical, thorough,
neurologic disease and to the memory of and up-to-date analysis
education of neurologistsVand Dr Autumn Klein, she provides to guide
other physicians who care for who was devoted to us in our management
women of childbearing ageV of these patients.
about these critically important the care of pregnant Drs Steven Feske and
and complex neurologic issues women with Aneesh Singhal discuss
that can impact both the moth- neurologic disease the cerebrovascular disor-
er and her child. As the guest ders that may complicate
editor of this issue, Dr Klein and to the education pregnancy. In this article,
carefully planned and organized of neurologists about the authors thoroughly
these topics and invited this these critically review ischemic and
group of expert authors to in- hemorrhagic strokes that
form us about the current and important and may be associated with
optimal diagnosis and manage- complex issues. pregnancy and provide
ment of the variety of neurologic us with an expert discus-
disorders that can affect women sion of preeclampsia/
during pregnancy and the postpartum period. eclampsia, the posterior reversible encepha-
The issue begins with an overview by lopathy syndrome (PRES), and the reversible
Drs Riley Bove and Joshua Klein of the cerebral vasoconstriction syndrome (RCVS);
considerations involved in the neuroradio- their state-of-the-art review will help us
logic investigation of women who are recognize and manage these overlapping
pregnant or breast-feeding. This article pro- syndromes. Next, Drs E. Wayne Massey and
vides an important introduction to the Amanda Guidon review the diagnosis and
diagnostic issues involved in nearly all of management of peripheral neuropathies in
the subsequent articles with regard to the pregnancy. These authors provide a thorough
risks and benefits of the various imaging discussion of the diagnosis and management
modalities available to us and their current of the variety of cranial neuropathies,
role in our diagnostic armamentarium in radiculopathies, mononeuropathies, and
these settings. Next, Dr Patricia Coyle polyneuropathies that can occur coincident
discusses multiple sclerosis in pregnancy with, or as a consequence of, pregnancy or
and the postpartum state, an issue that we delivery. The next article, by Drs Janice
deal with frequently given the prevalence of Massey and Carolina De Jesus-Acosta, is
multiple sclerosis in women of childbearing devoted to the specific neuromuscular disease
age. This comprehensive review will provide of myasthenia gravis, which may complicateV
readers with a very current understanding of or even occasionally present inVpregnancy.
the issues regarding risk of relapse and We will benefit from their expertise, which
current recommendations regarding the we can emulate as we counsel and manage
role, or lack thereof, of acute and preventive our patients with this potentially life-threaten-
therapy in these settings. Dr Cynthia Harden ing, but treatable, neuromuscular disease.
then discusses the commonly encountered Dr E. Anne MacGregor provides her
issue of epilepsy and pregnancy. All clini- expertise to help us counsel and manage

Continuum (Minneap Minn) 2014;20(1):21–22 www.ContinuumJournal.com 21

Copyright © American Academy of Neurology. Unauthorized reproduction of this article is prohibited.


Editor’s Preface

our pregnant patients with primary head- you need to earn credits specifically ap-
ache disorders, as well as to recognize and proved by the American Board of Psychiatry
manage those secondary headache and Neurology (ABPN) for self-assessment,
disordersVsome of which may carry signif- submit your answers to the multiple-choice
icant morbidity and mortality if not recog- questions in the Self-Assessment Pretest
nized and treated expeditiouslyVthat may crafted by Drs Eduardo Benarroch and
occur in association with (and in some cases Ronnie Bergen before you read the issue;
as a consequence of) pregnancy or the review your results to better tailor your
postpartum state. Drs Janis Miyasaki and learning needs; and then complete the
Amaal AlDakheel discuss the movement Postreading CME Test after reading the
disorders that may occur in pregnant wom- issue. By doing so you may earn up to 12
en. In their thorough review, these authors AMA PRA Category 1 CME CreditsTM toward
provide their approach to the counseling, self-assessment. Alternatively, you may wish
diagnosis, and management of patients to receive credits toward CME only, in which
whose movement disorders may specifically case, reading the issue and submitting the
arise during pregnancy (such as restless legs Postreading CME Test will allow you to earn
syndrome and chorea gravidarum), as well as up to 10 AMA PRA Category 1 CME Credits.
to the many management issues that arise The Patient Management Problem, written
among patients with preexisting movement by Dr Riley Bove, involves the management
disorders who become pregnant. In the final of a woman presenting to the emergency
review article of this issue, Drs Kathleen department with neurologic symptoms in
Digre and Krista Kinard review neuro- her third trimester of pregnancy. By
ophthalmic disorders that may occur in following her case and answering multiple-
association with pregnancy. These authors choice questions corresponding to
provide us with their very practical symptom- important diagnostic and therapeutic
and finding-based approach to the diagnosis decision points along the rest of her
and management of the neuro-ophthalmic pregnancy and delivery (reinforcing many of
disorders that can affect vision and eye the issues discussed in the previous articles),
movements, whether caused by, or just you will have the opportunity to earn up
coincident with, pregnancy. to 2 AMA PRA Category 1 CME Credits.
In this issue’s Ethical Perspectives section, I would like to give my special and
Drs Bethanie Morgan-Followell, Jacqueline sincere gratitude to Drs Steven Feske and
Nicholas, and Pedro Weisleder carefully Riley Bove, who provided their valuable
analyze the complex ethical considerations time and considerable expertise in
involved in the counseling of a postpartum assisting in the editing of this issue and
breast-feeding woman with multiple scle- in bringing Dr Klein’s volume to fruition
rosis in regard to the timing of restarting after her passing. My special thanks also
disease-modifying therapy. In the Practice to each of the contributors to this issue,
section, Dr Joseph Kass shares his neuro- who accepted Dr Klein’s invitation to
logic and legal expertise by providing impart their expertise to our readers and
practical case examples involving epilepsy whose outstanding contributions serve as
and pregnancy to illustrate several issues a lasting tribute to her memory.
involved in mitigating legal risk as we All of us on the editorial staff of
provide our highest level of care to are honored to have been
pregnant patients with neurologic disease. able to work with Dr Klein during the
Finally, Drs Mark Yerby and Laura Powers planning and creation of this issue, through
outline the issues involved in coding of the which each of us and our patients can
pregnant patient with neurologic disease, continue to benefit from the knowledge
including a focus on specific consider- and dedication of this remarkable physician.
ations related to coding of the pregnant
patient with epilepsy.
As with every issue, a VSteven L. Lewis, MD, FAAN
number of opportunities exist for CME. If Editor-in-Chief

22 www.ContinuumJournal.com February 2014

Copyright © American Academy of Neurology. Unauthorized reproduction of this article is prohibited.


Review Article

Neuroradiology in
Address correspondence to
Dr Joshua P. Klein, Department
of Neurology, Room AB-124,
Brigham and Women’s Hospital,

Women of Childbearing 75 Francis Street, Boston, MA


02115, jpklein@partners.org.
Relationship Disclosure:

Age Dr Bove reports no disclosure.


Dr Klein receives financial
compensation for serving on
the editorial boards of the
Riley M. Bove, MD; Joshua P. Klein, MD, PhD Journal of Neuroimaging and
AccessMedicine Neurology,
and royalties from McGraw-Hill
for Adams and Victor’s Principles
ABSTRACT of Neurology.
Purpose of Review: This review summarizes safety concerns associated with Unlabeled Use of
diagnostic neuroimaging in patients who are of childbearing age, focusing on Products/Investigational
Use Disclosure:
diagnostic modalities and radiologic features of neurologic conditions encountered Drs Bove and Klein report no
by pregnant women. disclosures.
Recent Findings: During pregnancy, women experience a range of physiologic * 2014, American Academy
changes that can affect neurologic function. These include endocrine, hemodynamic, of Neurology.
endothelial, immunologic, and coagulopathic changes that can alter susceptibility to
stroke, subarachnoid hemorrhage, demyelination, venous thrombosis, and other
neurologic conditions. Unique safety concerns are associated with imaging pro-
cedures performed to diagnose neurologic conditions that occur during pregnancy.
Summary: This review discusses the use of diagnostic neuroimaging, including
administration of IV contrast, in pregnant women and in nonpregnant women of
childbearing age.

Continuum (Minneap Minn) 2014;20(1):23–41.

INTRODUCTION: PREGNANCY SAFETY OF IMAGING PROTOCOLS


AND NEUROLOGIC FUNCTION Exposure to ionizing radiation from
During pregnancy, a range of physio- CT and strong magnetic fields from
logic changes occur that impact neuro- MRI, with or without use of contrast,
logic function, including hemodynamic, carry potential risks to the pregnant
endocrine, immune, metabolic, and patient and her conceptus. The choice
other adaptations to accommodate the of diagnostic modality to evaluate
needs of the fetus.1 These physiologic neurologic disorders in the pregnant
adaptations alter the risk of neurologic patient should aim to provide the
disorders during pregnancy. An appre- patient with the standard of care for
ciation of gestational physiology can diagnosis and treatment while mini-
aid the clinician in distinguishing mizing potential risks to her conceptus.
between more typical pregnancy- The evidence-based and comprehen-
associated changes in neurologic sive practice guidelines and recommen-
symptoms and concerning neurologic dations established by independent
changes requiring further evaluation. consensus panels of the American Col-
For example, while the gradual onset lege of Obstetricians and Gynecologists
of low back pain and urinary stress (ACOG),2 the American College of
incontinence may be common in the Radiology (ACR),3 and the European
later stages of pregnancy, an abrupt Society of Urogenital Radiology
onset of back pain and urinary reten- (ESUR)4 for the use of these diagnostic
tion may warrant imaging of the spine. modalities during pregnancy are largely
Continuum (Minneap Minn) 2014;20(1):23–41 www.ContinuumJournal.com 23

Copyright © American Academy of Neurology. Unauthorized reproduction of this article is prohibited.


Neuroradiology in Women

KEY POINTS
h Physiologic changes concordant. Specific indications, risks, 10,000 children, increases to 5 per
during pregnancy and benefits from any diagnostic mo- 10,000 after in utero exposures of 1 to
modulate the incidence dality should be discussed with the 2 rad (unit of absorbed dose equal to
and presentation of a pregnant patient and her other care- 0.01 Gy; the Gy, or gray, is the
number of neurologic givers (obstetrician and radiologist) International System of Units base unit
conditions. whenever possible, and this discussion for absorbed dose of ionizing radia-
h Elective imaging should, should be documented in the medical tion, in joules [J]/kg). A meta-analysis
when possible, be record. Elective imaging should be showed a 6% increase in risk of
deferred to the deferred to the postpartum period. childhood cancer per 100 rad.7 Even
postpartum period. with repeated imaging, however, such
When imaging is Computed Tomography exposures would not be achieved;
essential for the Harm to fetus. The risks of exposure therefore, the attributable risk of
evaluation and to ionizing radiation associated with childhood cancer from modern clini-
treatment of a pregnant CT can be categorized as stochastic cal imaging is believed to be low.
patient, careful review
versus deterministic. Stochastic ef- Rate of absorption. During the entire
of the indications for,
fects, including mutagenesis and child- gestation, fetal exposure to back-
risks and benefits of,
and alternatives to
hood malignancies, may theoretically ground environmental radiation is es-
neuroimaging should be occur after any amount of radiation timated at 0.23 rad.6 With CT, the dose
documented. exposure. Given these effects, the ‘‘as absorbed by the fetus varies based on
low as reasonably achievable’’ para- maternal size, examination parame-
h During head CT
examination of the
digm encourages limitation of radia- ters, and whether it receives direct
mother, the fetus is tion.2 In contrast, deterministic effects rather than indirect radiation.
exposed only to (such as cataract formation and infer- Fetal exposure to indirect radiation
radiation that is tility) are associated with specific occurs with imaging of the maternal
scattered through the exposure thresholds. head or cervical spine; the fetus is
mother’s body. Radiation threshold for conceptus exposed only to attenuated scattered
Therefore, shielding of injury. Three research settingsVall radiation through the mother’s body,
the abdomen, such as associated with much greater radia- with a dose estimated at less than 0.01
with a lead vest, does tion exposure than is used clinically rad.6 Fetal exposure to direct radiation
not significantly todayVhave provided most of the occurs with imaging of the maternal
reduce the minimal fetal
guidance regarding the risk of gesta- lumbar spine or pelvis. The fetus in
radiation exposure but
tional exposure. These settings in- this case may be exposed to direct
may help to alleviate
maternal anxiety.
clude (1) observational studies of radiation at higher doses: 0.28 rad to
human survivors of the Hiroshima 2.4 rad for a lumbar spine CT and up to
and Nagasaki atomic bombs, (2) ob- 3 rad for an abdomen/pelvis CT.6 Atten-
servational studies of patients exposed tion to patient positioning and beam
to radiation before the advent of collimation parameters can minimize
concerns about radiation safety during fetal exposure to direct radiation.
pregnancy, and (3) experimental stud- Gestational age considerations. Con-
ies in animals. These have shown that ception to implantation (days 0 to 15)
the effects of radiation depend on the is the period of highest risk, with an
dose of radiation absorbed, the rate of all-or-nothing effect: animals exposed
dose absorption, and fetal gestational to radiation during this period experi-
age (Table 1-1).3,5,6 ence either death or no conse-
Dose considerations. Low-dose irra- quences; atomic bomb survivors
diation to the fetus has been linked to exposed before 15 days gestational
an increased risk of childhood cancer, age had no sequelae. An increased
particularly leukemia.7 The baseline risk of miscarriages through week 4 is
rate of childhood leukemia, 3.6 per usually cited.3 During organogenesis
24 www.ContinuumJournal.com February 2014

Copyright © American Academy of Neurology. Unauthorized reproduction of this article is prohibited.


a
TABLE 1-1 Suspected In Utero Fetal Radiation Effects

Menstrual or G50 mGy 50Y100 mGy 9100 mGy


Gestational Age Conception Age (G5 rad)b (5Y10 rad)b (910 rad)b
0Y2 weeks Prior to conception None None None
(0Y14 days)
Third and fourth FirstYsecond weeks None Probably none Possible spontaneous
weeks (15Y28 days) (1Y14 days) abortion
FifthY10th weeks ThirdYeighth weeks None Potential effects Possible malformations
(29Y70 days) (15Y56 days) are scientifically increasing in likelihood
uncertain and as dose increases
probably too
subtle to be
clinically detectable
11thY17th weeks NinthY15th weeks None Potential effects Increased risk of
(71Y119 days) (57Y105 days) are scientifically deficits in IQ or mental
uncertain and retardation that increases
probably too subtle in frequency and severity
to be clinically with increasing dose
detectable
18thY27th weeks 16thY25th weeks None None IQ deficits not detectable
(120Y189 days) (106Y175 days) at diagnostic doses
927 weeks 925 weeks None None None applicable to
(9189 days) (9175 days) diagnostic medicine
a
Modified from American College of Radiology.3 Reprinted with permission of the American College of Radiology. No other representation
of this material is authorized without expressed, written permission from the American College of Radiology. Refer to the ACR website at
www.acr.org/Quality-Safety/Standards-Guidelines for the most current and complete version of the ACR Practice Guidelines.
b
Columns 3, 4, and 5 describe the suspected fetal effects due to the increasing amounts of fetal radiation exposure delineated in the
column headings.

(weeks 3 through 8), an increased risk than in the period from 15 to 25


of congenital malformations, transient weeks, beyond which the risk is
growth retardation, and neonatal negligible.3
death is noted in animal studies. Patient management and counseling.
Human fetuses exposed to medical Whenever possible, radiologists should
irradiation during this period also consider estimated radiation doses and
manifested malformations, but in either adjust preset parameters or
atomic bomb survivors, only dose- dynamically reduce doses based on
dependent microcephaly was noted.8 habitus to limit the radiation exposure
Although malformations have not in the pregnant patient. Advanced
been noted in the fetal period (week image reconstruction techniques may
6 until birth), the risk of mental offer alternative sources of diagnostic
retardation associated with irradiation images that limit the need for additional
is increased during this period. In the radiation exposure. Imaging parame-
most sensitive period, from 8 to ters, including estimated dose received,
15 weeks, a 12-rad to 20-rad dose should be included in all CT reports and
threshold (much higher than used become part of the patient’s medical
in clinical imaging) is associated record. If necessary, a radiation dosime-
with an increased risk of mental try expert can assist in calculating
retardation; this risk is 4 times greater the total dose to which the fetus was

Continuum (Minneap Minn) 2014;20(1):23–41 www.ContinuumJournal.com 25

Copyright © American Academy of Neurology. Unauthorized reproduction of this article is prohibited.


Neuroradiology in Women

KEY POINTS
h The radiologist or a or will be exposed.2 Eventually, investi- ‘‘pump and dump’’ interruption from
radiation dosimetry gations of these data may provide breast-feeding may be preferred by the
expert can assist the better information about the impact of nursing mother but is not indicated.
neurologist and current diagnostic imaging and fetal
obstetrician in deciding outcomes. Magnetic Resonance Imaging
on CT or MRI in the Pregnant women who have been Harm to fetus. To date, no conclusive
pregnant patient. exposed to radiation or for whom evidence has shown that MRI expo-
Some CT and MRI imaging is planned should be sure up to 3 Tesla is associated with
examinations can be counseled that no definitive associa- fetal harm.3,11 Theoretical concerns
modified to provide tion between radiation exposure of include noise exposure, positioning
diagnostically critical
less than 5 rad (50 mGy) and an within strong magnetic fields, and
information while
increased risk of spontaneous abor- increase in body temperature caused
exposing the conceptus
to as little risk as possible.
tion, developmental malformations, or by radiofrequency pulse energy depo-
mental retardation has been proven. A sition. Fetal MRI is routinely used
h Iodinated contrast is US very small association exists between when reliable fetal imaging cannot
Food and Drug
radiation and childhood malignancies. be obtained by ultrasonography,
Administration class B. If
use of iodinated
Fetal-absorbed doses of 5 rad to 15 rad according to practice guidelines
contrast cannot be (50 mGy to 150 mGy) may result in a established by the ACR and the Society
avoided during small but detectable increase in the risk of Pediatric Radiology.3 While MRI of
pregnancy, neonatal of congenital defects above the baseline the mother or fetus can be safely
thyroid testing should population risk of 5% to 10% of births.3 obtained when clinically indicated,
be performed during Safety of iodinated contrast. The elective imaging should be deferred
the first week. During use of IV iodinated contrast should be to the postpartum period if possible.
breast-feeding, avoided during pregnancy if possible. Safety of gadolinium-based con-
administration of Iodinated contrast is classified by the trast. During pregnancy, the use of
iodinated contrast is not US Food and Drug Administration gadolinium should be avoided unless
contraindicated.
(FDA) as class B (Appendix A). Animal it is likely to result in changes in
h MRI is the diagnostic studies have not shown teratogenic or management that would directly ben-
modality of choice mutagenic outcomes from iodinated efit the patient or fetus.3,4 Gadolinium
during pregnancy. contrast exposure,9 and well- contrast is classified by the FDA as
Nonetheless, whenever
controlled studies have not been per- class C. In animal studies, maternal
possible it should be
formed in humans; however, iodinated exposure to concentrations of gado-
delayed to the
postpartum period.
contrast instilled directly into the fetal linium higher than typically adminis-
cavity (as opposed to intravenously) tered in humans has been associated
has been associated with neonatal with abortion and developmental ab-
hypothyroidism. Therefore, if iodinated normalities, and gadolinium can enter
contrast administration cannot be fetal circulation.9 When gadolinium
deferred until after delivery, written use cannot be avoided, informed
informed consent should be obtained3 written consent, including a risk-
and neonatal thyroid function testing benefit analysis, should be obtained.
should be performed in the first week If gadolinium contrast is to be used,
of life.10 If contrast is to be used, one should be aware of the possibility
standard precautions pertaining to the of nephrogenic systemic fibrosis (a rare
risk of contrast-induced nephropathy condition that can occur in patients
should be followed. During lactation, with underlying renal insufficiency
no adverse effect on the infant of the who are exposed to gadolinium) devel-
low concentrations of iodinated con- oping in the mother.
trast transmitted in breast milk has During lactation, gadolinium is prob-
been proven.2,4 A 24-hour period of ably safe to use in the mother without
26 www.ContinuumJournal.com February 2014

Copyright © American Academy of Neurology. Unauthorized reproduction of this article is prohibited.


KEY POINTS
concern for direct toxicity or allergic Chronic infarctions appear hypodense on h Gadolinium contrast is
reaction to the neonate, as no adverse CT, with accompanying parenchymal US Food and Drug
outcome has been documented from volume loss. Administration class C
the very low estimated delivery of With MRI, diffusion-weighted imag- and should be avoided
gadolinium via breast milk.4 A 24-hour ing (DWI) sequences can detect re- during pregnancy when
period of ‘‘pump and dump’’ interrup- duced diffusivity of water due to possible. As an
tion from breast-feeding is suggested cytotoxic edema in acute infarctions alternative to contrast
by the ESUR4 but not by the ACOG or within minutes of the onset of ische- injection, imaging of the
ACR.2,3 mia. As an infarct evolves, mixed arterial and venous
cytotoxic and vasogenic edema ap- circulation can often
be performed using
RADIOGRAPHIC CHARACTERISTICS pears hyperintense on T2-weighted
time-of-flight
OF NEUROLOGIC CONDITIONS sequences and hypointense on
sequences. When
ASSOCIATED WITH PREGNANCY T1-weighted sequences. As with CT, gadolinium is essential,
petechial hemorrhage can be detected no specific monitoring
Ischemic Stroke in subacute infarcts, often appearing tests are required.
Changes to the cardiovascular, hyperintense on T1-weighted se-
h During breast-feeding,
hematologic, and endocrine systems, quences (due to methemoglobin) gadolinium
including volume expansion and estro- and hypointense on gradient echo administration is not
genic stimulation of blood-clotting and susceptibility-weighted sequences. contraindicated.
factors, occur during pregnancy. Con- Chronic infarctions appear hypointense
h Pregnancy-specific
sequently, risk of ischemic stroke is on T1-weighted sequences and hyper- causes of ischemic
increased in the peripartum and first 6 intense on T2-weighted sequences, stroke may include
weeks postpartum periods,12,13 but with accompanying parenchymal preeclampsia/eclampsia,
interestingly, not earlier during the volume loss. paradoxical embolism
gestational period. Pregnancy-specific Amniotic fluid embolism, arising from deep vein
causes of ischemic stroke may include from passage of amniotic fluid, fetal thrombosis through a
preeclampsia/eclampsia, paradoxical cells, and other organic matter into patent foramen ovale,
embolism from deep vein thrombosis the maternal circulation, occurs in 1 in trophoblastic
through a patent foramen ovale, tro- 20,000 deliveries and can lead to embolism, amniotic
phoblastic embolism, amniotic fluid cardiorespiratory collapse, anaphylaxis fluid embolism,
embolism, air embolism, and air embolism, and
(anaphylactoid syndrome of pregnancy),
cardioembolism from
cardioembolism from postpartum and coagulopathy. Neurologic sequelae
postpartum
cardiomyopathy. include focal deficits, seizures, and coma. cardiomyopathy.
The imaging appearance of ischemic Mortality occurs in at least one-quarter of
stroke is the same as in nonpregnant cases and accounts for 5% to 10% of
patients. On CT, the earliest changes maternal mortality in the United States.14
from ischemia include loss of gray-white MRI, while nonspecific, may reveal
matter differentiation and subtle swelling multiple foci of abnormal T2 hyper-
of infarcting tissue, mostly due to cyto- intensity and reduced diffusivity, as seen
toxic edema. As an ischemic stroke in thromboembolic infarctions.
evolves, hypoattenuation of infarcting
tissue due to mixed cytotoxic and Cerebral Venous Thrombosis
vasogenic edema is seen, often with mass The risk of cerebral venous thrombo-
effect on adjacent structures (ie, sulcal sis (CVT) in pregnancy is increased,
and ventricular effacement). During the although pregnancy-associated CVT
subacute period, petechial hemorrhage has a better prognosis than CVT from
(hyperattenuation) within the infarcted other etiologies. In the first trimester,
tissue can be seen, reflecting reperfusion the risk may be attributed to underly-
of infarcted tissue after clot dissolution. ing thrombophilia, but the risk is most
Continuum (Minneap Minn) 2014;20(1):23–41 www.ContinuumJournal.com 27

Copyright © American Academy of Neurology. Unauthorized reproduction of this article is prohibited.


Neuroradiology in Women

KEY POINT
h Contributing factors to increased in the first 4 to 8 weeks cerebral veins and dural sinuses with-
cerebral venous postpartum.15 Contributing factors in- out exposing the mother or conceptus
thrombosis include clude dehydration, cesarean delivery to IV contrast. Second, MRI sequences,
dehydration, cesarean or traumatic delivery, intracranial hy- particularly DWI, are more sensitive
delivery or traumatic potension from dural puncture during than CT for detecting acute infarctions.
delivery, intracranial neuraxial anesthesia, anemia, raised Finally, MRI spares the conceptus ex-
hypotension from dural homocysteine levels, or other etiolo- posure to ionizing radiation.
puncture during gies. Patients present with severe, Typical findings on MRI include the
neuraxial anesthesia, diffuse, and constant headaches; these following16:
anemia, raised are typically progressive but may also
homocysteine levels, or
& TOF venography allows for
present as thunderclap headaches. visualization of blood flow through
other etiologies.
Other manifestations include enceph- the cerebral veins and dural
alopathy, seizures, papilledema, and venous sinuses and is typically
focal neurologic deficits. Additionally, acquired in thin slices. If interruption
20% to 40% of patients with CVT have of flow through any of these
intracranial hypertension and thus can structures occurs, a lack of
present with symptoms mimicking flow-related signal will be evident
idiopathic intracranial hypertension.16 on this sequence. Since veins and
Infarctions may occur as a result of sinuses are under low pressure,
venous outflow obstruction with con- reduced or absent signal on TOF
sequent increase in venous pressure does not absolutely imply thrombotic
relative to arterial pressure. These occlusion.
infarctions do not respect arterial & Absence of normal hypointense
territories and may be peripheral; in vascular flow voids in the cerebral
the case of CVT in the deep cerebral veins and venous sinuses on
veins, bilateral thalamic infarctions T1-weighted and T2-weighted
may occur. Because of differences in sequences can be associated with
mechanism, venous infarctions are thrombosis or slow flow. The MRI
often surrounded by more edema appearance of thrombus within a
than would be expected from an cortical vein depends on its age,
arterial infarct. Parenchymal and sub- similar to the appearance of
dural hemorrhage may also occur as a an evolving intraparenchymal
result of CVT. Clinical symptoms from hematoma.
CVT may fluctuate as a result of & Focal hemorrhage appears
variability in collateral drainage, partial hypointense on gradient echo
recanalization of thrombosed veins or (GRE) and susceptibility-weighted
dural sinuses, and fluctuating paren- imaging (SWI) sequences.
chymal changes (vasogenic and cyto- & Focal ischemia appears
toxic edema). hyperintense on DWI and
Because of the potential risks of hypointense on the apparent
CVT to mother and fetus, adequate diffusion coefficient (ADC) sequence.
diagnostic workup is essential. MRI is & Variable signal may be present on
the preferred imaging modality to DWI and ADC, reflecting both
identify CVT in pregnant patients for vasogenic (enhanced diffusivity)
several reasons (Case 1-1). First, the and cytotoxic (reduced diffusivity)
use of noncontrast-based vessel imag- edema.
ing techniques such as time-of-flight If MRI cannot be used, a noncontrast
(TOF) MR angiography allows for an head CT, while often unrevealing, may
assessment of the patency of the show a focal hyperdense venous sinus

28 www.ContinuumJournal.com February 2014

Copyright © American Academy of Neurology. Unauthorized reproduction of this article is prohibited.


Case 1-1
Four days after cesarean delivery, a 33-year-old woman presented with left hemianesthesia followed
by a generalized seizure. Anticonvulsive treatment was started. As the differential for
new onset of neurologic deficits and seizures in the postpartum period included cerebral venous
thrombosis, posterior reversible encephalopathy syndrome, and eclampsia potentially complicated
by hemorrhage, MRI was selected as the most sensitive imaging modality of the brain parenchyma
and vasculature. Sagittal and axial T1 MRI (Figure 1-1A and Figure 1-1B) performed 1 day after
presentation showed focal thrombosis (T1 hyperintensity) of a cerebral vein overlying the right parietal
lobe. Subtle loss of cortical gray-white matter differentiation was evident in the gyrus adjacent to the
thrombosed vessel. On axial T2 fluid-attenuated inversion recovery (FLAIR) MRI (Figure 1-1C), the
thrombosed vessel appeared hyperintense, suggesting that the blood had degenerated into methemoglobin.
Abnormal T2 hyperintensity consistent with vasogenic edema was also noted within the juxtacortical
white matter of the adjacent postcentral gyrus. Gradient echo (GRE) imaging (Figure 1-1D) showed
abnormal hypointensity in the thrombosed cortical vein as well as within the postcentral gyrus and sulcus
that was consistent with blood products. No diffusion abnormalities were present to suggest venous
infarction. Patent arteries and veins should appear hypointense on both T1-weighted and T2-weighted
sequences as a result of flow voids due to movement of molecules during image acquisition.

FIGURE 1-1 Cerebral venous thrombosis. Sagittal (A) and axial (B) T1-weighted MRI sequences show focal thrombosis
(T1 hyperintensity) of a cerebral vein overlying the right parietal lobe (arrows). On axial T2 fluid-attenuated
inversion recovery (FLAIR) MRI (C), the thrombosed vessel appears hyperintense (arrow). Abnormal T2
hyperintensity is also noted within the juxtacortical white matter of the adjacent postcentral gyrus, consistent with
vasogenic edema. Gradient echo (GRE) imaging (D) shows abnormal hypointensity in the thrombosed cortical vein as well
as within the postcentral gyrus and sulcus, consistent with blood products.

Comment. The findings of focal venous thrombosis, surrounded by edema and some hemorrhage,
support the clinical picture of focal neurologic deficits and seizures, and are consistent with cerebral
venous thrombosis. The risk for cerebral venous thrombosis is increased during pregnancy,
particularly in the first 4 to 8 weeks postpartum. MRI is the most sensitive diagnostic modality.

or cortical vein, or occasionally focal blood and dura, producing the so-called
edema or parenchymal hemorrhage, empty delta sign as seen on axial
diffuse cerebral edema, or subdural and coronal images. Certain anatomic
hemorrhage. If CT venography is used variants complicate the diagnosis of
in the postpartum period, a thrombus in CVT on both MR venography and CT
the superior sagittal sinus will appear venography, such as sinus atresia/
hypodense relative to the surrounding hypoplasia, asymmetric sinus drain-
hyperdense contrast-enhanced flowing age, and normal sinus-filling defects

Continuum (Minneap Minn) 2014;20(1):23–41 www.ContinuumJournal.com 29

Copyright © American Academy of Neurology. Unauthorized reproduction of this article is prohibited.


Neuroradiology in Women

KEY POINTS
h Recent reviews have related to prominent arachnoid gran- catheter tip thrombosis exist. Primary
suggested that, contrary ulations or intrasinus septae.16 nonaneurysmal SAH remains a diagnosis
to conclusions from of exclusion.
prior studies, risk of Subarachnoid Hemorrhage
subarachnoid Subarachnoid hemorrhage (SAH) is the Preeclampsia and Eclampsia
hemorrhage is not third leading cause of maternal death Preeclampsia and eclampsia refer to a
increased during for reasons that are not related to multisystem condition that arises in 2%
pregnancy. obstetric complications. Recent reviews to 8% of pregnancies, probably from
h Preeclampsia and have suggested that, contrary to con- immune-induced endothelial and vascu-
eclampsia refer to a clusions from prior studies, risk of SAH lar dysregulation triggered by the pla-
multisystem condition is not increased during pregnancy.17,18 centa in susceptible pregnant women,
that arises in 2% to Additionally, a greater proportion of after week 20 (and usually after week 28)
8% of pregnancies. SAH occurring during pregnancy may of pregnancy and continuing until 6 to
be nonaneurysmal and triggered by 8 weeks postpartum. Contributing risk
hypertension, disrupted cerebral factors include prior preeclampsia,
autoregulation, or other etiologies. primiparity, maternal age, family history,
Contributing risk factors to SAH in preexisting hypertension, diabetes
pregnancy include advanced maternal mellitus or renal disease, and certain
age; advanced gestational age; under- autoimmune conditions, such as systemic
lying hypertension and disorders of lupus erythematosus. Clinical manifes-
coagulation; and tobacco, drug, or tations depend on the end organ
alcohol use. 17 Patients with SAH affected. The mildest form of the con-
typically present with acute onset of dition is pregnancy-induced hyperten-
severe thunderclap headache and sion. Preeclampsia is diagnosed by the
meningismus. Because rapid treatment combination of hypertension and pro-
of ruptured aneurysms improves both teinuria; peripheral edema often accom-
maternal and fetal outcomes, rapid panies these manifestations but is not
radiologic evaluation is essential. In necessary for diagnosis.
the acute setting, noncontrast head The hemolysis, elevated liver en-
CT has a greater than 90% sensitivity zymes, and low platelet-count (HELLP)
for detecting SAH18 and can be used to syndrome occurs in 10% to 20% of
monitor for expansion of the hemor- cases of severe preeclampsia. It prob-
rhage as well as for the presence of ably results from activation of the
hydrocephalus. Whenever possible, fibrinolytic cascade and presents with
however, three-dimensional TOF MRI malaise, epigastric pain, and nausea/
should be used to identify and monitor vomiting. Additional manifestations
aneurysms while limiting the concep- may include pulmonary edema, acute
tus’ exposure to ionizing radiation. renal failure, and disseminated intra-
Aneurysms are typically assessed for vascular coagulation. Neurologic signs
size, location, proximity to vessel and symptoms of preeclampsia include
origin, neck size (narrow versus bilateral throbbing headaches, confu-
wide neck), and orientation of the sion, visual blurring and scintillating
aneurysm apex in relation to its base. scotomata, photophobia, increased
If MRI cannot be performed, CT angi- deep tendon reflexes, and paresthesias.
ography provides a rapid assessment. Eclampsia, referring to seizures occur-
Catheter angiography is the gold stan- ring as a result of cerebral involvement,
dard, although risks associated with arises in 1% to 2% of severe preeclampsia
radiation, arterial puncture, contrast- cases and has an associated mortality rate
induced nephropathy or allergy, and of up to 14%. Seizures are usually
30 www.ContinuumJournal.com February 2014

Copyright © American Academy of Neurology. Unauthorized reproduction of this article is prohibited.


KEY POINTS
generalized tonic-clonic and last up to 1 prodrome, typically over 12 to h Most women with
minute. In up to one-third of cases, 48 hours. Additionally, patients may typical preeclampsia or
neither proteinuria nor elevated blood develop vasospasm and intracranial eclampsia do not
pressure (above 140/90 mm Hg) are hemorrhage. require neuroimaging,
present before seizure onset. Symptoms Most pregnancy-related PRES is but diagnostic imaging
may resolve in the hours after delivery likely to be a manifestation of should be performed
of the placenta, but women are at risk preeclampsia/eclampsia, even in pa- in the following
of preeclampsia up to 6 to 8 weeks tients who are normotensive and with- circumstances: seizures
postpartum.19 out proteinuria.20 Hemorrhage may be arising before week
Most women with typical preeclamp- exacerbated by concurrent thrombotic 20 of pregnancy,
postpartum eclampsia,
sia or eclampsia do not require neuro- thrombocytopenic purpura (TTP),
focal neurologic deficits,
imaging, but diagnostic imaging should which is also associated with preg-
persistent visual
be performed in the following circum- nancy, and presents with the pentad of symptoms, and
stances: seizures arising before week fever, microangiopathic hemolytic symptoms that are
20 of pregnancy, postpartum eclamp- anemia, renal insufficiency, thrombocy- refractory to magnesium
sia, focal neurologic deficits, persistent topenia, and neurologic dysfunction. infusion and
visual symptoms, and symptoms that Many patients with TTP do not manifest antihypertensive
are refractory to magnesium infusion all these symptoms. In addition to therapy.
and antihypertensive therapy. In these eclampsia and TTP, other causes h The radiologic
cases, posterior reversible encephalop- of PRES in nonpregnant patients in- appearance of posterior
athy syndrome (PRES) and reversible clude malignant hypertension, hyper- reversible encephalopathy
cerebral vasoconstriction syndrome calcemia, and use of drugs such as syndrome, probably due
(RCVS) should be considered, as both cyclosporine, tacrolimus, and other to a shared underlying
pregnancy-associated PRES and RCVS cancer chemotherapeutics. pathophysiology, is
are likely to share an underlying The radiologic appearance of PRES, similar to eclampsia,
pathophysiologic mechanism with probably due to a shared underlying hypertensive
encephalopathy,
preeclampsia/eclampsia. Additionally, pathophysiology, is similar to eclamp-
and thrombotic
imaging should be performed if con- sia, hypertensive encephalopathy, and
thrombocytopenic
cern exists for injuries sustained during TTP (Case 1-2).20Y22 purpura.
seizure activity. On CT, nonspecific patchy white
matter hypoattenuation is noted, pre-
Posterior Reversible dominantly in the parieto-occipital
Encephalopathy Syndrome lobes. Radiologic manifestations typi-
PRES is a clinical and imaging cally normalize after clinical resolu-
syndrome in which, perhaps as a tion, unless infarction or hemorrhage
result of disrupted posterior cerebral have occurred.22
autoregulation and increased vascular On MRI, patchy areas of T2-hyper-
endothelial permeability, vasogenic intense and T1-hypointense edema
edema arises diffusely but pre- involving cortical and subcortical
dominantly in the occipital and parie- structures and not strictly adhering to
tal lobes. Patients present with dull arterial territories are seen.20 The
headaches, encephalopathy, cortical lesions can be distinguished from
visual changes (eg, hallucinations, posterior cerebral artery infarctions be-
blurring, hemianopsia, diplopia, corti- cause they often spare the medial
cal blindness, and visuospatial dys- occipital lobe and calcarine cortex.
function), and seizures that are Occasionally, the lesions extend into
usually generalized tonic-clonic but the frontal lobes, basal ganglia, and
may have focal onset. The onset of brainstem structures. With contrast,
symptoms is rapid and without a enhancement of the lesions is variable,
Continuum (Minneap Minn) 2014;20(1):23–41 www.ContinuumJournal.com 31

Copyright © American Academy of Neurology. Unauthorized reproduction of this article is prohibited.


Neuroradiology in Women

Case 1-2
A 36-year-old woman in her third trimester of pregnancy, with no relevant medical history,
developed nausea, headache, and visual disturbances. On examination in the emergency department,
she was noted to be hypertensive with a blood pressure of 140/90. An urgent head CT was obtained
and revealed subtle hypoattenuation posteriorly. An MRI was subsequently obtained to better
characterize this finding. Axial CT and T2 fluid-attenuated inversion recovery (FLAIR) MRIs are shown
in Figure 1-2. Subtle CT hypoattenuation is noted within the bilateral parietal and occipital and
posterior temporal gray matter and subcortical white matter, corresponding to abnormal T2
hyperintensity seen on MRI. These findings are consistent with edema. No diffusion abnormality or
evidence of hemorrhage was present.

FIGURE 1-2 Posterior reversible encephalopathy syndrome. Axial CT (A and C) and T2 fluid-attenuated inversion
recovery (FLAIR) MRI (B and D) are shown. Subtle CT hypoattenuation is noted within the bilateral parietal
(A) and occipital and posterior temporal (C) gray matter and subcortical white matter, corresponding to
abnormal T2 hyperintensity seen on MRI (B and D). No diffusion abnormality or evidence of hemorrhage
is present.

Comment. The MRI findings of edema in the white and gray matter, more prominent posteriorly,
without associated hemorrhage or diffusion abnormality, are consistent with posterior reversible
encephalopathy syndrome (PRES). Most pregnancy-related PRES is likely to be a manifestation of
preeclampsia/eclampsia; in this case, the blood pressure was only marginally elevated but concerning
in a pregnant woman without a history of hypertension. The patient received supportive treatment
for her headaches, as well as antihypertensive therapy. Radiologic manifestations typically
normalize after clinical resolution, unless infarction or hemorrhage have occurred.

reflecting focal areas of blood-brain Postpartum Angiopathy/


barrier disruption.20 Reversible Cerebral
Hemorrhage, when it occurs, can Vasoconstriction Syndrome
be detected on GRE and SWI se- Postpartum angiopathy (PPA)/RCVS in
quences. Diffusion abnormalities con- pregnancy is likely to be a manifesta-
sistent with ischemic infarction are tion of the preeclampsia/eclampsia
sometimes seen. Lastly, vasospasm of syndrome, even though patients
medium and large cerebral arteries may be normotensive and without
(particularly the basilar artery) may proteinuria or other features of
be observed with angiography. eclampsia. PPA (occasionally referred

32 www.ContinuumJournal.com February 2014

Copyright © American Academy of Neurology. Unauthorized reproduction of this article is prohibited.


KEY POINT
to as postpartum cerebral angiopathy) can be used as a noninvasive means h Postpartum angiopathy/
denotes the form of RCVS that occurs of monitoring the resolution of reversible cerebral
postpartum.23,24 Other triggers for RCVS.23 vasoconstriction
RCVS (sometimes referred to as Call- syndrome in pregnancy
Fleming syndrome) include iatrogenesis is likely to be a
(use of ergot alkaloids, sympathomi- DEMYELINATION manifestation of the
metics, and selective serotonin re- Demyelinating diseases, such as mul- preeclampsia/eclampsia
uptake inhibitors), uncontrolled tiple sclerosis (MS) and neuromyelitis syndrome, even though
hypertension, or endocrine dysregula- optica (NMO), commonly present in patients may be
tion. Pathophysiologically, multifocal young women of childbearing age. normotensive and
without proteinuria or
segmental constriction of large- and Gonadal hormones influence immu-
other features of
medium-sized cerebral arteries occurs. nologic function; thus, the hormonal
eclampsia.
Patients with PPA typically present changes associated with pregnancy
with acute onset of severe thunder- appear to influence disease course.
clap headache, usually in the first This section focuses on MS, but it
4 weeks postpartum. Patients may also should be noted that the risk of relapse
manifest vomiting, photophobia, en- of NMO is likely to be increased in the
cephalopathy, seizures, and focal neu- postpartum period, as well.25
rologic deficits. The clinical course of
PPA is typically uniphasic and revers- Multiple Sclerosis
ible.23,24 In the most severe cases, MS affects 3 times more women than
patients can develop subarachnoid men and, with a peak onset in women
and intraparenchymal hemorrhage at age 24 years, primarily manifests in
as well as cerebral infarction and women of childbearing age. Pregnancy
occasionally cervicocranial arterial (particularly the third trimester) is a
dissections. time of relative quiescence for MS,
Brain MRI may reveal patchy foci of with a rebound in relapses noted in
abnormal T2 hyperintensity, predomi- the first few months postpartum.
nantly in the white matter and most During pregnancy, the radiologic
often at the border zones between assessment of new or evolving neuro-
arterial territories. These lesions may logic symptoms pertaining to demye-
show reduced diffusivity, indicating lination should consist of brain or
acute ischemic infarction, as well as spine MRI without gadolinium. On
hemorrhage. Vascular imaging should MRI, demyelinating lesions appear as
be performed; however, CT or focal T2 hyperintensities, characteris-
catheter-based angiography may be tically ovoid and periventricular or
replaced in the pregnant patient by intracallosal. In addition to periventric-
TOF MRI to optimize maternal and ular and juxtacortical locations, lesions
fetal safety (Case 1-3). Imaging may may be found in the brainstem, cere-
be initially unremarkable; however, it bellar peduncles, optic nerves, cere-
is positive in 80% of patients and bellum, and spinal cord. ‘‘Black
reveals multifocal and segmental con- holes,’’ or hypointensities seen on
striction and post-stenotic dilation of T1-weighted sequences, reflect axonal
large- and medium-sized intracranial loss and have a similar morphology and
arteries in a ‘‘string of beads’’ pattern distribution to the lesions seen on T2-
(Figure 1-3). The constriction is re- weighted sequences.
versible, even in the setting of infarc- The avoidance of contrast has four
tion. Transcranial Doppler ultrasound implications. First, it is important to
measurements of arterial flow velocity compare the current lesion burden to
Continuum (Minneap Minn) 2014;20(1):23–41 www.ContinuumJournal.com 33

Copyright © American Academy of Neurology. Unauthorized reproduction of this article is prohibited.


Neuroradiology in Women

Case 1-3
Eight weeks after vaginal delivery, a 28-year-old woman experienced
sudden-onset right occipital headaches. She was referred to the emergency
department, where she was found to be hypertensive. Because she had no history
of headaches or hypertension, urgent neuroimaging was indicated. MRI was
selected because of its increased sensitivity. Three-dimensional time-of-flight
intracranial magnetic resonance (MR) angiogram (Figure 1-3) showed diffuse
segmental arterial narrowing involving the anterior, middle, and posterior
cerebral arteries. No diffusion abnormality or evidence of hemorrhage was
present on MRI. A repeat MR angiogram 6 weeks later showed resolution of
vasoconstriction.

FIGURE 1-3 Reversible cerebral vasoconstriction syndrome. A, Three-dimensional time-of-flight


intracranial MR angiogram shows diffuse segmental arterial narrowing involving
the anterior, middle, and posterior cerebral arteries (arrows). B, A repeat
angiogram 6 weeks later showed resolution of vasoconstriction.

Comment. The findings of diffuse segmental arterial narrowing of the


intracerebral arteries in a postpartum patient presenting with hypertension
and headaches is consistent with postpartum angiopathy/reversible cerebral
vasoconstriction syndrome.

KEY POINT that on a prior MRI in assessing for diagnostic imaging, a careful documen-
h Pregnancy-related inflammatory activity. Second, it is not tation of risks and benefits should be
hormonal changes have possible to confirm the diagnosis of outlined and signed by the clinician,
trophic effects on
MS by making the radiologic diagnosis radiologist, and patient.
intracranial neoplasms,
of dissemination in time based on the
most commonly
simultaneous appearance of enhanc- NEOPLASMS
meningiomas and
pituitary adenomas, but ing and nonenhancing lesions on the Pregnancy-related hormonal changes
also ependymomas, same MRI, per 2010 McDonald have trophic effects on intracranial
hemangioblastomas, Criteria.26 Third, clinicians and radiolo- neoplasmsVmost commonly meningi-
and schwannomas, as gists should maintain a high index of omas and pituitary adenomas but also
well as on metastases suspicion for alternative diagnoses, such ependymomas, hemangioblastomas,
from breast cancer and as progressive multifocal leuko- and schwannomas, as well as metastases
melanoma. encephalopathy in a pregnant patient from breast cancer and melanoma.27
previously treated with natalizumab or
other immunosuppressive therapies, in Meningiomas
a patient with MS. Finally, in select cases Meningiomas, the most common pri-
in which contrast is deemed essential to mary intracranial tumor, arise from the

34 www.ContinuumJournal.com February 2014

Copyright © American Academy of Neurology. Unauthorized reproduction of this article is prohibited.


KEY POINTS
arachnoid cap cells in the meningeal tumors. It is characterized by markedly h Meningiomas arise from
arachnoid villi, and the presence of elevated serum "-human chorionic the arachnoid cap cells
hormonal receptors causes them to gonadotropic hormone (hCG) levels. in the meningeal
increase in size during pregnancy. Choriocarcinoma exerts local invasive arachnoid villi, and the
Meningiomas most commonly arise effects and metastasizes to the spine, presence of hormonal
from the parasellar and petroclival liver, lungs, and (in 15% to 20% of cases) receptors causes them
dura, the tentorium and falx, and the brain.28 Intracerebral manifestations of to increase in size during
dura overlying the cerebral hemi- an isolated lesion include headaches, pregnancy.
spheres. Presenting symptoms are encephalopathy, seizures, and focal h It has been estimated
attributable to mass effect and, neurologic deficits. In the setting of that the pituitary gland
depending on tumor location, may massive hemorrhage, signs of elevated enlarges by 136%
include headache and symptoms related intracranial pressure may be present.29 during pregnancy,
to elevated intracranial pressure, In patients with acute or catastrophic exceeding 10 mm in
seizures, visual impairment from optic neurologic presentations, a noncontrast height and regressing
by 6 months
nerve compression and atrophy, or head CT can reveal hemorrhagic me-
postpartum. Other,
other focal deficits. tastases with marked surrounding
more serious changes
On MRI, meningiomas most often edema. MRI, which is the imaging to the pituitary include
appear as rounded lesions that arise modality of choice for choriocarcinoma apoplexy, pituitary
from dura and in some cases have a in nonemergent situations, will re- tumor growth, and
dural tail. The margins of a meningioma veal mass lesions with variable size, lymphocytic hypophysitis.
are well defined, and meningiomas associated edema, hemorrhage, and
displace but do not infiltrate into enhancement.29 GRE and SWI se-
surrounding brain parenchyma. On quences will reveal blood products in
T1-weighted images, meningiomas are these lesions and can potentially detect
typically isointense to hypointense smaller hemorrhagic metastases that
compared to cortex, and, if contrast is are not visible on T1- and T2-weighted
given, avid homogenous enhancement sequences.
is seen. Hypercellular meningiomas
can demonstrate reduced diffusivity PITUITARY LESIONS
on DWI. A rim of T2-hyperintense Gestational hormonal changes lead to a
parenchymal edema may surround a variety of pituitary and parasellar changes.
meningioma and can be an indicator of Most generally, during pregnancy the
the tumor’s rate of growth. Similarly, pituitary’s normally concave upper mar-
but conversely, T2-hyperintense gliosis gin may become convex, mimicking
of adjacent parenchyma with volume pituitary hyperplasia and at times causing
loss may indicate a more chronic optic chiasm compression leading to
lesion, and the effects of long-term bitemporal hemianopia. It has been
parenchymal compression. estimated that the pituitary gland en-
larges by 136% during pregnancy, ex-
Gestational Trophoblastic ceeding 10 mm in height and regressing
Disease (Choriocarcinoma) by 6 months postpartum. Other, more
Gestational trophoblastic diseases en- serious changes to the pituitary include
compass a range of malignancies, from apoplexy, pituitary tumor growth, and
benign hydatidiform molar pregnancy to lymphocytic hypophysitis.30
invasive molar pregnancy and choriocar-
cinoma. Choriocarcinoma, a highly vas- Pituitary Apoplexy and
cular and aggressive tumor prone to Sheehan Syndrome
hemorrhage, is the most malignant Pituitary apoplexy refers to an acute
form of these gestational trophoblastic infarction of the pituitary, arising as
Continuum (Minneap Minn) 2014;20(1):23–41 www.ContinuumJournal.com 35

Copyright © American Academy of Neurology. Unauthorized reproduction of this article is prohibited.


Neuroradiology in Women

the pituitary gland grows and outstrips surrounding and in some cases com-
its vascular supply, which leads to pressing the cavernous carotid arteries.
hemorrhagic and/or ischemic changes. During pregnancy, 1.5% of micro-
Patients may develop sudden head- adenomas and 26% of macroadenomas
ache, nausea and vomiting, encepha- become symptomatic. The most com-
lopathy, and endocrine dysfunction.30 mon functioning pituitary adenomas
Focal visual and oculomotor deficits are prolactinomas, followed by those
may arise, given the proximity of the causing acromegaly.30
pituitary to the optic chiasm and cranial On MRI, both microadenomas and
nerves III, IV, and VI. Noncontrast head macroadenomas are discrete mass
CT reveals an enlarged pituitary gland lesions arising from the pituitary
and may reveal the associated hemor- gland. They often cause asymmetric
rhage, best seen in sagittal or coronal enlargement of the pituitary gland and
reconstructed images. MRI is the diag- are best seen in the coronal and
nostic modality of choice. The intensity sagittal planes. If gadolinium is given,
of blood products on T1-weighted and adenomas typically enhance less avidly
T2-weighted images will, as with any than surrounding pituitary tissue.
hemorrhage in the brain, vary according
to temporal evolution.31 Lymphocytic Hypophysitis
Sheehan syndrome refers to clinical Lymphocytic hypophysitis refers to
hypopituitarism that results in deficits autoimmune lymphocytic infiltration
in lactation and postpartum resump- of the adenohypophysis, neurohy-
tion of menses and is precipitated by pophysis, or infundibulum, most com-
pituitary ischemia in the setting of monly occurring in women during late
maternal hemorrhage, shock, or other pregnancy and the first 2 months
obstetric clinical event. Necrosis of the postpartum. The lymphocytic infiltrate
pituitary gland subsequently occurs. may cause the pituitary gland to
Initially, MRI reveals an enlarged pitu- expand, thus compressing adjacent
itary gland without associated hemor- tissue and clinically mimicking a pitu-
rhage. Over time, loss of tissue volume itary adenoma.30
results in an empty sella, best seen on On MRI, the pituitary gland and/or
sagittal MRI or CT.30 the infundibulum appear enlarged,
and findings can be indistinguishable
Pituitary Tumors from a pituitary adenoma. Contrast is
Pituitary adenomas are classified as often required to adequately visualize
microadenomas (ie, less than 10 mm) the stalk, which will enhance avidly
or macroadenomas (ie, equal to or and homogenously.
greater than 10 mm). Clinical manifesta-
tions may include headaches and visual BACK PAIN
field abnormalities from compression of During pregnancy, physiologic changes,
optic pathways, as well as hormonal including increased laxity of joints
alterations (eg, in nonpregnant patients, and spinal ligaments and increasing
prolactinomas may manifest with uterine pressure on the lumbosacral
amenorrhea and galactorrhea). plexus and lumbar spine, commonly
Microadenomas are often slow growing result in exaggerated lordosis and
with a benign course and minimal lower back pain. However, in cases of
clinical manifestations. Macroadenomas more severe pain or loss of motor,
can cause chiasmal compression and sensory, and bowel/bladder func-
expand into the cavernous sinuses, tion, neuroimaging should be used to
36 www.ContinuumJournal.com February 2014

Copyright © American Academy of Neurology. Unauthorized reproduction of this article is prohibited.


KEY POINT
evaluate specific and potentially more Spinal Tumors h MRI is the preferred
severe etiologies.32 MRI is the preferred The growth of several spinal tumors imaging modality for
imaging modality for pregnancy- may be accelerated during pregnancy pregnancy-related back
related back pain, except in cases as a consequence of hormonal signal- pain, except in cases of
of acute trauma. CT should be used ing, causing progressive neurologic acute trauma.
in the setting of trauma to evaluate deficits. As described above, this is
for vertebral fracture and other blunt most commonly seen with meningio-
injuries. mas but can also occur in spinal hem-
angiomas and giant cell tumors.35,39 On
Epidural Processes MRI, spinal meningioma appears as a
Epidural abnormalities can cause ef- discrete soft-tissue mass extrinsic to
facement or displacement of the nor- the spinal cord, often with a dural
mally T1-hyperintense spinal epidural tail.36 Spinal hemangiomas demon-
fat, narrowing of the thecal sac, and strate elevated signal on T1-weighted
displacement of the thecal sac’s dural and T2-weighted sequences. They may
margin. Acute intervertebral disc her- contain intralesional fat, which on fat-
niation, which may rarely occur during suppression sequences (such as short
pregnancy, may present with acute T1 inversion recovery [STIR]) will
weakness, sensory loss, radicular and appear hypointense.37 Spinal giant cell
back pain, and sphincter dysfunction, tumors, while rare, may expand dra-
and in the lumbosacral spine may matically during pregnancy and pres-
progress to a cauda equina syndrome. ent as an expansile mass with
MRI can reveal a prolapsed disc caus- heterogeneous low to intermediate
ing narrowing of the spinal canal or signal intensity on T2-weighted images,
neural foramen.33 a curvilinear area of low signal intensity
An epidural abscess, whether aris- on T1-weighted and T2-weighted im-
ing spontaneously or resulting from ages, cystic changes within the mass,
instrumentation, leads to spinal ten- and heterogeneous enhancement.38
derness, weakness, and fever. On MRI, Spinal metastases from gestational
abnormal T2 hyperintensity may cor- trophoblastic disease may also occur.
respond to a phlegmon, which en-
hances diffusely and homogeneously, OTHER
or to an abscess, which enhances Epilepsy
peripherally. The epidural space may Women with primary seizure disor-
circumferentially enhance.34 ders may experience hormonal mod-
Epidural hematomas may occur ulation of seizure frequency or
either spontaneously during labor or severity. Seizure disorders may be
as a complication of epidural or spinal influenced both by hormonal changes
anesthesia.35 On sagittal MRI, the relating to pregnancy as well as altered
hematoma is most often found in the pharmacokinetics of antiepileptic
posterior epidural space with well- drugs during pregnancy. In a large
defined borders that taper superiorly European registry of pregnant women
and inferiorly. Acutely, within hours to with epilepsy, 63.6% of the women
days after onset, the hematoma will experienced no change in seizure
appear hypointense to isointense on activity; 17% had an increase and
T1-weighted images and with mixed 16% had a decrease in frequency.39
signal on T2-weighted imaging. With In a systematic review, seizure free-
temporal evolution, signal intensity on dom for at least 9 months before
MRI varies. pregnancy was associated with a

Continuum (Minneap Minn) 2014;20(1):23–41 www.ContinuumJournal.com 37

Copyright © American Academy of Neurology. Unauthorized reproduction of this article is prohibited.


Neuroradiology in Women

high likelihood (84% to 92%) of PostYdural puncture headache occurs


remaining seizure free during preg- in approximately 1% of obstetric
nancy.40 The main concerns regarding neuraxial anesthesia cases but also
seizure management during pregnancy after other spinal interventions, such
may relate to antiepileptic drug choice as lumbar puncture and spinal surgery
and dosing, given concerns of terato- during pregnancy. Usual symptoms
genesis and altered pharmacokinet- include acute onset of nuchal or
ics41; and neuroimaging can often be occipital positional headache when
deferred. upright that improves after lying flat
Neuroimaging concerns arise, how- for 15 minutes, and that may be
ever, in the workup of a pregnant accompanied by neck stiffness, tinnitus/
patient presenting with new-onset sei- hyperacusia, diplopia, photophobia, and
zures. In this population, the differential nausea.42
diagnosis for new-onset seizures may, In cases with classic symptoms that
in addition to usual causes, include do not resolve after fluids, caffeine, and
systemic processes such as hypoglyce- pain medications, empiric treatment
mia, but also neurologic processes such with an autologous epidural blood patch
as eclampsia, CVT, PRES, intracerebral may be considered, with usual resolu-
hemorrhage, RCVS, TTP, enlarging in- tion of symptoms within 48 hours.
tracranial tumors, meningitis, encepha- However, in cases where the diagnosis
litis, or neuroinflammation. Except in is delayed as a result of atypical symp-
patients with typical manifestations of toms, MRI may reveal low-lying cerebel-
eclampsia, neuroimaging is often nec- lar tonsils, ‘‘brain sag,’’ and convexity
essary. Standard radiologic evaluation subdural hematoma arising from the
for epilepsy may be performed without tearing of bridging veins. If it is necessary
contrast and includes evaluation of to administer gadolinium, diffuse
anatomy, cortical gyral patterns, and smooth pachymeningeal enhancement
heterotopic gray matter on thin- is often seen.
section volumetric T1-weighted images;
and evaluation of mesial temporal, Wernicke Encephalopathy
hippocampal, and other parenchymal During pregnancy, hyperemesis
signal abnormalities on T2-weighted gravidarum as well as increased fetal
sequences. However, given additional metabolic demand for thiamine may
diagnostic considerations in pregnant lead, like chronic alcoholism and mal-
patients, gadolinium may improve diag- nutrition, to thiamine deficiency. This
nostic accuracy; the advantages it pro- deficiency can result in Wernicke
vides for diagnosis and management of encephalopathy due to impaired
maternal seizures must be weighed neuronal metabolism, lactic acid accu-
against the potential risks to the con- mulation, impaired blood-brain bar-
ceptus. In suspected cases of CVT or rier, and neuronal death in areas
RCVS, TOF MRI angiography may be of high thiamine-dependent glucose
performed, thus avoiding the use of metabolism. Only rarely do patients
contrast. present with the full clinical triad of
acute encephalopathy, ataxia, and
Intracranial Hypotension ophthalmoplegia; they may occasionally
Intracranial hypotension arising from present in coma.43
CSF leaks results most often from On MRI, abnormal T2 hyper-
spinal anesthesia or inadvertent dural intensities in the paramedian thalamic
puncture during epidural anesthesia. nuclei, in the mammillary bodies,

38 www.ContinuumJournal.com February 2014

Copyright © American Academy of Neurology. Unauthorized reproduction of this article is prohibited.


KEY POINT
around the third and fourth ventricles, www.acog.org/Resources_And_ h Pregnancy engenders a
and in the periaqueductal gray matter Publications/Committee_Opinions/ series of physiologic
and superior cerebellar vermis may be Committee_on_Obstetric_Practice/ changes that increase
noted. These lesions may enhance Guidelines_for_Diagnostic_Imaging_ the risk of certain
with gadolinium. On DWI, reduced During_Pregnancy neurologic conditions,
diffusivity indicating cytotoxic edema American College of Radiology and
such as preeclampsia,
may be noted. High-dose IV thiamine stroke, hemorrhage,
Society for Pediatric Radiology Prac- and back pain. These
can rapidly reverse this condition if tice Guideline for Imaging Pregnant or
administered early, and therefore, in changes should be
Potentially Pregnant Adolescents and considered, along with
cases with high clinical suspicion, treat- Women With Ionizing Radiation the typical range of
ment should not await imaging confir- www.acr.org/~/media/ACR/Documents/ neurologic conditions
mation. Recognition of the clinical PGTS/guidelines/Pregnant_Patients.pdf affecting women of
symptoms and associated radiologic reproductive age, in any
findings is essential to prompt life- American Academy of Pediatrics Com- pregnant woman
saving therapy in pregnant women. mittee on DrugsVMaternal Medications presenting with
Usually Compatible With Breastfeeding neurologic symptoms.
pediatrics.aappublications.org/
SUMMARY content/108/3/776/T6.full
Pregnancy engenders a series of phys-
University of California, San Francisco
iologic changes that increase the risk
Guidelines for the Use of CT and MRI
of certain neurologic conditions, such
During Pregnancy and Lactation
as preeclampsia, stroke, hemorrhage,
www.radiology.ucsf.edu/patient-
and back pain. These changes should
care/patient-safety/ct-mri-pregnancy
be considered, along with the typical
range of neurologic conditions affect-
ing women of reproductive age, in any REFERENCES
pregnant woman presenting with neu- 1. Cunningham FG, Leevno KJ, Bloom SL,
rologic symptoms. Hauth JC, Rouse DJ, Spong CY, editors.
Williams obstetrics, 23rd ed. The
Pregnant women presenting with McGraw-Hill Companies, Inc., 2010.
neurologic symptoms may require
2. American College of Obstetricians and
diagnostic imaging. When this cannot Gynecologists Committee on Obstetric
be delayed to the postpartum period, Practice. ACOG Committee Opinion. Number
the choice of appropriate imaging 299, September 2004 (replaces No. 158,
September 1995). Guidelines for diagnostic
modality should reflect a trade-off imaging during pregnancy. Obstet Gynecol
between the most sensitive test for 2004;104(3):647Y651.
the pregnant patient and the safest 3. American College of Radiology Committee
test for her conceptus. At this time, on Drugs and Contrast Media. ACR manual
MRI without gadolinium is most advis- on contrast media, Version 9. www.acr.org/~/
media/ACR/Documents/PDF/QualitySafety/
able in many cases. When performed Resources/Contrast%20Manual/2013_
in a timely and safe fashion, diagnostic Contrast_Media.pdf. Accessed December 5, 2013.
imaging can be a highly useful com- 4. Webb JA, Thomsen HS, Morcos SK. The use
ponent of the prompt evaluation of a of iodinated and gadolinium contrast media
pregnant patient presenting with neu- during pregnancy and lactation. Eur Radiol
2005;15(6):1234Y1240.
rologic symptoms.
5. Hall EJ, Giaccia AJ. Radiobiology for the
USEFUL WEBSITES Radiologist. Philadelphia, PA: Lippincott,
Williams, and Wilkins, 2006.
American College of Obstetricians and
6. Osei EK, Faulkner K. Fetal doses from
Gynecologists Guidelines for Diagnostic radiological examinations. Br J Radiol
Imaging During Pregnancy 1999;72(860):773Y780.

Continuum (Minneap Minn) 2014;20(1):23–41 www.ContinuumJournal.com 39

Copyright © American Academy of Neurology. Unauthorized reproduction of this article is prohibited.


Neuroradiology in Women

7. Wakeford R, Little MP. Risk coefficients brain CT and MRI findings in 12 patients.
for childhood cancer after intrauterine Neurology 1999;52(6):1285Y1288.
irradiation: a review. Int J Radiat Biol
22. Bartynski WS. Posterior reversible
2003;79(5):293Y309.
encephalopathy syndrome, part 1: fundamental
8. Yamazaki JN, Schull WJ. Perinatal loss imaging and clinical features. AJNR Am J
and neurological abnormalities among Neuroradiol 2008;29(6):1036Y1042.
children of the atomic bomb. Nagasaki
23. Ducros A. Reversible cerebral vasoconstriction
and Hiroshima revisited, 1949 to 1989.
syndrome. Lancet Neurol 2012;11(10):906Y917.
JAMA 1990;264(5):605Y609.
24. Fugate JE, Ameriso SF, Ortiz G, et al.
9. Nelson JA, Livingston GK, Moon RG.
Variable presentations of postpartum
Mutagenic evaluation of radiographic
angiopathy. Stroke 2012;43(3):670Y676.
contrast media. Invest Radiol 1982;17(2):
183Y185. 25. Bourre B, Marignier R, Zephir H, et al.
Neuromyelitis optica and pregnancy.
10. Gruters A, Krude H. Detection and
Neurology 2012;78(12):875Y879.
treatment of congenital hypothyroidism.
Nat Rev Endocrinol 2011;8(2):104Y113. 26. Polman CH, Reingold SC, Banwell B, et al.
Diagnostic criteria for multiple sclerosis:
11. Kanal E, Shellock FG, Talagala L. Safety
2010 revisions to the McDonald criteria.
considerations in MR imaging. Radiology
Ann Neurol 2011;69(2):292Y302.
1990;176(3):593Y606.
27. Doyle S, Messiou C, Rutherford JM, Dineen
12. Salonen Ros H, Lichtenstein P, Bellocco R,
RA. Cancer presenting during pregnancy:
et al. Increased risks of circulatory diseases
radiological perspectives. Clin Radiol
in late pregnancy and puerperium.
2009;64(9):857Y871.
Epidemiology 2001;12(4):456Y460.
28. Smith HO, Kohorn E, Cole LA.
13. Wilterdink JL, Easton JD. Cerebral ischemia
Choriocarcinoma and gestational
in pregnancy. Adv Neurol 2002;90:51Y62.
trophoblastic disease. Obstet Gynecol
14. Gilbert WM, Danielsen B. Amniotic fluid Clin North Am 2005;32(4):661Y684.
embolism: decreased mortality in a
29. Mamelak AN, Withers GJ, Wang X.
population-based study. Obstet Gynecol
Choriocarcinoma brain metastasis in a
1999;93(6):973Y977.
patient with viable intrauterine pregnancy.
15. Coutinho JM, Ferro JM, Canhao P, et al. Case report. J Neurosurg 2002;97(2):
Cerebral venous and sinus thrombosis in 477Y481.
women. Stroke 2009;40(7):2356Y2361.
30. Motivala S, Gologorsky Y, Kostandinov J,
16. Saposnik G, Barinagarrementeria F, Brown Post KD. Pituitary disorders during pregnancy.
RD Jr, et al. Diagnosis and management of Endocrinol Metab Clin North Am 2011;40(4):
cerebral venous thrombosis: a statement for 827Y836.
healthcare professionals from the American
31. Turgut M, Ozsunar Y, Basak S, Guney E,
Heart Association/American Stroke Association.
Kir E, Meteoglu I. Pituitary apoplexy: an
Stroke 2011;42(4):1158Y1192.
overview of 186 cases published during
17. Bateman BT, Olbrecht VA, Berman MF, the last century. Acta Neurochir (Wien)
et al. Peripartum subarachnoid hemorrhage: 2010;152(5):749Y761.
nationwide data and institutional
32. Han IH. Pregnancy and spinal problems.
experience. Anesthesiology 2012;116(2):
Curr Opin Obstet Gynecol 2010;22(6):
324Y333.
477Y481.
18. Kotsenas AL, Roth TC, Hershey BL, Yi JK.
33. Del Grande F, Maus TP, Carrino JA. Imaging
Imaging neurologic complications of
the intervertebral disk: age-related changes,
pregnancy and the puerperium. Acad Radiol
herniations, and radicular pain. Radiol Clin
1999;6(4):243Y252.
North Am 2012;50(4):629Y649.
19. Steegers EA, von Dadelszen P, Duvekot JJ,
34. Sandhu FS, Dillon WP. Spinal epidural abscess:
Pijnenborg R. Pre-eclampsia. Lancet
evaluation with contrast-enhanced MR
2010;376(9741):631Y644.
imaging. AJNR Am J Neuroradiol 1991;12(6):
20. Roth C, Ferbert A. The posterior reversible 1087Y1093.
encephalopathy syndrome: what’s certain,
35. Wang P, Xin XT, Lan H, et al. Spontaneous
what’s new? Pract Neurol 2011;11(3):
cervical epidural hematoma during
136Y144.
pregnancy: case report and literature
21. Bakshi R, Shaikh ZA, Bates VE, Kinkel PR. review. Eur Spine J 2011;20(suppl 2):
Thrombotic thrombocytopenic purpura: S176YS179.

40 www.ContinuumJournal.com February 2014

Copyright © American Academy of Neurology. Unauthorized reproduction of this article is prohibited.


36. Gezen F, Kahraman S, Canakci Z, Beduk A. change in seizure frequency: Report of
Review of 36 cases of spinal cord meningioma. the Quality Standards Subcommittee and
Spine (Phila Pa 1976) 2000;25(6):727Y731. Therapeutics and Technology Assessment
Subcommittee of the American Academy
37. Vijay K, Shetty AP, Rajasekaran S.
of Neurology and the American Epilepsy
Symptomatic vertebral hemangioma in
Society. Epilepsia 2009;50(5):1229Y1236.
pregnancy treated antepartum. A case
report with review of literature. Eur Spine J 41. Tomson T, Landmark CJ, Battino D.
2008;17(suppl 2):S299YS303. Antiepileptic drug treatment in pregnancy:
Changes in drug disposition and their
38. Kwon JW, Chung HW, Cho EY, et al. clinical implications. Epilepsia 2013;54(3):
MRI findings of giant cell tumors of the spine. 405Y414.
AJR Am J Roentgenol 2007;189(1):246Y250.
42. Chang LY, Carabuena JM, Camann W.
39. EURAP Study Group. Seizure control and Neurologic issues and obstetric anesthesia.
treatment in pregnancy: observations from Semin Neurol 2011;31(4):374Y384.
the EURAP epilepsy pregnancy registry.
Neurology 2006;66(3):354Y360. 43. Chiossi G, Neri I, Cavazzuti M, Basso G,
Facchinetti F. Hyperemesis gravidarum
40. Harden CL, Hopp J, Ting TY, et al. complicated by Wernicke encephalopathy:
Management issues for women with background, case report, and review of the
epilepsy-Focus on pregnancy (an evidence-based literature. Obstet Gynecol Surv 2006;61(4):
review): I. Obstetrical complications and 255Y268.

Continuum (Minneap Minn) 2014;20(1):23–41 www.ContinuumJournal.com 41

Copyright © American Academy of Neurology. Unauthorized reproduction of this article is prohibited.


Review Article

Multiple Sclerosis in
Address correspondence to
Dr P. K. Coyle, Department of
Neurology, Stony Brook
University, HSC, T-12, Room 020,
Stony Brook, NY 11794-8121,
Patricia.Coyle@
stonybrookmedicine.edu.
Pregnancy
Relationship disclosure: Patricia K. Coyle, MD, FAAN
Dr Coyle has consulted
for Acorda Therapeutics;
Accordant Health Services;
Bayer AG; Biogen Idec; ABSTRACT
Genentech, Inc; Genzyme
Corporation; Merck KGaA; Purpose of Review: This article reviews the current understanding of the
Mylan Inc; Novartis Corporation; interactions between multiple sclerosis (MS) and pregnancy, and implications for
and Teva Pharmaceuticals. reproductive counseling. This is a key topic in MS because the typical patient is a
Dr Coyle receives clinical
trial support from Actelion young woman of childbearing age.
Pharmaceuticals; Novartis Recent Findings: It has been known for some time that MS disease activity
Corporation; and Opexa markedly reduces during the last trimester of pregnancy, then markedly increases
Therapeutics, Inc.
Unlabeled Use of
in the 3 months postpartum before returning to the prepregnancy baseline.
Products/Investigational High relapse rate or disability before pregnancy, as well as relapse during
Use Disclosure: pregnancy, have been associated with increased risk for postpartum attacks. Recent
Dr Coyle discusses the
unlabeled use of IV
data continue to support the conclusion that long-term disease progression is
immunoglobulin postpartum. not worsened (and may actually be lessened) with pregnancy in patients with
* 2014, American Academy relapsing MS; the data are not so clear for those with progressive MS. Among the
of Neurology. MS disease-modifying therapies, the only one that requires contraception use by
men is the new oral agent teriflunomide, because the drug is present in semen. It is
reassuring that, to date, no human teratogenic effects have been documented for
any of the MS disease-modifying therapies.
Summary: Pregnancy has a profound effect on MS disease activity. Identification of
the responsible mechanisms for this effect should lead to new disease insights and
therapeutic strategies.

Continuum (Minneap Minn) 2014;20(1):42–59.

MULTIPLE SCLEROSIS BASICS have MS. This is probably the tip of the
Multiple sclerosis (MS) is the major iceberg. Autopsy studies suggest that
acquired neurologic disease of young the number of affected individuals can
adults, short of trauma, and has some be doubled. Silent MS is the mildest end
interesting characteristic features. MS of the disease spectrum.
affects young people: 90% of patients MS takes two major forms. At least
have their clinical onset between the 85% to 90% of patients present with
ages of 15 and 50 years. Less than 1% relapsing disease, characterized by
present before age 10 or after age 60. intermittent attacks (also called re-
It is strongly female predominant lapses, exacerbations, or flare-ups).
(70% to 75%) and highly variable in Between attacks, patients are clinically
its expression. No two patients are stable. This is the major clinical MS
alike. MS is on the rise among young subtype. The first attack is also re-
women. The reasons for this are unclear ferred to as a clinically isolated syn-
but are not explained by enhanced drome (CIS). Not all CIS patients turn
awareness or diagnostic testing. out to have relapsing MS. CIS patients
At least 400,000 people in the United are classified as low or high risk for MS
States and over 2.5 million worldwide based on whether silent brain MRI

42 www.ContinuumJournal.com February 2014

Copyright © American Academy of Neurology. Unauthorized reproduction of this article is prohibited.


KEY POINTS
lesions are present. CIS patients at tolerant state, in which the maternal h Multiple sclerosis is on
high risk for MS can be offered immune system adapts to an alloge- the rise among young
disease-modifying therapy (DMT). neic pregnancy.2 Mother and fetus women.
The other major form of MS is pro- have active communication, with bidi-
h The prototypical
gressive MS, with slow worsening rectional exchange of cells (leading to multiple sclerosis patient
independent of relapses. Only 10% to fetomaternal microchimerism) and remains the young
15% of patients show a progressive other immune components.3 Four woman with relapsing
course from onset. They differ from important biological changes occur disease.
most MS patients because of their during pregnancy that are probably h Pregnancy is
later age at clinical disease onset (late pertinent for MS. First, a number of considered not an
thirties, early forties) and equal gen- hormone levels increase markedly immunosuppressive
der ratio (no female predominance). during pregnancy and then fall dra- state but an
This unusual group is divided into matically postpartum, including estro- immune-tolerant state.
primary progressive MS (by definition, gens (especially estriol), progesterone, h Insights into how the
they never experience a relapse) and prolactin, and glucocorticoids. These immune system
progressive relapsing MS (they have hormones have very strong effects on operates during
occasional relapses superimposed on the immune system. They shift cyto- pregnancy may
their slow worsening). The most com- kines, decrease adhesion molecules hopefully provide ideas
mon progressive subtype of MS, how- and matrix metalloproteinases, de- about novel treatment
ever, is secondary progressive MS, crease antigen presentation, and boost strategies for multiple
made up of relapsing patients who numbers of regulatory T cells (T-regs).2 sclerosis.
ultimately transition to a slow worsen- The net result is a decrease in inflam- h Multiple sclerosis has
ing phase in which attacks become matory processes. Second, signif- little to no direct effect
less frequent and ultimately stop. The icant immune cell shifts occur during on pregnancy.
natural history of untreated MS is to pregnancy involving T-regs, T-helper
start out as the relapsing subtype 17 cells, and natural killer cells. Third,
and then to evolve after some time into fetal antigens interact with and mo-
the secondary progressive subtype. dulate the maternal immune system.
Progressive MS leads to disability. This Peripheral T-regs appear that are specific
clinical subtype classification, created for fetal antigens. Fetal antigens released
by expert consensus, is expected to into small microvesicles are taken up by
change shortly to include neuroimaging maternal dendritic cells, which are very
characteristics as well as at-risk individ- potent antigen-presenting cells. Finally,
uals who are identified radiologically. recent studies (still very sketchy) suggest
The prototypical MS patient remains that pregnancy may result in positive
the young woman with relapsing dis- benefits to the maternal CNS by promot-
ease. Pregnancy is a major topic at some ing endogenous recovery mechanisms
point for most of these patients, and at and enhancing the ability to respond to
least 20% to 33% will go on to have immune-mediated injury. Insights into
children after they are diagnosed with how the immune system operates dur-
MS.1 Fortunately, much is known about ing pregnancy may hopefully provide
MS and pregnancy, so patients can be ideas about novel treatment strategies
given valuable information and helpful for MS.
reproductive counseling (Table 2-1).
MULTIPLE SCLEROSIS IMPACT
THE IMMUNE SYSTEM IN ON PREGNANCY
PREGNANCY MS has little to no direct effect on
Pregnancy is considered not an immu- pregnancy.4 Patients can be told with
nosuppressive state but an immune- confidence that their disease has no
Continuum (Minneap Minn) 2014;20(1):42–59 www.ContinuumJournal.com 43

Copyright © American Academy of Neurology. Unauthorized reproduction of this article is prohibited.


Multiple Sclerosis in Pregnancy

TABLE 2-1 Reproductive Counseling Topics for Multiple Sclerosis

Topic Counseling
General
Fertility Not affected by multiple sclerosis (MS).
Contraceptive use No evidence for negative
effect in relapsing MS.
Genetic risk MS is not an inherited disease.
Disease risk is slightly increased when
a parent has MS (2%Y2.5%); risk is
increased to 31% when both parents
have MS.
Sexual dysfunction Increased in both men and women with MS,
especially over time; may require specific
counseling and treatments.
Pregnancy-related
In vitro fertilization Use of gonadotropin-releasing
hormone agonists may be associated
with temporary increased risk
for relapses.
Pregnancy prognosis: No increase in ectopic pregnancies,
outcome birth defects, miscarriage, or stillbirths.
Pregnancy prognosis: Marked decrease in disease activity
short term during third trimester, followed by
marked rebound increase in the
3 months postpartum.
Pregnancy impact: No negative impact on relapsing MS;
long term suggestion of positive long-term benefit
in decreasing disability.
Treatments during Generally disease-modifying therapies are not
pregnancy used. Symptomatic treatments are confined to
necessary drugs. Glucocorticoids can be used
to treat relapses after the first trimester.
Delivery and postpartum
Delivery options (natural, Obstetric decision; MS is not a factor
vaginal assisted, cesarean) except with very disabled patients.
Anesthetic Obstetric decision; MS is not a factor
except with very disabled patients.
Breast-feeding No negative effect on MS. Conflicting
data indicate exclusive breast-feeding
probably decreases disease activity.

meaningful impact on ability to conceive, or major neonatal or obstetric complica-


pregnancy itself, ability to deliver, or tions has been proven.
fetal status and well-being. No The one exception is the very dis-
convincing increase in spontaneous abled patient who becomes pregnant,
abortions, assisted vaginal deliveries, but this turns out to be quite unusual.1
ectopic pregnancies, cesarean deliveries, Approximately 97% of pregnant MS
44 www.ContinuumJournal.com February 2014

Copyright © American Academy of Neurology. Unauthorized reproduction of this article is prohibited.


KEY POINTS
patients have the relapsing subtype. reassuring. No single gene produces h Approximately 97% of
Most are in the earlier stages of relaps- MS. This is an important topic to bring pregnant patients with
ing MS and do not have much in the up with patients early on and should multiple sclerosis have
way of accumulated baseline disability. always be included in reproductive the relapsing subtype.
They are in good shape physically. counseling (see Table 2-1). Risk for h The most current data
Pregnancy is quite uncommon in pa- MS in the general population is about support that pregnancy
tients with progressive MS, who often 0.13%. When a parent has MS, risk to a is not detrimental and
have significant motor problems that child is about 2% to 2.5%. Surprisingly, may actually be a
might interfere with the delivery process. the risk is a little greater to develop MS favorable prognostic
Occasional reports have associated MS when a sibling has the disease (2.7%) feature for multiple
with lower birth weight, higher rates of than when a parent does.8 Such data sclerosis.
induced labor, operative intervention, suggest the importance of environmen- h Reliable information is
cesarean delivery, and rehospitalization, tal over genetic factors in MS. available about genetic
although most studies have failed to Reproductive counseling for MS risk of multiple sclerosis,
confirm these associations. A recent patients not only encompasses a dis- and it is very reassuring.
meta-analysis of the literature to date cussion of the risk of passing their h When a parent has
concluded that women with MS were disease on to future children, but also multiple sclerosis, risk to
not at significantly higher risk for obstet- addresses the impact the disease a child is about 2% to
ric or neonatal complications.5 might have on use of contracep- 2.5%.
Any young woman just diagnosed tion, pregnancy management and out- h The latter part of
with MS is facing many unknowns. She come, use of therapies during pregnancy, pregnancy confers
is going to be concerned that she may and postpartum issues including profound reduction in
pass MS on to her children. She may be breast-feeding. Most of these issues multiple sclerosis
afraid that MS is going to disable her are covered in this review; with regard disease activity.
and put her in a wheelchair. It to use of oral contraception in MS, the
is common for such patients to ask limited available data suggest no nega-
their neurologist whether they should tive effect and hint at the possibility
avoid getting pregnant. Newly diag- that use may delay onset of MS and
nosed patients need to be informed may mitigate symptoms for patients
that the most current data support that with known MS.9,10
pregnancy is not detrimental and may
actually be a favorable prognostic fea- PREGNANCY AND DISEASE
ture for MS. They need comprehensive ACTIVITY
reproductive counseling. Much more is It is unusual for MS to present during
known about MS and pregnancy now pregnancy. Relapse can occur, but less
than in prior years, and a lot of stress often than outside of pregnancy.
can be alleviated by providing accurate, The latter part of pregnancy confers
clear information. profound reduction in MS disease
activity. This was most clearly shown
GENETIC RISK AND REPRODUCTIVE in the Pregnancy in Multiple Sclerosis
COUNSELING (PRIMS) study, the best large-scale
As highlighted in Case 2-1, many MS prospective analysis published to
patients worry that they may pass date.11 PRIMS included 254 women
their disease on to their children. This (246 with relapsing MS), and 269
has been reported to be an important pregnancies. Each patient’s first
reason that some patients choose not pregnancy leading to a live birth was
to have a child after their diagnosis.7 included for analysis, for a total of 227
Reliable information is available about pregnancies. Patients were followed
genetic risk of MS, and it is very for at least 12 months postpartum.
Continuum (Minneap Minn) 2014;20(1):42–59 www.ContinuumJournal.com 45

Copyright © American Academy of Neurology. Unauthorized reproduction of this article is prohibited.


Multiple Sclerosis in Pregnancy

KEY POINT
h The most consistently
identified markers
Case 2-1
A 25-year-old right-handed woman developed left eye monocular vision loss
associated with
with pain shortly after returning from her honeymoon. Neurologic examination
postpartum activity
abnormalities were confined to cranial nerve II. Workup showed abnormal brain
are high relapse rate
MRI. Enhancement of the left optic nerve was evident, consistent with her optic
in the year before
neuritis, and four hyperintense T2/fluid-attenuated inversion recovery (FLAIR)
pregnancy, higher
brain lesions (two periventricular, two juxtacortical) were present, ranging in
disability level before
size from 4 mm to 10 mm. One of the brain lesions enhanced. Spinal MRI showed
pregnancy, and relapse
a nonenhancing thoracic lesion at T2. CSF contained 18 white blood cells per
during pregnancy.
mm3 and was oligoclonal band positive. Workup was otherwise unremarkable.
The patient was diagnosed with relapsing multiple sclerosis (MS) and treated
with 3 days of high-dose glucocorticoids (1 g/d of IV methylprednisolone).
Three weeks later, her neurologic examination was normal. On this follow-up
visit, she asked her physician whether it would be better to try to become
pregnant right away or delay pregnancy in order to go on treatment for her
MS. She had always dreamed of having several children. She voiced concerns
about becoming disabled in the future. She also verbalized her fear that she
might pass MS on to her children. She was reassured that MS is not an inherited
disease; that most patients with MS in the treatment era are doing well for
years after they are diagnosed; and that the timing of a pregnancy could be
based on the personal decisions she and her husband made, as her MS process
was mild enough that it should not be a factor.
Comment. This is a fairly typical story for a newly diagnosed relapsing
MS patient. Optic neuritis is a clinically isolated syndrome that carries
about a 70% likelihood of being due to relapsing MS. Despite a single
attack, this young woman meets the formal 2010 diagnostic criteria for
relapsing MS, based on MRI dissemination in time and space.6

Compared to prepregnancy, annual- have been made to identify risk


ized relapse rate fell by 70% during factors for postpartum attacks. The
the third trimester. During the first 3 most consistently identified markers
months postpartum, the relapse rate associated with postpartum activity
rebounded to 70% above the are high relapse rate in the year before
prepregnancy level, then came down pregnancy, higher disability level be-
and stayed down at the prepregnancy fore pregnancy, and relapse during
rate. This suppression of clinical at- pregnancy.
tacks in later pregnancy has been Pregnancy may prevent MS. In a
confirmed in several prospective clin- recent Australian study of 282 women
ical trials and has also been backed up with CIS versus 542 matched controls,
by much more limited MRI data, a higher number of pregnancies/births
showing decreases in silent brain le- decreased the likelihood of experienc-
sion development. Postpartum activity ing such a first attack, consistent with
was confirmed in a recent analysis of a cumulative protective effect from
19 women with MS, in which 11 (58%) pregnancy.13 Radiologically isolated
had increase in postpartum versus syndrome (RIS) involves detection of
third trimester MRI lesion activity.12 a very abnormal brain MRI suggestive
The 3-month period of increased risk of MS in otherwise clinically normal
postpartum has also been a consistent individuals. It may represent asymp-
observation. About 30% of MS patients tomatic or presymptomatic MS. In a
will relapse during this period. Attempts study of 60 women with RIS, seven

46 www.ContinuumJournal.com February 2014

Copyright © American Academy of Neurology. Unauthorized reproduction of this article is prohibited.


KEY POINT
became pregnant.14 In this small RIS born in November, which suggests that h Birth month appears to
cohort, pregnancy was associated with effects on the developing immune system be a risk factor for
increased clinical and MRI disease might be the responsible mechanism. multiple sclerosis.
activity, as though silent disease was Higher maternal milk consumption
activated. Further studies on CIS and and vitamin D levels have been
RIS will be needed to clarify these suggested to protect against develop-
points. ing MS,18 but such a link is far from
proven.19 A northern Swedish study
PREGNANCY-RELATED RISK evaluated two banked serum samples:
FACTORS the first was from a large cohort of
Birth month appears to be a risk factor women, 192 of whom later developed
for MS. In a review of 17 previous MS (at a median time of 9 years after
studies (all but one of which focused sample collection); the second, smaller
on regions in the northern hemi- cohort involved women who gave
sphere), 80% reported that children birth to offspring who eventually
born in the spring were significantly developed MS (n = 37). They were
more likely to have MS.15 Those born compared to control women whose
in the fall were significantly less likely children did not develop MS. The
to have MS. The single study from the study found that high vitamin D levels
southern hemisphere found this effect in the preceding years (but not gesta-
in the corresponding seasons at the tional levels) decreased risk for MS.20
opposite time of yearVthat is, the It will be important to determine
same seasonal effect with excess risk whether correcting low vitamin D can
for November births (spring in the prevent MS.
southern hemisphere) and decreased
risk for April births (fall in the PROGNOSIS
southern hemisphere). In another Up until the 1950s, women with MS
extensive analysis of over 150,000 MS were told that pregnancy was bad for
patients born in the northern hemi- their disease and were actively dis-
sphere, excess risk for MS was couraged from becoming pregnant.
reported for those born in April, and This belief turned out to be wrong.
decreased risk for those born in Since then, accumulating data suggest
October and November.16 This study that pregnancy not only carries a
also found a higher rate of MS with favorable prognosis in the short term
increasing latitude in the northern during the last trimester but may also
hemisphere. The widely held explana- be associated with more favorable
tion for such a link between MS risk, long-term prognosis.21,22 In a recent
latitude, and birth month has been a cross-sectional study of 973 women with
relationship between ultraviolet light MS, relapsing-onset patients with two or
exposure and maternal vitamin D more pregnancies were less likely to
levels. In a recent study from London, reach a standardized disability marker
cord blood was collected from 50 (an Expanded Disability Status Scale
healthy babies born in May and 50 [EDSS] score of 6, which means that a
babies born in November.17 In this cane is required to walk 100 meters).23
region, being born in May increases However, this observation did not hold
risk for MS by 20% compared to being true for women with progressive-onset
born in November. The May babies MS. In the smaller progressive cohort,
had significantly higher levels of T cells early oral contraceptive use increased
and lower vitamin D levels than those risk for reaching EDSS 6, while higher
Continuum (Minneap Minn) 2014;20(1):42–59 www.ContinuumJournal.com 47

Copyright © American Academy of Neurology. Unauthorized reproduction of this article is prohibited.


Multiple Sclerosis in Pregnancy

KEY POINTS
h None of the published age at onset of menses decreased this remain on therapy. Yet DMTs can be
reports or ongoing risk. This raises a provocative and considered for patients with very severe
pregnancy registries for disturbing possibility that pregnancy or highly active MS.26
any of the and hormonal states may have a differ- Several different contraceptive tech-
disease-modifying ent impact on women with progressive niques can be used to avoid pregnancy
therapies has ever versus relapsing MS. However, the data (Table 2-3 27). It is important to make
shown teratogenic are far from clear. In a population-based sure that patients are aware of their
effects. study from British Columbia, pregnancy varying failure rates.
h No uniform guidelines had no effect on reaching EDSS 6; No uniform guidelines exist to direct
exist to direct the rather, that disability milestone was the stopping of a DMT before trying to
stopping of a related to age at MS onset.24 A recent become pregnant. One month is reason-
disease-modifying New York State Multiple Sclerosis Con- able for the interferon "s, glatiramer
therapy before trying to sortium analysis found just the oppo- acetate, and BG-12/dimethyl fumarate,
become pregnant. site: pregnancy delayed time to EDSS and 2 months for fingolimod. Although
6.25 Clearly, further studies are needed 3 months have been recommended
to clarify these issues. Such studies will with natalizumab, it may be that 1
have to employ large databases to month is sufficient, and the antibody
collect sufficient case numbers. does not need to be totally gone from
the system before a patient tries to
TREATMENTS DURING become pregnantVthe level is much
PREGNANCY lower at 1 month, longer washout
There is always a concern about is associated with risk of activity,
medication use in pregnant women. the patient is not likely to get preg-
Very few pregnancy drugs have been nant immediately, and no negative
rated category A by the US Food and pregnancy effect has been documented.
Drug Administration (FDA). For the Teriflunomide should always be washed
MS patient, treatment concerns can be out. MS patients can be counseled,
divided into three areas: use of DMTs, based on general population data, that
treatment of acute relapses, and use of 68% of couples actively trying for
symptomatic therapies. pregnancy are successful by 3 months,
and 90% by 12 months.28
Disease-Modifying Therapies
Pregnancy ratings for the 10 FDA- Interferon "
approved DMTs, as well as drugs in The interferon "s have shown dose-
development that may be approved in dependent first trimester abortifacient
the next few years, are shown in Table 2-2. effects in primate models.29,30 This is
These ratings are due to change shortly, at higher doses than are used in
as the FDA is in the process of dropping humans. No animal-model evidence
the letter rating and moving to more exists for teratogenicity, and interferon
descriptive commentary. The FDA and " as a large macromolecule has no
the National MS Society consensus significant passage across the placenta.
statement have said that DMTs should Some small-scale studies have reported
not be used in MS patients who are both increased abortion rate and low
pregnant, trying to become pregnant, birth weight with interferon " exposure
or breast-feeding. In fact, none of the in early pregnancy, while larger regis-
published reports or ongoing pregnancy tries have not confirmed this. For IM
registries for any of the DMTs has ever interferon "1a, an American registry of
shown teratogenic effects. Nevertheless, 302 pregnant women and a global
only a minority of pregnant MS patients database involving over 500 known
48 www.ContinuumJournal.com February 2014

Copyright © American Academy of Neurology. Unauthorized reproduction of this article is prohibited.


TABLE 2-2 US Food and Drug Administration Pregnancy Category
Ratings for Current and Potential Disease-Modifying
Therapies for Multiple Sclerosisa

FDA
Pregnancy Animal Model
Drug Category Issues
Approved disease-modifying therapies
Glatiramer acetate B None
Interferon " C Abortifacient activity
Fingolimod C Fetal malformations
BG-12/dimethyl fumarate C Embryo toxicity,
testicular/body
weight issues,
neurobehavioral
issues
Natalizumab C Hematologic,
survival issues
Mitoxantrone D Decreased fetal
weight, increased
prematurity
Teriflunomide X Fetal malformations
and death
Potential disease-modifying therapies
Anti-CD 20s (rituximab, ofatumumab) C B-cell lymphoid tissue
reduced for 6 months
in offspring
Alemtuzumab C No data available
Daclizumab C Early prenatal loss
in monkeys
Laquinimod Unknown No data available
FDA = US Food and Drug Administration.
a
Please see Appendix A for the US Food and Drug Administration Pregnancy Category descriptions.

outcomes found no specific pattern or abortion.33 The bottom line is that no


concern for either birth defects or evidence exists of teratogenicity with
spontaneous abortions.31 A large global interferon ".
database of 425 pregnancies exposed Interferon " shows minimal levels
to subcutaneous interferon "1a found (around 0.006% of the maternal dose)
no increase in spontaneous abortions in breast milk.34 In this study, no
or birth defects.32 In a recent system- impact was appreciated on the six
atic review of the literature up to breast-feeding infants.
February 2012, based on 761 pregnan-
cies, interferon " exposure was associ- Glatiramer Acetate
ated with lower mean birth weight, Glatiramer acetate has the most favor-
shorter mean birth length, and pre- able pregnancy rating (class B). It has
term (less than 37 weeks) birth, but been safe in animal models at doses
not low birth weight (less than 2500 g), far above those used to treat MS, and
congenital anomaly, or spontaneous both postmarketing surveillance data

Continuum (Minneap Minn) 2014;20(1):42–59 www.ContinuumJournal.com 49

Copyright © American Academy of Neurology. Unauthorized reproduction of this article is prohibited.


Multiple Sclerosis in Pregnancy

a
TABLE 2-3 Contraception Options for Multiple Sclerosis Patients

Methods Failure Rate


Abstinence 0%
Natural family planning/rhythm method 25%
Spermicide 30%
Barrier methods
contraceptive sponge with spermicide 16%Y32%
diaphragm, cervical cap, cervical shield 15%
(recommended with spermicide)
male condom 11%Y16%
female condom 20%
Hormonal methods
oral contraceptives (combined pill) 5% (increased for the overweight)
oral contraceptives (progestin only mini pill) 5% (increased for the overweight)
patch 5%
shot/injection G1%
vaginal ring 5%
Implantable device (rod, intrauterine device) G1%
Permanent methods (sterilization G1%
implant, surgical sterilization)
Emergency contraception (must be used 1%
within 72 hours)
a
Adapted from Womenshealth.gov. Office on Women’s Health, US Department of Health and Human
Services.27 womenshealth.gov/publications/our-publications/fact-sheet/birth-control-methods.pdf.

and other published data have identi- was at higher doses than used in
fied no apparent risks. In the recent humans. The prospective early preg-
systematic review of 97 cases, no asso- nancy exposure registry, with 362 out-
ciation with lower mean birth weight, comes, has not identified any issues.36
congenital anomaly, preterm delivery, In a systematic review of 35 pregnan-
or spontaneous abortion was found.33 cies, no association with shorter mean
In fact, some neurologists treat preg- birth length, lower mean birth weight,
nant women with glatiramer acetate.35 or lower mean gestational age was
Glatiramer acetate is another large evident.33 Nine women with rapid
macromolecule that is not likely to disease progression were treated with
cross the placental barrier. It should natalizumab during their pregnancy.
be compatible with breast-feeding; Reversible, mild to moderate hemato-
small-scale studies of breast-feeding logic abnormalities were present in
during treatment have reported no eight of the newborns.37 Alpha 4
issues.29,30 integrin receptors, the target of
natalizumab, are expressed on human
Natalizumab uterine epithelium and on embryonic
In guinea pigs, natalizumab was asso- issue. Natalizumab crosses the pla-
ciated with decreased pup survival, centa in the second trimester and is
and in primates it was associated with secreted at low levels in breast milk.29
fetal hematologic disturbances. This Natalizumab takes 3 months to clear

50 www.ContinuumJournal.com February 2014

Copyright © American Academy of Neurology. Unauthorized reproduction of this article is prohibited.


after discontinuation but can be re- not identified any teratogenic con-
moved quickly using a series of plasma cerns.39,40 The teriflunomide clinical
exchanges. development program has likewise
not identified any teratogenicity
Fingolimod signal in humans.41 A pregnancy test is
Fingolimod penetrates the CNS quite required before starting therapy.
well. In animal models, fingolimod is Teriflunomide can persist in the body
associated with fetal malformations for as long as 24 months but can be
(ventricular septal defect, persistent eliminated with a washout protocol
truncus arteriosus, and embryonic/fetal (Table 2-4). Teriflunomide crosses the
deaths). Pregnant rabbits also experi- placenta and is secreted in the breast
enced increased embryonic/fetal milk of rats.
deaths and fetal growth retardation.
Sphingosine-1 phosphate receptors, BG-12/Dimethyl Fumarate
the target of fingolimod, are involved BG-12, an oral formulation of dimethyl
in fetal vascular development. The fumarate, is the most recently ap-
most recent pregnancy exposure data proved oral agent. It has about 25%
from clinical trials and postmarketing penetration of the CNS. In pregnant
showed 59 healthy newborns, 39 in- rats, at very high doses, it is associated
duced and 21 spontaneous abortions, with embryotoxicity and delayed ossi-
five congenital anomalies, and three fication. In male rats, it increased
other pregnancy anomalies (two ec- nonmotile sperm. In mice, rats, and
topic pregnancies and one blighted dogs, it showed testicular toxicity.
ovum).38 Fingolimod takes 2 months Limited pregnancy exposures in 35
to be eliminated after discontinuation; women involved 15 live births, three
there is no way to rapidly clear it. spontaneous abortions, and seven
Fingolimod crosses the placenta and is elective abortions, with results in the
excreted into breast milk in rats. other 10 women pending.42 A preg-
nancy registry is ongoing. Whether
Teriflunomide dimethyl fumarate is excreted in
Teriflunomide does not penetrate the breast milk is currently unknown.
CNS. Teriflunomide in pregnant rats
causes a high rate of fetal malformations Mitoxantrone
(craniofacial, axial, appendicular skele- Mitoxantrone, a chemotherapeutic
tal) and embryofetal death. Preg- anthracenedione, is now hardly ever
nant rabbits show similar findings. used in MS. It can cause amenorrhea
Teriflunomide also gets into semen (at in humans and has been associated with
a mean level of 10.6 2g/mL) and is the fetal growth retardation as well as
only DMT for which consideration must premature delivery. Treatment requires
be made for men who are on the a pregnancy test before each dosing. It
therapy. However, the potential dose is typically given at 3-month intervals
for a female partner would be 100 times with a lifetime cap at 11 doses (cumu-
lower than from taking the medication lative lifetime dose of 140 mg/m2)
itself. Teriflunomide is the active because of cardiotoxicity. Pregnancy
metabolite of leflunomide, an oral should not be attempted for 6 months
agent that has been available to after discontinuing therapy. Placental
treat rheumatoid arthritis for several passage of the drug is little, but it is
years. The limited experience with secreted in breast milk at significant
leflunomide pregnancy exposure has concentrations of 18 ng/mL.26
Continuum (Minneap Minn) 2014;20(1):42–59 www.ContinuumJournal.com 51

Copyright © American Academy of Neurology. Unauthorized reproduction of this article is prohibited.


Multiple Sclerosis in Pregnancy

KEY POINTS
h Steroids are used fairly TABLE 2-4 Teriflunomide Washout Protocol
broadly in obstetrics and
are probably safe in b Oral cholestyramine 8 g every 8 hours for 11 days; can reduce to 4 g every 8 hours if
the second and third not tolerated
trimesters. b Alternative regimen is oral activated charcoal 50 g every 12 hours for 11 days
h The fact that a woman b Ultimate goal is to achieve teriflunomide plasma level G0.02 mg/L
has multiple sclerosis b This may take less than 11 days; washout days do not have to be consecutive
should have little to no
b Important to supplement folate during washout in pregnant women
impact on delivery
issues.
h Multiple sclerosis should
not affect choice of ACUTE RELAPSES Contrast is not recommended because
anesthetic. The standard symptomatic treatment gadolinium compounds cross the pla-
to speed up recovery from an acute centa and enter the fetal bloodstream. It
attack is a self-limited course of high- should be given only with extreme
dose glucocorticoids. Typically, the caution. See ‘‘Neuroradiology in Women
equivalent of 1 g of methylpredniso- of Childbearing Age’’ by Drs Riley Bove
lone is given daily for 3 to 7 days, and Joshua Klein in this issue of
depending on the severity of the for more on this topic.
attack. Because of excellent bioequiv-
alence, it probably does not matter SYMPTOMATIC THERAPIES
whether it is given intravenously or As a general principle, symptomatic
orally. No oral taper is used. In therapies are minimized in pregnant
general, it is best not to treat during patients. Only those that are absolutely
the first trimester. Corticosteroids necessary to contain problematic issues
cross the placental barrier and may should be used, at the minimum effec-
increase risk for cleft palate and lower tive dose. Alternative nonpharmacologic
birth weight. However, steroids are strategies are encouraged. It is estimated
used fairly broadly in obstetrics and that over 85% of women use some medi-
are probably safe in the second and cation during pregnancy. If a medication
third trimesters. As noted in Case 2-2, is needed to prevent suffering or to
a clinician can certainly consider maintain quality of life, then it should be
treating pregnant MS patients who used. However, the least risky option
experience a relapse. A rationale exists based on pregnancy rating is preferred.
for using prednisone, prednisolone, or
methylprednisolone, which are metab- DELIVERY ISSUES
olized to inactive forms by the placenta, Neither prematurity nor birth weight
rather than betamethasone or dexa- has any impact on the infant’s risk to
methasone, which cross the placenta develop MS. Likewise, the fact that a
with minimal metabolism.29 Case 2-2 woman has MS should have little to no
highlights that patients who relapse impact on delivery issues. The only
during pregnancy are at increased risk exception is the small proportion of
of relapse postpartum. A clear plan patients with severe, advanced disease
should be outlined to handle the that limits their mobility or ability to
postpartum period. push during labor or that creates
No fetal effects of MRI during respiratory problems. It would be the
pregnancy have been documented. extraordinary pregnant patient with MS
Guidelines are precautionary, however, who fits into this group. MS should not
and recommend MRI only if essential. affect choice of anesthetic. Epidural
52 www.ContinuumJournal.com February 2014

Copyright © American Academy of Neurology. Unauthorized reproduction of this article is prohibited.


KEY POINT

Case 2-2 h Conflicting data indicate


either no effect from
A 32-year-old woman presented 23 weeks into her first pregnancy with a
breast-feeding or a
3-day history of numbness and paresthesia that began in her feet, then
positive benefit in
ascended to her lower abdomen. She denied fever or other systemic
reducing multiple
symptoms. She had been diagnosed with relapsing multiple sclerosis (MS)
sclerosis activity.
5 years ago and started on glatiramer acetate; she stopped therapy shortly
before trying to become pregnant. Neurologic examination found a T11
sensory level to pin, with blunted vibration in her toes. Heel-to-shin
was minimally clumsy. She also showed a mild paraparesis and poor
tandem gait.
Comment. This patient with relapsing MS presented with new neurologic
symptoms and findings during her second trimester. The differential was MS
relapse, pseudorelapse, or an unrelated non-MS issue. No evidence for a
pseudorelapse (no fever, infection, or metabolic abnormality) existed. The
most likely explanation was a new MS attack involving the spinal cord.
Although unusual during pregnancy, relapses certainly can occur.
The first issue that came up for her neurologist was whether to perform
MRI to confirm new lesions and contrast lesion activity. It was decided that
this was not necessary, and would be done after delivery to determine
whether a new spinal cord lesion was present. The second issue was
whether to treat this relapse. The decision was made to give a short course
(3 days) of IV methylprednisolone at 1 g/d. No oral taper was used, because
the proven benefit of steroids for MS relapses are only for high-dose
exposure. In addition, such short courses do not cause adrenal suppression.
The decision to treat was based in part on the fact that the patient was
very bothered by her attack and wanted to get over it as quickly as possible.
The decision to treat was also based on the presence of both motor and
coordination involvement, which are considered a more severe attack with
poorer prognosis. She was also counseled that she was at increased risk to have
a postpartum relapse in the 3 months after delivery and that a plan of action
for resuming a disease-modifying therapy would therefore need to be made.

injections, even spinal anesthesia, are has suggested that breast-feeding


not contraindicated. MS should almost worsens MS. Conflicting data indicate
never determine whether a vaginal either no effect from breast-feeding or
delivery or cesarean delivery is chosen; a positive benefit in reducing MS
this should be based on obstetric activity. In a prospective Kaiser
factors. In a recent Italian study, nei- Permanente study of 32 pregnant
ther epidural anesthesia nor cesarean women with MS and 29 matched
delivery had any impact on MS post- pregnant controls, 14 (48%) exclusively
partum relapse or disability rate.43 breast-fed for at least 2 months.44 A
postpartum relapse occurred in 36%
POSTPARTUM ISSUES of these patients versus 87% of those
Breast-feeding who did not exclusively breast-feed.
Case 2-3 outlines the first postpartum Women with MS who breast-fed were
decision a pregnant MS patient faces: fivefold less likely to relapse in the
whether she is going to breast-feed or first year postpartum. The authors
not. Breast-feeding means she will suggested that breast-feeding caused
probably not start a DMT. No study a prolonged lactational amenorrhea
Continuum (Minneap Minn) 2014;20(1):42–59 www.ContinuumJournal.com 53

Copyright © American Academy of Neurology. Unauthorized reproduction of this article is prohibited.


Multiple Sclerosis in Pregnancy

KEY POINTS
h Patients with multiple
sclerosis should be told
Case 2-3
A 28-year-old woman with relapsing multiple sclerosis (MS) was seen in her
there may well be a
fifth month of pregnancy. She wanted to discuss the pros and cons of
benefit in suppressing
breast-feeding versus going back on her MS disease-modifying therapy
postpartum disease
(DMT) after delivery. She wanted to know whether she could breast-feed
activity by
and still be on a therapy. She had read that IV immunoglobulin (IVIg) had
breast-feeding, but it is
been used to treat women with MS postpartum and wanted to know if
not certain.
that was an option for her.
h It is probably best to try Comment. Both the US Food and Drug Administration (FDA) and the
exclusive breast-feeding. National MS Society have said that MS patients who are breast-feeding
h Multiple sclerosis should not be on therapy. The available data on breast-feeding have never
disease severity and shown a negative effect on MS, but how significant a benefit it has on
prognosis should be a suppressing disease activity is unclear. Data are conflicting. Therefore, MS
factor in deciding how disease severity and prognosis should be a factor in deciding how rapidly
rapidly to institute to institute therapy postpartum. IVIg has been used as monotherapy in
therapy postpartum. relatively small-scale studies to treat postpartum MS with reported benefit.
IVIg is not FDA-approved for MS. It is a therapy that can be used during
pregnancy or while breast-feeding and theoretically could be added on to
a primary DMT. However, no studies have been performed to examine
such a combination. This might be an appealing option, at least for
high-risk patients in the 3- to 6-month postpartum period, when risk for
postpartum disease activity is at a maximum. Some neurologists have felt
comfortable using in particular glatiramer acetate during pregnancy and
breast-feeding because of its category B pregnancy rating, but this is not
common practice.

with ovarian suppression, high prolac- pitfalls in these studies.47 A recent


tin, and low nonpulsatile luteinizing review tried to reconcile conflicting
hormone levels that might be benefi- studies by suggesting that only exclusive
cial. A larger German database includ- breast-feeding (for at least 2 months)
ed 170 women with MS who breast- was protective. The authors commented
fed exclusively, and they did show a that if trials did not sort out exclusive
lower postpartum relapse rate.45 How- breast-feeding, they would not see a
ever, the larger, pivotal PRIMS trial benefit.48 The bottom line is that MS
showed no benefit from breast- patients should be told there may well
feeding.11 A recent Italian study eval- be a benefit in suppressing postpartum
uated 302 full-term deliveries in disease activity by breast-feeding, but it
298 women with MS followed for at is not certain. Therefore, predictors of
least 1 year postpartum.46 Postpartum postpartum relapses, disease progno-
relapse rate was not affected by sis, and current clinical status should all
breast-feeding. In this study, only factor into the decision on whether to
higher relapse rate in the year before delay therapy to breast-feed. It is
pregnancy predicted postpartum re- probably best to try exclusive breast-
lapse. A meta-analysis of 869 women feeding. MS patients who are breast-
with MS who breast-fed versus 689 feeding and undergo contrast MRI
who did not found that those who did have been advised to suspend breast-
not breast-feed were almost twice as feeding for 24 hours. However, it is
likely to relapse, but the authors also probably safe, and patients may choose
emphasized all of the limitations and to resume it immediately.29 For more

54 www.ContinuumJournal.com February 2014

Copyright © American Academy of Neurology. Unauthorized reproduction of this article is prohibited.


KEY POINTS
on this topic, refer to ‘‘Neuroradiology ated, but there is no theoretical con- h Breast-feeding while
in Women of Childbearing Age’’ by traindication to such a combination. taking interferon " or
Drs Riley Bove and Joshua Klein in this glatiramer acetate is
issue of . IN VITRO FERTILIZATION/ likely to be safe.
With regard to breast-feeding, most ASSISTED REPRODUCTIVE
h Use of
patients will not be taking a DMT, TECHNOLOGY
gonadotropin-releasing
since that is the formal recommenda- MS has no effect on fertility. Of course, hormone agonists
tion. Breast-feeding while taking inter- occasional patients will face difficulty during the in vitro
feron " or glatiramer acetate is likely getting pregnant, just as is true for fertilization/assisted
to be safe, and a minority of physi- women without MS. Infertility is de- reproductive technology
cians use glatiramer acetate in partic- fined as the inability to get pregnant procedure puts women
ular in this situation. NonYFDA- after trying for a year (or 6 months, for with multiple sclerosis at
approved therapies that can be used those 35 years and older). About 10% risk for relapses and
in a breast-feeding patient include of women are estimated to have diffi- new MRI lesion activity.
monthly pulse steroids or IV immuno- culty getting pregnant. In certain cases,
globulin (IVIg), which has even been assisted reproductive technology
pulsed during pregnancy. (ART) can be used. This is a group of
For women receiving high gluco- different methods that involve remov-
corticoid doses, a delay in breast- ing eggs, mixing them with sperm to
feeding for 24 to 48 hours is the create an embryo, and then implanting
conservative approach. A breast pump the embryo back into the woman. The
can be used to remove milk during success rate is as high as 39% for
the delay period, and saved breast patients under 35 years of age but
milk can be used for this short period. drops off markedly for older women.
In vitro fertilization (IVF) is the most
Therapy effective form of ART and can be used if
The second major decision the post- the fallopian tubes are blocked or
partum patient with MS faces is when sperm count is deficient. IVF/ART strat-
to start or resume a DMT. The DMTs egies involve hormonal therapy to
can be started virtually immediately cause multiple eggs to be produced.
postpartum, as long as the patient is Two recent studies highlight that
not breast-feeding. When patients use of gonadotropin-releasing hor-
choose to breast-feed, they typically mone (GnRH) agonists during the
forego use of a DMT. IVF/ART procedure puts women with
For patients who choose not to MS at risk for relapses and new MRI
breast-feed but to begin therapy with lesion activity in the 3 months after the
a DMT, it takes at least 2 months to fertility treatments, particularly if they
achieve meaningful efficacy. Some do not become pregnant. In a 1-year
physicians have used combination review of a consortium of French
therapy during this period, adding centers, 32 women with MS underwent
monthly pulse IVIg to a DMT. This is 70 IVF/ART treatments.50 Forty-eight
based on the Gammaglobulin Post of these treatments used GnRH ago-
Partum (GAMPP) trial data, which nists, and 19 used GnRH antagonists.
reported benefit of either 60 g or 10 The annualized relapse rate (ARR) in
g IVIg shortly after delivery, followed the 3 months before and after treat-
by 10 g monthly for an additional five ment doubled from 0.80 to 1.60. The
doses.49 In this trial, IVIg was used as ARR in the year before therapy was
monotherapy. Combination with a 0.68. Posttreatment relapses were as-
DMT has never been rigorously evalu- sociated with agonist as opposed to
Continuum (Minneap Minn) 2014;20(1):42–59 www.ContinuumJournal.com 55

Copyright © American Academy of Neurology. Unauthorized reproduction of this article is prohibited.


Multiple Sclerosis in Pregnancy

antagonist use and failure to become 8 mg/d) in 12 women with MS took


pregnant. A second prospective Ar- place; the relapsing cohort but not the
gentine study focused on 16 women secondary progressive cohort showed
who received 26 IVF/ART cycles that clinical and MRI benefit.52 This has led
involved GnRH agonists.51 Post cycle, to a current phase II trial of combina-
women with MS showed a sevenfold tion glatiramer acetate plus estriol or
increase in relapses and a ninefold placebo in about 130 women with
increase in new brain MRI lesion relapsing MS; the trial has not yet
activity. This was associated with im- reported data.
munologic changes that might be Postpartum Progestin and Estriol in
expected to enhance autoimmune Multiple Sclerosis (POPARTMUS) is a
disease. Although the number of pa- French and Italian multicenter placebo-
tients in these two studies is low, the controlled trial that evaluated a daily oral
pattern is consistent enough to coun- progesterone (10 mg/d) plus weekly
sel patients about this risk. percutaneous estradiol (100 2g) versus
placebo, given for 3 months postpartum.
IMPLICATIONS FOR THERAPY Patients were followed for 6 months
There is no doubt that pregnancy postpartum. A total of 202 pregnant
suppresses MS disease activity in the women with MS entered the study.
last trimester, followed by a self-limited Unfortunately, postpartum hormones
postpartum rebound in activity. This did not reduce either clinical or MRI
has led to interest in determining the disease activity compared to placebo.53
responsible pregnancy factors, be- Although this trial is a blow to sex
cause doing so could suggest novel hormone therapy for MS, it did use a
treatment strategies. The most likely relatively low dose of estrogen along
pregnancy factors would seem to be with progesterone.
sex hormones, especially estrogen and In summary, the role of sex hormones
progesterone. These hormones have to treat MS remains unclear. If benefit
immunologic and neurologic effects in could be proven, then development of a
addition to their endocrinologic ef- designer hormone that provides the
fects. Several years ago, a small proof- important immune or neurologic effects
of-principle study of oral estriol (at without the undesirable endocrinologic

TABLE 2-5 Future Needs to Clarify Multiple Sclerosis and Pregnancy


Interactions

b Find out why multiple sclerosis (MS) is increasing in women


b Identify the pregnancy factors that suppress MS disease activity
b Resolve whether pregnancy results in a poorer prognosis for progressive MS
b Determine whether treating low vitamin D levels in women can prevent MS
b Clarify the value of breast-feeding and whether it is comparable to disease-modifying
therapy use
b Identify which patients will experience postpartum relapses
b Determine whether there are safe therapies to use during pregnancy and in
postpartum breast-feeding patients
b Determine why gonadotropin-releasing hormone agonists might worsen MS
disease activity

56 www.ContinuumJournal.com February 2014

Copyright © American Academy of Neurology. Unauthorized reproduction of this article is prohibited.


effects would be the preferred thera- diagnosis of multiple sclerosis (MS).
Mult Scler 2013;19(3):351Y358.
peutic product.
8. O’Gorman C, Lin R, Stankovich J, Broadley
SA. Modelling genetic susceptibility to
FUTURE NEEDS multiple sclerosis with family data.
A whole series of interesting un- Neuroepidemiology 2012;40(1):1Y12.
resolved issues come up when dis- 9. Jobin C, Larochelle C, Parpal H, et al. Gender
cussing MS and pregnancy (Table 2-5). issues in multiple sclerosis: an update.
Womens Health (Lond Engl) 2010;6(6):
Solving these issues would have direct 809Y832.
impact on the understanding and treat-
10. McCombe PA, Greer JM. Female reproductive
ment of MS and could lead to major issues in multiple sclerosis. Mult Scler 2013;
breakthroughs in this disease. How- 19(4):392Y402.
ever, important practical needs also 11. Confavreux C, Hutchinson M, Hours MM,
exist. Every MS patient deserves et al. Rate of pregnancy-related relapse in
to know that pregnancy probably im- multiple sclerosis. N Engl J Med 1998;339(5):
285Y291.
proves the long-term prognosis
for relapsing MS, that the risk of 12. Paavilainen T, Kurki T, Farkkila M, et al.
Lower brain diffusivity in postpartum period
passing MS on to children is very compared to late pregnancy: results from a
small, and that MS has little or no prospective imaging study of multiple
impact on fertility and pregnancy out- sclerosis patients. Neuroradiology 2012;54
(8):823Y828.
come. Hopefully, all physicians who
take care of patients with MS will make 13. Ponsonby AL, Lucas RM, van der Mei IA,
et al. Offspring number, pregnancy, and risk
sure they are familiar with the current of a first clinical demyelinating event. The
approach to these pregnancy-related AusImmune Study. Neurology 2012;78(12):
issues. 867Y874.
14. Lebrun C, Le Page E, Kantarci O, et al.
REFERENCES Impact of pregnancy on conversion to
clinically isolated syndrome in a radiologically
1. van der Kop ML, Pearce MS, Dahlgren L,
isolated syndrome cohort. Mult Scler 2012;18
et al. Neonatal and delivery outcomes in
(9):1297Y1302.
women with multiple sclerosis. Ann Neurol
2011;70(1):41Y50. 15. Torkildsen O, Grytten N, Aarseth J, et al.
Month of birth as a risk factor for multiple
2. Patas K, Broder Engler J, Friese MA,
sclerosis: an update. Acta Neurol Scand
Gold SM. Pregnancy and multiple sclerosis:
Suppl 2012;(195):58Y62.
feto-maternal immune cross talk and its
implications for disease activity. J Reprod 16. Dobson R, Giovannoni G, Ramagopalan S.
Immunol 2013;97(1):140Y146. The month of birth effect in multiple
3. Zenclussen AC. Adaptive immune responses sclerosis: systematic review, meta-analysis
during pregnancy. Am J Reprod Immunol and effect of latitude. J Neurol Neurosurg
2013;69(4):291Y303. Psychiatry 2013;84(4):427Y432.

4. Coyle PK. Pregnancy and multiple sclerosis. 17. Disanto G, Watson CT, Meier UC, et al.
Neurol Clin 2012;30(3):877Y888. Month of birth and thymic output. JAMA
Neurol 2013;70(4):527Y528.
5. Finkelsztejn A, Brooks JB, Paschoal FM Jr,
18. Mirzaei F, Michels KB, Munger K, et al.
Fragoso YD. What can we really tell
Gestational vitamin D and the risk of multiple
women with multiple sclerosis regarding
sclerosis in offspring. Ann Neurol 2011;70(1):
pregnancy? A systematic review and
30Y40.
meta-analysis of the literature. BJOG
2011;118(7):790Y797. 19. Cutter G, Salter A. Got milk? Ann Neurol
2011;70(1):3Y4.
6. Polman CH, Reingold SC, Banwell B, et al.
Diagnostic criteria for multiple sclerosis: 20. Salzer J, Hallmans G, Nystrom M, et al.
2010 revisions to the McDonald criteria. Vitamin D as a protective factor in multiple
Ann Neurol 2011;69(2):292Y302. sclerosis. Neurology 2012;79(21):2140Y2145.
7. Alwan S, Yee IM, Dybalski M, et al. 21. Verdru P, Theys P, D’Hooghe MB, et al.
Reproductive decision making after the Pregnancy and multiple sclerosis: the

Continuum (Minneap Minn) 2014;20(1):42–59 www.ContinuumJournal.com 57

Copyright © American Academy of Neurology. Unauthorized reproduction of this article is prohibited.


Multiple Sclerosis in Pregnancy

influence on long term disability. Clin Neurol pregnancy. Neurology 2012;79(11):


Neurosurg 1994;96(1):38Y41. 1130Y1135.
22. Runmarker B, Andersen O. Pregnancy is 34. Hale TW, Siddiqui AA, Baker TE. Transfer of
associated with a lower risk of onset and a interferon "1a into human breast milk.
better prognosis in multiple sclerosis. Brain Breastfeed Med 2012;7(2):123Y125.
1995;118(pt 1):253Y261.
35. Fragoso YD, Finkelsztejn A, Kaimen-Maciel
23. D’hooghe MB, Haentjens P, Nagels G, et al. DR, et al. Long-term use of glatiramer
Menarche, oral contraceptives, pregnancy acetate by 11 pregnant women with
and progression of disability in relapsing multiple sclerosis: a retrospective, multicentre
onset and progressive onset multiple case series. CNS Drugs 2010;24(11):969Y976.
sclerosis. J Neurol 2012;259(5):855Y861.
36. Cristiano L, Friend S, Bozic C, et al. Evaluation
24. Ramagopalan S, Yee I, Byrnes J, et al. Term of pregnancy outcomes from the TYSABRI
pregnancies and the clinical characteristics (natalizumab) pregnancy exposure registry.
of multiple sclerosis: a population based Presented at: 65th Annual Meeting of the
study. J Neurol Neurosurg Psychiatry American Academy of Neurology; March
2012;83(8):793Y795. 2013; San Diego, CA.

25. Teter B, Kavak KS, Kolb C, et al. Parity 37. Haghikia A, Bochum G, Rolfes E, et al.
associated with long-term disease progression Natalizumab in active MS during pregnancy:
in women with multiple sclerosis. Presented efficacy, safety and the consequences for
at: 65th Annual Meeting of the American fetal hematopoiesis. Presented at: 65th
Academy of Neurology; March 2013; Annual Meeting of the American Academy
San Diego, CA. of Neurology; March 2013; San Diego, CA.

26. Lu E, Wang BW, Guimond C, et al. Safety 38. Geissbuhler Y, Butzkueven H, Hernandez-
of disease-modifying drugs for multiple Diaz S, et al. Pregnancy outcomes from
sclerosis in pregnancy: current challenges fingolimod clinical trials and post-marketing
and future considerations for effective experience and the need for a multinational
pharmacovigliance. Expert Rev Neurother Gilenya (fingolimod) pregnancy exposure
2013;13(3):251Y260. registry in multiple sclerosis. Presented at:
28th Annual Meeting of the European
27. Adapted from Womenshealth.gov. Office on Committee for Treatment and Research in
Women’s Health, US Department of Health Multiple Sclerosis.
and Human Services. Birth Control Methods.
womenshealth.gov/publications/our- 39. Cassina M, Johnson DL, Robinson K, et al.
publications/fact-sheet/birth-control- Pregnancy outcome in women exposed to
methods.pdf. Accessed November 26, 2013. leflunomide before or during pregnancy.
Arthritis Rheum 2012;64(7):2085Y2094.
28. Gnoth C, Godehardt D, Godehardt E, et al.
Time to pregnancy: results of the German 40. Chambers CD, Johnson DL, Robinson LK,
prospective study and impact on the et al. Birth outcomes in women who have
management of infertility. Hum Reprod taken leflunomide during pregnancy.
2003;18(9):1959Y1966. Arthritis Rheum 2010;62(5):1494Y1503.

29. Cree BA. Update on reproductive safety of 41. Henson LJ, Stuve O, Kieseler B, et al.
current and emerging disease-modifying Pregnancy outcomes from the
therapies for multiple sclerosis. Mult Scler Teriflunomide clinical development
2013;19(7):835Y843. program: retrospective analysis of the
Teriflunomide clinical trial database.
30. Houtchens MK, Kolb CM. Multiple sclerosis Presented at: 65th Annual Meeting of the
and pregnancy: therapeutic considerations.
American Academy of Neurology; March
J Neurol 2013;260(5):1202Y1214.
2013; San Diego, CA.
31. Tomczyk S, Sperling B. Pregnancy outcomes
42. Gold R, Bochum G, Phillips T, et al. BG-12
in patients exposed to intramuscular
(dimethyl fumarate) and pregnancy:
interferon beta-1a (IM IFN"-1a) (S30.006).
preclinical and clinical data from the clinical
AAN 2013 Annual Meeting.
development program. Presented at:
32. Sandberg-Wollheim M, Alteri E, Moraga MS, 65th Annual Meeting of the American
Kornmann G. Pregnancy outcomes in Academy of Neurology; March 2013;
multiple sclerosis following subcutaneous San Diego, CA.
interferon beta-1a therapy. Mult Scler
43. Pasto L, Portaccio E, Ghezzi A, et al. Epidural
2011;17(4):423Y430.
analgesia and cesarean delivery in multiple
33. Lu E, Wang BW, Guimond C, et al. Disease- sclerosis post-partum relapses: the Italian
modifying drugs for multiple sclerosis in cohort study. BMC Neurol 2012;12:165.

58 www.ContinuumJournal.com February 2014

Copyright © American Academy of Neurology. Unauthorized reproduction of this article is prohibited.


44. Langer-Gould A, Huang SM, Gupta R, et al. 49. Haas J, Hommes OR. A dose comparison
Exclusive breastfeeding and the risk of study of IVIG in postpartum
postpartum relapses in women with relapsing-remitting multiple sclerosis. Mult
multiple sclerosis. Arch Neurol 2009;66(8): Scler 2007;13(7):900Y908.
958Y963.
50. Michel L, Foucher Y, Vukusic S, et al.
45. Hellwig K, Haghikia A, Rockhoff M, Gold R. Increased risk of multiple sclerosis relapse
Multiple sclerosis and pregnancy: experience after in vitro fertilization. J Neurol
from a nationwide database in Germany. Neurosurg Psychiatry 2012;83(8):796Y802.
Ther Adv Neurol Disord 2012;5(5):
51. Correale J, Farez MF, Ysrraelit MC, et al.
247Y253.
Increase in multiple sclerosis activity after
46. Portaccio E, Ghezzi A, Hakiki B, et al. assisted reproduction technology. Ann
Breastfeeding is not related to postpartum Neurol 2012;72(5):682Y694.
relapse in multiple sclerosis. Neurology
52. Sicotte NL, Liva SM, Klutch R, et al. Treatment
2011;77(2):145Y150.
of multiple sclerosis with the pregnancy hormone
47. Pakpoor J, Disanto G, Lacey MV, et al. estriol. Ann Neurol 2002;52(4):421Y428.
Breastfeeding and multiple sclerosis
53. Vukusic S, El-Etr M, Ionescu I, et al. The
relapses: a meta-analysis. J Neurol 2012;
POPARTMUS French-Italian multicentric
259(10):2246Y2248.
trial of post partum progestin and estriol
48. Langer-Gould A, Beaber BE. Effects of in multiple sclerosis; final results. Presented
pregnancy and breastfeeding on the at: 28th Annual Meeting of the European
multiple sclerosis disease course. Clin Immunol Committee for Treatment and Research in
2013.doi:10.1016/j.clim.2013.01.008. Multiple Sclerosis.

Continuum (Minneap Minn) 2014;20(1):42–59 www.ContinuumJournal.com 59

Copyright © American Academy of Neurology. Unauthorized reproduction of this article is prohibited.


Review Article

Pregnancy and Epilepsy


Address correspondence to
Dr Cynthia L. Harden, North
Shore-Long Island Jewish
Health System, 611 Northern
Boulevard, Suite 150, Great Cynthia L. Harden, MD
Neck, NY 11021,
charden@nshs.edu.
Relationship Disclosure:
Dr Harden has received ABSTRACT
personal compensation Purpose of Review: This review encompasses issues regarding the management
for activities with
GlaxoSmithKline; Lundbeck; of women with epilepsy and pregnancy, including preconception planning,
UCB SA; and Upsher-Smith antiepileptic drug (AED) effects on the exposed offspring, and consequences of
Laboratories, Inc. Dr Harden seizures during pregnancy, with an emphasis on counseling points and risk
has served in an editorial
capacity for UpToDate and management.
received research support Recent Findings: In utero valproate exposure continues to show the highest risk of
from the Epilepsy Therapy congenital malformations and of adverse cognitive outcomes, including autism,
Project.
Unlabeled Use of
compared to other AEDs. In utero topiramate exposure is associated with facial
Products/Investigational clefts. In utero exposure to lamotrigine, carbamazepine, phenytoin, and leveti-
Use Disclosure: racetam has been evaluated in large numbers of offspring, and all of these AEDs
Dr Harden reports no
disclosure.
have a low risk of major congenital malformations, near 2.5%. The risk of major
* 2014, American Academy congenital malformations due to in utero AED exposure is dose associated. Folic
of Neurology. acid taken at the time of conception decreases the risk of adverse cognitive
outcomes. AED polytherapy may adversely affect fertility, and in utero exposure of
AED polytherapy is associated with infants who are small for their gestational age.
Seizure freedom before pregnancy is a good predictor of remaining seizure free
during pregnancy.
Summary: Counseling points are complex for managing women with epilepsy
contemplating pregnancy, including evaluation of AED choice and dose. The
physician must have knowledge of the issues to enable an honest discussion and
appropriate decision making in partnership with the patient.

Continuum (Minneap Minn) 2014;20(1):60–79.

INTRODUCTION appropriate AED choices are based on


The majority of people with epilepsy risk to the offspring and expectations
experience prolonged seizure remis- of seizure control. Because about half
sion, usually while taking antiepileptic of pregnancies are unplanned, this
drugs (AEDs). It follows, then, that discussion should be initiated by the
many women with epilepsy who in clinician and touched upon at every
general feel little affected by their office visit with women with epilepsy
illness on a day-to-day basis have a of childbearing potential, so that the
similar outlook toward family planning patient’s therapy is optimized for
as do women without chronic illness, pregnancy during childbearing years.
and therefore hope to bear children. The decision about bearing children
The medical community must provide is very personal, emotional, and funda-
informed guidance for these patients. mental to family life. The multiple risks
In the conversation with the patient, to women with epilepsy around preg-
discussion points should address (1) nancy vary in their severity and timing,
whether it is reasonable and safe to as well as in whether the patient or the
become pregnant based on the in- baby is at greater risk. This review will
dividual’s epilepsy severity and cur- discuss risks to women with epilepsy
rent treatment and (2) what the most and their offspring during pregnancy
60 www.ContinuumJournal.com February 2014

Copyright © American Academy of Neurology. Unauthorized reproduction of this article is prohibited.


KEY POINTS
and delivery and conclude with a used, with taking three or more AEDs h Low birth rates are not
summary of these considerations. imparting a 20-fold risk of not conceiv- the same as infertility.
ing. However, this data set also implies
INFERTILITY IN EPILEPSY h Psychosocial factors play
an important nonbiological impact on
a role in lower birth
Birth rates to women with epilepsy birth rates as well, in that less than 10 rates to women with
have been estimated using large years of education was associated with a epilepsy.
population-based registries. However, doubled risk of not conceiving.2
h Medical factors
birth rates do not directly assess Psychosocial factors certainly contrib-
contributing to lower
infertility, which is defined as lack of ute to these outcomes, but reasonable birth rates to women
conception after 1 year of frequent biological hypotheses exist for repro- with epilepsy include
sexual activity without the use of birth ductive endocrine disruption in women severe neurologic
control. Birth rate statistics cannot be with epilepsy. For example, AEDs alter disability and
directly translated to fertility since the endogenous hormonal profile and antiepileptic drug
they do not account for the many thereby affect menstrual cycle length polytherapy.
patients and couples who simply and ovulation rates. Additionally, sei-
choose not to have children because zures and epilepsy themselves may pro-
of the risks of epilepsy and AEDs. For duce a central reproductive hormonal
example, in a study using national dysregulation and cause pituitary-
insurance data in Finland, the birth mediated anovulation.3
rates to women with epilepsy were
17% lower than the general popula- TAKING ANTIEPILEPTIC
tion of women.1 However, approxi- DRUGS DURING PREGNANCY:
mately 15% to 18% of women with A COMPLEX DECISION
epilepsy in this population had severe Ideally, women should not take med-
comorbidity and therefore were un- ications during pregnancy. Epilepsy,
likely to choose to bear children; this however, is a rather unforgiving illness
could, in great part, account for the dif- in that physically dangerous seizures
ference between groups.1 The Kerala can occur without provocation for
Registry of Epilepsy and Pregnancy many people with epilepsy, and AEDs
from southern India provides some decrease the chance of seizure occur-
nuance to the question of fertility in rence. This seizure risk, however low
epilepsy. The investigators evaluated in many patients with epilepsy, be-
factors associated with not achieving comes of even greater concern when
pregnancy in their cohort of 375 wom- the patient is a pregnant woman with
en with epilepsy prospectively followed epilepsy. Hence, the physician faces
as they actively tried to conceive.2 The the dilemma of balancing the risks of
follow-up period was at least 1 year, seizures to the mother and develop-
with a mean of 3 years. While 61% did ing fetus versus the risks of in utero
become pregnant, an important factor AED exposure to the developing fetus.
associated with not conceiving was The mother is the patient and argu-
using more than one AED (P=.001; ably the physician’s primary responsi-
odds ratio [OR] 1.33, 95% confidence bility in this risk assessment. She, on
interval [CI] 1.11 to 1.60). While taking the other hand, may feel a greater
one AED did not increase the risk of responsibility to the fetal well-being
not conceiving compared to the small than to herself. It is imperative for
group of women with epilepsy not maintaining a therapeutic alliance
taking AEDs (n = 14), a significant that her concerns and opinions be
trend was found for an increase in risk addressed and that she participate in
correlating with the number of AEDs this often emotional discussion. An
Continuum (Minneap Minn) 2014;20(1):60–79 www.ContinuumJournal.com 61

Copyright © American Academy of Neurology. Unauthorized reproduction of this article is prohibited.


Pregnancy and Epilepsy

KEY POINTS
h The assessment of informed, acceptable consensus must exists between AED in utero exposure
structural teratogenesis be reached that incorporates medically and adverse outcomes in the offspring.
is much different than informed decisions regarding taking The information in these documents
the assessment for AEDs altogether and decision points for appears to have stood the test of time
cognitive teratogenesis. increasing AED doses based on clinical thus far, but major additions to the field
h Structural teratogenesis factors and blood levels during pregnan- have occurred since then that can help
is due to first trimester cy. Since many women with epilepsy will guide management. Important factors
exposure, while be taking AEDs during pregnancy, a used in the practice parameters for
cognitive teratogenesis discussion of the risks associated with determining unbiased studies for AED-
is a likely risk with in utero AED exposure follows. associated teratogenesis are put forth
exposure throughout in Table 3-1.5 Key considerations
pregnancy. RECENT INFORMATION are that major congenital malformations
REGARDING RISKS OF MAJOR were considered objective outcomes,
CONGENITAL MALFORMATIONS and studies were therefore not required
IN ANTIEPILEPTIC DRUG USE to have investigators be blinded to the
Contribution of the American AED exposure in order to be considered
Academy of Neurology strong, high-level studies. Major congen-
Evidence-Based Practice ital malformations are defined as life-
Parameters threatening malformations or those that
The American Academy of Neurology need surgical treatment. This was not
(AAN) evidence-based practice param- the case for the more subtle outcome of
eters on management of women with cognitive impairment, in which the high-
epilepsy regarding pregnancy syn- level studies required the investigators
thesized the available evidence at evaluating the children to be blinded to
the time of publication in 2009.4Y6 the specific AED exposure. Further,
The methodology involves stratifying the maternal IQ must have been accounted
risk of bias in the studies to find for in the cognitive assessment of the
the strongest scientific evidence to offspring, since this factor is highly
determine whether a causal relationship associated with the child’s IQ.

TABLE 3-1 Criteria for Unbiased Studies Used in 2009 Women With
Epilepsy Practice Guidelinesa

b Studies used for structural teratogenesis results were based on first


trimester exposure
Population representative
Well-described outcome measures; no masking needed for clear
outcomes such as spina bifida, facial clefts, and limb deformities
Women with epilepsy not on antiepileptic drugs used as a control group, not healthy
controls to account for the potential effect of epilepsy itself on increasing risk
b Studies used for cognitive outcomes based on up to 9 months of exposure to
antiepileptic drugs
Control for maternal IQs
Evaluated children at least 2 years old
Outcomes were measured by investigators masked to treatments
a
Reprinted with permission from Harden C, et al, Neurology.5 B 2009, American Academy of Neurology.
www.neurology.org/content/73/2/133.long.

62 www.ContinuumJournal.com February 2014

Copyright © American Academy of Neurology. Unauthorized reproduction of this article is prohibited.


KEY POINTS
The practical essence of the AAN pared to other AEDs is approximately h Dose-related risks are
practice parameters regarding AED tera- threefold, and the absolute risk is 6% likely to be present with
togenesis are summarized in Table 3-2. to 9% of pregnancies exposed. The most antiepileptic drug
The absolute rate of teratogenesis high risk of major congenital intrauterine exposure.
with valproate is 6% to 9%, which is malformations with valproate has h The most clearly
approximately threefold the risk of been recognized since 19848 and is demonstrated
untreated women with epilepsy and confirmed in several large pregnancy dose-related risk is a
those treated with lamotrigine or registries.9Y11 A dose-associated risk marked increase in risk
carbamazepine. Spina bifida is specif- clearly exists, with the risk below 700 with maternal use of
ically associated with valproate use. A mg of valproate per day being around valproate at more than
small but increased risk exists with 4%, similar to the rate of lamotrigine 700 mg/d.
polytherapy versus monotherapy ex- at more than 300 mg/d and carbamaz-
posure; this risk is increased when epine at more than 400 mg/d in this
valproate is part of the polytherapy. study.12 The escalation of risk with
Valproate is associated with a risk of first trimester use of valproate at
neurocognitive deficits, specifically a greater than approximately 700 mg/d
lower-than-expected verbal IQ.5 has been observed by other investiga-
tors,13 and both of these reports show
Teratogenic Risks of Specific rates of major congenital malforma-
Antiepileptic Drugs tions at greater than 20% when the dose
Valproate. Valproate is well recog- is 1500 mg/d or more.12,13 This dose
nized to be associated with a higher information is useful for those women
risk of major congenital malformations whose epilepsy absolutely cannot be
and adverse cognitive outcomes than controlled on other AEDs; lower daily
other AEDs. Its use should be avoided doses of valproate in these women
if possible in women with epilepsy of may optimize risk management.
childbearing potential. While some The recent publication from the
major congenital malformations ap- North American Antiepileptic Drug
pear to be a class effect of AEDs in Pregnancy Registry (NAAEDPR) pro-
general, such as spinal bifida, 7 vides a snapshot of the risks of major
valproate has a higher risk of spina congenital malformations associated
bifida than the other AEDs and is also with each AED as first trimester mono-
specifically associated with hypospadias.8 therapy exposure (Figure 3-1).14 These
The relative risk of major congen- data are consistent with worldwide
ital malformations with in utero first experience. They also show the impor-
trimester valproate exposure com- tant new finding that phenobarbital

TABLE 3-2 Major Points Regarding Teratogenesis From AAN Practice


Parametersa

b Valproate exposure in the first trimester is associated with midline birth


defects, including spina bifida, at a rate of 6% to 9%
b Increased risk of birth defects with polytherapy versus monotherapy
b Valproate exposure during pregnancy is associated with 6 to 10 points lower
verbal IQ in children
a
Reprinted with permission from Harden C, et al, Neurology.5 B 2009, American Academy of Neurology.
www.neurology.org/content/73/2/133.long.

Continuum (Minneap Minn) 2014;20(1):60–79 www.ContinuumJournal.com 63

Copyright © American Academy of Neurology. Unauthorized reproduction of this article is prohibited.


Pregnancy and Epilepsy

KEY POINTS
h For antiepileptic drugs
with large numbers of
exposures, such as
carbamazepine,
lamotrigine, phenytoin,
and levetiracetam, the
estimates of risks for
major congenital
malformations is
precise. For less
frequently used or
newer antiepileptic
drugs, such as
zonisamide,
oxcarbazepine,
clonazepam, North American AED Pregnancy Registry between 1997 and 2011. Percent of
topiramate, and FIGURE 3-1
major congenital malformations and 95% confidence interval with
phenobarbital, the monotherapy in first trimester exposure.
estimates are less CBZ = carbamazepine; VPA = valproate; LTG = lamotrigine; PHT = phenytoin;
precise, and larger OXC = oxcarbazepine; GBP = gabapentin; PB = phenobarbital; ZNS = zonisamide;
numbers of exposure TPM = topiramate; CZP = clonazepam; LVT = levetiracetam.
must be evaluated in
Data from Hernández-Dı́az S, et al, Neurology.14 www.neurology.org/content/78/21/
order to determine the 1692.abstract?sid=4a54bb28-bec2-496a-8fcb-e348d3da2b98.
actual risk.
h Little information also has a significantly increased risk of gabapentin, zonisamide, and clonaze-
regarding lacosamide major congenital malformations, most pam in the large NAAEDPR study to
is available at this time. often cardiac malformations. Phenobar- determine precise estimates, the upper
bital should also be avoided for women limits of the 95% CIs for three of these
of childbearing potential who have agents (gabapentin, oxcarbazepine, and
epilepsy. While it is not widely used zonisamide) are still low. With these
in developed countries, it is a mainstay four widely used AEDs for which the
of therapy in many areas of the world, data are most precise (carbamazepine,
with continuing impact on pregnancy lamotrigine, phenytoin, and leveti-
outcomes. racetam), the major congenital malfor-
The point estimates of the percent- mation rates all cluster closely around
age of major congenital malformations 2% to 2.5%. This study also showed
with valproate and phenobarbital expo- that, compared to the risk with
sure are greater than with the other lamotrigine, the risk ratio was fivefold
AEDs. Further, the CIs with valproate with valproate, threefold with pheno-
do not overlap with most of the other barbital, and twofold with topiramate.14
AEDs (specifically, carbamazepine, Topiramate. One of the most im-
lamotrigine, phenytoin, oxcarbazepine, portant findings since the publication
gabapentin, zonisamide, and leveti- of the AAN practice parameters in
racetam), which indicates that valproate 2009 is the association of topiramate
carries a significant increase in risk first trimester exposure with an in-
compared to these AEDs. These results creased risk of facial clefts. The asso-
are reliable estimates for carbamaze- ciation prompted the US Food and
pine, lamotrigine, phenytoin, and Drug Administration (FDA) to issue a
levetiracetam. While there were not report changing the pregnancy risk
enough exposures to oxcarbazepine, category from C to D in March 2011,
64 www.ContinuumJournal.com February 2014

Copyright © American Academy of Neurology. Unauthorized reproduction of this article is prohibited.


KEY POINT
meaning that topiramate is a known topiramate use and hypospadias in h The risk of midline
teratogen in humans. This has impor- exposed offspring.8,17 defects, such as facial
tant implications for both epilepsy and Carbamazepine. Although spina clefts, spina bifida, and
headache treatment. bifida has been strongly associated hypospadias, may be
The FDA categories provide a com- with in utero valproate exposure, increased with in utero
mon language for understanding tera- new evidence confirms that carbamaz- exposure to several
togenic risks (see Appendix A); epine exposure is also associated with antiepileptic drugs.
however, the clinical decision-making spina bifida.7,18 In a 2010 study using However, the risk of
pathway for AED use usually incorpo- the European Surveillance of Con- these defects is much
rates additional factors based on genital Anomalies (EUROCAT) Anti- greater with valproate
exposure.
emerging evidence, such as dose epileptic Study database and
effects and family history of major incorporating other published reports,
congenital malformations. Jentink and colleagues found an asso-
The association of facial clefts with ciation between spina bifida and in
in utero topiramate exposure has been utero carbamazepine exposure com-
reported from several populations. pared to no exposure (OR 2.6, 95% CI
The rate of oral clefts with topiramate 1.2 to 5.3).7 Notably, the risk with
first trimester exposure was 1.4% in carbamazepine was approximately
NAAEDPR, which is an approximately 80% less than with valproate, but was
10-fold increase compared to the local not different from the risk of exposure
control population prevalence of to other AEDs, specifically lamotrigine,
0.11%.14 In this report, oral cleft levetiracetam, phenobarbital, and clo-
prevalence was 0.5% with lamotrigine, nazepam. The EUROCAT database
carbamazepine, and phenytoin; 1.2% found no evidence of an association
with valproate; and 4% with pheno- between lamotrigine exposure and
barbital. The UK AED Pregnancy Reg- orofacial clefts.19
ister reported a 2.2% rate of facial Levetiracetam. Information on the
clefts with first trimester topiramate teratogenic risk of one of the most
exposure.15 Compared to the general widely used AEDs, levetiracetam, is
UK population prevalence of 0.2%, emerging, and the information thus
topiramate increased the risk of facial far is reassuring. In a recent report
clefts at a similar magnitude as the from the UK pregnancy registry of 671
NAAEDPR study. This increased risk levetiracetam first trimesterYexposed
was also found in two large birth pregnancies, 304 were monotherapy
defect registries using a different exposures. Of these, two major con-
method. Two large registries located genital malformations occurred (0.70%
in the United States, the Slone Epide- of the total; 95% CI 0.19% to 2.51%).20
miology Center Birth Defects Study The specific malformations were an
and the National Birth Defects Preven- inguinal hernia with maternal use of
tion Study, were analyzed from 1997 4000 mg/d and reflux requiring sur-
to 2007 for the association of cleft lip gery with maternal use of 2000 mg/d.
with or without cleft palate in off- These results also hint at a dose
spring exposed in utero to topiramate effect; the mean daily dose for preg-
compared to no AED use. The OR nancies with normal outcomes was
from pooled data for periconceptional 1680 mg. The NAAEDPR also confirms
topiramate use and cleft lip with or a low risk of major congenital
without cleft palate in these registries malformations with levetiracetam
was 5.4 (1.5Y20.1).16 Emerging find- at 2.4% (95% CI 1.2 to 4.3) with 450
ings also suggest an association with exposures.14
Continuum (Minneap Minn) 2014;20(1):60–79 www.ContinuumJournal.com 65

Copyright © American Academy of Neurology. Unauthorized reproduction of this article is prohibited.


Pregnancy and Epilepsy

Risk of Repeated Major dose effects have been debated in the


Congenital Malformations literature, the large sample sizes in this
Vajda and colleagues recently reported prospective international study are
on the outcomes of second pregnancies convincing, especially in light of the
in women with epilepsy after the occur- biological plausibility. Family history of
rence of a major congenital malforma- major congenital malformations was
tion in the first pregnancy while taking also a significant factor in the occurrence
the same AED for both pregnancies.21 of major congenital malformations in
The risk of a major congenital malfor- exposed offspring (Table 3-3).12
mation in the second pregnancy was
35.7% if the first pregnancy showed a COGNITIVE TERATOGENESIS
major congenital malformation versus WITH ANTIEPILEPTIC DRUGS
3.1% if the first pregnancy was normal Developmental delays associated with in
(OR 17.6, 95% CI 4.5Y68.7). This effect utero AED exposure were initially de-
was most marked with valproate, with scribed as part of the fetal antiepileptic
which the second pregnancy had a drug syndrome. This syndrome is char-
major congenital malformation rate of acterized by a variable set of malfor-
57% if the first pregnancy also showed mations but always includes ‘‘minor’’
a MCM. The authors suggest that a congenital malformations. The minor
maternal genetic influence predisposes malformations are cosmetic rather than
to teratogenicity and compounds the life-threatening. Their existence should
AED risk. Interestingly, the types of be determined by careful measurement
malformations varied within families.21 using comparisons to population nor-
mative values, because it has been
Antiepileptic Drug Teratogenic demonstrated that relying on obser-
Dose Effects vation alone is not a sensitive method.22
The clearest evidence for dose- These malformations include craniofa-
associated risk of major congenital cial dysmorphic features, such as a flat
malformations is from the International nasal bridge, epicanthic folds, a promi-
Registry of Antiepileptic Drugs and Preg- nent upper lip over wide mouth, and
nancy (EURAP) study.12 In this study, hypertelorism. Fingernail and distal pha-
dose effects were present for four lanx hypoplasia may be present. Micro-
widely used AEDs for which large cephaly and facial clefts (a major
sample sizes were available for first malformation) are also described in
trimester monotherapy exposures: phe- association with the fetal AED syndrome,
nobarbital, valproate, carbamazepine, as well as developmental delay.23
and lamotrigine. The rates of major While the initial reports of minor
congenital malformations were the low- congenital malformations in babies
est with less than 300 mg/d of exposed in utero to AEDs are from
lamotrigine (at 1.7%) and with less than the 1970s, more recent reports con-
400 mg/d of carbamazepine (at 2%). firm that minor congenital abnormali-
Although the major congenital malfor- ties are highly correlated with the
mation rates were higher with pheno- occurrence of decreased verbal IQ,
barbital and valproate at all doses, each performance IQ, and full-scale IQ.22,24
AED showed a significant dose effect. From a 2005 study of 76 children whose
For carbamazepine at or above 1000 mg/d, mothers took AEDs in pregnancy,
the rate of major congenital malformations the investigators concluded that either
was 7.7%, and for lamotrigine at or above midface hypoplasia or digital hypopla-
300 mg/d it was 3.6%.12 Although these sia in a neonate exposed to AEDs in
66 www.ContinuumJournal.com February 2014

Copyright © American Academy of Neurology. Unauthorized reproduction of this article is prohibited.


TABLE 3-3 Author’s Stratification of Risk for Antiepileptic
Drugs Structural Teratogenesis (Occurrence of Major
Congenital Malformations)

b Overall
Family history of major congenital malformations will increase risk of major
congenital malformations with antiepileptic drug (AED) exposure
b Probably safest AEDs
Lamotrigine
Carbamazepine
Phenytoin
Levetiracetam
b Probably have risk lower than valproate
Oxcarbazepine
Zonisamide
Gabapentin
b Have significant risk greater than some other AEDs
Topiramate (oral clefts)
Phenobarbital (cardiac defects)
Valproate (spina bifida, hypospadias)

utero should prompt a developmental This report also led to the apprecia-
evaluation.24 tion that subtle but clinically impor-
Another influential study in this tant cognitive teratogenesis occurs in
field was put forth by Reinisch and the absence of any dysmorphism.
colleagues in 1995.25 The investigators Conversely, major congenital malfor-
evaluated adult men exposed to phe- mations have not been shown to be
nobarbital in utero and an unexposed, associated with lower verbal IQ.26
age-matched control group, and found Two subsequent retrospective stud-
that phenobarbital exposure during ies showed a risk of significantly lower
gestation was associated with a verbal IQ in children exposed in utero
seven-point reduction in verbal IQ to valproate as compared to other
compared to controls. Notably, the AED exposures or to nonexposed
mothers of the study subjects did not children.27,28 In the study by Vinten
have epilepsy and used phenobarbital and colleagues in 2005, 249 children
during pregnancy for other condi- between the ages of 6 and 16 years
tions, such as blood pressure control with in utero AED exposures were
and anxiety treatment.25 The investi- evaluated. Valproate-exposed children
gators’ important findings include the were more likely to have a verbal IQ
specific vulnerability of verbal IQ com- lower than 69. Importantly, maternal
pared to other cognitive domains, IQ was controlled for in this study,
which has been borne out in subse- indicating that the adverse effect oc-
quent studies with other AED expo- curred independently of the influence
sures. Further, they found that third of maternal IQ.27 The 2004 study by
trimester exposure imparted greater Gaily and colleagues was performed with
risk than first trimester exposure.25 the investigators masked to the exact AED

Continuum (Minneap Minn) 2014;20(1):60–79 www.ContinuumJournal.com 67

Copyright © American Academy of Neurology. Unauthorized reproduction of this article is prohibited.


Pregnancy and Epilepsy

KEY POINT
h In the Neurodevelopmental exposure; they evaluated 182 school-aged except valproate, for which there was
Effects of Antiepileptic children whose mothers were treated for a clear dissociation with the children’s
Drugs study, in utero epilepsy.28 Again, verbal IQ was signifi- IQ falling below the predicted range
valproate exposure was cantly lower than expected in children based on the maternal IQ.29
associated with impaired with a history of valproate exposure as The primary outcome of the NEAD
cognitive development; either polytherapy or monotherapy. The study was IQ at age 6 years. After a
IQ at 6 years of age was average verbal IQ in these groups was 82 multivariate analysis, the finding is that
lower after exposure and 85, respectively, while carbamazepine- children exposed in utero to valproate
to valproate than to exposed children had an average verbal IQ had a significantly lower IQ (mean 97,
carbamazepine, of 95Vnot different from a control group, 95% CI 94 to 101) than those exposed to
lamotrigine, or
which implied no risk with carbamazepine carbamazepine (mean 105, 95% CI 102
phenytoin.
exposure.28 Doses and serum AED levels to 108; P=.0015), lamotrigine (mean
were evaluated in this study; notably, the 108, 95% CI 105 to 110; P=.0003) or
median valproate dose was 950 mg/d, phenytoin (mean 108, 95% CI 104 to
which is thought to be a relatively safe 112; P=.0006). A dose effect, with worse
dose for structural teratogenesis. cognitive scores associated with higher
The need for prospective, well- maternal medication doses, was present
controlled studies to evaluate the for valproate but not for the other AEDs.
effects of AEDs on cognitive outcomes The valproate dose effect was found
in offspring exposed to AEDs in utero across all cognitive domains including
was recognized and funded by the IQ, verbal ability, nonverbal ability, mem-
National Institute of Neurological Dis- ory and executive function.31
orders and Stroke (NINDS). The The vulnerability of verbal IQ develop-
Neurodevelopmental Effects of Anti- ment to in utero AED exposure was
epileptic Drugs (NEAD) study is an supported by the NEAD study findings.
ongoing, prospective, observational, Although the study had no unexposed
nonrandomized but assessor-masked, control group to provide comparison,
multicenter study performed at 25 verbal abilities were lower than nonverbal
sites in the United States and United abilities for all AED exposures. Left-
Kingdom. The offspring of women handedness was more frequent in the
with epilepsy taking one of four overall study group and was most mark-
monotherapy AEDs (phenytoin, edly increased in the valproate- and
lamotrigine, valproate, and carbamaz- lamotrigine-exposed groups compared
epine) during pregnancy were evalu- to expected rates. These interesting find-
ated. Three reports on the outcomes ings prompted the investigators to spec-
of this study are available, tracking the ulate that AED exposure may alter the
cognitive development of 224 enrolled development of cerebral lateralization.31
children at 3 years,29 4.5 years,30 and 6 Determining a preventive effect of
years of age.31 Findings in this study folate supplementation on AED-
have been consistent as the children associated major congenital malfor-
have grown, with an overall trend of mations has been elusive,12 but this
improvement in cognitive abilities in was not the case for cognitive terato-
all groups across the evaluation period.31 genesis according to the NEAD study
In all NEAD reports, valproate exposure findings. The mean IQs were higher in
was associated with adverse cognitive children exposed to periconceptional
outcomes. This finding emerged at the folate (mean 108, 95% CI 106 to 111)
earliest interim analysis point, where than they were in unexposed children
maternal IQ was highly associated with for all AEDs (mean 101, 95% CI 98 to
the children’s IQ for all exposures 104; P=.0009).31
68 www.ContinuumJournal.com February 2014

Copyright © American Academy of Neurology. Unauthorized reproduction of this article is prohibited.


Levetiracetam is one of the most women with epilepsy and an un-
widely used AEDs, including in wom- treated nonepileptic control group,
en of childbearing potential who have 6.3% of valproate-exposed children
epilepsy. This is appropriate because had autism spectrum disorder or key
emerging evidence indicates a low risk symptoms of this disorder (language
for structural teratogenesis. Regarding impairment, reduced attention, social
the evidence for risk of cognitive difficulties) compared to 0.9% of the
teratogenesis with levetiracetam, a control children, a sevenfold increase
recent report suggests that leveti- in occurrence.34 In the NEAD study,
racetam exposure also carries less risk behavioral problems were also found
of developmental delay than valproate in the valproate-exposed children;
and no risk increase over unexposed parents of these children reported a
controls. 32 These data should be decline in social skills and the pres-
considered preliminary; the AED ence of hyperactive behaviors, sug-
groups were small, consisting of gesting a risk for the development of
51 levetiracetam-exposed and 44 attention deficit hyperactivity disor-
valproate-exposed children each, and der.35 The autism and valproate asso-
the children were tested at less than 2 ciation was confirmed in a recent
years of age using a developmental population-based retrospective study
scale, since IQ tests cannot be used at using Danish national medical regis-
this early age.32 The Maternal Outcomes ters.36 In this population, children
and Neurodevelopmental Effects of exposed to valproate and other AEDs
Antiepileptic Drugs (MONEAD) study33 in utero, as well as unexposed chil-
follows on the findings of the NEAD dren, were identified for occurrence
study and will include a levetiracetam- of autism. In valproate-exposed chil-
exposed group of offspring, which will dren, the absolute risk of autism
augment these findings (Table 3-4). spectrum disorder was 4.42% and of
childhood autism was 2.95%. By com-
Autism and In Utero Valproate parison, in the overall population, the
Exposure absolute risk of autism spectrum disor-
One of the most convincing initial der was 1.53% and of childhood autism
reports on the association between in was 0.48%. No increased risk was
utero valproate exposure and autism evident with other AED exposures,
spectrum disorder was by Bromley and no significant increase in risk was
and colleagues in 2008. 34 In this imparted by maternal epilepsy itself.36
prospective study of the offspring of These two studies34,36Vthe Bromley

TABLE 3-4 Summary of Risk Management for Cognitive


Teratogenesis

b Use folate supplementation for women with epilepsy of childbearing


potential
b Avoid valproate and phenobarbital, if possible
b Valproate exposure during pregnancy increases risk of autism and autism
spectrum disorders in offspring
b Be cognizant of continued risk in all trimesters when managing women with
epilepsy with ongoing seizures

Continuum (Minneap Minn) 2014;20(1):60–79 www.ContinuumJournal.com 69

Copyright © American Academy of Neurology. Unauthorized reproduction of this article is prohibited.


Pregnancy and Epilepsy

and colleagues report and the Danish The EURAP study enrolled patients
study performed using national medi- from Europe, India, and Australia and
cal registriesVperformed by differing reported on 1956 pregnancies in
methods show similar absolute and women with epilepsy who were
relative risks of autism spectrum dis- followed prospectively. Status epilep-
order and autism in children exposed ticus occurred in 36 pregnancies (1.8%)
to valproate in utero. Further, as with and was convulsive in 12. It resulted in
other teratogenic outcomes, an in- one stillbirth and no miscarriages or
creased risk with valproate compared maternal deaths.40
to other AEDs is present (Table 3-5). It is not the case, however, that
seizures during pregnancy are without
RISKS OF SEIZURES DURING adverse consequences. One intriguing
PREGNANCY study showed that 5 or more convul-
The immediate risk of a seizure to the sive seizures during pregnancy was
developing fetus is difficult to quantify. independently associated with lower
Certainly, a generalized seizure could verbal IQ in the offspring; children
cause trauma that would put the preg- were tested at age 6 years and older.
nancy at risk. However, prevention of Verbal IQ was approximately seven
trauma, injury, drowning, and sudden points below that of offspring of
unexpected death due to seizures is mothers without seizures during preg-
important for the nonpregnant as nancy.26 The mechanism for this effect
well as the pregnant epilepsy popula- is not known.
tion and is in great part the motivation Another important study from the
for treatment. Case reports have been Taiwanese birth certificate registry
published of fetal heart rate decele- showed that, for women with epilepsy,
rations in association with focal seizures during pregnancy were asso-
seizures.37,38 However, systematic eval- ciated with an increased risk of off-
uations of this issue must be extrapo- spring who were small for gestational
lated from the obstetric literature, in age compared with women with epi-
which fetal heart rate changes follow- lepsy who did not have seizures during
ing eclamptic convulsions are well pregnancy (OR 1.34; 95% CI 1.01 to
described.39 These include fetal bra- 1.84).41 This study is remarkable be-
dycardia, transient late decelerations, cause none of the women with epi-
decreased beat-to-beat variability, and lepsy in this study took AEDs 41 ;
compensatory tachycardia. These therefore, a major confounder for the
changes are thought to be caused by occurrence of small-for-gestational-age
maternal hypoxia and reverse within 3 offspring42 was not present.
to 10 minutes after the convulsion The pregnancies in women with
terminates.39 epilepsy are also at risk simply due to

TABLE 3-5 Autism and Valproate: Autism Spectrum Disorder Risk

In Utero Increased Risk


Valproate General With Valproate
Study Exposure Population Exposure
Bromley et al, 200834 6.30% 0.90% 7-fold
Christenson et al, 201336 4.42% 1.53% 3-fold

70 www.ContinuumJournal.com February 2014

Copyright © American Academy of Neurology. Unauthorized reproduction of this article is prohibited.


KEY POINT
the fact that women with epilepsy rence.40 Further, AED dose increases h Adverse effects of
appear to have a much higher rate of occurred more frequently in the seizures in pregnancy
death either during pregnancy or oxcarbazepine- and lamotrigine-treated include (1) maternal
within 42 days of pregnancy termina- subjects, probably reflecting the need death from seizures
tion, according to data from the UK for treatment of ongoing seizures with when antiepileptic
Confidential Inquiry Into Maternal use of these AEDs. drugs are abruptly
Deaths.26 The rate of maternal deaths In another recent publication dur- stopped; (2) infants
for women with epilepsy is estimated ing which seizure frequency in women who are small for
to be 10 times higher than expected in with epilepsy was documented before their gestational age;
the general population. Available de- pregnancy as well as during pregnancy, (3) decreased verbal IQ
in offspring when five or
tails of some cases point to the the prepregnancy seizure occurrence
more convulsive seizures
cessation of medication, poor compli- was highly predictive of the course of
occur during pregnancy;
ance, and seizures as possible causes. seizure activity during pregnancy.43 and (4) fetal heart rate
These sobering findings point up the Seizure relapse was most likely during alterations during
importance of seizure prevention with the second and third months and convulsive seizures.
effective treatments in women with again in the sixth month.43 The in-
epilepsy. They also underscore the vestigators found that oxcarbazepine
need for an open understanding be- and other AEDs used as monotherapy
tween the patient and the provider were associated with an increased risk
that the patient will discuss any AED of seizure occurrence, but that val-
changes before undertaking them on proate was associated with a lower risk
her own, especially in the case of an of seizure occurrence. Overall, poly-
unexpected pregnancy. therapy was associated with the risk
of seizures during pregnancy, consis-
SEIZURE CONTROL DURING tent with a more severe underlying
PREGNANCY seizure disorder.43 Both of these stud-
Using retrospective studies, with pa- ies showed that seizures occurring
tients as their own control, the AAN around the time of delivery were
practice parameters reported that 80% highly associated with prior seizure
to 90% of women with epilepsy who occurrence.40,43 Seizures during labor
were seizure free for 9 to 12 months occurred in 3.5% of subjects in the
before pregnancy were likely to re- EURAP study.40 Seizures during the
main so during pregnancy.4 These month before conception was associ-
findings are reassuring and suggest ated with a 3.7-fold increase in the risk
that the physiologic changes of preg- of seizures in the peripartum period in
nancy do not in general adversely the second study.43
affect seizure threshold. Similar results
were reported from the EURAP study, MANAGEMENT OF
in which data were prospectively ANTIEPILEPTIC DRUG LEVELS
obtained, using the first trimester DURING PREGNANCY
seizure frequency as a comparison It is important that patients who are
for the second and third trimesters.40 pregnant or planning pregnancy un-
These investigators did report small derstand and anticipate that serum
proportions of either seizure frequency AED levels should be checked during
increase (17.3%) or decrease (15.9%) pregnancy, when levels of AEDs gen-
during pregnancy, consistent with pre- erally tend to decrease.6 Ideally, an
vious information.4 In the EURAP study, individual therapeutic level at which
use of oxcarbazepine as monotherapy the patient is doing well is docu-
was associated with seizure occur- mented, thereby providing a target
Continuum (Minneap Minn) 2014;20(1):60–79 www.ContinuumJournal.com 71

Copyright © American Academy of Neurology. Unauthorized reproduction of this article is prohibited.


Pregnancy and Epilepsy

KEY POINTS
h Women with epilepsy level to maintain during pregnancy. ied for pregabalin, lacosamide, and
should be advised that Induction of AED metabolism by the retigabine.
folic acid use does not high hormone levels of pregnancy is
eliminate the risk of the main cause of decreased levels, and
major congenital this will affect both total and unbound FOLIC ACID USE DURING
malformations compartments of the circulating PREGNANCY
associated with AEDs.44 Glucuronidation is markedly Folic acid supplementation during
antiepileptic drug use. induced by pregnancy, which is the pregnancy has been associated with
h Folic acid use during main metabolic avenue for lamotrigine a risk reduction in the occurrence
preconception and and oxcarbazepine. This probably ex- of congenital malformations, includ-
pregnancy in women plains the increased risk of seizures ing neural tube defects, of approxi-
with epilepsy decreases reported with use of these AEDs mately 50%.45 The importance of folic
the risk of lowered during pregnancy and also indicates acid supplementation is a widely
verbal IQ caused by that frequent monitoring and dose accepted general health measure.
antiepileptic drug
adjustment should be undertaken Since 1998, by US federal mandate,
exposure. It also
when women are using these AEDs. folic acid has been added to cold
decreases the risk
of spontaneous
Carbamazepine levels remain little cereals, flour, breads, pasta, bakery
miscarriage. c ha n g e d du r i n g pr e g nancy. 6 , 4 4 items, cookies, and crackers. The
Topiramate and valproate also decline American College of Obstetricians
less than other AEDs during pregnancy, and Gynecologists (ACOG) advocates
from 10% to 30%. Other AEDs for which that women with epilepsy take a folic
there is information regarding a decline acid dose of 4 mg/d,46 although the
in levels during pregnancy from 40% effectiveness of this dose versus lower
to 70% include levetiracetam, phenytoin, doses in the absence of a family
phenobarbital, and zonisamide.44 history of neural tube defects has
These changes during pregnancy not been established. Two impor-
have much interindividual variability. tant studies have not shown a benefit
Within-individual changes may be of folic acid supplementation in
more predictable as a result of genet- reducing the risk of major con-
ically imparted metabolic capacity. genital malformations. 12,47 These
Therefore, a simple AED monitoring findings imply that the mechanism
recommendation is to monitor AED of AED-associated major congenital
levels monthly (or every 2 weeks at malformations does not primarily
the beginning of pregnancy for ox- involve folate mechanism. However,
carbazepine and lamotrigine), aiming the NEAD study showed a clear
to maintain the predetermined thera- benefit in reduction of cognitive
peutic level. Ongoing monitoring can teratogenesis with folic acid sup-
be based on this initial information; plementation. In another recent
however, obtaining levels monthly publication using a subset of the
throughout pregnancy is reasonable, EURAP population, folic acid supple-
keeping in mind that the maximum mentation before pregnancy re-
period of metabolic induction is in the duced the risk of spontaneous
third trimester.44 Allowing the level to abortion in women with epilepsy.48
drift below the therapeutic target Folic acid supplementation is re-
level may be risky, since many women commended for women of child-
with epilepsy are titrated to the lowest bearing potential who have epilepsy,
effective level before pregnancy.44 and in keeping with ACOG guidelines,
At this time, changes in levels 4 mg/d is often used during pre-
during pregnancy have not been stud- conception and pregnancy.
72 www.ContinuumJournal.com February 2014

Copyright © American Academy of Neurology. Unauthorized reproduction of this article is prohibited.


OBSTETRIC RISKS including a low 5-minute Apgar score
The evidence cited in the AAN and and the need for transfer to a neonatal
American Epilepsy Society practice intensive care unit.50
parameters showed that for women
with epilepsy taking AEDs, there was ADVISING A PATIENT WHO IS
probably no substantially increased CONTEMPLATING PREGNANCY
risk (meaning that the risk was not Several clear tenets serve to assist in
more than twice the expected risk) of answering the question of how best to
cesarean delivery or late-pregnancy provide care for a patient contemplat-
bleeding, and probably no moderately ing pregnancy. First, the patient’s
increased risk (meaning that the risk seizures should be well controlled,
was not more than 1.5 times the particularly generalized convulsions.
expected risk) of premature contrac- An individualized therapeutic level
tions or premature labor and deliv- should be established with at least
ery.4 However, in a population-based two serum values before pregnancy.
study published subsequent to the Secondly, the AED used should not be
AAN report,49 women with epilepsy valproate or phenobarbital, if possible.
who took AEDs had an increased risk If valproate or phenobarbital must be
of mild preeclampsia (OR 1.8, 95% CI used because of failure or intolerabil-
1.3 to 2.4), gestational hypertension ity of other meds, the lowest effective
(OR 1.5, 95% CI 1.0 to 2.2), vaginal dose should be used (ideally below
bleeding late in pregnancy (OR 1.9, 750 mg/d for valproate) and folic acid
95% CI 1.1 to 3.2), and delivery before supplementation should be given. In
34 weeks of gestation (OR 1.5, 95% CI general, to decrease risk, the lowest
1.1 to 2.0) compared to women with dose of the most streamlined regimen
epilepsy not taking AEDs, who in turn should be achieved, with mono-
did not differ from the general popu- therapy if possible. Thirdly, the physi-
lation.49 Although these analyses do cian and patient should have a clear
not greatly differ in that the obstetric understanding of the risks given the
risks are still relatively low, the individual circumstances, and a plan
strength of evidence imparted by for how to manage them. A plan for
large, population-based studies is monitoring AED levels and what to do
likely to tip the scales toward an in the setting of declining levels
increase in obstetric risks. These find- should be discussed. Some women
ings support a recommendation that with epilepsy may not be taking AEDs,
women with epilepsy taking AEDs and discussion of how to manage
who are contemplating pregnancy seizure recurrence should be dis-
should be counseled that they are at cussed. Women with epilepsy should
increased risk for pregnancy compli- hear that no health care provider can
cations, including premature delivery. guarantee that the pregnancy will
Recent evidence from the NEAD result in a healthy baby, since many
study also indicates a risk for offspring factors, including genetic influences,
who are small for gestational age with are outside the scope of medical
in utero valproate exposure42 com- management. They should also hear
pared to the other AED exposures. A that in almost 2% of pregnancies in
study using the Finnish national health the general population, in women not
register also indicates an association taking medications, babies are born
between AED use in women with with a major congenital malformation.
epilepsy and increased neonatal risks, If the patient is taking topiramate,
Continuum (Minneap Minn) 2014;20(1):60–79 www.ContinuumJournal.com 73

Copyright © American Academy of Neurology. Unauthorized reproduction of this article is prohibited.


Pregnancy and Epilepsy

a
TABLE 3-6 Management of Epilepsy During Pregnancy

b Prior to Planning Pregnancy


Review current seizure frequency and severity. If seizures persist (in particular, generalized tonic-clonic seizures),
consider further seizure management before endorsing pregnancy at that time. The potential teratogenic effects
of antiepileptic drugs (AEDs) need to be carefully weighed against the efficacy of the drug to control seizures.
If epilepsy is in remission, consider suggesting gradual withdrawal of treatment before pregnancy if recurrence risk is
low and the patient is willing to take the risk.
Review history for congenital malformations with previous pregnancies or family history of such. Should congenital
malformations be present, discuss the higher risk of malformations, consider genetics consultation, consider a switch
from the AED used in previous pregnancy that resulted in malformations, and encourage more intensive
monitoring in future pregnancies.
Review all medications. If the patient is taking valproate, change in medication should be considered.
Review history for, or try out, lowest effective dose.
If the patient is taking the lowest effective dose of an appropriate AED, obtain serum level preferably on
two occasions weeks apart, and as trough level before morning dose in order to obtain range of effective
levels and to assess stability of levels within the patient.
Recommend taking 4 mg of folic acid per day.
All changes in AED therapy should be completed and fully evaluated before conception.
b During Pregnancy
Continue folic acid and prenatal vitamins throughout pregnancy.
Avoid changes in AEDs unless prompted by poor maternal seizure control. Avoid AED withdrawals during
pregnancy because of the risk of seizure relapse and since the potential benefits for fetal exposure are
questionable with the necessary slow tapering.
Monitor for seizure occurrence regularly; the frequency of office visits depends on seizure control before and
during pregnancy.
Where methods are available, obtain serum AED levels at the time of pregnancy confirmation and at regular
intervals throughout pregnancy. In the case of carbamazepine and valproate, this may be less frequent; in the case
of lamotrigine, obtain levels monthly and consider increase in dose in a proportional manner to restore individual
prepregnancy therapeutic level. Obtain free and total levels if the woman is taking phenytoin or valproate,
if available.
Monitor for vomiting of AEDs, and if it is occurring, consider methods to reduce vomiting with assistance
from an obstetrician.
Follow obstetric outcomes, including ultrasound results. Suggest high-level ultrasound at 14 to 18 weeks for a
detailed view of fetal structures.
If a malformation is recognized in ultrasonography, the pregnancy would require more detailed evaluation.
Confirmatory tests such as fetal MRI, amniocentesis, and other procedures are part of the evaluation in some
regions. Further counseling on the possible impact of the suspected malformation on the health of the baby should
be discussed by people experienced in this field. The couple may require assistance in deciding on further course of
action such as continuation or termination of the pregnancy. Undue attribution of all complications to the use of
AEDs or occurrence of seizures during pregnancy can lead to severe emotional and social stress and should be avoided.
All seizures that occur during pregnancy may not be epilepsy. Eclampsia and other acute symptomatic
seizures may occur, and new-onset epilepsy can manifest during pregnancy. Clinicians should ascertain
the type of epilepsy or seizure disorder, as the treatment may differ according to the cause of seizures.
If seizures occur late in pregnancy with therapeutic AED levels, monitor for preeclampsia and discuss
delivery induction, if appropriate.
Continued on next page

74 www.ContinuumJournal.com February 2014

Copyright © American Academy of Neurology. Unauthorized reproduction of this article is prohibited.


a
TABLE 3-6 Management of Epilepsy During Pregnancy (continued)

b After Delivery
If the patient is stable and the dose has been increased during pregnancy, decrease the AED dose over
the first 10 days after delivery to a dose slightly above prepregnancy maintenance dose, to compensate
for fatigue and postpartum sleep deprivation. A more rapid return to a lower dose may be needed for
lamotrigine to avoid concentration-dependent adverse effects in the postpartum period.
Encourage breast-feeding but indicate that adverse effects could occur occasionally if the mother is
taking phenobarbital.
Counsel the patient about getting adequate sleep and to report adverse mood effects.
a
Adapted with permission from Thomas SV, et al, Wiley-Blackwell.51 B 2013, John Wiley and Sons.

she should understand the small but factors such as response to topira-
increased risk of facial cleft, and mate treatment, family history, and
discuss whether the AED should be previous AEDs used. Both the patient
changed before conception, based on and the caregiver need to have a

Case 3-1
A 27-year-old right-handed woman with a history of juvenile myoclonic epilepsy since 17 years of age was
considering becoming pregnant. She was in a long-term relationship and had been using condoms for
birth control. She took low-dose valproate at 750 mg/d and had no seizures for 2 years and rare myoclonus
premenstrually. Her previous generalized tonic-clonic seizures only occurred when she missed her medication
or was sleep deprived. Although extremely reluctant to consider changing medications, she was gradually
cross-tapered to levetiracetam because of teratogenic concerns. She tolerated levetiracetam at a dose of
1500 mg twice per day but had another generalized tonic-clonic seizure. She was then restarted on valproate,
and lamotrigine was added to her regimen. She gradually tapered off valproate and was maintained on
lamotrigine at 500 mg/d with levels of 6 2g/mL to 7 2g/mL. She had no seizures and generally felt well.
She continued on lamotrigine monotherapy, became pregnant after stopping use of condoms, and did well
during the first two trimesters of pregnancy, with an increase in her dose of lamotrigine from 500 mg/d to
800 mg/d. However, she had several generalized tonic-clonic seizures in 1 day at 26 weeks when her
lamotrigine level had decreased to 5 2g/mL. She was also tired and stressed during pregnancy. Her lamotrigine
dose was gradually increased to 1000 mg/d based on levels checked twice per month. On this dose, at
34 weeks, her level had increased to 12 2g/mL. She began to experience dizziness and mild discoordination,
thought to be adverse effects of lamotrigine. She had several more generalized tonic-clonic seizures at
35 weeks, and levetiracetam was added to her regimen at 500 mg twice per day, while lamotrigine was
decreased to 800 mg/d. She had no further seizures and delivered a healthy baby at 39 weeks.
Comment. This case illustrates that with primary generalized epilepsy, such as juvenile myoclonic
epilepsy, for which valproate is often a very effective medication, the patient can be successfully
transitioned to a less teratogenic antiepileptic drug. With careful monitoring of her lamotrigine
levels, she did well during most of the pregnancy.
However, this case also illustrates the pitfalls of lamotrigine use during pregnancy, as well as the
susceptibility of some women to seizures late in pregnancy. Her lamotrigine levels were monitored
frequently and followed by dose adjustment, but the usual proportional dose-to-level relationship
became unpredictable, as might be expected with the complex influence of pregnancy reproductive
hormones on lamotrigine metabolism. She began to have seizures again, even with increased
lamotrigine levels to the point of mild toxicity. With the addition of levetiracetam, a medication she
had previously tolerated, she safely continued the pregnancy.

Continuum (Minneap Minn) 2014;20(1):60–79 www.ContinuumJournal.com 75

Copyright © American Academy of Neurology. Unauthorized reproduction of this article is prohibited.


Pregnancy and Epilepsy

certain amount of confidence in the Case 3-1 demonstrates manage-


available data to navigate the decisions ment strategies and pitfalls for a woman
and to accept the risks and a willingness with primary generalized epilepsy.
to work hard together for a most worthy Case 3-2 demonstrates the challenges
outcome, a child. The management of managing seizures that have not been
paradigm is presented in Table 3-6. well controlled early in pregnancy.

Case 3-2
A 24-year-old right-handed woman presented after having her first
generalized tonic-clonic seizure during the daytime. Further history revealed
that she had been having abnormal nocturnal behavior for the previous
6 months. Her video-EEG showed left frontal low-amplitude fast activity
progressing to hemispheral rhythmic slowing during the nocturnal events.
Her behavior consisted of raising her right arm and striking it against the
pillow, as well as flexor posturing of the left arm. The patient smiled during
the events but did not vocalize and did not always remember them, although
in general she could count how many spells had occurred in the night. The
events occurred out of sleep without warning. Her brain MRI was normal. She
was recently married and using condoms for birth control.
She had difficulty accepting the diagnosis of epilepsy and reluctantly took
oxcarbazepine at a dose of 1200 mg/d. Because of inadequate seizure control,
levetiracetam was briefly added to her regimen but caused severe sedation.
Lamotrigine was added at 25 mg twice per day with a plan to escalate the dose.
She was not planning pregnancy but, on the day of a scheduled
positron emission tomography (PET) scan as part of a presurgical
evaluation, discovered that she was 5 weeks pregnant. Her seizures had
increased from two to three per night to six to seven per night at this
point. The lamotrigine was stopped and oxcarbazepine increased to 1500
mg/d in an attempt to control the seizures on monotherapy, but her
seizure frequency remained unchanged. At 9 weeks’ gestation, she had
another secondarily generalized tonic-clonic seizure and was hospitalized.
She was having 10 nocturnal seizures per day on video-EEG monitoring.
Phenytoin was added to her regimen with a 1000 mg oral loading dose in
divided doses. The seizures decreased to two per night, and a maintenance
dose of 200 mg phenytoin twice daily was started, with a phenytoin level
of 15 2g/mL. Since she was responding to the phenytoin, an attempt was
made to decrease the oxcarbazepine, but the seizures again increased to
8 per night. She was then maintained on oxcarbazepine at 600 mg twice
per day and phenytoin 200 mg twice per day, with levels of phenytoin at
17 2g/mL to 18 2g/mL and free levels of 1.8 2g/mL. She remained seizure
free on this regimen and delivered a healthy baby girl at 39 weeks’ gestation.
Comment. This case illustrates the challenges of poorly controlled
seizures before pregnancy and the risk of worsening seizures at the
beginning of pregnancy, probably due to a decrease in the level of
oxcarbazepine caused by pharmacokinetic effects of reproductive hormones.
While phenytoin is a well-known antiepileptic drug used readily when
seizures are escalating in the nonpregnant patient, this case prompts a review
of existing data on the teratogenic effects of phenytoin. The recent data from
pregnancy registries and cognitive outcome studies are reassuring regarding
phenytoin. Further, one of the main issues for using phenytoin during
pregnancy, which is management of levels, are stable for this patient.

76 www.ContinuumJournal.com February 2014

Copyright © American Academy of Neurology. Unauthorized reproduction of this article is prohibited.


REFERENCES congenital malformations. N Engl J Med
2010;362(23):2185Y2193.
1. Viinikainen K, Heinonen S, Eriksson K, 9. Morrow J, Russell A, Guthrie E, et al.
Kälviäinen R. Fertility in women with active Malformation risks of antiepileptic drugs
epilepsy. Neurology 2007;69(22):2107Y2108. in pregnancy: a prospective study from
2. Sukumaran SC, Sarma PS, Thomas SV. the UK Epilepsy and Pregnancy Register.
Polytherapy increases the risk of infertility in J Neurol Neurosurg Psychiatry 2006;77(2):
women with epilepsy. Neurology 2010; 193Y198.
75(15):1351Y1355. 10. Wide K, Winbladh B, Kallen B. Major
3. Harden CL, Pennell PB. Neuroendocrine malformations in infants exposed to
considerations in the treatment of men antiepileptic drugs in utero, with emphasis
and women with epilepsy. Lancet Neurol on carbamazepine and valproic acid: a
2013;12(1):72Y83. nation-wide, population-based register
study. Acta Paediatr 2004;93(2):174Y176.
4. Harden CL, Hopp J, Ting TY, et al; American
Academy of Neurology; American Epilepsy 11. Artama M, Auvinen A, Raudaskoski T, et al.
Society. Practice parameter update: Antiepileptic drug use of women with
management issues for women with epilepsy and congenital malformations
epilepsyVfocus on pregnancy (an in offspring. Neurology 2005;64(11):
evidence-based review): obstetrical 1874Y1878.
complications and change in seizure 12. Tomson T, Battino D, Bonizzoni E, et al;
frequency: report of the Quality Standards EURAP study group. Dose-dependent risk of
Subcommittee and Therapeutics and malformations with antiepileptic drugs: an
Technology Assessment Subcommittee of analysis of data from the EURAP epilepsy
the American Academy of Neurology and and pregnancy registry. Lancet Neurol
American Epilepsy Society. Neurology 2011;10(7):609Y617.
2009;73(2):126Y132.
13. Vajda FJ, Graham J, Roten A, et al.
5. Harden CL, Meador KJ, Pennell PB, et al; Teratogenicity of the newer antiepileptic
American Academy of Neurology; American drugsVthe Australian experience. J Clin
Epilepsy Society. Practice parameter update: Neurosci 2012;19(1):57Y59.
management issues for women with
epilepsyVfocus on pregnancy (an 14. Hernández-Dı́az S, Smith CR, Shen A, et al;
evidence-based review): teratogenesis and North American AED Pregnancy Registry.
perinatal outcomes: report of the Quality Comparative safety of antiepileptic drugs
Standards Subcommittee and Therapeutics during pregnancy. Neurology 2012;78(21):
and Technology Assessment Subcommittee 1692Y1699.
of the American Academy of Neurology and
15. Hunt S, Russell A, Smithson WH, et al;
American Epilepsy Society. Neurology
Topiramate in pregnancy: preliminary
2009;73(2):133Y141.
experience from the UK Epilepsy and
6. Harden CL, Pennell PB, Koppel BS, et al; Pregnancy Register. Neurology 2008;71(4):
American Academy of Neurology; American 272Y276.
Epilepsy Society. Practice parameter update:
16. Margulis AV, Mitchell AA, Gilboa SM, et al;
management issues for women with
National Birth Defects Prevention Study. Use
epilepsyVfocus on pregnancy (an
of topiramate in pregnancy and risk of oral
evidence-based review): vitamin K, folic acid,
clefts. Am J Obstet Gynecol 2012;207(5):
blood levels, and breastfeeding: report of
405.e1Ye7.
the Quality Standards Subcommittee and
Therapeutics and Technology Assessment 17. Vajda FJ, O’Brien TJ, Graham J, et al.
Subcommittee of the American Academy of Associations between particular types of
Neurology and American Epilepsy Society. fetal malformation and antiepileptic drug
Neurology 2009;73(2):142Y149. exposure in utero. Acta Neurol Scand
2013;128(4):228Y234.
7. Jentink J, Dolk H, Loane MA, et al; EUROCAT
Antiepileptic Study Working Group. 18. Werler MM, Ahrens KA, Bosco JL, et al;
Intrauterine exposure to carbamazepine National Birth Defects Prevention Study. Use
and specific congenital malformations: of antiepileptic medications in pregnancy in
systematic review and case-control study. relation to risks of birth defects. Ann
BMJ 2010;341:c6581. Epidemiol 2011;21(11):842Y850.
8. Jentink J, Loane MA, Dolk H, et al; EUROCAT 19. Dolk H, Jentink J, Loane M, et al; EUROCAT
Antiepileptic Study Working Group. Valproic Antiepileptic Drug Working Group. Does
acid monotherapy in pregnancy and major lamotrigine use in pregnancy increase

Continuum (Minneap Minn) 2014;20(1):60–79 www.ContinuumJournal.com 77

Copyright © American Academy of Neurology. Unauthorized reproduction of this article is prohibited.


Pregnancy and Epilepsy

orofacial cleft risk relative to other 32. Shallcross R, Bromley RL, Irwin B, et al;
malformations? Neurology 2008;71(10): Liverpool Manchester Neurodevelopment
714Y722. Group and the UK Epilepsy and Pregnancy
Register. Child development following in
20. Mawhinney E, Craig J, Morrow J, et al.
utero exposure: levetiracetam vs sodium
Levetiracetam in pregnancy: results from the
valproate. Neurology 2011;76(4):383Y389.
UK and Ireland epilepsy and pregnancy
registers. Neurology 2013;80(4):400Y405. 33. MONEAD. Maternal Outcomes and
21. Vajda FJ, O’Brien TJ, Lander CM, et al. Neurodevelopmental Effects of Antiepileptic
Teratogenesis in repeated pregnancies in Drugs. web.emmes.com/study/monead/
antiepileptic drug-treated women. Epilepsia index.htm. Accessed October 21, 2013.
2013;54(1):181Y186. 34. Bromley RL, Mawer G, Clayton-Smith J,
22. Kini U, Adab N, Vinten J, et al; Liverpool and Baker GA; Liverpool and Manchester
Manchester Neurodevelopmental Study Neurodevelopment Group. Autism spectrum
Group. Dysmorphic features: an important disorders following in utero exposure to
clue to the diagnosis and severity of fetal antiepileptic drugs. Neurology 2008;71(23):
anticonvulsant syndrome. Arch Dis Child 1923Y1924.
Fetal Neonatal Ed 2006;91(2):F90YF95. 35. Cohen MJ, Meador KJ, Browning N, et al.
23. Kini U. Chapter 14. In: Epilepsy in women. Fetal antiepileptic drug exposure: motor,
Harden CL, Thomas SV, Tomson T, eds. adaptive, and emotional/behavioral
West Sussex, UK; Willey-Blackwell, 2013; functioning at age 3 years. Epilepsy Behav
164Y169. 2011;22(2):240Y246.
24. Holmes LB, Coull BA, Dorfman J, 36. Christensen J, Grønborg TK, Sørensen MJ,
Rosenberger PB. The correlation of deficits et al. Prenatal valproate exposure and risk
in IQ with midface and digit hypoplasia in of autism spectrum disorders and childhood
children exposed in utero to anticonvulsant autism. JAMA 2013;309(16):1696Y1703.
drugs. J Pediatr 2005;146(1):118Y122.
37. Nei M, Daly S, Liporace J. A maternal
25. Reinisch JM, Sanders SA, Mortensen EL, complex partial seizure in labor can affect fetal
Rubin DB. In utero exposure to phenobarbital heart rate. Neurology 1998;51(3):904Y906.
and intelligence deficits in adult men. JAMA
38. Sahoo S, Klein P. Maternal complex partial
1995;274(19):1518Y1525.
seizure associated with fetal distress. Arch
26. Adab N, Kini U, Vinten J, et al. The longer Neurol 2005;62(8):1304Y1305.
term outcome of children born to mothers
39. Sibai BM. Diagnosis, prevention, and
with epilepsy. J Neurol Neurosurg Psychiatry
management of eclampsia. Obstet Gynecol
2004;75(11):1575Y1583.
2005;105(2):402Y410.
27. Vinten J, Adab N, Kini U, et al.
40. EURAP Study Group. Seizure control and
Neuropsychological effects of exposure
treatment in pregnancy: observations from
to anticonvulsant medication in utero.
the EURAP epilepsy pregnancy registry.
Neurology 2005;64(6):949Y954.
Neurology 2006;66(3):354Y360.
28. Gaily E, Kantola-Sorsa E, Hiilesmaa V, et al.
Normal intelligence in children with 41. Chen YH, Chiou HY, Lin HC, Lin HL. Affect of
seizures during gestation on pregnancy
prenatal exposure to carbamazepine.
outcomes in women with epilepsy. Arch
Neurology 2004;62(1):28Y32.
Neurol 2009;66(8):979Y984.
29. Meador KJ, Baker GA, Browning N, et al;
NEAD Study Group. Cognitive function at 3 42. Pennell PB, Klein AM, Browning N, et al;
NEAD Study Group. Differential effects of
years of age after fetal exposure to
antiepileptic drugs. N Engl J Med 2009; antiepileptic drugs on neonatal outcomes.
360(16):1597Y1605. Epilepsy Behav 2012;24(4):449Y456.

30. Meador KJ, Baker GA, Browning N, et al; 43. Thomas SV, Syam U, Devi JS. Predictors of
NEAD Study Group. Effects of fetal seizures during pregnancy in women with
antiepileptic drug exposure: outcomes at epilepsy. Epilepsia 2012;53(5):e85Ye88.
age 4.5 years. Neurology 2012;78(16): 44. Tomson T, Landmark CJ, Battino D.
1207Y1214. Antiepileptic drug treatment in pregnancy:
31. Meador KJ, Baker GA, Browning N, et al; changes in drug disposition and their clinical
NEAD Study Group. Fetal antiepileptic implications. Epilepsia 2013;54(3):405Y414.
drug exposure and cognitive outcomes at 45. Czeizel AE. Prevention of congenital
age 6 years (NEAD study): a prospective abnormalities by periconceptional
observational study. Lancet Neurol 2013; multivitamin supplementation. BMJ
12(3):244Y252. 1993;306(6893):1645Y1648.

78 www.ContinuumJournal.com February 2014

Copyright © American Academy of Neurology. Unauthorized reproduction of this article is prohibited.


46. Cheschier N; American College of Obstetricians antiepileptic therapy. J Neurol 2008;255(12):
and Gynecologists Committee on Practice 1926Y1931.
BulletinsVObstetrics. ACOG practice bulletin.
49. Borthen I, Eide MG, Veiby G, et al.
Neural tube defects. Number 44, July 2003.
(Replaces committee opinion number 252, Complications during pregnancy in women
March 2001). Int J Gynaecol Obstet 2003;83(1): with epilepsy: population-based cohort
study. BJOG 2009;116(13):1736Y1742.
123Y133.
47. Morrow JI, Hunt SJ, Russell AJ, et al. 50. Artama M, Gissler M, Malm H, Ritvanen A;
Folic acid use and major congenital Drug and Pregnancy Group. Effects of
malformations in offspring of women maternal epilepsy and antiepileptic drug use
with epilepsy: a prospective study from during pregnancy on perinatal health in
the UK Epilepsy and Pregnancy Register. offspring: nationwide, retrospective cohort
J Neurol Neurosurg Psychiatry 2009;80(5): study in Finland. Drug Saf 2013;36(5):359Y369.
506Y511. 51. Thomas SV, Tomson T, Harden CL.
48. Pittschieler S, Brezinka C, Jahn B, et al. Management of Epilepsy and Pregnancy.
Spontaneous abortion and the prophylactic In: Harden CL, Thomas SV, and Tomson T,
effect of folic acid supplementation eds. Epilepsy in women. West Sussex, UK:
in epileptic women undergoing Wiley-Blackwell, 2013;196Y200.

Continuum (Minneap Minn) 2014;20(1):60–79 www.ContinuumJournal.com 79

Copyright © American Academy of Neurology. Unauthorized reproduction of this article is prohibited.


Review Article

Cerebrovascular
Address correspondence to
Dr Steven K. Feske, Neurology
Department, Brigham and
Women’s Hospital, 75 Francis
Street, Boston, MA 02115,
sfeske@partners.org.
Relationship Disclosure:
Disorders Complicating
Dr Feske has received
royalties from Elsevier for
his role as editor of Office
Pregnancy
Practice of Neurology, 2nd
Edition, and receives Steven K. Feske, MD; Aneesh B. Singhal, MD
research support from the
National Institute of
Neurological Disorders and
Stroke. Dr Singhal has
ABSTRACT
served as a consultant for Purpose of Review: This article discusses the physiologic changes of pregnancy and how
Biogen Idec and as a medical they affect risk of ischemic and hemorrhagic stroke and then reviews epidemiology, diagnosis,
expert witness in cases of
stroke. Dr Singhal’s spouse and treatment of ischemic and hemorrhagic stroke in pregnancy and the puerperium.
holds stock or stock options Recent Findings: This article updates our understanding of the relationship of
greater than 5% of the preeclampsia/eclampsia to the posterior reversible encephalopathy syndrome and the
company or greater than
$10,000 in value in reversible cerebral vasoconstriction syndrome, emphasizing their shared pathogenesis. It
Biogen Idec and Vertex reviews the most recent data and offers recommendations concerning the use of
Pharmaceuticals Incorporated. thrombolytic and other revascularization therapies for pregnancy-related strokes.
Dr Singhal has received
research support from the Summary: Although cerebrovascular complications are uncommon occurrences during
National Institute of pregnancy and the puerperium, stroke is still the most common seriously disabling
Neurological Disorders and complication of pregnancy. Therefore, stroke and other vascular issues raise questions
Stroke, and his institution has
received research support about the best evaluation and management that is safe for mother and child.
from Pfizer Inc and
PhotoThera, Inc, for clinical
Continuum (Minneap Minn) 2014;20(1):80–99.
trial participation.
Unlabeled Use of
Products/Investigational
Use Disclosure:
Drs Feske and Singhal report PHYSIOLOGY AND ume, and heart rate increase 30% to
no disclosures. PATHOPHYSIOLOGY OF 50% as a result of the increased
* 2014, American Academy PREGNANCY IN RELATION demand of the developing fetus and
of Neurology.
TO VASCULAR DISEASE placenta and maternal hypervolemia.
Hemodynamic Changes This begins as early as the fifth
Pregnancy is a state of high metabolic gestational week and reaches a pla-
demand. The cardiovascular changes teau in the late second or third
of pregnancy prepare the maternal trimester. Heart rate continues to rise
circulation to meet that demand. Es- until term. Increase in prostacyclin
trogen and other hormones cause an and redistribution of high flow in the
increase in renin activity, leading to low-resistance uteroplacental circula-
retention of sodium and water. This tion and breasts and kidneys cause
supports an increase in plasma vol- systemic vascular resistance (SVR) to
ume beginning in the first trimester begin to fall around the fifth gesta-
around 6 weeks of gestation. Some tional week. This drop in SVR is
studies suggest that this increase in accompanied by a fall in the systolic
plasma volume reaches a plateau in and diastolic blood pressure. It
the third trimester. Others suggest a reaches a nadir in the third or late
progressive increase until term.1 Red second trimester, about 20 to 32
blood cell mass also increases, but weeks, after which it rises until term
proportionally less, resulting in a mild to levels at or slightly above the
physiologic hemodilutional anemia of normal nonpregnant blood pressure.
pregnancy. Cardiac output, stroke vol- There is increased venous capacitance

80 www.ContinuumJournal.com February 2014

Copyright © American Academy of Neurology. Unauthorized reproduction of this article is prohibited.


KEY POINTS
and reduced blood flow (ie, relative and lysis are altered by pregnancy h Pregnancy results in
venous stasis), often accompanied by resulting in a state of increased increased metabolic
orthostatic intolerance. The volume hypercoagulability in late pregnancy. demand, sodium and
increase tends to increase preload. Levels of procoagulant factors I, VII, water retention, and
However, preload is heavily depen- VIII, IX, X, XII, and XIII increase decrease in systemic
dent upon position, with reduced during pregnancy. There is little vascular resistance,
preload and orthostasis resulting from change in levels of factors II, V, and leading to expansion of
the higher venous capacitance and XI. The levels of some coagulation plasma volume; mild
from compression on the inferior vena inhibitors fall during pregnancy. The anemia; increased
cava in the supine position. coagulation inhibitor antithrombin III stroke volume, heart
rate, and cardiac
falls and is at its nadir in the third
Vascular and Connective output; and decreased
trimester. Total and free levels of the
systolic and diastolic
Tissue Changes coagulation inhibitor cofactor protein blood pressures.
Pregnancy causes remodeling of the S are significantly decreased as well.
heart and all blood vessels. The walls Although levels of protein C remain h Changes in vascular
structure and the
of systemic arteries show a reduction unchanged, almost a third of women
coagulation system,
in collagen and elastin content and a have functional activated protein C resis- although adaptive,
loss of distensibility. Animal models tance during the third trimester. These also lead to a relative
have shown increased contractile changes of venous flow and the molec- vulnerability to
force, decreased stiffness, and in- ular mediators of thrombosis are greatest hemorrhage and
creased relaxation response in the during the late third trimester and early ischemic stroke,
middle cerebral artery.2 The roles of puerperium. Along with iron deficiency especially during the
the molecular promoters underlying and the acute phase responses of the postpartum period.
these physiologic responses to preg- trauma and hemorrhage of delivery, they
nancy and the full effects of these create the greatest hypercoagulability
adaptive changes are complex and during the early puerperium.
poorly understood. Their relationship
to stroke is not certain; however, one Preeclampsia/Eclampsia
might expect the hemodynamic Preeclampsia is a multisystem disorder
changes near term combined with of mid- to late pregnancy traditionally
these structural changes of blood characterized by pregnancy-induced
vessels to result in a state in which hypertension, edema, and proteinuria;
more vulnerable vascular walls experi- eclampsia is the development of sei-
ence greater hemodynamic stress, zures in a patient with preeclampsia.
possibly contributing to the risk of Preeclampsia and eclampsia can infre-
hemorrhage in late pregnancy. quently occur after childbirth, usually
within 48 hours (postpartum pre-
Changes in the eclampsia), or up to 4 to 6 weeks
Coagulation System after childbirth (late postpartum pre-
Both hemodynamic and biochemical eclampsia).3 Postpartum preeclampsia
changes make pregnancy a state of can manifest de novo postpartum or
hypercoagulability. Decreased venous result from preeclampsia, preexisting
compliance results in venous stasis chronic hypertension, or persistent
and congestion. Compression of the gestational hypertension (systolic
inferior vena cava, aorta, and blood blood pressure greater than 140 mm
supply of the gravid uterus can cause Hg or diastolic blood pressure greater
vascular injury. Levels of procoagulant than 90 mm Hg without proteinuria).
factors, coagulation inhibitors, and The criteria for diagnosis depend on
other mediators of clot formation the definition of the clinical syndrome.
Continuum (Minneap Minn) 2014;20(1):80–99 www.ContinuumJournal.com 81

Copyright © American Academy of Neurology. Unauthorized reproduction of this article is prohibited.


Cerebrovascular Disorders Complicating Pregnancy

KEY POINT
h Preeclampsia/eclampsia Modern consensus definitions all en- relevant clinical complications of pre-
is a state of dorse pregnancy-induced hypertension eclampsia are endothelial dysfunc-
hypertension, beginning after the 20th week with tion, absence of the normal stimulation
endothelial and proteinuria as preeclampsia, and gener- of the renin-angiotensin system despite
platelet dysfunction, ally accept the inclusion of women with hypovolemia, hypersensitivity to angio-
and enhanced pregnancy-induced hypertension with- tensin II leading to increased systemic
coagulability with out proteinuria but with other common vascular resistance and hypertension,
many pathologic manifestations, including (1) cerebral augmentation of the normal thrombox-
consequences. symptoms, (2) epigastric or right upper ane A2/prostacyclin ratio, increased
quadrant pain with nausea or vomiting, platelet activation, and increased throm-
or (3) thrombocytopenia and abnormal bin formation and fibrin generation.
liver enzymes.4 It has been claimed that, Endothelial dysfunction contributes to
although pregnancy-induced hyperten- increased capillary permeability that un-
sion is the cornerstone of clinical derlies proteinuria, edema, and, most
diagnosis, a significant proportion of important among our concerns, brain
women who develop hemolysis, ele- edema at relatively modest elevations
vated liver enzymes, and low platelet of blood pressure.
count (HELLP) syndrome or eclamptic
seizures do not have hypertension.5,6 ISCHEMIC STROKE ASSOCIATED
Since the precise diagnosis of pre- WITH PREGNANCY
eclampsia is currently limited by the Epidemiology
lack of a specific biomarker, it is often No long-term prospective studies of
desirable to entertain a permissively the incidence and types of pregnancy-
broad definition for use in clinical associated stroke have yet been done.
practice. Although much has been Available studies handle spontaneous
learned about preeclampsia over the and therapeutic abortions and still-
last decade, a clear understanding of births differently and use different
the relationship of the many underlying definitions of the puerperium and
risk factors and various features of different methods of stroke classifica-
abnormal physiology is still lacking. tion. This lack of consistency of data
There is strong evidence that immune limits comparisons, but we can esti-
maladaptation is central to its cause. mate the impact of stroke from these
While abnormal placentation and pla- studies. Table 4-18 summarizes nine
cental hypoperfusion play an important major studies published since
role, hypoperfusion does not seem to be 1985.9Y17 From the three population-
primary, because markers of preeclamp- based studies, we can estimate the
sia are present in the first trimester, incidence of stroke. In these three
before placental hypoperfusion.7 Al- studies, the incidence of all types of
though the triggers for the abnormal stroke ranges from four to seven cases
placentation are not clearly understood, per 100,000 pregnancies. However,
there is strong evidence for dysregula- data from the National Inpatient
tion of angiogenic and vasoactive factors, Sample of the Healthcare Cost and
such as vascular endothelial growth Utilization Project suggest that the
factor (VEGF) and placental growth incidence of pregnancy-associated
factor (PlGF) and nitric oxide and, stroke has risen since the 1990s. These
ultimately, antagonism of these factors data estimate the incidence of all types
by binding to soluble VEGF receptor-1 of pregnancy-associated stroke, in-
(also known as Fms-like tyrosine kinase cluding ischemic and hemorrhagic
1 [sFlt1]). Fundamental to many of the strokes, subarachnoid hemorrhage,
82 www.ContinuumJournal.com February 2014

Copyright © American Academy of Neurology. Unauthorized reproduction of this article is prohibited.


and cerebral venous sinus thrombosis, (from 0.12 to 0.22 per 1000 deliveries),
to be 25 to 34 per 100,000 deliveries. with the increases largely explained by
This analysis compares data from 1994 concurrent hypertensive disorders or
and 1995 to data from 2006 and 2007. heart disease.18 In comparison, the
Between these intervals, antenatal hos- incidence of stroke in nonpregnant
pitalizations increased by 47% (from women in the 15- to 45-year age group
0.15 to 0.22 per 1000 deliveries), and is 11 per 100,000.19 Kittner found
postpartum hospitalizations by 83% no increased risk of ischemic stroke

a
TABLE 4-1 Studies of Stroke in Pregnancy

Time Number of Number of Strokes


Study Location Methods Interval Pregnancies Ischemic Hemorrhagic
Feske et al, Boston Retrospective 1996Y2005 101,570 17 36
2009 hospital
Postpartum
period
(PP) = 3 months
Liang et al, Taiwan Retrospective 1992Y2004 66,781 11 21
2006 hospital
PP = 6 weeks

Jeng et al, Taiwan Retrospective 1984Y2002 Data not 27 22


2004 hospital reported
15Y40-year-olds
PP = 6 weeks
Jaigobin et al, Toronto Retrospective 1980Y1997 50,711 21 13
2000 hospital
PP = 6 weeks

Witlin et al, Memphis Retrospective 1985Y1995 79,301 14 6


1997 hospital
PP = not
reported
Kittner et al, Maryland and Retrospective 1988Y1991 234,023 17 14
1996 Washington DC
(46 hospitals) Population-
based
PP = 6 weeks
Sharshar et al, Ile de France Retrospective 1989Y1991 348,295 15 16
1995 (63 hospitals)
Population-
based
PP = 2 weeks
Awada et al, Saudi Arabia Retrospective 1983Y1993 Data not 9 3
1995 hospital reported
pp = 15 days
Wiebers et al, Rochester, MN Retrospective 1955Y1979 26,099 1 0
1985 hospitals (live births)
Population-
based
PP not included
a
Modified with permission from Feske SK, Semin Neurol.8 B 2007, Thieme Medical Publishers. www.thieme-connect.com/DOI/
DOI?10.1055/s-2007-991126.

Continuum (Minneap Minn) 2014;20(1):80–99 www.ContinuumJournal.com 83

Copyright © American Academy of Neurology. Unauthorized reproduction of this article is prohibited.


Cerebrovascular Disorders Complicating Pregnancy

KEY POINTS
h The risk of ischemic during pregnancy (relative risk 0.7) but risk. In studies in which preeclampsia/
stroke is increased a large relative increase during the eclampsia is reported, it is present in
during the postpartum 6-week postpartum period (relative 11% to 47% of cases of stroke. The
period. risk 8.7). Figure 4-1 shows a similar contribution of preeclampsia/eclampsia
h Although preponderance of events in the post- to the cause of strokes is presumed to
pregnancy-associated partum period in the authors’ study. The be complex and related to the various
ischemic stroke is rare, it higher proportions of strokes in the vasculopathic and prothrombotic ef-
is a major contributor to other studies are probably due to referral fects discussed above. Other well-
long-term disability bias, since these represent series from established causes of stroke in young
resulting from single large referral hospitals. Despite its patients, such as arterial dissection
pregnancy. low overall incidence, stroke contributes and moyamoya syndrome, can present
h Cardioembolism, a major proportion of the long-term during pregnancy and should be consid-
preeclampsia/eclampsia, disability resulting from pregnancy. ered (Case 4-1). Other pregnancy-
and cerebral venous specific causes, such as peripartum
sinus thrombosis Mechanisms cardiomyopathy, choriocarcinoma,
account for most Although differences and limitations and embolization of amniotic fluid
pregnancy-related in methods of case assessment com- or air, are very rare and should be
ischemic stroke. promise interpretations, the ischemic considered based on the clinical pre-
strokes can be broken down by types sentations. Amniotic fluid embolism
to assess the mechanisms and con- should be considered when evidence of
tributing causes of pregnancy-related diffuse or multifocal brain ischemia is
strokes (Table 4-28). Here, the au- present and accompanied by features of
thors have classified venous sinus throm- pulmonary embolism.
bosis with ischemic-thrombotic stroke,
although many will have components of
hemorrhage. When mechanisms of ische- Evaluation
mic stroke are identified, the major ones Stroke is suspected clinically based on
are cardioembolism and venous sinus the sudden onset of a neurologic
thrombosis. Preeclampsia/eclampsia ap- deficit suggestive of a focal lesion
pears to be a major contributor to stroke and without an alternative cause.

FIGURE 4-1 Timing of events during pregnancy and the puerperium. Each dot represents
the time during pregnancy (in weeks) or the puerperium (in days) of a single
event as color-coded.

Reprinted with permission from Feske SK, et al, International Stroke Conference.17

84 www.ContinuumJournal.com February 2014

Copyright © American Academy of Neurology. Unauthorized reproduction of this article is prohibited.


a,b,c
TABLE 4-2 Causes of Ischemic Strokes in Pregnancy (% of All Ischemic Strokes)

Preeclampsia/
Eclampsia Venous Sinus
Study Cardioembolism Angiopathy Thrombosis Unknown Other
Feske et al, 35 30 39 I 22
2009
Liang et al, 36 18 27 I I
2006
Jeng et al, 44 I 22 22 I
2004
Jaigobin et al, 20 20 40 20 15
2000
Witlin et al, I I 64 I I
1997
Kittner et al, I 38d 6 38 19
1996
Sharshar et al, I 54d I 27 20
1995
Awada et al, 33 11 I 44 11
1995
a
Modified with permission from Feske SK, Semin Neurol.8 B 2007, Thieme Medical Publishers. www.thieme-connect.com/DOI/
DOI?10.1055/s-2007-991126.
b
Totals of the rows may exceed 100% because multiple contributing causes may be counted.
c
Ellipses indicate that data was not reported.
d
These authors reported preeclampsia/eclampsia-associated and CNS angiopathy separately (see text).

Properly timed neuroimaging will con- infarctions, and should include agitated
firm the great majority of ischemic saline injection (bubble study) to investi-
strokes. Concerns arise when planning gate for a right-to-left shunt from a patent
neuroimaging during pregnancy. foramen ovale, the presence of which
These are discussed in detail in the may suggest paradoxical embolism and
article ‘‘Neuroradiology in Women of warrant further testing for lower extrem-
Childbearing Age’’ by Drs Riley Bove ity or pelvic deep vein thrombus.22 As
and Joshua Klein in this issue of with any patient with stroke, blood tests
. In general, with prop- such as lipid panel, hemoglobin A1C,
er precautions, CT and MRI should be erythrocyte sedimentation rate, C-
used as with nonpregnant patients to reactive protein, and others should be
identify areas of infarction and to routinely performed, with additional tests
investigate the cerebral vasculature. such as hemoglobin electrophoresis for
Cerebral vessel imaging with magnetic sickle cell disease or antiphospholipid
resonance (MR) angiography, CT antibody panel performed on a case by
angiography, or transfemoral catheter case basis. Genetic testing for throm-
angiography is indicated to assess bophilia (prothrombin G20210A muta-
for cerebral arterial dissection, revers- tion, factor V Leiden mutation,
ible cerebral vasoconstriction syn- methylenetetrahydrofolate reductase
drome, moyamoya disease, or other mutation) can be performed during
arteriopathies. Cardiac ultrasound should pregnancy, but testing for other hyper-
be performed in patients with embolic coagulable states (protein C, protein S,

Continuum (Minneap Minn) 2014;20(1):80–99 www.ContinuumJournal.com 85

Copyright © American Academy of Neurology. Unauthorized reproduction of this article is prohibited.


Cerebrovascular Disorders Complicating Pregnancy

Case 4-1
A 20-year-old woman developed right hemichorea during the second
trimester of her first pregnancy. Brain imaging showed subcortical infarctions
predominantly in the left hemisphere and severe stenosis of the right and left
middle cerebral arteries (Figure 4-2). No headache or fever or segmental arterial
narrowing or other evidence of cerebral arteritis or infection was present.
Hemoglobin electrophoresis was normal, ruling out sickle cell anemia.

FIGURE 4-2 Brain imaging of a 20-year-old primigravida with right hemichorea. Note the
subcortical infarctions predominantly in the left hemisphere (A, fluid-attenuated
inversion recovery [FLAIR] MRI) and severe stenosis of the right and left middle
cerebral arteries (B, head magnetic resonance [MR] angiogram,
three-dimensional time-of-flight image).

Comment. This patient was diagnosed with moyamoya disease,


an idiopathic noninflammatory cerebral arteriopathy. The choreiform
movements resolved after a short course of steroids. The patient declined
the option of surgical intervention with the encephaloduroarteriosynangiosis
procedure, which has been shown to reduce the risk for future stroke.20,21
She was treated with aspirin 325 mg/d for stroke prevention. She went on to
have three vaginal deliveries without further neurologic symptoms such as
headache, chorea, or weakness. Follow-up brain imaging studies showed no
evidence for new stroke.

and antithrombin III deficiency) should Administration (FDA)Yapproved


be performed at least 6 weeks after thrombolytic drug that remains the
delivery when physiologic changes due only proven efficacious therapy for
to pregnancy will have normalized. acute ischemic stroke.23,24 Many large
stroke centers offer intra-arterial throm-
Therapies and Outcomes bolysis as a salvage therapy in severe
IV recombinant tissue plasminogen stroke cases. Intra-arterial thromboly-
activator (tPA) is a US Food and Drug sis typically involves mechanical clot

86 www.ContinuumJournal.com February 2014

Copyright © American Academy of Neurology. Unauthorized reproduction of this article is prohibited.


retrieval using FDA-approved devices. literature of six women who have
Recent phase III clinical trials showing received IV tPA for stroke while preg-
no benefit of intra-arterial over IV nant, although many more have been
thrombolysis25 will prompt a treated as this has become accepted
reassessment of the role of catheter- practice.26Y31 Of the six women who
based therapies, but evidence exists received IV tPA, three suffered no
that improved outcomes depend hemorrhagic complications, one had
on early recanalization of occluded minor hemorrhagic transformation of
arteries, and FDA-approved devices the cerebral infarct, and one had an
will continue to find use as research intrauterine hematoma. Of these six
proceeds on this issue. Because preg- cases, no fetal complications occurred
nant women were excluded from all in three, and in two cases the preg-
of these trials, there has been no nancy was terminated allowing no
controlled study of the use of such further analysis. The sixth patient and
agents in pregnancy. Although there fetus died as a result not of a direct
are case reports of successful IV and effect of the systemic tPA, but from
intra-arterial thrombolytic use in arterial dissection complicating angio-
pregnant women,26Y31 questions for plasty. Of five women treated with
clinicians remain: Should these intra-arterial thrombolysis for acute
therapies be applied in pregnant arterial occlusion (three tPA, two uro-
women suffering acute ischemic kinase), none had serious complica-
strokes? Are they safe and effective tions from the procedure; two had
for the mother? Are they safe for the hemorrhagic transformation of the
fetus? stroke with good neurologic out-
IV tPA has a very short serum half- comes, and one had a minor buttock
life of less than 5 minutes. However, it hematoma.26,29,32 Four of the five
binds to newly formed fibrin clots, women delivered healthy babies; one
where its lytic effect lasts for many pregnancy, in which the stroke
hours. It is a large molecule that does resulted from bacterial endocarditis,
not cross the placenta in animals, and ended in spontaneous abortion. It
so it should not be expected to place should be noted that urokinase, un-
the fetus at risk of teratogenicity. The like tPA, does cross the placenta.
potential risks of real concern are Minor hemorrhagic transformation is
maternal hemorrhage, placental hem- common after thrombolysis in gener-
orrhage and abruption, fetal loss, and al, and it does not appear to worsen
preterm delivery. Although there are outcomes. In fact, it has been associ-
theoretical reasons to question com- ated with better outcomes, possibly
parisons to its use in nonpregnant because it is a marker of early recan-
patients, mainly that pregnancy is a alization. To summarize, of these 11
state of relative hypercoagulability women who received IV or intra-
characterized by decreased intrinsic arterial thrombolysis and were
tPA and increased plasminogen activa- reported in the literature, 10 had no
tor inhibitor, the ultimate effects of major complications from thromboly-
these changes on the clinical efficacy sis, and the patient who died had a
of tPA are speculative and unlikely to major complicating illness; of the 11
be answered by clinical trials. There- reported fetal outcomes, seven were
fore, more empirical clinical data must delivered without complications, two
be used to estimate the risk. The were terminated therapeutically, one
authors have found reports in the patient with bacterial endocarditis had
Continuum (Minneap Minn) 2014;20(1):80–99 www.ContinuumJournal.com 87

Copyright © American Academy of Neurology. Unauthorized reproduction of this article is prohibited.


Cerebrovascular Disorders Complicating Pregnancy

KEY POINT
h Although data on the a spontaneous abortion, and one fetus chemic strokes in pregnancy. Second,
use of thrombolytic died along with the mother. one direct consequence of preeclampsia/
therapies during With such limited and uncontrolled eclampsia is the posterior reversible
pregnancy are scarce, data, it is not possible to draw firm encephalopathy syndrome (PRES), a
limited experience conclusions; however, it is reasonable form of the syndrome of hypertensive
suggests that these to judge that pregnancy does not encephalopathy characterized by re-
agents can be given appear to present a decisive added versible brain edema, often associated
with safety comparable risk to thrombolytic therapy. There- with elevated blood pressure, seizures,
to that in nonpregnant fore, thrombolysis should be consid- brain hemorrhage, and ischemic
patients. ered for all potentially disabling strokes.
strokes during pregnancy, and it
should not be excluded based on the Preeclampsia/Eclampsia
fact of pregnancy alone. As in all and Posterior Reversible
patients, the details of the case should Encephalopathy Syndrome
be carefully weighed, and patients or Pathophysiologic mechanisms. The
proxy decision makers should be well pathophysiology of preeclampsia/
informed of risks. Obstetric consulta- eclampsia is discussed briefly above.
tion should be sought from the outset The features of preeclampsia/eclampsia
for careful monitoring and decision directly relevant to PRES and probably
making. Care in facilities with experi- also to eclamptic seizures are (1) an
ence both in advanced stroke care and abnormal increase in vascular tone and
high-risk obstetrics is optimal, and the (2) dysfunction of endothelial cells.
cause and mechanism of the stroke Patients with preeclampsia/eclampsia
should be carefully determined to have heightened sensitivity to media-
the extent possible before therapy is tors of vasoconstriction, such as angio-
prescribed. For example, women with tensin II. This resultant increase in
stroke as a result of amniotic fluid vascular tone is responsible for system-
embolism would not benefit from ic hypertension and for the vasomotor
tPA. Additionally, given the known risk instability that underlies vasospasm.
of cerebral hemorrhage from hyper- Endothelial dysfunction is in part re-
tensive encephalopathy in the setting sponsible for the instability of vascular
of preeclampsia/eclampsia, patients tone, and it also results in increased
who have strokes complicating pre- vascular permeability that underlies the
eclampsia or eclampsia should not development of edema and protein-
receive tPA. uria that characterize preeclampsia/
eclampsia. The syndrome commonly
PREECLAMPSIA/ECLAMPSIA, called PRES results from the develop-
HYPERTENSIVE ENCEPHALOPATHY, ment of cerebral edema. Fluid crosses
AND POSTPARTUM CEREBRAL from the intravascular to the interstitial
ANGIOPATHY space as a result both of an increase in
capillary filtration pressure caused by
Overview hypertension and of loss of integrity
Preeclampsia/eclampsia contributes to of the blood-brain barrier caused
cerebrovascular events in two major by endothelial dysfunction. Animal
ways. First, as noted above, preeclampsia/ models of the blood-brain barrier’s
eclampsia causes many pathophysio- response to severe acute hypertension
logic changes in blood vessels and have shown both increased pinocytosis
the thrombotic system and in this way and flow across impaired endothelial
accounts for a large proportion of is- gap junctions.33
88 www.ContinuumJournal.com February 2014

Copyright © American Academy of Neurology. Unauthorized reproduction of this article is prohibited.


KEY POINTS
Postpartum Cerebral our understanding of this disorder is h Preeclampsia/eclampsia
Angiopathy well developed. For example, there can lead to several
Postpartum angiopathy is one of several may be important pathophysiologic cerebrovascular
conditions included in the spectrum of differences in classic preeclampsia/ syndromes, including
the reversible cerebral vasoconstriction eclampsia and postpartum syndromes posterior reversible
syndromes (RCVS).34 Postpartum an- that lack proteinuria or even hyper- encephalopathy
giopathy is characterized by severe tension.4 Nonetheless, it has been syndrome, reversible
headaches and reversible narrowing of recognized in recent years that the cerebral vasoconstriction
basic preeclampsia/eclampsia patho- syndrome, and ischemic
intracerebral arteries, often com-
physiology may account for these late and hemorrhagic
plicated by seizures, reversible brain
strokes.
edema, lobar hemorrhage, convexity pregnancy and postpartum syn-
(nonaneurysmal) subarachnoid hemor- dromes when they do not fit the h The major CNS
traditional definition of preeclampsia/ complications of
rhage, and ischemic strokes. In the
eclampsia with proteinuria and when preeclampsia/eclampsia
past, patients with this syndrome were
are a form of
often misinterpreted as having inflam- they develop up to many weeks after
hypertensive
matory cerebral vasculitis because the delivery.4,38
encephalopathy.
latter can also manifest with headache, Eclamptic hypertensive encepha- They should be treated
stroke, and cerebral angiographic ab- lopathy (ie, PRES) typically presents aggressively with
normalities; however, postpartum with headache, visual symptoms refer- rapid control of blood
angiopathy is a noninflammatory, vaso- able to the occipital lobes, and sei- pressure and IV
constrictive condition. Approximately zures. Eclamptic RCVS presents with magnesium sulfate.
one-third of patients with postpartum thunderclap headache, seizures, and
angiopathy are noted to have revers- focal neurologic deficits. Imaging typ-
ible cerebral edema and clinical fea- ically shows posterior white and often
tures (headaches, seizures) which are gray matter change consistent with
very similar to patients with PRES, and vasogenic cerebral edema (hypodense
more than half the patients with PRES on CT and hyperintense on T2-
show evidence of cerebral artery nar- weighted MRI), the findings typical of
rowing on vascular imaging.35 Hence, PRES, or segmental narrowing and
postpartum angiopathy and PRES are dilation of large and medium-sized
considered overlapping conditions.36 cerebral arteries, the findings typical
Case 4-2 illustrates this overlap in a of RCVS. Both or either of these
single patient. The pathophysiologic imaging patterns may be seen, and
mechanisms whereby preeclampsia/ these imaging findings have limited
eclampsia is related to PRES are prob- sensitivity, so imaging may be un-
ably also applicable to postpartum revealing in otherwise clinically con-
angiopathy. vincing cases. Although most lesions
are limited to edema and are there-
Evaluation fore reversible, hemorrhages and focal
The syndromes mentioned aboveV ischemic strokes may also occur.
variously called eclampsia, hyperten-
sive encephalopathy, PRES, RCVS, and Therapies and Outcomes
postpartum angiopathyVcan then be The goals of therapy are to control
considered as various presentations of elevated blood pressure, control sei-
a similar fundamental underlying path- zures, and minimize vasospasm and risk
ophysiology. This clinical lumping is of secondary infarct and hemorrhage.
not meant to oversimplify the com- Because the authors interpret these
plex pathophysiology of preeclampsia/ syndromes in association with preg-
eclampsia, nor to make the claim that nancy as manifestation of eclampsia,
Continuum (Minneap Minn) 2014;20(1):80–99 www.ContinuumJournal.com 89

Copyright © American Academy of Neurology. Unauthorized reproduction of this article is prohibited.


Cerebrovascular Disorders Complicating Pregnancy

Case 4-2
A 36-year-old woman developed severe headaches associated with new-onset hypertension 10 days
after delivery of twins by cesarean delivery. The initial brain MRI and CT examinations were normal.
Headaches persisted despite antihypertensive medications. A seizure and an episode of aphasia and
hemiparesis occurred. Repeat MRI on day 18 (Figure 4-3A37) showed hyperintense regions in both
parietalYoccipital lobes with elevated diffusionVfindings consistent with vasogenic edema. These
clinical-imaging features are consistent with the posterior reversible encephalopathy syndrome (PRES).
Magnetic resonance (MR) angiography of the circle of Willis showed multifocal stenoses in the

FIGURE 4-3 Brain imaging of a 36-year-old woman with severe headaches associated with postpartum hypertension.
A, fluid-attenuated inversion recovery (FLAIR) image shows hyperintense regions in both parietal and occipital
lobes (arrows) with elevated diffusion (not shown), findings that are consistent with vasogenic edema.
B, Magnetic resonance (MR) angiography of the circle of Willis shows multifocal stenoses in the proximal anterior, middle, and
posterior cerebral arteries. This finding is consistent with postpartum angiopathy. C and D show hyperintense lesions (arrows)
on FLAIR and diffusion-weighted images, respectively, from MRI performed 1 day later, a finding consistent with ischemic
stroke. E, a follow-up MR angiogram shows worsening of the multifocal cerebral arterial stenosis. F, FLAIR shows bilateral
cerebral infarction with edema and hemorrhage.

Reprinted with permission from Singhal AB et al, N Engl J Med.37 Copyright B 2009, Massachusetts Medical Society. www.nejm.org/doi/full/10.1056/NEJMcpc0809063.

Continued on page 91

90 www.ContinuumJournal.com February 2014

Copyright © American Academy of Neurology. Unauthorized reproduction of this article is prohibited.


Continued from page 90
proximal anterior, middle, and posterior cerebral arteries. This finding is consistent with
postpartum angiopathy. MRI performed on postpartum day 19 showed hyperintense lesions on FLAIR
and diffusion-weighted images, a finding consistent with ischemic stroke. Despite multiple attempts to
dilate the cerebral arteries with intracerebral vasodilator injections, the patient showed clinical and
angiographic progression over the course of 1 week. A follow-up MR angiogram showed worsening of
the multifocal cerebral arterial stenosis, and FLAIR showed bilateral cerebral infarction with edema and
hemorrhage. The patient eventually died. On autopsy, the cerebral arteries were normal, with no
evidence of inflammation.
Comment. This is a classic example of postpartum eclampsia with postpartum angiopathy and
features of posterior reversible encephalopathy and reversible cerebral vasoconstriction syndromes,
illustrating that these causes of postnatal ischemic and hemorrhagic stroke are interrelated
conditions. These syndromes are difficult to predict or prevent. Considering these as manifestations
of preeclampsia/eclampsia, the authors treat with magnesium sulfate, based on the clinical trials
discussed in the text. It is also important to control blood pressure when elevated, as with other forms
of hypertensive encephalopathy. No treatment has proven efficacy for the cerebral artery narrowing
of postpartum angiopathy. Calcium channel blockers are a reasonable, if untested, choice. While
90% of patients have a self-limited course and recover within days to weeks, some patients
(as in this example) may have a progressive course and even a fatal outcome.

treatment with IV magnesium sulfate and maintenance. Although many prac-


along with other therapies directed at titioners have used calcium channel
blood pressure and seizures is blockers and glucocorticoids in these
recommended. Several studies have patients, this use is not supported by
shown the superiority of magnesium clinical data. No evidence has proven
sulfate over commonly used anticon- that calcium channel blockers are more
vulsants for prevention and treatment effective than other antihypertensive
of eclamptic seizures.39Y41 The authors agents. In a study of 139 patients with
give a loading dose of 4 g to 6 g of RCVS, of which 12 were postpartum,
magnesium sulfate over 20 to 30 corticosteroids were associated with a
minutes followed by continuous infu- trend toward poorer outcomes.34 This
sion at 2 g per hour with an additional series is weighted toward the RCVS
2 g bolus if seizures occur during this presentation, so it is not representative
therapy. Patients should be monitored of eclampsia in general. A third of the
closely and magnesium sulfate patients in this series presented with
stopped if deep tendon reflexes are seizures, and a third suffered ischemic
lost, if respirations are depressed, or if strokes. Ninety percent had good out-
urine output falls below 100 mL in 4 comes (Modified Rankin Scale 0 to
hours. Calcium gluconate 1 g slow IV 3).34 Most patients with pregnancy-
push can be given to reverse severe associated PRES and RCVS have a self-
toxicity. In addition, patients with limited clinical course with benign
eclamptic syndromes should be treated outcome and resolution of brain and
urgently with IV antihypertensive vascular imaging abnormalities within
agents and with additional antiepileptic days to weeks; however, 5% to 12%
agents if needed for seizure control. can have a fulminant course with prog-
Most women with preeclampsia/ ressive vasoconstriction, brain edema,
eclampsia are volume contracted and and strokes, culminating in persistent
will benefit from volume replacement severe neurologic deficits or death.37,42

Continuum (Minneap Minn) 2014;20(1):80–99 www.ContinuumJournal.com 91

Copyright © American Academy of Neurology. Unauthorized reproduction of this article is prohibited.


Cerebrovascular Disorders Complicating Pregnancy

KEY POINT
CEREBRAL VENOUS SINUS hemorrhage of labor and delivery may
h Cerebral venous
thrombosis, especially
THROMBOSIS contribute to the propensity for abnor-
postpartum, is one mal thrombosis. This timing of risk is
of the most common
Epidemiology comparable to lower extremity deep
cerebrovascular This review includes the cases of venous thrombosis in pregnancy.43
complications of cerebral venous thrombosis among Although often called ‘‘venous infarc-
pregnancy. those of arterial ischemic infarction tion,’’ with large collecting sinus
above in discussing the rate of ische- thrombosis, the brain lesions typically
mic stroke in pregnancy because it is begin as areas of brain edema without
often not clearly distinguished from infarction as a result of impaired venous
arterial stroke in large series of preg- drainage and increased venous pres-
nancy and stroke. However, cerebral sures. Ultimately, stasis of flow may
venous sinus thrombosis is a disorder cause these lesions to progress to in-
very different from arterial occlusion clude areas of infarction and hemor-
with different pathophysiology, ther- rhage. In addition, hemorrhage may
apy, and outcomes. Cerebral venous extend to other compartments, includ-
thrombosis accounts for 6% to 64% of ing the subarachnoid, subdural, and
all pregnancy-associated strokes in intraventricular spaces. Because the pri-
large reported series and 17% in the mary process is edema, much of the
authors’ series.16 Venous thrombosis visualized lesion (hypodensity on CT or
may present with imaging findings of hyperintensity on T2-weighted MR) is
thrombus within a cerebral vein or reversible with treatment, and outcomes
venous sinus without parenchymal are typically very good, much better than
changes or with evidence of cerebral for comparable-sized arterial strokes.
edema, apparent ischemic stroke, or
hemorrhage, and as a result, this Evaluation
disorder is classified differently by Women with cerebral venous throm-
different authors. bosis may present with headaches,
focal neurologic deficits, depressed
Mechanisms level of consciousness, or seizures,
Thrombosis in the venous circulation, and the pregnant state should greatly
including the cerebral venous sinuses heighten the index of suspicion for
and veins, is presumed to be the this diagnosis. Cerebral venous sinus
outcome of the underlying hyper- thrombosis can be detected on
coagulable state of pregnancy, promoted noncontrast CT as hyperdensity in
by the various pathophysiologic the region of thrombosis or as paren-
changes of pregnancy described above. chymal hypodensity from edema or
These effects reach their peak during infarction or hyperdensity from hem-
the early postpartum period, the time orrhage. Contrast CT may show a
when most cases of cerebral venous filling defect within the thrombosed
thrombosis present. Figure 4-1 shows sinus surrounded by the enhancing
the time during pregnancy of the dura of the sinus wall (empty delta
diagnosis of cerebral venous thrombo- sign). Contrast CT venography may
sis in the authors’ patients.17 In addi- show the thrombosis as a filling defect
tion to the known alterations in in the region of the affected sinus. On
platelet function and prothrombotic MRI, venous sinus thrombosis can be
and antithrombotic proteins, iron defi- seen directly as thrombus with signal
ciency anemia and the adaptive re- characteristics appropriate to the
sponse to the acute trauma and time since onset (T1-isodense and
92 www.ContinuumJournal.com February 2014

Copyright © American Academy of Neurology. Unauthorized reproduction of this article is prohibited.


KEY POINT
T2-hypodense when acute, with T1 degree for the small difference be- h Patients with cerebral
turning hyperintense followed by T2 tween groups. Cerebral hemorrhage venous sinus
turning hyperintense so that the occurs in nearly half of patients with thrombosis benefit
thrombus is bright on both T1- and cerebral venous thrombosis, so the from anticoagulation.
T2-weighted images when in the late question of the safety of anticoagula-
subacute phase). Since cerebral venous tion in this subset is important. In the
thrombosis may have been present for Einhäupl study, three of 10 patients
some time before symptoms lead to treated with heparin had experienced
diagnosis, it is common to see later- hemorrhage before treatment. None
phase characteristics upon initial diag- of these patients had expansion of
nosis. MR venography can also show their hemorrhage or new hemor-
the thrombosis as a filling defect. This rhage, and all recovered fully. This
can be done without contrast, an advan- study also included a retrospective
tage during pregnancy and nursing. As review of 102 patients with cerebral
with arterial stroke, with proper pre- venous thrombosis. Among these pa-
cautions, it is possible to obtain good tients, 27 of 43 with hemorrhage
imaging confirmation safely during received full-dose heparin, while 13
pregnancy. Cerebral cortical vein throm- received no heparin after hemor-
bosis without venous sinus thrombosis rhages were found. Those who re-
can be more difficult to confirm but is ceived heparin had lower mortality
commonly visible as an expanded tubu- (15% versus 56%). In the Dutch study
lar vein on the cortical surface, often as well, no worsening occurred in
with T2-weighted signal hyperintensity those receiving anticoagulation de-
in the adjacent parenchyma. spite the presence of hemorrhage.
The authors recommend full heparin
Therapy anticoagulation during the acute
The best available data support, if phase of cerebral venous thrombosis
weakly, the use of anticoagulation to whether hemorrhage is present or
treat cerebral venous thrombosis, in- not, and then a period of approxi-
cluding in those patients with hemor- mately 3 to 6 months of ambulatory
rhagic lesions. Meta-analysis and the anticoagulation. This extended period
authors’ experience treating many of anticoagulation is typically accom-
such patients are consistent with this plished with warfarin postpartum.
recommendation from the litera- During pregnancy, when warfarin is
ture.44,45 A randomized, controlled contraindicated, low-molecular-weight
trial by Einhäupl was small but seemed heparin is given and then held
to show a clear benefit.46 In fact, it was during the period of labor and deliv-
small because it was terminated early ery (Case 4-3).
due to the evidence of benefit in favor
of heparin anticoagulation after only HEMORRHAGIC STROKE AND
20 patients had been enrolled. A larger VASCULAR MALFORMATIONS
Dutch study of low-molecular-weight Epidemiology
heparin was negative but showed a Hemorrhagic, like ischemic, stroke is
trend in favor of early anticoagula- uncommon during pregnancy and the
tion.46 This study was limited by the puerperium. The proportion ranges
fact that patients in both groups from five to 35 per 100,000 in the
received warfarin anticoagulation after reported series (see Table 4-1), and
the first 3 weeks of the study treat- estimating the incidence from the
ment, possibly accounting to some three population-based series, it
Continuum (Minneap Minn) 2014;20(1):80–99 www.ContinuumJournal.com 93

Copyright © American Academy of Neurology. Unauthorized reproduction of this article is prohibited.


Cerebrovascular Disorders Complicating Pregnancy

Case 4-3
A 40-year-old woman developed progressively worsening headaches and nausea in the first trimester
of her third pregnancy. She had a medical history of depression and chronic hypertension; two
previous pregnancies had been uneventful. Her blood pressure was 120/78 mm Hg. The neurologic
and systemic examination findings were unremarkable. On brain imaging (Figure 4-4), MRI showed
hyperintense signal in the region of the right transverse sinus, and magnetic resonance (MR)
venogram showed absence of flow-related signal within the right transverse sinus and decreased
flow-related signal within the right sigmoid sinus and internal jugular veinVresults consistent with
cerebral venous sinus thrombosis. Laboratory tests showed an elevated D-dimer and a low protein S
level. She was treated with low-molecular-weight heparin, and the headaches resolved within 5 days.
A follow-up MR venogram performed after 2 weeks showed complete recanalization of the venous
sinuses. She went on to have an uncomplicated vaginal delivery. Follow-up blood tests showed normal
D-dimer and protein S levels. Six weeks after delivery, low-molecular-weight heparin was discontinued,
and she began treatment with aspirin.

FIGURE 4-4 Brain imaging of a 40-year-old pregnant woman with progressively worsening headaches and nausea. MRI
revealed hyperintense signal in the region of the right transverse sinus (A, fluid-attenuated inversion recovery
[FLAIR] image), and magnetic resonance (MR) venogram (B) showed absence of flow-related signal within
the right transverse sinus and decreased flow-related signal within the right sigmoid sinus and internal jugular vein. These
imaging results are consistent with cerebral venous sinus thrombosis. A follow-up MR venogram performed after 2 weeks of
treatment with low-molecular-weight heparin showed complete recanalization of the venous sinuses (C).

Comment. This case illustrates the association between pregnancy and cerebral venous sinus thrombosis.
Several mechanisms, including low levels of protein S as documented in this patient, contribute to a
transient hypercoagulable state during pregnancy. MR venography was preferred over CT venography to
avoid radiation risks during pregnancy. This patient was treated with low-molecular-weight heparin and
not warfarin because warfarin is teratogenic and can cause bleeding in the fetus.

KEY POINT ranges from 0 to 6 per 100,000. 28.5).9 Despite its rarity, because of the
h Pregnancy increases the However, despite the low absolute severe implications of cerebral hemor-
risk of hemorrhagic risk, pregnancy increases the risk for rhage, hemorrhagic stroke is also an
stroke. This increased hemorrhagic much more than for important cause of pregnancy-related
risk is greatest in the ischemic stroke. This risk increase is mortality. The major established causes
postpartum period.
substantial during pregnancy (relative of pregnancy-related cerebral hemor-
risk 2.5) and very great during the rhage are preeclampsia/eclampsia, fol-
early postpartum period (relative risk lowed by arteriovenous malformations

94 www.ContinuumJournal.com February 2014

Copyright © American Academy of Neurology. Unauthorized reproduction of this article is prohibited.


a,b
TABLE 4-3 Causes of Hemorrhagic Strokes in Pregnancy (% of All Hemorrhagic Strokes)

Cerebral Arteriovenous Cavernous Preeclampsia/


Study Aneurysm Malformation Malformation Eclampsia Unknown Other
Feske 14 14 3 42 22 11
et al, 2009
Liang 10 19 I 24 24 24
et al, 2006
Jeng et al, 14 23 I 32 I I
2004
Jaigobin 23 38 I I 23 15
et al, 2000
Witlin 50c I I 50 I
et al, 1997
Kittner I 23 I 15 31 31
et al, 1996
Sharshar 13 13 13 44 19 I
et al, 1995
Awada I I I I 100 I
et al, 1995
a
Totals of the rows may exceed 100% because multiple contributing causes may be counted.
b
Ellipses indicate that data was not reported.
c
This value combines both cerebral aneurysm and arteriovenous malformation.

KEY POINTS
and aneurysms. Preeclampsia/eclampsia The risk of aneurysmal rupture h The major causes of
probably contributes even a larger por- appears to increase severalfold, rising pregnancy-associated
tion than is apparent from the series with gestational age until it peaks at 30 hemorrhage are
reported in Table 4-3, since it is likely to 34 weeks.48 Dias and Sakhar48 preeclampsia/eclampsia
that cases that present in late pregnancy reported the mortality of pregnancy- and cerebral vascular
or the puerperium without proteinuria associated aneurysmal subarachnoid malformations, such
are often diagnosed and classified as of hemorrhage to be 35%, with a fetal as aneurysms and
unknown cause. Other potential causes, mortality of 17%. If a ruptured aneu- arteriovenous
such as disseminated intravascular co- rysm is left unsecured surgically, rates malformations.
agulation, have not been reported com- of recurrent hemorrhage and maternal h Aneurysmal subarachnoid
monly in these series. and fetal mortality are very high. This hemorrhage during
The physiologic changes of pregnancy mortality may be greatly reduced by pregnancy confers a
reviewed above include expansion of early surgery. In one study, subarach- high risk of death to
blood volume, increased stroke volume noid hemorrhage without early sur- both mother and baby.
and cardiac output, rise of blood pres- gery resulted in a maternal mortality h Women with subarachnoid
sure from its nadir in the late second or of 63% and fetal mortality of 27%; hemorrhage should be
early third trimester to near or slightly these mortalities were lowered to 11% seen by a neurosurgeon
above normal as term approaches, and and 5%, respectively, by early sur- and undergo vascular
imaging to look for
remodeling of vascular tissue with loss gery.48 With evidence that early sur-
aneurysm, arteriovenous
of collagen and elastin content and loss gery, open or endovascular, to secure
malformation, or other
of distensibility. One might expect ruptured aneurysms leads to better vascular lesions.
these changes to underlie an increased maternal and fetal outcomes, it is rec-
risk of hemorrhage near term. The ommended that therapy for women
strain and trauma of labor might be after aneurysmal rupture proceed as it
expected to add to the increased risk. does for all patients as dictated by

Continuum (Minneap Minn) 2014;20(1):80–99 www.ContinuumJournal.com 95

Copyright © American Academy of Neurology. Unauthorized reproduction of this article is prohibited.


Cerebrovascular Disorders Complicating Pregnancy

KEY POINT
h Aneurysmal neurosurgical principles. Therefore, if practitioners followed the analysis of
subarachnoid aneurysmal subarachnoid hemorrhage Robinson and colleagues, which
hemorrhage should occurs during pregnancy, the patient suggested that pregnancy increased
be treated with early should proceed to surgery immediately, the rate of hemorrhage of AVMs.50 A
surgery or endovascular if feasible. Endovascular coiling may later influential analysis found a back-
techniques to secure the be an alternative with proper shielding ground annual rate of hemorrhage of
ruptured aneurysm and to minimize fetal radiation exposure.49 3.5% in women with AVM and no prior
minimize the risk of If urgent obstetric issues (such as ac- hemorrhage and 5.8% in those with
recurrent hemorrhage. tive labor, eclampsia, or fetal distress) prior hemorrhage, with no increase
prevent immediate surgery, then the conferred by pregnancy.51 However,
woman should undergo urgent cesar- an analysis of the risk of rupture per
ean delivery followed by surgical con- day found a severalfold increase in risk
trol of the aneurysm. Because of the on the day of delivery.52,53 Also,
severe morbidity and high mortality although overall hemorrhage rates
rate of subarachnoid hemorrhage and appear to be comparable to nonpreg-
the increased risk of rupture near nant women, evidence suggests that
term, it is recommended that un- when an AVM bleeds during pregnancy,
ruptured aneurysms at significant risk the rebleeding rate is higher than in
of rupture be secured before preg- nonpregnant women. In one study of
nancy, whenever possible. With the 27 women with intracerebral hemor-
high rate of screening by MR angiogra- rhage due to AVM during pregnancy
phy for headaches and other common who were not treated with immediate
disorders, it is not uncommon to find resection, seven had recurrent hemor-
small, asymptomatic, unruptured aneu- rhage during or immediately after
rysms. In general, the risk of rupture pregnancy. This 26% rate of recurrent
depends on size and morphology. The hemorrhage in the first year is signifi-
risk is low for small, uncomplicated cantly higher than the roughly 6%
aneurysms. Systematically evaluated rate in nonpregnant women. Well-
clinical experience that would dictate controlled data on which to base ther-
the best policy for management of such apeutic decisions concerning AVMs
aneurysms is lacking; however, it is discovered during pregnancy are lack-
often considered prudent to deliver ing; however, based on the above
such women by cesarean delivery or considerations, expert recommenda-
by vaginal methods that interrupt the tions are that (1) if a woman with
second stage of labor. No clear data known AVM anticipates pregnancy, the
have been published to argue against AVM should be treated before preg-
vaginal delivery for women who have nancy; (2) if an AVM is discovered
surgically secured aneurysms, and most during pregnancy and has not bled
such women can be delivered vaginally during the pregnancy, conservative
with close monitoring. observation is usually recommended,
Data are conflicting concerning the with plans to proceed to definitive
influence of pregnancy on arteriove- treatment after delivery; (3) if an AVM
nous malformations (AVMs). Hemor- bleeds during pregnancy, consideration
rhage is the most common presenting should be given to treatment during the
manifestation of AVM, and AVMs that pregnancy, taking into account the
present with hemorrhage are more grade of the lesion and the expected
likely to bleed again than those dis- timing of benefit in lowering risk
covered as a result of seizures or focal (immediate for low-grade lesions ame-
neurologic deficits. For many years, nable to complete surgical excision or
96 www.ContinuumJournal.com February 2014

Copyright © American Academy of Neurology. Unauthorized reproduction of this article is prohibited.


KEY POINT
embolization but delayed by up to 1 to 7. Thadhani R, Mutter WP, Wolf M, et al.
First trimester placental growth factor and h Women with
3 years for higher-grade lesions requir- soluble fms-like tyrosine kinase 1 and risk arteriovenous
ing radiosurgery and combination ther- for preeclampsia. J Clin Endocrinol Metab malformations should
apies).54 Although no study has shown 2004;89(2):770Y775. be managed in
an advantage to cesarean delivery, 8. Feske SK. Stroke in pregnancy. Semin Neurol consultation with a
based on the suggestion of higher 2007;27(5):442Y452. neurosurgeon. In most
rates of hemorrhage on the day of 9. Kittner SJ, Stern BJ, Feeser BR, et al. cases, they can
delivery, many obstetricians will favor Pregnancy and the risk of stroke. N Engl J be treated with
Med 1996;335(11):768Y774. conservative
this approach to minimize risk.
10. Wiebers DO, Whisnant JP. The incidence observation until after
of stroke among pregnant women in delivery. However,
CONCLUSION Rochester, Minn 1955 through 1979. JAMA
when hemorrhage
Ischemic and hemorrhagic strokes are 1985;254(21):3055Y3057.
occurs during pregnancy,
uncommon but serious complications 11. Awada A, al Rajeh S, Duarte R, Russell N. patients with low-grade
of late pregnancy and the puerperium, Stroke and pregnancy. Int J Gynaecol Obstet
1995;48(2):157Y161.
arteriovenous
and when they occur, they confer malformations may
a major risk of long-term disability 12. Sharshar T, Lamy C, Mas JL. Incidence and
benefit from early
causes of stroke associated with pregnancy
or death. Knowledge of the risks and puerperium. A study in public hospitals definitive surgery or
of pregnancy-associated stroke and of Ile de France. Stroke in Pregnancy Study endovascular
the neurologic manifestations of Group. Stroke 1995;26(6):930Y936. embolization.
preeclampsia/eclampsia will support 13. Witlin AG, Friedman SA, Egerman RS, et al.
and encourage early diagnosis and Cerebrovascular disorders complicating
pregnancyVbeyond eclampsia. Am J Obstet
optimal management decisions. With Gynecol 1997;176(6):1139Y1145.
proper precautions to minimize risk to
14. Jaigobin C, Silver FL. Stroke and pregnancy.
the fetus, women can generally undergo Stroke 2000;31(12):2948Y2951.
diagnostic evaluations and be treated 15. Jeng JS, Tang SC, Yip PK. Incidence and
with aggressive measures appropriate to etiologies of stroke during pregnancy and
the severity of the condition. puerperium as evidenced in Taiwanese
women. Cerebrovasc Dis 2004;18(4):
290Y295.
REFERENCES
1. Silversides CK, Colman JM. Physiological 16. Liang CC, Chang SD, Lai SL, et al. Stroke
changes in pregnancy. In: Oakley C, complicating pregnancy and the puerperium.
Warnes CA, eds. Heart Disease in Pregnancy. Eur J Neurol 2006;13(11):1256Y1260.
Malden, MA: Blackwell Publishing, 2007: 17. Feske SK, Klein AM, Ferrante KL. Clinical
6Y17. risk factors predict pregnancy-associated
2. Hull AD, Long DM, Longo LD, Pearce WJ. strokes. Poster presented at: International
Pregnancy-induced changes in ovine Stroke Conference, February 18Y19, 2009
cerebral arteries. Am J Physiol 1992; San Diego, CA.
262(1 pt 2):R137YR143. 18. Kuklina EV, Tong X, Bansil P, et al. Trends in
3. Sibai BM. Etiology and management of pregnancy hospitalizations that included a
postpartum hypertension-preeclampsia. Am stroke in the United States from 1994 to
J Obstet Gynecol 2012;206(6):470Y475. 2007: reasons for concern? Stroke 2011;
42(9):2564Y2570.
4. Sibai B, Dekker G, Kupferminc M.
Pre-eclampsia. Lancet 2005;365(9461): 19. Grear KE, Bushnell CD. Stroke and pregnancy:
785Y799. clinical presentation, evaluation, treatment,
and epidemiology. Clin Obstet Gynecol
5. Sibai BM. Diagnosis, controversies, and 2013;56(2):350Y359.
management of the syndrome of hemolysis,
elevated liver enzymes, and low platelet 20. Starke RM, Komoter RJ, Hickman ZL, et al.
count. Obstet Gynecol 2004;103(5 pt 1): Clinical features, surgical treatment, and
981Y991. long-term outcome of adult moyamoya
patients. J Neurosurg 2009;111(5):936Y942.
6. Douglas KA, Redman CW. Eclampsia in the
United Kingdom. BMJ 1994;309(6966): 21. Guzman R, Lee M, Achrol A, et al. Clinical
1395Y1400. outcome after 450 revascularization

Continuum (Minneap Minn) 2014;20(1):80–99 www.ContinuumJournal.com 97

Copyright © American Academy of Neurology. Unauthorized reproduction of this article is prohibited.


Cerebrovascular Disorders Complicating Pregnancy

procedures for moyamoya disease. Clinical 35. Bartynski WS, Boardman JF. Catheter
article. J Neurosurg 2009;111(5):927Y935. angiography, MR angiography, and
MR perfusion in posterior reversible
22. Colletti PM, Lee KH, Elkayam U.
encephalopathy syndrome. AJNR Am J
Cardiovascular imaging of the pregnant
Neuroradiol 2008;29(3):447Y455.
patient. AJR Am J Roentgenol 2013;200(3):
515Y521. 36. Singhal AB. Postpartum angiopathy with
reversible posterior leukoencephalopathy.
23. Tissue plasminogen activator for acute
Arch Neurol 2004;61(3):411Y416.
ischemic stroke. The National Institute of
Neurological Disorders and Stroke rt-PA 37. Singhal AB, Kimberly WT, Schaefer PW,
Stroke Study Group. N Engl J Med 1995; Hedley-Whyte ET. Case records of the
333(24):1581Y1587. Massachusetts General Hospital. Case
8-2009. A 36-year-old woman with
24. Hacke W, Kaste M, Bluhmki E, et al.
headache, hypertension, and seizure 2
Thrombolysis with alteplase 3 to 4.5 hours
weeks post partum. N Engl J Med 2009;
after acute ischemic stroke. N Engl J Med
360(11):1126Y1137.
2008;359(13):1317Y1329.
38. Raps EC, Galetta SL, Broderick M, Atlas SW.
25. Broderick JP, Palesch YY, Demchuk AM,
Delayed peripartum vasculopathy: cerebral
et al. Endovascular therapy after
eclampsia revisited. Ann Neurol 1993;33(2):
intravenous t-PA versus t-PA alone for
222Y225.
stroke. N Engl J Med 2013;368(10):
893Y903. 39. Which anticonvulsant for women with
eclampsia? Evidence from the Collaborative
26. Johnson DM, Kramer DC, Cohen E, et al.
Eclampsia Trial. Lancet 1995;345(8963):
Thrombolytic therapy for acute stroke
1455Y1463.
in late pregnancy with intra-arterial
recombinant tissue plasminogen activator. 40. Lucas MJ, Leveno KJ, Cunningham FG. A
Stroke 2005;36(6):e53Ye55. comparison of magnesium sulfate with
phenytoin for the prevention of eclampsia.
27. Wiese KM, Talkad A, Mathews M, Wang D. N Engl J Med 1995;333(4):201Y205.
Intravenous recombinant tissue plasminogen
activator in a pregnant woman with 41. Altman D, Carroli G, Duley L, et al;
cardioembolic stroke. Stroke 2006;37(8): Magpie Trial Collaboration Group. Do
2168Y2169. women with pre-eclampsia, and their
babies, benefit from magnesium sulfate?
28. Leonhardt G, Gaul C, Nietsch HH, et al. The Magpie Trial: a randomised
Thrombotic therapy in pregnancy. J Thromb placebo-controlled trial. Lancet 2002;
Thrombolysis 2006;21(3):271Y276. 359(9321):1877Y1890.
29. Murugappan A, Coplin WM, Al-Sadat AN, 42. Fugate JE, Wijdicks EF, Parisi JE, et al.
et al. Thrombolytic therapy of acute Fulminant postpartum cerebral vasoconstriction
ischemic stroke during pregnancy. syndrome. Arch Neurol 2012;69(1):111Y117.
Neurology 2006;66(5):768Y770.
43. Ray JG, Chan WS. Deep vein thrombosis
30. Del Zotto E, Giossi A, Volonghi I, et al. during pregnancy and the puerperium: a
Ischemic stroke during pregnancy and meta-analysis of the period of risk and the
puerperium. Stroke Res Treat 2011;2011: leg of presentation. Obstet Gynecol Surv
606Y780. 1999;54:169Y175.
31. Dapprich M, Boessenecker W. Fibrinolysis 44. Bousser MG. Cerebral venous thrombosis:
with alteplase in a pregnant women with diagnosis and management. J Neurol
stroke. Cerebrovasc Dis 2002;13(4):290. 2000;247(4):252Y258.
32. Elford K, Leader A, Wee R, Stys PK. Stroke 45. Stam J, de Bruijn SFTM, de Veber G.
in ovarian hyperstimulation syndrome Anticoagulation for cerebral sinus
in early pregnancy treated with thrombosis. Cochrane Database Systemic
intra-arterial rt-PA. Neurology 2002; Reviews 2002;(4):CD002005.
59(8):1270Y1272.
46. Einhäupl KM, Villringer A, Meister W,
33. Feske SK. Posterior reversible encephalopathy et al. Heparin treatment in sinus venous
syndrome: a review. Semin Neurol 2011; thrombosis. Lancet 1991;338(8767):597Y600.
31(2):202Y215.
47. de Bruijn SF, Stam J. Randomized,
34. Singhal AB, Hajj-Ali RA, Topcuoglu MA, placebo-controlled trial of anticoagulant
et al. Reversible cerebral vasoconstriction treatment with low-molecular-weight-
syndromes: analysis of 139 cases. Arch heparin for cerebral sinus thrombosis.
Neurol 2011;68(8):1005Y1012. Stroke 1999;30(3):484Y488.

98 www.ContinuumJournal.com February 2014

Copyright © American Academy of Neurology. Unauthorized reproduction of this article is prohibited.


48. Dias MS, Sekhar LN. Intracranial hemorrhage 52. Parkinson D, Bachers G. Arteriovenous
from aneurysms and arteriovenous malformations: summary of 100 consecutive
malformations during pregnancy and the supratentorial cases. J Neurosurg 1980;53(3):
puerperium. Neurosurgery 1990;27(6):855Y865. 285Y299.

49. Meyers PM, Halbach VV, Malek AM, et al. 53. Weir B, Macdonald RL. Management of
Endovascular treatment of cerebral artery intracranial aneurysms and arteriovenous
aneurysms during pregnancy: report of malformations during pregnancy. In:
three cases. AJNR Am J Neuroradiol 2000; Wilkins RH, Rengachary SS, eds.
21(7):1306Y1311. Neurosurgery. New York, NY: McGraw-Hill,
1996:2421Y2427.
50. Robinson JL, Hall CS, Sedzimir CB.
54. Ogilvy CS, Stieg PE, Awad I, et al.
Arteriovenous malformations, aneurysms,
Recommendations for the management
and pregnancy. J Neurosurg 1974;41(1):63Y70.
of intracranial arteriovenous malformations:
51. Horton JC, Chambers WA, Lyons SL, et al. a statement for health care professionals
Pregnancy and the risk of hemorrhage for a special writing group of the Stroke
from cerebral arteriovenous malformations. Council, American Stroke Association. Stroke
Neurosurgery 1990;27(6):867Y872. 2001;32(6):1458Y1471.

Continuum (Minneap Minn) 2014;20(1):80–99 www.ContinuumJournal.com 99

Copyright © American Academy of Neurology. Unauthorized reproduction of this article is prohibited.


Review Article

Peripheral Neuropathies
Address correspondence to
Dr E. Wayne Massey, Division
of Neurology, Department of
Medicine, Duke University
Medical Center, DUMC 3909,
Durham, NC 27710,
masse010@mc.duke.edu.
in Pregnancy
Relationship Disclosure: E. Wayne Massey, MD, FAAN; Amanda C. Guidon, MD
Drs Massey and Guidon
report no disclosure.
Unlabeled Use of
Products/Investigational ABSTRACT
Use Disclosure:
Drs Massey and Guidon
Purpose of Review: This article provides an overview of the most common
report no disclosure. peripheral neuropathic disorders in pregnancy with a focus on clinical recognition,
* 2014, American Academy diagnosis, and treatment.
of Neurology. Recent Findings: The literature on this topic consists primarily of case reports, case
series, and retrospective reviews. Recent work, particularly in carpal tunnel syn-
drome, brachial neuritis, and inherited neuropathies in pregnancy, has added to our
knowledge of this field. Awareness of diabetic polyneuropathy with associated
autonomic dysfunction in pregnancy has grown as the incidence of diabetes
mellitus increases in women of childbearing age.
Summary: Women may develop mononeuropathy, plexopathy, radiculopathy, or
polyneuropathy during pregnancy or postpartum. Pregnancy often influences con-
sideration of etiology, treatment, and prognosis. In women of childbearing age with
known acquired or genetic neuromuscular disorders, pregnancy should be anticipated and
appropriate counseling provided. An interdisciplinary approach with other medical
specialties is often necessary.

Continuum (Minneap Minn) 2014;20(1):100–114.

INTRODUCTION of EMG and nerve conduction studies


Disorders affecting peripheral nerves, (NCS), both safe in pregnancy, further
plexus, or nerve roots are generally refine the clinical localization and
rare in pregnancy and the postpartum provide additional information about
period. However, they may be com- severity, chronicity, and prognosis. In
monly encountered in both inpatient some cases, further laboratory or
and outpatient neurology practice. imaging evaluation is warranted.
Acquired and inherited peripheral When choosing therapy, clinicians
nerve dysfunction affects the mother should consider potential effects on
and fetus in various ways. This article the fetus during pregnancy and
reviews the most commonly encoun- whether the patient is breast-feeding
tered peripheral nerve disorders in postpartum.
pregnancy (Table 5-1) with the goal
of facilitating rapid recognition, treat- UPPER EXTREMITY
ment, symptom management, appro- Median Neuropathy at the
priate counseling, and interdisciplinary Wrist (Carpal Tunnel Syndrome)
collaboration. Carpal tunnel syndrome (CTS) is the
After initial history and focused most common mononeuropathy during
symptom-specific neurologic examina- pregnancy. Estimates of the incidence
tion, the clinician should strive to of pregnancy-related CTS vary from 1%
localize the problem to one nerve, to 60% depending on study methodol-
multiple nerves, plexus, or nerve root. ogy. Incidence is approximately 17%
Electrodiagnostic studies, consisting in prospective studies describing
100 www.ContinuumJournal.com February 2014

Copyright © American Academy of Neurology. Unauthorized reproduction of this article is prohibited.


KEY POINT

TABLE 5-1 Most Common Peripheral Neuropathic Disorders h Carpal tunnel syndrome
in Pregnancy is common in
pregnancy. Treatment
b Upper Extremity is typically conservative,
particularly when
Mononeuropathies (median, ulnar, radial)
symptoms start during
Brachial neuritis/neuralgic amyotrophy the third trimester.
b Lower Extremity
Mononeuropathies (femoral, obturator, lateral femoral cutaneous, fibular)
Lumbosacral plexopathy
Lumbosacral radiculopathy
b Facial Neuropathy (Bell Palsy)
b Intercostal Neuralgia
b Polyneuropathy
Immune-mediated neuropathies (eg, acute inflammatory demyelinating
polyradiculoneuropathy, chronic inflammatory demyelinating
polyradiculoneuropathy, multifocal motor neuropathy)
Diabetic polyneuropathy
Polyneuropathy due to nutritional deficiency
Genetic causes of polyneuropathy (eg, Charcot-Marie-Tooth, hereditary
neuropathy with liability to pressure palsies)

only electrodiagnostically confirmed cause edema in the tissues of the carpal


pregnancy-related CTS.1 EMG and tunnel. Most women with pregnancy-
NCS can distinguish pregnancy-related related CTS develop symptoms in the
CTS from other forms of pregnancy- third trimester when they have more
related hand pain. generalized edema, which supports this
Key presenting features include hypothesis. Other hypotheses include
nocturnal or activity-related hand par- the effect of relaxin and other hor-
esthesias, which can be painful. Sen- mones on ligamentous laxity, changes
sory disturbance classically occurs in a in sleep position, and increased adipose
median distribution in the hand; how- tissue in pregnancy. Although diabetes
ever, more widespread hand and arm mellitus is a known risk factor for CTS in
symptoms are commonplace. Patients the general population, an association
often describe hand stiffness and between gestational diabetes mellitus
clumsiness, and in severe cases, weak- and pregnancy-related CTS is not
ness of the abductor pollicis brevis known. Pregnancy-related CTS may
may occur.2 occur postpartum and is most typically
Although it is a common disorder in associated with breast-feeding, possibly
pregnancy, the etiology of pregnancy- due to altered hand positioning.3
related CTS is poorly understood. In Given the high likelihood that
general, CTS is caused by compression pregnancy-related CTS will resolve in
of the median nerve at the wrist due to the postpartum period, management
reduced space in the carpal tunnel. is typically conservative, as illustrated
Pregnancy-induced fluid retention may by Case 5-1. Pregnancy-related CTS

Continuum (Minneap Minn) 2014;20(1):100–114 www.ContinuumJournal.com 101

Copyright © American Academy of Neurology. Unauthorized reproduction of this article is prohibited.


Peripheral Neuropathies in Pregnancy

KEY POINTS
h While carpal tunnel
syndrome in pregnancy
Case 5-1
A 35-year-old right-handed woman developed severe bilateral hand
is often less severe than
pain and paresthesia during her third trimester. Her symptoms intermittently
nonYpregnancy-related
extended to her forearms and were particularly severe at night. She
carpal tunnel syndrome,
described hand clumsiness and swelling. Carpal tunnel syndrome (CTS)
a significant percentage
was suspected. A trial of wrist splints recommended by her obstetrician
of women may have
provided no relief. Neurologic examination was notable for normal
ongoing symptoms
strength, decreased sensation in a median distribution in the right hand,
postpartum.
and a Tinel sign over the median nerve at both wrists. Electrodiagnostic
Electrodiagnostic
studies showed mild, bilateral, right greater than left median neuropathies
studies are
at the wrist. No evidence of more widespread polyneuropathy was present.
recommended to
Given the symptom severity and failed trial of wrist splints, the option of
support the diagnosis
injections for CTS was discussed. The patient declined but stated that she
in cases of severe,
was relieved to know the cause of her symptoms, which resolved
atypical, or persistent
3 months postpartum.
symptoms.
Comment. This patient was diagnosed with pregnancy-related CTS.
h Carpal tunnel syndrome By electrodiagnostic criteria, pregnancy-related CTS is typically mild,
may present or worsen although patients may describe severe symptoms with significant
postpartum. functional impairment. As this case illustrates, symptoms often resolve
postpartum, particularly when they first appear in the third trimester.
Recent longitudinal data, however, suggest that a larger percentage of
women than previously suspected may have persistent symptoms after
delivery, particularly when symptom onset occurs early in pregnancy.
Conservative measures and anticipatory guidance are typically
recommended for pregnancy-related CTS.

is typically less severe than nonY years. However, among women not
pregnancy-related CTS, thus justifying treated surgically, at 3 years 50% of
more conservative management. Ini- those with pregnancy-related CTS still
tial recommendations include using had symptoms, in comparison with 83%
wrist splints to maintain a neutral of women in the control group.4 Pa-
position and avoiding repetitive ma- tients with symptom onset earlier in
neuvers that may worsen symptoms. pregnancy and with greater pregnancy-
Few data compare the effects of associated weight gain were more
various nonoperative therapies (eg, likely to have persistent symptoms.
analgesics, low-salt diet, rest, splinting, Surgical therapy should be consid-
local steroid or lidocaine injections). ered if conservative measures fail,
The belief that symptoms resolve symptoms are severe, or progression
in 75% of patients in the first month occurs after delivery. Surgery can
postpartum was recently challenged. A nearly always be postponed until after
3-year longitudinal study that included delivery. It is important to remember
a control group of age-matched, that surgery may restrict hand and arm
nonpregnant women showed that per- use in the postpartum period, when
sistent symptoms were more common newborns require handling and care.
than previously estimated. Approxi-
mately 50% of all patients with Ulnar and Radial
pregnancy-related CTS had improve- Neuropathies
ment of symptoms in 1 year, and Ulnar and radial neuropathies occur
between 60% and 70% improved in 3 infrequently in pregnancy. If severe or

102 www.ContinuumJournal.com February 2014

Copyright © American Academy of Neurology. Unauthorized reproduction of this article is prohibited.


KEY POINTS
atypical or if localization is uncertain nerves outside the brachial plexus, and h Neuralgic amyotrophy
clinically, electrophysiologic evalua- the presence of mild dysmorphic fea- has both idiopathic and
tion may be considered. Therapy for tures. Dysmorphic features in hereditary autosomal dominant
ulnar neuropathy first involves care neuralgic amyotrophy include short hereditary forms. Both
not to traumatize the olecranon stature, hypotelorism, epicanthic folds, forms may occur
groove or other site of compression, redundant cervical skin folds, and dys- postpartum. The
such as the wrist.3 Radial neuropathy morphic ears.6 In one series of 246 inherited variety can
near the spiral grove associated with patients, four of the 10 cases of neural- recur with subsequent
prolonged use and inappropriate po- gic amyotrophy associated with preg- pregnancies, and no
sitioning of a birthing bar during labor nancy were hereditary, whereas the known method exists
to prevent attacks.
has been reported.5 A birthing bar remaining six were idiopathic.7 The
attaches to the labor bed and creates a method of delivery (eg, vaginal or h Attention to positioning
shelf on which a woman may rest her cesarean delivery) does not appear to in labor helps prevent
upper arms. It is sometimes used to influence the incidence of postpartum lower extremity
mononeuropathies.
facilitate a squatting position during attacks in hereditary neuralgic amyo-
Femoral, fibular, and
the second stage of labor. trophy, and no proven way to prevent
(less commonly)
episodes exists. Anecdotally, early treat- obturator neuropathies
Neuralgic Amyotrophy ment with corticosteroids in hereditary may occur.
Neuralgic amyotrophy, also known as neuralgic amyotrophy and idiopathic
Parsonage-Turner syndrome or brachial neuralgic amyotrophy may stop progres-
neuritis, has both an idiopathic and sion, limit pain, or improve outcome.6
autosomal dominant hereditary form.
Both forms are characterized by attacks LOWER EXTREMITY
of severe neuropathic pain, typically in During pregnancy or after delivery,
the shoulder, followed by multifocal women may develop lower extremity
upper limb weakness and atrophy. The weakness or sensory loss. Postpartum
upper trunk of the brachial plexus is peripheral nerve injuries are now un-
usually affected, scapular winging is common as a result of prevention strat-
frequently present, and sensory symp- egies during labor.8 In general, frequent
toms can be patchy. However, the leg position changes, shortened active
clinical presentation is often heteroge- labor time, and avoidance of prolonged
neous; it may be bilateral or involve hip flexion, extreme thigh abduction, or
nerves outside the brachial plexus, such hip external rotation are protective.
as the phrenic or spinal accessory nerves. Particular attention is warranted in
Progression is rapid initially, and recov- women receiving epidural anesthesia,
ery may take months to years. which is associated with a longer second
Altered immunity and resulting in- stage of labor. Sensory blockade makes
flammation are thought to underlie women less likely to sense pressure
both the idiopathic and hereditary and change their own position.9
forms of neuralgic amyotrophy. He-
reditary neuralgic amyotrophy is ge- Femoral and Obturator
netically heterogenous, but mutations Neuropathies
in the SEPT9 gene on chromosome The femoral nerve is formed by the
17q25 have been reported in the L2 to L4 nerve roots and arises from
majority of affected families. Features the lumbar plexus. The nerve passes
suggesting hereditary rather than idio- between the psoas and iliacus muscles
pathic neuralgic amyotrophy include and under the inguinal ligament. Dur-
repeated episodes (especially post- ing delivery, the femoral nerve may be
partum), family history, involvement of compressed at the inguinal ligament
Continuum (Minneap Minn) 2014;20(1):100–114 www.ContinuumJournal.com 103

Copyright © American Academy of Neurology. Unauthorized reproduction of this article is prohibited.


Peripheral Neuropathies in Pregnancy

KEY POINT
h The etiology of or stretched through hip abduction names; both terms are still used
compression or stretch and external rotation.10 In femoral interchangeably by many texts. During
should be evaluated neuropathy, patients report difficulty pregnancy or following labor, women
when women develop standing and may describe a buckling who have a fibular (peroneal) neurop-
lower extremity sensation in the leg. This results from athy may report paresthesia along the
mononeuropathies weakness of the quadriceps femoris lateral aspect of the leg and may notice
or plexopathy and, in lesions proximal to the ingui- foot drop; examination reveals weakness
postpartum. nal ligament, the iliopsoas. Sensory of ankle dorsiflexion and eversion, and
Electrodiagnostic studies loss and paresthesia may be seen in toe extension. Sensory loss may involve
confirm localization and the distribution of the femoral nerve the lateral aspect of the leg and dorsum
assist with assessment of
over the distal anteromedial thigh of the foot. The deep or superficial
severity and prognosis,
and, if involved, the saphenous nerve branches of the fibular nerve may be
which is typically
favorable.
over the anteromedial aspect of the affected together or in isolation. Me-
lower leg. The patellar reflex can be chanical injury, neuroma, fat, or cysts
diminished or absent. Weakness of can compress the common fibular nerve
both hip flexion and adduction sug- at the fibular head. Cysts may enlarge
gests a plexus or root lesion instead of during pregnancy. Electrodiagnostic
an isolated femoral neuropathy.11 evaluation can confirm fibular neuropa-
Although obturator neuropathy is rare thy and exclude lumbosacral plexopathy
postpartum, it should be considered in a and L5 radiculopathy. Peripheral nerve
patient with isolated hip adduction weak- ultrasound may further aid in localiza-
ness and numbness over the proximal tion or in identification of structural
medial thigh. The obturator nerve may causes of focal compression. Prognosis
be affected by the lithotomy position. A for recovery from pregnancy-related
cadaver study suggested that hip abduc- lower extremity nerve injury is typically
tion, as is used in the lithotomy position, good; however, the number of patients
increases strain on the obturator nerve with peroneal neuropathy in one study
to a degree that has been associated of postpartum nerve injuries was small.9
consistently with subsequent nerve dys- The potential etiology of injury and
function. This strain was lessened by severity of deficits should be considered
using concomitant hip flexion.12 The in each case to help guide assessment
nerve may also be compressed against of prognosis. Patients should be
the pelvic brim during forceps delivery or counseled to avoid pressure at the
due to hematoma from pudendal nerve fibular head from activities such as
block.9 In both femoral and obturator prolonged squatting or crossing the
neuropathy in the postpartum, pelvic legs. Ankle-foot orthoses may be re-
imaging is often indicated if symptoms quired to assist with ambulation if
are persistent or compression from hem- severe ankle dorsiflexion weakness is
orrhage is suspected.13 If no cause is present.3
found, treatment is supportive. Symptoms
often improve within 2 to 6 months.9 Lateral Femoral Cutaneous
Neuropathy (Meralgia
Fibular (Peroneal) Neuropathy Paresthetica)
Following reviewed terminology pub- Meralgia paresthetica is a sensory
lished in 1998 by the Federative mononeuropathy that occurs from in-
Committee on Anatomical Termi- jury to the lateral cutaneous nerve of
nology (FCAT), the peroneal nerve is the thigh (also called the lateral femo-
known as the fibular nerve, to distin- ral cutaneous nerve). The course of the
guish it from other nerves with similar nerve is highly variable, and locations
104 www.ContinuumJournal.com February 2014

Copyright © American Academy of Neurology. Unauthorized reproduction of this article is prohibited.


KEY POINT
of trauma include the psoas or tensor and malpresentation are risk factors.9 h Meralgia paresthetica
fascia lata muscles, the iliacus compart- Symptoms include pelvic and proxi- involves sensory loss
ment in the pelvis, the inguinal liga- mal leg weakness and pain with asso- and paresthesia of the
ment, or the thigh. No cause is ciated weakness in ankle dorsiflexion, anterior and lateral
identified in many cases. Symptoms inversion, and eversion. Muscles in- thigh. While symptoms
may be bilateral. Physical examination nervated by L4 and L5 roots are most are bothersome to the
reveals sensory loss in the anterolateral affected. Sensory impairment predom- patient, it is typically a
thigh in the distribution of the lateral inates in an L5 distribution. The benign disorder, which
cutaneous nerve of the thigh. Impor- Achilles (S1) reflex is often spared. is diagnosed clinically.
tantly, because the lateral cutaneous Electrodiagnostic studies show a plex- Electrodiagnostic
studies can be
nerve of the thigh is an exclusively us lesion (ie, sparing the paraspinal
performed if the
sensory nerve, strength and deep ten- muscles).3 Outcome data are limited,
localization is in
don reflexes are normal. Standing, and after excluding an ongoing com- question. Normal
walking, and flexion or extension ma- pressive lesion, management is typi- strength and deep
neuvers of the hip can exacerbate cally conservative. tendon reflexes help
symptoms.11 distinguish this from
During pregnancy and delivery, the Low Back Pain and Lumbosacral other causes of sensory
lateral cutaneous nerve of the thigh Radiculopathy disturbance of the
can be stretched or compressed. This Low back pain from musculoskeletal thigh.
nerve is unlikely to be damaged causes is reported in approximately
during regional anesthesia because it 50% of pregnancies. However, low
originates more caudal and lateral back pain due to herniated disc is rare
than the site typically used for epidu- and occurs in an estimated 1 in 10,000
ral placement.2,14 Presence of meralgia pregnancies. Pregnancy does not in-
paresthetica should not preclude the crease the risk of disc herniation.16
use of regional anesthesia. Man- Back pain without neurologic deficits
agement of meralgia paresthetica in can be managed symptomatically.
pregnancy focuses on symptom man- Changes in sensation, strength, or
agement. Most cases resolve sponta- reflexes, however, should prompt in-
neously within several months of vestigation for radiculopathy with
delivery. Avoiding aggravating posi- noncontrasted MRI and electro-
tions and tight-fitting clothing is diagnostic studies. Most common
recommended.15 Tricyclic antidepres- causes of radiculopathy can be inves-
sants or anticonvulsants, which are tigated without contrast. Gadolinium
first-line medications in nonpregnant agents are FDA pregnancy category C
patients, are all FDA pregnancy cate- and should only be used if the poten-
gory C or D.10 These are not recom- tial clinical benefit outweighs the risk
mended for use in pregnancy because (for more on this topic, see ‘‘Neuro-
the risk to the fetus outweighs po- radiology in Women of Childbearing
tential benefit to the patient with Age’’ by Drs Riley Bove and Joshua
meralgia paresthetica. Klein in this issue of ).
Contrast is typically indicated only if
Lumbosacral Plexopathy concern exists for infection, inflamma-
Injuries to the lumbosacral plexus can tion, or malignancy causing radi-
occur during the third trimester or culopathy.17 Conservative measures
delivery, due to compression from the such as rest, physical therapy, postural
fetal head or forceps. The plexus is education, and lumbar support are
relatively unprotected at the pelvic generally recommended.18 Surgery is
brim. Therefore, fetal macrosomia typically avoided in pregnancy except
Continuum (Minneap Minn) 2014;20(1):100–114 www.ContinuumJournal.com 105

Copyright © American Academy of Neurology. Unauthorized reproduction of this article is prohibited.


Peripheral Neuropathies in Pregnancy

KEY POINTS
h Back pain with focal in cases of significant progressive which is a poor prognostic factor.20
neurologic deficits neurologic deficits or cauda equina Whether this difference is due to the
should prompt further syndrome.18 disease process or treatment-related
investigation for The risk of neurologic complication factors is unknown.
radiculopathy, which from epidural anesthesia is low, esti- Pregnant and breast-feeding women
occurs only rarely in mated at 0.1%. Potential complications have traditionally been excluded from
pregnancy. include epidural hematoma, drug corticosteroid and antiviral trials for
h Epidural anesthesia toxicity, chemical radiculitis, arach- Bell palsy. The decision to treat preg-
very uncommonly noiditis, and direct needle injection nant women or not involves con-
results in neurologic injury into the nerve root. Therefore, sideration of the potential risks and
complications, including urgent MRI is recommended in women benefits. Recent evidence-based guide-
radiculitis, arachnoiditis, who develop radiculopathy or myelop- lines published by the American
or epidural hematoma. athy after epidural anesthesia.19 Al- Academy of Neurology (AAN) recom-
h Women have an though electrodiagnostic studies have mended using corticosteroids for Bell
increased risk of Bell limited ability to detect abnormalities palsy in the general population when
palsy in the third in the acute setting, studies may be started within 3 to 7 days of symptom
trimester and postpartum. performed to establish a baseline for onset to maximize the chance for
Pregnancy-related Bell future comparison if symptoms persist. facial nerve recovery.21 Steroids are
palsy may be more severe
FDA pregnancy category C and prob-
than nonYpregnancy- Idiopathic Facial Nerve Palsy ably safe during lactation. As in the
related Bell palsy. The
(Bell Palsy) general population, comorbid condi-
risks versus benefits of
treatment with steroids Bell palsy is the most common disor- tions need to be considered when
should be considered for der of the facial nerve. Pregnant making the decision to start these
each patient. women, particularly in the third tri- agents in pregnancy or postpartum. If
mester and in the first 2 weeks the patient has poorly controlled hy-
postpartum, have 3 times the risk for pertension or hyperglycemia, the risk
developing Bell palsy over their may outweigh the benefit. The same
nonpregnant counterparts. The path- AAN guidelines found that antiviral
ophysiology of Bell palsy in pregnancy agents have not conclusively shown
is poorly understood. Relative immu- efficacy in Bell palsy, and their benefit
nosuppression hypothetically may may only be modest. These agents are
lower the threshold for reactivation FDA pregnancy category B and con-
of herpes viruses in the geniculate sidered generally safe in breast-
ganglion. Increased extracellular vol- feeding. No clear evidence-based
ume, hypertension, hypercoagulable guidelines for treating pregnant or
states, and changes in hormonal levels breast-feeding patients who have Bell
have also been suggested. Retrospec- palsy have been published. Typically,
tive studies have shown a significant in an otherwise uncomplicated preg-
association with chronic or gestational nancy, the potential benefit of treat-
hypertension, preeclampsia, and dia- ment with corticosteroids after the
betes mellitus.10 first trimester is likely to outweigh
Bell palsy presents identically in the risk, in the authors’ opinion.
pregnant and nonpregnant women, Because of the uncertain benefit and
but the course may be more severe possible risk, antiviral agents can
in pregnant women. Although most probably be forgone in pregnancy
patients regain normal or near-normal when no additional manifestations of
function, retrospective data suggest viral infection are present. In all pa-
that pregnant women are more likely tients, the eye should be lubricated to
to have complete facial paralysis, prevent corneal abrasion. Patients can be
106 www.ContinuumJournal.com February 2014

Copyright © American Academy of Neurology. Unauthorized reproduction of this article is prohibited.


KEY POINTS
reassured that recurrence of Bell palsy polyneuropathy with particular rele- h Intercostal neuralgia
in future pregnancies is rare.3 vance to pregnancy and the postpar- results in pain and
tum period. sensory change in the
Intercostal Neuralgia distribution of one or
Intercostal neuralgia occurs rarely in AUTOIMMUNE NEUROPATHIES two thoracic roots. The
pregnancy (when it is also called Acute Inflammatory etiology in pregnancy
thoraconeuralgia gravidarum). Patients Demyelinating is unknown, and
experience mild to severe pain and Polyradiculoneuropathy symptoms typically
sensory change in the distribution of Acute inflammatory demyelinating resolve within hours
one to two thoracic roots, typically in polyradiculoneuropathy (AIDP), or of delivery.
the lower thoracic region. Pain can Guillain-Barré syndrome, is an acute h The approach to
worsen with palpation of the paraspinal neuropathy that may affect women evaluation of
muscles or radiate to the abdomen. The during pregnancy or the postpartum polyneuropathy is not
etiology is unknown and may relate to period. The incidence of AIDP is ap- significantly changed
by pregnancy.
stretch injury. Because symptoms typi- proximately 0.75 to 2 in 100,000 per year
cally remit within hours of delivery, a and increases with age. During preg-
physiologic rather than structural cause nancy, the age-adjusted incidence is
of nerve injury has been hypothesized. identical to that of the general popula-
EMG may reveal abnormal spontaneous tion. Nearly all of these cases, however,
activity in the paraspinal muscles of the occur in the second or third trimesters
corresponding symptomatic level. This or the first month postpartum.25
abnormality is not required for diagno- The clinical presentation of AIDP
sis.22 Intra-abdominal and spinal (spinal and approach to therapy in pregnancy
cord or radicular) etiologies should be is the same as in the general popula-
considered in the differential diagnosis, tion. Patients develop progressive
and imaging of the thoracic spine may weakness, sensory disturbance, and
be considered. Herpes zoster should loss of deep tendon reflexes that often
be considered in the presence of a skin follow an ascending pattern. Extra-
rash. Treatment with topical lidocaine ocular movement abnormalities, facial
patches, FDA pregnancy category B, weakness, ataxia, respiratory compro-
may be considered after a discussion of mise, and autonomic dysfunction may
the risk of treatment to the fetus versus occur. Diagnosis is made based on the
potential benefit to the patient. Ex- clinical history, the presence of
tended epidural anesthetic until delivery cytoalbuminologic dissociation in CSF,
has been reported in a severe case.23 electrodiagnostic studies that support
an acute neuropathy, and the exclusion
POLYNEUROPATHY of mimic diagnoses. Importantly, CSF
The general approach to the evalua- and electrodiagnostic studies may be
tion of polyneuropathy is not sig- normal very early in the disease course.
nificantly changed by pregnancy. AIDP is most frequently demyelinating;
Clinicians use the history, physical however, some autoimmune polyneu-
examination, and electrodiagnostic ropathy variants are axonal. Pregnant
studies to characterize the polyneu- women may be less likely than the
ropathy systematically and narrow the general population to have serologic
differential diagnosis of potential etiol- evidence of associated Campylobacter
ogies. A series of questions helps focus jejuni infection. The 10% incidence of
the evaluation (Table 5-224). With this cytomegalovirus infection in pregnant
framework in mind, the authors focus women with AIDP mirrors the general
this discussion on several forms of population. Testing for these agents is
Continuum (Minneap Minn) 2014;20(1):100–114 www.ContinuumJournal.com 107

Copyright © American Academy of Neurology. Unauthorized reproduction of this article is prohibited.


Peripheral Neuropathies in Pregnancy

KEY POINT
h Acute inflammatory a
TABLE 5-2 Characterization of Polyneuropathy
demyelinating
polyradiculoneuropathy b What is the time course?
can be treated with
plasma exchange or IV Acute or chronic
immunoglobulin in Progressive or relapsing/remitting
pregnancy and does not
b What is the distribution?
appear to have adverse
perinatal or neonatal Length-dependent or independent
outcomes. Testing for Multifocal
cytomegalovirus should
be performed because b What types of nerve fibers are affected?
cytomegalovirus can be Motor and/or sensory
transmitted to the fetus.
Large and/or small
Somatic and/or autonomic
b What portion of the nerve is affected?
Axon, myelin, or both
b Is there a family history or other features that would suggest a hereditary
neuropathy?
Lack of positive sensory symptoms
Associated skeletal abnormalities (eg, scoliosis, high-arched feet)
Early age at onset or very slowly progressive course
a
Data from Alport AR, Sander HW, Continuum (Minneap Minn).24 journals.lww.com/continuum/
Fulltext/2012/02000/Clinical_Approach_to_Peripheral_Neuropathy_.6.aspx.

recommended because their presence is of the patients, labor was induced for
often associated with more severe dis- maternal neurologic decline. Sixty-one
ease and residual disability. Cytomegalo- percent of patients delivered via ce-
virus is also important to identify because sarean delivery. However, vaginal de-
it may be transmitted to the fetus.26 liveries were achieved even in patients
The treatment for AIDP is either a with severe weakness and ventilator
course of plasma exchange or IV dependence. Therefore, in general,
immunoglobulin. One review of a AIDP does not appear to affect uterine
small series of treatment outcomes contractility, and operative delivery
and complications in pregnant pa- can be reserved for obstetric indica-
tients with AIDP found no treatment- tions. General anesthesia has been used
related complications with either without complication but may be com-
therapy.25 Termination of pregnancy plicated by autonomic instability. One
because of AIDP is not recommended, report of succinylcholine precipitating
as it has no proven benefit in short- severe hyperkalemia and maternal
ening disease course or improving death in AIDP has been published27;
maternal outcome. Additionally, peri- depolarizing neuromuscular blocking
natal and neonatal outcomes appear agents should be avoided in patients
reasonable, without defined risk due with AIDP. Epidural anesthesia has
to AIDP. In this review,25 35% of patients generally been used in this group
had preterm delivery; however, in most without complications, although one

108 www.ContinuumJournal.com February 2014

Copyright © American Academy of Neurology. Unauthorized reproduction of this article is prohibited.


KEY POINTS
case of worsening weakness has been polyneuropathy. This neuropathy is of- h Communication with
reported.28,29 The authors recommend ten painful and may be accompanied by the obstetrician and
communication with the obstetrician autonomic abnormality. Manifestations anesthesiologist
and anesthesiologist regarding method of autonomic neuropathy include regarding the method
of delivery in AIDP. Additional important gastroparesis, vomiting, constipation, di- of delivery in women
treatment considerations include deep arrhea, urinary frequency or retention, with acute inflammatory
vein thrombosis prophylaxis and preven- and postural hypotension. Many of these demyelinating
tion of secondary infections, such as symptoms are common during preg- polyradiculoneuropathy
pneumonia or urinary tract infection. nancy in nondiabetics and thus may go is recommended.
unrecognized as related to diabetic h Autonomic dysfunction
Chronic Inflammatory neuropathy by women and their doc- in diabetic
Demyelinating tors. Diabetes mellitus also predisposes polyneuropathy may
Polyradiculoneuropathy patients to cranial, nerve root, and focal cause gastroparesis or
Chronic inflammatory demyelinating entrapment neuropathies. cardiac complications
during pregnancy and
polyradiculoneuropathy (CIDP) is a Limited information, mostly from case
delivery. Diabetes also
group of immune-mediated neuropa- reports, exists about whether symptoms
predisposes pregnant
thies with a relapsing or progressive of diabetic polyneuropathy or autonomic women to focal
course. Because they usually present in neuropathy worsen with pregnancy. compressive
older adults, with a peak incidence Moreover, the effect of diabetic poly- neuropathies.
between the ages of 40 and 60, informa- neuropathy on pregnancy outcomes is
tion about pregnancy is limited. In case difficult to separate from other known
reports and a small series of nine women risk factors, such as poor metabolic
with CIDP who became pregnant, an control, hyperemesis, inadequate nutri-
increased risk of relapse during the third tion, or coexisting vascular disease. Preg-
trimester and postpartum period was nancy does not appear to increase the
observed.30 A similar pattern of worsen- risk for development or progression of
ing has been reported in multifocal diabetic polyneuropathy or autonomic
motor neuropathy (MMN), another un- neuropathy. Adjustments in pharmaco-
common disease in pregnancy. Risk to therapy for painful diabetic neuropathies
the fetus is not well established in may be indicated during pregnancy. Partic-
either disorder. Interestingly, one case ular attention to positioning and padding
of neonatal transmission of MMN asso- during delivery may help avoid superficial
ciated with anti-ganglioside M1 (anti- nerve injuries at sites of compression.32
GM1) antibodies and monoclonal Gastroparesis and cardiac compli-
gammopathy has been reported.31 cations from autonomic neuropathy
pose significant challenges during
METABOLIC AND NUTRITIONAL pregnancy. Gastroparesis exacerbates
NEUROPATHIES nausea and vomiting in pregnancy,
Diabetic Polyneuropathy resulting in inadequate absorption of
Diabetic polyneuropathy is estimated to nutrients and erratic blood glucose
affect 50% of patients with either type 1 control. Patients with gastroparesis
or type 2 diabetes mellitus. Risk factors may benefit from treatment with
include duration of disease and poor metoclopramide or erythromycin, both
glycemic control. Other vascular risk FDA category B medications in preg-
factors, such as obesity and hyperlipid- nancy. Total parenteral nutrition may be
emia, may play a role. The most com- necessary in cases of severe gastro-
mon form of diabetic polyneuropathy is paresis with nausea and vomiting. Di-
a length-dependent, small and large minished counter-regulatory response
fiber, sensory greater than motor to hypoglycemia may be present.33 This
Continuum (Minneap Minn) 2014;20(1):100–114 www.ContinuumJournal.com 109

Copyright © American Academy of Neurology. Unauthorized reproduction of this article is prohibited.


Peripheral Neuropathies in Pregnancy

KEY POINTS
h Deficiency of B vitamins may result in inadequate release of ciency, patients may have ophthal-
should be considered glucagon, norepinephrine, epineph- moparesis or nystagmus or other
as a cause of rine, and cortisol, which would typi- features of Wernicke encephalopathy.
polyneuropathy in cally restore normoglycemia.34 Patients Vitamin B12 deficiency can be manifested
pregnancy, particularly should be assessed for this and edu- by myelopathy (including posterior col-
in women with cated on strategies to minimize the umn and corticospinal tract abnormali-
hyperemesis gravidarum. occurrence of hypoglycemia.33 ties). Neuropathy due to B12 deficiency
h In women presenting Cardiac autonomic neuropathy is may also manifest as a large fiber sensory
with polyneuropathy estimated to affect 11% to 33% of young neuropathy or small fiber neuropathy
postpartum, a detailed adults with diabetes mellitus and de- with normal nerve conduction studies.36
history of emesis, diet, pends on the quality of glycemic control. In addition to low serum B12 levels,
and supplementation It may be accompanied by left ventricu- assessing for increased methylmalonic
during pregnancy is lar hypertrophy and diastolic dysfunc- acid and homocysteine levels increases
essential. tion. Cardiac autonomic neuropathy the diagnostic yield. B6 supplementation
manifests as exercise intolerance, ortho- has sometimes been used to treat
static hypotension, cardiac arrhythmias, nausea and vomiting in pregnant wom-
silent myocardial ischemia, or intrao- en. Therefore, both B6 toxicity and B6
perative cardiovascular lability and in- deficiency should be considered poten-
creased cardiac events.33 Hypotension tial causes of polyneuropathy in this
may worsen as a result of blunting of group. B6 intoxication manifests as a
the normal compensatory response of sensory neuropathy or neuronopathy;
increased heart rate. Volume expan- strength is preserved, although signifi-
sion in pregnancy, however, may also cant sensory ataxia may be present.
reduce baseline orthostasis. Obstetric Serum B6 and whole blood thiamine
anesthetists should be aware of the can be reliably measured. In patients
possibility of labile blood pressures in with presumed thiamine deficiency, one
response to general anesthesia.32 should emergently provide thiamine
re p l a c e m e n t t o t r e a t p o ss i b l e
Neuropathy due to Nutritional Wernicke encephalopathy and not
Deficiency delay treatment while the results of
When evaluating pregnant or postpartum the blood level are pending.
patients with symptoms of poly-
neuropathy, it is important to consider HEREDITARY DISORDERS OF
nutritional etiologies. Nausea and PERIPHERAL NERVE
vomiting are common and affect 50% Charcot-Marie-Tooth Disease
of pregnancies. In approximately 0.3% Hereditary motor and sensory neuropa-
to 1% of pregnant women, these thies, which is a term used interchange-
symptoms are severe and meet criteria ably with Charcot-Marie-Tooth disease
for hyperemesis gravidarum. In these (CMT), are the most common inherited
settings, women are at risk for polyneuropathies. Symptoms often be-
polyneuropathy related to nutritional gin in the first to third decades and
deficiency, particularly thiamine (vita- progress slowly. Symmetric distal limb
min B1), pyridoxine (vitamin B6), and weakness, pes cavus, sensory loss with-
cobalamin (vitamin B12).35 This is a out positive sensory symptoms, and
point well illustrated by Case 5-2. imbalance are typical. Significant genetic
Polyneuropathy due to nutritional and phenotypic variability exists. In some
deficiency typically follows a length- forms of CMT, postural tremor, dyspho-
dependent, axonal, and sensory greater nia from vocal cord paralysis, respiratory
than motor pattern. In thiamine defi- insufficiency, pupillary dysfunction, or
110 www.ContinuumJournal.com February 2014

Copyright © American Academy of Neurology. Unauthorized reproduction of this article is prohibited.


KEY POINT

Case 5-2 h Treatment of


Charcot-Marie-Tooth
A 30-year-old woman was seen 3 months postpartum for numbness and
disease is heavily
painful paresthesia in her distal lower extremities, which started during
focused on adaptive
the second trimester of pregnancy. Pregnancy was complicated by
equipment. Functional
hyperemesis gravidarum and 30-pound weight loss. Physical examination
needs or abilities may
was notable for loss of pinprick and light touch sensation distal to the
change during
ankles with intact vibration, proprioception, and strength. Deep tendon
pregnancy or postpartum,
reflexes were brisk throughout. Plantar responses were flexor. EMG and
and reassessment
nerve conduction studies, including autonomic testing of R-R interval
is warranted.
variability with deep breathing and posture, were normal. Laboratory
evaluation demonstrated normal whole blood thiamine, vitamin E, vitamin
B6, copper, erythrocyte sedimentation rate, extractable nuclear antigens,
angiotensin-converting enzyme, antinuclear antibody, serum protein
electrophoresis, and thyroid function testing. Hemoglobin A1C and 2-hour
glucose tolerance testing were normal. Zinc levels were mildly low.
Vitamin B12 was 199 pg/mL and had decreased from 600 pg/mL 2 months
earlier. Methylmalonic acid was elevated due to B12 deficiency. HIV and
hepatitis C testing had previously been negative. She was treated for
small fiber neuropathy in association with B12 deficiency with oral B12
supplementation. Follow-up B12 and methylmalonic acid levels were
normal, and the patient’s symptoms greatly improved.
Comment. This patient was diagnosed with small fiber neuropathy due
to B12 deficiency. While B12 deficiency typically causes a myeloneuropathy
with findings of a large fiber sensory and motor axonal neuropathy, it may
also cause small fiber neuropathy. Routine electrodiagnostic studies are
normal in small fiber neuropathy. In this case, B12 deficiency developed
due to hyperemesis gravidarum. In her evaluation, it was important to
exclude other significant nutritional deficiencies and other non-nutritional,
common causes of small fiber neuropathy. She was not breast-feeding,
and gabapentin was successfully used for symptomatic relief. In the
postpartum period, breast-feeding status may influence the clinician’s
decision to treat symptomatically and the choice of medications.

scoliosis may be manifest.37 Classification ally favorable. In the most recent review,
of the CMT variant as axonal or demye- which included 33 patients undergoing a
linating, combined with any available total of 63 pregnancies, pregnancy out-
clues regarding pattern of inheritance, comes were good. No increase in the
directs genetic testing. Inheritance is rate of miscarriage, pregnancy complica-
mainly autosomal dominant, although tions, preterm delivery, delivery by ce-
X-linked and autosomal recessive forms sarean delivery or instrumentation,
exist. Life expectancy is normal. Patients abnormal presentation, or adverse neo-
rarely (approximately 5%) become natal outcome was documented.38 This
wheelchair dependent. Care is focused study did not replicate prior data on
on maintaining function and mobility. increased risk of presentation abnor-
Important areas for discussion re- malities, instrumentation, cesarean deliv-
garding pregnancy for women with ery, or postpartum bleeding. As with
CMT, as with other preexisting periph- prior studies, however, transient wors-
eral neuropathic disorders, are outlined ening of CMT symptoms was reported
(Table 5-3).10 Retrospective data on in approximately 32% of pregnancies,
pregnancy outcomes in CMT are gener- and persistent worsening in an additional

Continuum (Minneap Minn) 2014;20(1):100–114 www.ContinuumJournal.com 111

Copyright © American Academy of Neurology. Unauthorized reproduction of this article is prohibited.


Peripheral Neuropathies in Pregnancy

KEY POINT
h Pregnancy outcomes are TABLE 5-3 Key Considerations for Pregnancy in Women With
typically favorable in Preexisting Neuropathya
Charcot-Marie-Tooth
disease. Transient b Will the course of maternal disease change with pregnancy?
worsening of
b Will treatment need to be adjusted, and, if so, how?
Charcot-Marie-Tooth
disease occurs in b What are the potential effects of therapy on the fetus?
approximately 30%
b Will there be complications during labor and delivery?
of pregnancies, while
persistent worsening is b Are there additional implications for the fetus?
seen in an additional a
Data from Guidon AC, Massey EW, Neurol Clin.10 www.sciencedirect.com/science/article/pii/
22% of pregnancies. S0733861912000199.

22% of pregnancies.38 Worsening may be CMT1A: 85% to 90% of cases are due to a
more common with earlier-onset disease. PMP22 gene deletion and 10% to a
Deterioration in one pregnancy generally PMP22 point mutation. It is a multifocal
predicts deterioration in subsequent demyelinating neuropathy that presents
pregnancies. Women with CMT who are as recurrent painless mononeuro-
ambulatory before pregnancy, however, pathies. These mononeuropathies occur
typically remain ambulatory. Case reports at typical sites of compression (eg,
have documented safe use of regional fibular neuropathy at the knee, ulnar
anesthesia during delivery.39 neuropathy at the elbow) and with
The authors recommend optimiz- everyday activities such as leg crossing,
ing functional status before pregnancy squatting, or leaning on an elbow.37
and reevaluating patients for any Although HNPP mononeuropathies gen-
new needs during pregnancy and post- erally recover spontaneously, fixed defi-
partum. It is important to address the cits may occur. Women with HNPP may
generally good pregnancy outcomes develop such neuropathies during preg-
and to allow the opportunity for genetic nancy or delivery: axillary and fibular
counseling. Carrier and prenatal testing neuropathies have been reported.40,41
and preimplantation genetic diagnosis Women with HNPP should take partic-
are available for some forms of ular precautions to avoid compression.
CMT. Fatigue and excessive daytime A personal or family history of multiple
sleepiness are common in CMT. prior compression neuropathies prompts
Physicians should be mindful of poten- consideration of HNPP. EMG/NCS is
tial worsening of baseline fatigue in utilized to confirm the mononeuropathy
pregnant and postpartum patients with and site of compression.
CMT.37 The authors recommend that
patients work with physical and occu- CONCLUSION
pational therapists to anticipate the Although rare, acquired peripheral
challenges of holding, feeding, and neuropathic disorders in pregnancy
carrying the newborn. are seen in practice. Neurologists can
greatly assist the patient and often
Hereditary Neuropathy With the rest of the medical team by
Liability to Pressure Palsies facilitating localization of the problem,
Hereditary neuropathy with liability to treatment, symptom management,
pressure palsies (HNPP) is an autosomal and appropriate counseling. Diagnos-
dominant disorder, which is allelic to tic evaluation and treatment may be

112 www.ContinuumJournal.com February 2014

Copyright © American Academy of Neurology. Unauthorized reproduction of this article is prohibited.


influenced in key ways by pregnancy position: a report of 5 cases with a review
of literature. Muscle Nerve 1993;16(9):
and consideration of fetal outcomes. 891Y895.
For neurologists who see women with
14. Eltzschig HK, Lieberman ES, Camann WR.
preexisting disorders, an early and Regional anesthesia and analgesia for labor
open dialogue about pregnancy can and delivery. N Engl J Med 2003;348(4):
allow for anticipatory guidance and 319Y332.
planning. Current data in this field are 15. Van Diver T, Camann W. Meralgia
limited, and additional study of pe- paresthetica in the parturient. Int J Obstet
Anesth 1995;4(2):109Y112.
ripheral neuropathies in pregnancy is
needed. 16. Ng J, Kitchen N. Neurosurgery and
pregnancy. J Neurol Neurosurg Psychiatry
2008;79(7):745Y752.
REFERENCES
1. Padua L, Aprile I, Caliandro P, et al. Carpal 17. Chen M, Coakley F, Kaimal A, Laros RK Jr.
tunnel syndrome in pregnancy. Neurology Guidelines for computed tomography and
2002;59(10):1643Y1646. magnetic resonance imaging use during
pregnancy and lactation. Obstet Gynecol
2. Stewart JD. Focal peripheral neuropathies. 2008;112(2 pt 1):333Y340.
4th ed. West Vancouver, Canada: JBJ
Publishing; 2010:224Y225, 556. 18. Darwich AA, Diwan SA. Management of
back pain in pregnancy. Tech Reg Anesth
3. Massey EW, Stolp KA. Peripheral neuropathy
in pregnancy. Phys Med Rehabil Clin N Am Pain Manag 2009;13(4):251Y254.
2008;19(1):149Y162. 19. Sax TW, Rosenbaum RB. Neuromuscular
4. Mondelli M, Rossi S, Monti E, et al. Prospective disorders in pregnancy. Muscle Nerve
study of positive factors for improvement of 2006;34(5):559Y571.
carpal tunnel syndrome in pregnant women. 20. Gillman GS, Schaitkin BM, May M, Klein SR.
Muscle Nerve 2007;36(6):778Y783. Bell’s palsy in pregnancy: a study of recovery
5. Roubal P, Chavinson A, LaGrandeur R. outcomes. Otolaryngol Head Neck Surg
Bilateral radial nerve palsies from use of 2002;126(1):26Y30.
the standard birthing bar. Obstet Gynecol 21. Gronseth GS, Paduga R; American Academy
1996;87(5 pt 2):820Y821. of Neurology. Evidence-based guideline
6. van Alfen N. Clinical and pathophysiological update: steroids and antivirals for Bell
concepts of neuralgic amyotrophy. Nat Rev palsy: report of the Guideline Development
Neurol 2011;7(6):315Y322. Subcommittee of the American Academy
of Neurology. Neurology 2012;79(22):
7. van Alfen N, van Engelen B. The clinical
2209Y2213.
spectrum of neuralgic amyotrophy in 246
cases. Brain 2006;129(2):438Y450. 22. Skeen MB, Eggleston M. Thoraconeuralgia
gravidarum. Muscle Nerve 1999;22(6):
8. Pridjian G. Safe maternal positioning during
779Y780.
labor and delivery. Obstet Gynecol 2011;
118(2 pt 2):413Y414. 23. Samlaska S, Dews T. Long-term epidural
9. Wong C, Scavone B, Dugan S, et al. Incidence analgesia for pregnancy-induced intercostal
of postpartum lumbosacral spine and lower neuralgia. Pain 2003;62(2):245Y248.
extremity nerve injuries. Obstet Gynecol 24. Alport AR, Sander HW. Clinical approach
2003;101(2):279Y288. to peripheral neuropathy: anatomic
10. Guidon AC, Massey EW. Neuromuscular localization and diagnostic testing.
disorders in pregnancy. Neurol Clin 2012; Continuum (Minneap Minn) 2012;18
30(3):889Y911. (1 Peripheral Neuropathy):13Y38.

11. Stewart JD. Focal peripheral neuropathies. 25. Chan LY, Tsui MH, Leung TN. Guillain-Barré
3rd ed. Philadelphia PA: Lippincott Williams syndrome in pregnancy. Acta Obstet
& Wilkins; 2000:211Y213, 457Y466, 475Y478. Gynecol Scand 2004;83(4):319Y325.

12. Litwiller JP, Wells RE, Halliwill JR, et al. Effect 26. Rees J, Soudain S, Gregson N, Hughes RA.
of lithotomy positions on strain of the Campylobacter jejuni infection and
obturator and lateral femoral cutaneous Guillain-Barré syndrome. N Engl J Med
nerves. Clin Anat 2004;17(1):45Y49. 1995;333(21):1374Y1379.
13. Hakim al M, Katirji B. Femoral 27. Feldman JM. Cardiac arrest after succinycholine
mononeuropathy induced by the lithotomy administration in a pregnant patient recovered

Continuum (Minneap Minn) 2014;20(1):100–114 www.ContinuumJournal.com 113

Copyright © American Academy of Neurology. Unauthorized reproduction of this article is prohibited.


Peripheral Neuropathies in Pregnancy

from Guillain-Barre syndrome. Anesthesiology response to hypoglycemia. Diabetes


1990;72(5):924Y924. 2010;59(9):2271Y2280.
28. Kocabas S, Karaman S, Firat V, Bademkiran F. 35. Niebyl JR. Clinical practice: nausea and
Anesthetic management of Guillain-Barré vomiting in pregnancy. New Engl J Med
syndrome in pregnancy. J Clin Anesth 2010;363(16):1544Y1550.
2007;19(4):299Y302.
36. Saperstein DS, Wolfe GI, Gronseth GS, et al.
29. Wiertlewski S, Magot A, Drapier S, et al. Challenges in the identification of
Worsening of neurologic symptoms after cobalamin-deficiency polyneuropathy. Arch
epidural anesthesia for labor in a Neurol 2003;60(9):1296Y1301.
Guillain-Barre patient. Anesth Analg
37. Puwanant A, Herrmann DN. Hereditary
2004;98(3):825Y827.
motor sensory neuropathies (Charcot-Marie-
30. McCombe PA, McManis PG, Frith JA, et al. Tooth Disease). Tawil RN, Venance S, editors.
Chronic inflammatory demyelinating neurology in practice: neuromuscular
polyradiculoneuropathy associated with disorders. 1st ed. West Sussex, UK:
pregnancy. Ann Neurol 1987;21(1): Wiley-Blackwell, 2011:162Y169.
102Y104.
38. Awater C, Zerres K, Rudnik-Schöneborn S.
31. Attarian S, Azulay JP, Chabrol B, et al. Pregnancy course and outcome in women
Neonatal lower motor neuron syndrome with hereditary neuromuscular disorders:
associated with maternal neuropathy comparison of obstetric risks in 178 patients.
with anti-GM1 IgG. Neurology 2004;63(2): Eur J Obstet Gynecol Reprod Biol 2012;
379Y381. 162(2):153Y159.
32. Myers JE. Autonomic neuropathy in diabetes 39. Kuczkowski KM, Fernández CL, Drobnik L,
in pregnancy. McCance DR, Maresh M, Sacks Chandra S. Anesthesia for cesarean section
DA, editors. A practical manual of diabetes in a parturient with Charcot-Marie-Tooth
in pregnancy. 1st ed. West Sussex, UK: disease: unresolved controversies. Arch
Wiley-Blackwell, 2010:176Y183. Gynecol Obstet 2010;282(3):347Y348.
33. Kitzmiller JL, Block JM, Brown FM, et al. 40. Simonetti S. Lesion of the anterior branch of
Managing preexisting diabetes for axillary nerve in a patient with hereditary
pregnancy: summary of evidence and neuropathy with liability to pressure palsies.
consensus recommendations for care. Eur J Neurol 2000;7(5):577Y579.
Diabetes Care 2008;31(5):1060Y1079.
41. Stögbauer F, Young P, Kuhlenbäumer G,
34. Diggs-Andrews KA, Zhang X, Song Z, et al. et al. Hereditary recurrent focal neuropathies:
Brain insulin action regulates hypothalamic clinical and molecular features. Neurology
glucose sensing and the counterregulatory 2000;54(3):546Y551.

114 www.ContinuumJournal.com February 2014

Copyright © American Academy of Neurology. Unauthorized reproduction of this article is prohibited.


Review Article

Pregnancy and
Address correspondence to Dr
Janice M. Massey, Division of
Neurology, Department of
Medicine, Duke University

Myasthenia Gravis Medical Center, DUMC 3403,


Durham, NC 27710,
Janice.massey@duke.edu.
Janice M. Massey, MD, FAAN; Carolina De Jesus-Acosta, MD Relationship Disclosure:
Dr Massey has received
educational grants from
Allergan, Inc; and Merz
ABSTRACT Pharma. Dr De Jesus-Acosta
reports no disclosure.
Purpose of Review: Myasthenia gravis (MG) is an acquired autoimmune disorder Unlabeled Use of
characterized by fluctuating ocular, limb, or oropharyngeal muscle weakness due to Products/Investigational
an antibody-mediated attack at the neuromuscular junction. The female incidence Use Disclosure:
Drs Massey and De Jesus-Acosta
of MG peaks in the third decade during the childbearing years. A number of discuss the use of drugs for
exacerbating factors may worsen MG, including pregnancy. When treatment is the treatment of myasthenia
needed, it must be carefully chosen with consideration of possible effects on the gravis, none of which are
labeled by the US Food and
mother with MG, the pregnancy, and the fetus. Drug Administration for use
Recent Findings: Decisions are complex in the treatment of women with MG in pregnancy.
contemplating pregnancy or with presentation during pregnancy. While data is * 2014, American Academy
largely observational, a number of characteristic patterns and issues related to risk of Neurology.
to the patient, integrity of the pregnancy, and risks to the fetus are recognized.
Familiarity with these special considerations when contemplating pregnancy is
essential to avoid potential hazards in both the patient and the fetus. Use of
immunosuppressive agents incurs risk to the fetus. Deteriorating MG with respiratory
insufficiency poses risk to both the mother and the fetus.
Summary: This article reviews available information regarding expectations and
management for patients with MG in the childbearing age. Treatment decisions
must be individualized based on MG severity, distribution of weakness, coexisting
diseases, and welfare of the fetus. Patient participation in these decisions is
essential for successful management.

Continuum (Minneap Minn) 2014;20(1):115–127.

INTRODUCTION changes in thyroid function, general


Myasthenia gravis is a chronic disorder anesthesia, certain medications, emo-
manifested by fluctuating weakness and tional or physical stress, menses, preg-
rapid fatigue of voluntary muscles. The nancy, and the postpartum state.2
estimated prevalence of MG in the Caring for the female patient of
United States is 20/100,000 population.1 childbearing age involves anticipation
Acquired MG is an autoimmune disor- of pregnancy, as well as care through-
der in which antibodies at the neuro- out pregnancy and the postpartum
muscular junction produce impaired period. In preparation for pregnancy,
neuromuscular transmission and fatiga- women with MG need education and
ble weakness in skeletal muscle counseling to address special thera-
(periocular, limb, or oropharyngeal peutic issues, including the choice and
muscles). Symptoms can present at risks of treating or not treating, effects
any age, but the highest incidence in of MG on the pregnancy, and risks to
female patients occurs during the third the fetus and newborn. Women with
decade of life. MG is more common in MG benefit from a personalized inter-
women than in men, with a ratio of 3:2. disciplinary approach to care during
Known triggers for MG include infection, pregnancy and the postpartum period,

Continuum (Minneap Minn) 2014;20(1):115–127 www.ContinuumJournal.com 115

Copyright © American Academy of Neurology. Unauthorized reproduction of this article is prohibited.


Pregnancy and Myasthenia Gravis

KEY POINTS
h Women with including neuromuscular, high-risk onstrate a defect in neuromuscular
myasthenia gravis obstetric, and neonatal pediatric spe- transmission are repetitive nerve
benefit from a cialists.2 Recognizing risks for both the stimulation studies and the more
personalized mother and the baby requires careful sensitive single-fiber EMG, both safely
interdisciplinary monitoring and attention during both performed in pregnant patients. Pa-
approach to care during pregnancy and delivery. Even after the tients may undergo chest CT imaging
pregnancy and the successful delivery of a healthy infant, without contrast to assess the thymus
postpartum period, MG may impact the new mother. A gland, however, postponement until
including neuromuscular, woman with MG should be fully after delivery is preferable, particularly
high-risk obstetric, and informed and aware that a contem- in antibody negative patients. The
neonatal pediatric
plated pregnancy is a physical com- risk of radiation is eliminated with
specialists.
mitment that may be affected by MG chest MRI, but it does not visualize
h Myasthenia gravis is but also requires additional ability to the anterior mediastinum as well as
unmasked or worsened cope with both the demands of par- CT imaging, which is the preferred
in approximately
enting and the ongoing disease. This technique.
one-third of patients
article discusses the initial approach to Thymoma is uncommon in this age
during their pregnancy.
female patients of childbearing age group, particularly if AChR-antibody
h Elevated serum levels of with MG, including diagnosis and testing is negative.6,7 The decision to
antiYacetylcholine
management of the many challenging perform thymic imaging can usually
receptor-binding
questions that arise for the patient and be postponed until after delivery in
antibodies or
antiYmuscle-specific
the treating physicians. recognition of the potential risk to the
kinase (MuSK) fetus, unless there is strong clinical
antibodies in patients
DIAGNOSIS OF MYASTHENIA suspicion for thymoma.
with clinical signs GRAVIS DURING PREGNANCY
and symptoms of MG is unmasked or worsened in CHANGES IN MYASTHENIA
myasthenia gravis approximately one-third of patients GRAVIS DURING PREGNANCY
confirm the diagnosis. during their pregnancy.3Y5 Symptoms Pregnancy may change the course of
In the seronegative of fluctuating weakness are the hall- MG, often in unpredictable ways.8 The
patient, mark of this condition, and when severity of weakness at the beginning
electrophysiologic associated with evident weakness typ- of pregnancy does not predict either
demonstration of
ical of MG (ie, fatigable ptosis, diplo- remission or exacerbation,3,4 and in
an abnormality of
pia, dysarthria, dysphagia, and/or limb fact disease exacerbations, myasthenic
neuromuscular
transmission establishes
weakness), they should prompt fur- crisis, or even disease remission may
the diagnosis. ther diagnostic studies (in the as-yet- each occur during pregnancy. Patients
undiagnosed patient), including can develop hypoventilation secondary
h Patients may undergo
electrodiagnostic studies and acetyl- to respiratory muscle weakness. The
chest CT imaging
without contrast to
choline receptor (AChR)Ybinding anti- growing fetus may also restrict the
assess the thymus bodies. If AChR antibodies are not diaphragm and compromise respiratory
gland; however, detectable, antiYmuscle-specific kinase function; late in the pregnancy, in-
postponement until (MuSK) should be measured (Case 6-1). creased abdominal pressure and dia-
after delivery is Elevated serum levels of antiYAChR- phragm elevation reduce the capacity
preferable, particularly binding antibodies or anti-MuSK anti- for the lungs to inflate fully. At some
in antibody-negative bodies in patients with clinical signs point during pregnancy, approximately
patients. and symptoms of MG confirm the 20% of patients develop respiratory
diagnosis. In the seronegative patient crisis requiring mechanical ventila-
electrophysiologic demonstration of tion. Close monitoring for respiratory
an abnormality of neuromuscular difficulties is essential throughout
transmission establishes the diagnosis. pregnancy to maintain the welfare of
The electrophysiologic tests that dem- both mother and fetus.
116 www.ContinuumJournal.com February 2014

Copyright © American Academy of Neurology. Unauthorized reproduction of this article is prohibited.


KEY POINTS

Case 6-1 h The severity of


weakness at the
Five weeks postpartum, a 29-year-old woman developed proximal upper
beginning of pregnancy
extremity weakness. The weakness lasted for 4 weeks and was not
does not predict
associated with changes in sensation. Pregnancy, cesarean delivery, and
either remission or
the immediate postpartum period were uncomplicated. Two years later,
exacerbation, and,
she developed fatigable right ptosis and horizontal, binocular diplopia
in fact, disease
that lasted 1 to 2 weeks. Neuro-ophthalmic evaluation noted ptosis and
exacerbations,
extraocular motility abnormalities. Myasthenia gravis (MG) did not appear
myasthenic crisis, or
to have been considered at that time, and the patient was diagnosed with
even disease remission
a right Horner syndrome. MRI of the brain and magnetic resonance
may each occur during
angiogram of the intracranial and extracranial vessels were negative.
pregnancy.
A year later she became pregnant and had an uneventful pregnancy.
On the morning of her planned cesarean delivery, she was hypertensive, h Close monitoring for
and possible preeclampsia was treated with magnesium sulfate without respiratory difficulties is
event. She delivered a healthy daughter without weakness, feeding essential throughout
difficulty, or respiratory distress. Two weeks postpartum, she developed pregnancy to maintain
nasal dysarthria and dysphagia followed by right ptosis. Over the next the welfare of both
few weeks, she had gradual worsening of generalized weakness. mother and fetus.
Examination showed severe bilateral fatigable ptosis and limited
bilateral eye abduction with diplopia. Eye closure, cheek puff, tongue
protrusion, and the palate showed fatigable weakness. Neck flexion,
proximal arm, and hip flexion were weak bilaterally. Electrodiagnostic
studies showed abnormal decrement on 3-Hz repetitive nerve stimulation
consistent with MG. AChR-binding antibodies were elevated. Given the
distribution of weakness with oropharyngeal muscle predominance,
intervention seemed necessary. Prednisone was considered but not chosen
as patients often worsen in the first 2 weeks of therapy before receiving
benefit, and worsening of her oropharyngeal weakness may have required
intubation. She received five sessions of plasma exchange without
complication and with significant improvement of her condition.
Comment. This patient represents a case of MG unmasked by her
pregnancies. A high index of suspicion with focused history taking, paying
special attention to the patient’s medical history, and correlation with the
patient’s physical examination should prompt the diagnosis. Appropriate
management includes caution with some medications that may contribute
to worsening of MG symptoms.

Rare complications of MG during antibodies. Patients may develop worsen-


pregnancy, including bone marrow ing weakness especially during the second
suppression, have been reported. It stage of labor, when striated muscle is
has been postulated that suppression involved; some may become exhausted
could be due to an autoimmune and require assistance for delivery.
reaction against the megakaryocyte An association between MG and
colony-forming unit.9,10 Patients on preeclampsia has been suggested. If
immunosuppressive therapy also may treatment is indicated, magnesium
develop infections secondary to de- sulfate should be used with extreme
creased immunity. caution due to its direct deleterious
During labor, the uterine smooth mus- effect on neuromuscular transmis-
cle is not compromised, because unlike sion.11,12 Exacerbation of weakness
striated muscle, it is not affected by AChR may require respiratory support.
Continuum (Minneap Minn) 2014;20(1):115–127 www.ContinuumJournal.com 117

Copyright © American Academy of Neurology. Unauthorized reproduction of this article is prohibited.


Pregnancy and Myasthenia Gravis

KEY POINTS
h No evidence has been FETUS DURING PREGNANCY port and nasogastric feedings, when
published that babies AND DELIVERY needed. Pyridostigmine (0.5 mg/kg to
born to mothers with Rare circumstances affecting the fetus 1.0 mg/kg) in divided doses adminis-
myasthenia gravis have also occur in pregnant women with tered 30 minutes before feeding may
any increased risk MG. Transplacental passage of mater- be useful to improve suck and reduce
of developing nal autoantibodies may lead to fetal risk of aspiration.
autoimmune-mediated muscle weakness in utero, thus reduc- Rarely, patients with transient neo-
myasthenia gravis. ing fetal movements, producing natal MG may develop more permanent
h The risk of generalized polyhydramnios, and resulting in still- complications, including persistent bul-
myasthenia gravis is birth. Fetal difficulty has been de- bar and facial weakness and hearing
highest in the first 2 to scribed even in mothers with mild or loss. Inactivation of the fetal subunit of
3 years after onset. asymptomatic disease, who produce the AChR during a critical period of fetal
During these years, it is antibodies against the fetal AChRs. In muscle development has been pro-
advisable for a patient posed as the cause of this phenotype.19
rare cases, babies of mothers with MG
to delay pregnancy,
may develop arthrogryposis multiplex The maternal fetal/adult AChR antibody
thereby reducing
congenita, a disorder characterized by ratio was reported as useful in pre-
potential worsening
provoked by pregnancy multiple joint contractures and other dicting the severity of these manifesta-
and clarifying her anomalies.13 This condition most likely tions.19Y21 Case reports suggest that
severity and response is secondary to decreased fetal move- plasma exchange and possibly predni-
to treatment. ment in utero, which can be moni- sone during pregnancy may reduce
tored with ultrasound. Having a child phenotypic severity in offspring, but
affected with neonatal complications of further studies are needed.19,22
MG may be predictive of subsequent
offspring being affected. No evidence TREATMENT DECISIONS BEFORE
has been published that babies born PREGNANCY
to mothers with MG have any in- In general, the severity and distribu-
creased risk of developing autoimmune- tion of weakness should guide therapy
mediated MG.14,15 decisions for women with MG who are
Approximately 10% to 20% of in- planning a pregnancy. Patients with
fants born to mothers with MG de- recently diagnosed ocular or mild MG
velop transient neonatal MG.4 While have an increased risk of conversion to
more common with AChR-positive severe generalized MG, particularly
mothers, transient neonatal MG may within the first 2 years after onset of
occur with anti-MuSK antibodyY symptoms. Also, the MG patient cur-
positive mothers16,17 and rarely even rently using immunosuppressive med-
with seronegative mothers. Maternal ication presents another challenge.
antibodies are presumed to transfer Initiation of immunosuppressive
across the placenta to the infant. agents other than prednisone before
Although most infants have detectable or during pregnancy is typically
maternal antibodies, only a small per- avoided. Table 6-1 lists medications
centage of infants develop symptoms. used in the treatment of MG with their
Common symptoms include general- associated US Food and Drug Admin-
ized hypotonia as well as respiratory, istration (FDA) pregnancy category
feeding, and swallowing problems. and reported teratogenic risks.23
Symptoms of transient neonatal MG The risk of generalized MG is
usually develop a few hours after birth highest in the first 2 to 3 years after
and typically resolve within 1 month onset. During these years, it is advis-
(range of 1 to 7 weeks).18 Treatment is able for a patient to delay pregnancy,
supportive, including ventilator sup- thereby reducing potential worsening
118 www.ContinuumJournal.com February 2014

Copyright © American Academy of Neurology. Unauthorized reproduction of this article is prohibited.


provoked by pregnancy and clarifying immunosuppressive treatment may
her severity and response to treat- be contrary to the patient’s desires
ment. In some women with severe and necessitates a trusting physician-
disease, pregnancy would be danger- patient relationship. Without patient
ous and is therefore considered to acceptance, this recommendation may
be contraindicated. Abandoning the lead to patient abandonment of care.
idea of pregnancy and continuing Consideration of pregnancy must be

TABLE 6-1 Therapeutic Interventions in Myasthenia Gravis

FDA
Pregnancy
Intervention Side Effects Categorya Teratogenicity
Pyridostigmine Muscle twitching, diarrhea, cough C No clear data.
with increased mucus, bradycardia
Prednisone Weight gain, hyperglycemia, C Animal studies have yielded
hypertension, gastrointestinal an increased incidence of cleft
upset and ulceration, mood changes, palate in the offspring.
osteoporosis, and myopathy
Plasma exchange Hypotension, tachycardia, n/a No known data. Plasma exchange
electrolyte imbalances, sepsis, has been used successfully
allergic reaction, nausea, vomiting, during human pregnancy.24
venous thrombosis, and hematoma
Immunoglobulins Headache, aseptic meningitis C Animal studies have not been
dermatitis, pulmonary edema, reported. IVIg has been used
allergic/anaphylactic reactions, successfully during human
acute kidney injury, venous pregnancy.
thrombosis, stroke, and hepatitis
Cyclosporine Renal toxicity, hypertension, C Human data have revealed
seizures, myopathy, increased evidence of premature birth
risk of infections and low birth weight for
gestational age.
Mycophenolate Increased risk of infections, D Pregnancy loss in first trimester
mofetil possible increased risk of and congenital malformations
lymphoma and other in the face and distal limbs,
malignancies such as skin heart, esophagus, and kidney
cancer have been reported.
Azathioprine Hepatotoxicity, bone marrow D Sporadic congenital defects
suppression, nausea, vomiting, such as cerebral palsy,
diarrhea, possible increased risk cardiovascular defects,
of lymphoma and leukemia hypospadias, cerebral
hemorrhage, polydactyly, and
hypothyroidism. Reported
chromosomal aberrations
in utero.
Rituximab Fever, asthenia, headache, C B-cell lymphocytopenia
abdominal pain, hypotension, generally lasting less than
thrombocytopenia, progressive 6 months can occur in infants
multifocal encephalopathy exposed to rituximab in utero.
FDA = US Food and Drug Administration; n/a = not applicable.
a
Please see Appendix A for the US Food and Drug Administration Pregnancy Category descriptions.

Continuum (Minneap Minn) 2014;20(1):115–127 www.ContinuumJournal.com 119

Copyright © American Academy of Neurology. Unauthorized reproduction of this article is prohibited.


Pregnancy and Myasthenia Gravis

KEY POINTS
h Treatment is stepwise dealt with sympathetically, and the pa- appropriate drug-risk pregnancy registry
and depends on the tient should be reassured that regardless once pregnancy is confirmed (eg, www.
clinical scenario. of her decision, she will receive our care. mycophenolatepregnancyregistry.com).
With only minimal With close monitoring, an otherwise
manifestations of the healthy woman with well-controlled TREATMENT OPTIONS DURING
disease, pyridostigmine MG can have an uneventful pregnancy. PREGNANCY
for symptomatic Treatment is stepwise and depends During pregnancy, MG improves in
treatment before on the clinical scenario. With only approximately 30% to 40% of patients,
contemplating pregnancy minimal manifestations of the disease, remains unchanged in 30% to 40%,
may be considered. pyridostigmine for symptomatic treat- and worsens in 20% to 30%.3,5,8 The
h Corticosteroids, plasma ment before contemplating pregnancy greatest percentages of exacerbations
exchange, and IV may be considered. Given the uncer- occur during the first trimester, in the
immunoglobulin have tainty of the course of MG, therapy in final 4 weeks of gestation, or puerpe-
been used safely during an asymptomatic myasthenic patient is rium. Patients with only mild disease
pregnancy and are
not suggested. Patients with moderate may not require treatment but need
agents often chosen
weakness may benefit from steroids, a close follow-up with assessment for
for treatment of
exacerbation of
medication with lower teratogenic weakness. When weakness is mild, no
weakness. profile. A prior response to steroids treatment may be necessary. When
or other comorbid conditions aids in needed, medications that have less
the decision to choose this therapy. teratogenic effects are recommended.
However, if a patient is on other therapy Potential treatment alternatives for
(eg, steroid-sparing immunosuppres- symptomatic relief, including pyrido-
sive agents), she may have had a stigmine, can be used safely in recom-
previous incomplete response to ste- mended doses during pregnancy.
roids. Depending on the distribution of Anticholinesterase medications are
weakness and severity of involvement, pregnancy category C. Because of the
steroid use may be a reasonable alter- changes in intestinal absorption and
native. Side effects should be closely renal function during pregnancy, the
monitored. If thymectomy is consid- dose may need frequent adjustments.
ered, it should be performed before The overuse of cholinesterase inhibi-
pregnancy or after a stable postpartum tors may induce uterine contractions,
period because of the delayed thera- premature labor, and increase oral
peutic effect and surgical risks. secretions, which can be difficult for
Another, less desirable alternative patients with oropharyngeal weakness.
would be to continue immunosuppres- Corticosteroids, plasma exchange,
sive therapy while attempting preg- and IV immunoglobulin (IVIg) have
nancy, during pregnancy, and delivery. been used safely during pregnancy
The risk of precipitating myasthenic and are agents often chosen for treat-
exacerbation or crisis by withdrawing ment of exacerbation of weakness.
immunosuppressive therapy must be These treatments are generally very
weighed against potential harm to the well tolerated, although they are not
fetus. In this scenario, the mother will innocuous. Prednisone, prednisolone,
have the greatest likelihood of main- and IVIg are pregnancy category C. All
taining her strength and overall health, have been used frequently during
but risk of teratogenicity to the fetus is pregnancy in many other autoimmune
increased. If benefits are significant diseases. However, a small increase in
enough to outweigh the risk, it is impor- cleft palate with use of prednisone in
tant that the parents be well informed the first trimester is reported.25 In addi-
and the patient be registered in the tion, high doses of prednisone have

120 www.ContinuumJournal.com February 2014

Copyright © American Academy of Neurology. Unauthorized reproduction of this article is prohibited.


KEY POINTS
been associated with premature rupture necessary. Cholinesterase inhibitors h Azathioprine and
of membrane. With plasma exchange can minimize fatigable weakness dur- mycophenolate mofetil
or IVIg, a theoretical risk of inducing ing labor. No correlation has been are US Food and
abortion during the postY24-hour proven between the mode of preg- Drug Administration
period of coagulopathy is present.26 nancy delivery and the rate of exacer- pregnancy category D,
Given this risk, their use is reserved bation in the puerperium.3 and methotrexate is
for the management of more severe During labor, regional anesthesia category X. The use of
MG symptoms or myasthenic crisis. can be used safely and lessens the these medications is
When using IVIg, hyperviscosity and risk of medication-induced neuro- not recommended in
volume overload should be monitored muscular blockade from nondepo- pregnancy because they
pose significant risk to
carefully. Hypotension is a serious side larizing anesthetic or curare-like
the fetus.
effect associated with plasma exchange. agents (Table 6-2). It is also recom-
To protect against hypotension, the mended for patients undergoing ce- h Myasthenia gravis
patient should be placed in a left lateral sarean delivery. Those who receive a typically does not hinder
the early stages of labor,
decubitus position and her fluid status high level of spinal or epidural anesthe-
as smooth muscle
carefully monitored during treatment. sia may experience decreased respira-
contraction is involved
During the third trimester, fetal moni- tory function, especially if they have had in the first stage
toring is recommended during plasma- previous respiratory weakness or sig- of labor.
pheresis. Benefit from plasma exchange nificant oropharyngeal symptoms.
h During labor, regional
or IVIg is short-lived, and retreatment Nondepolarizing agents worsen neuro-
anesthesia can
may be required. muscular transmission and are therefore be used safely and
Other medications routinely used in avoided in MG. Immediate-acting drugs, lessens the risk of
MG pose a greater risk to pregnant carefully titrated, are recommended if medication-induced
mothers, and their use is usually dis- general anesthesia is needed.28 neuromuscular
couraged during pregnancy.27 Although Treating eclampsia with magnesium blockade from
cyclosporine is pregnancy category C, sulfate in a woman with MG should be nondepolarizing
its use during pregnancy is not approached with caution. Magnesium anesthetic or curare-like
recommended because of increased blocks calcium entry at the nerve agents.
risk of spontaneous abortions, prema- terminal and inhibits acetylcholine re- h Infections, electrolyte
turity, and low birth weight. Azathio- lease, further disrupting neuromuscu- disturbances, and
prine and mycophenolate mofetil are lar transmission. If the potential benefit numerous drugs have
pregnancy category D, and methotrex- of administering magnesium sulfate been found to unmask
ate is category X. The use of these outweighs the risks, the physician and latent myasthenia gravis
medications is not recommended in patient should be prepared for poten- or trigger a myasthenic
crisis.
pregnancy because they pose significant tial worsening of MG and be prepared
risk to the fetus.23 Case 6-2 demon- to provide ventilator support. Phenyt-
strates decisions in the treatment of a oin is an accepted alternative for the
patient with MG throughout pregnancy. treatment of eclampsia.29
Infections, electrolyte disturbances,
LABOR AND DELIVERY and numerous drugs have been found
MG typically does not hinder the early to unmask latent MG or trigger a
stages of labor, as smooth muscle myasthenic crisis. Additionally, the
contraction is involved in the first issue of appropriate vaccines may
stage of labor. In the second stage of arise. As a general rule, live virus
labor, fatigability may be pronounced vaccines should be avoided in any pa-
as striated muscle contraction be- tient with MG, particularly in the setting
comes more important. The obstetri- of immunosuppressive therapy.30,31
cian should be prepared to assist the Table 6-2 summarizes medications that
delivery with vacuum or forceps when may exacerbate MG.
Continuum (Minneap Minn) 2014;20(1):115–127 www.ContinuumJournal.com 121

Copyright © American Academy of Neurology. Unauthorized reproduction of this article is prohibited.


Pregnancy and Myasthenia Gravis

KEY POINT
h The American Academy
of Pediatrics considers
Case 6-2
A 17-year-old girl was diagnosed with oculobulbar myasthenia gravis (MG)
pyridostigmine,
based on clinical presentation, elevated acetylcholine receptor antibodies,
prednisone, and
abnormal repetitive stimulation, and single-fiber EMG. Her first symptoms
prednisolone
were ptosis and diplopia followed by dysphagia. Mycophenolate mofetil
compatible with
therapy was initiated and pre-pregnancy counseling was provided. She
lactation.
responded well with only minimal stable signs of MG. Mycophenolate
mofetil therapy was continued.
At 26 years of age, she was referred after discovering that she was in
her fifth week of gestation. She reported recent decreased energy but
denied weakness. Physical examination demonstrated mild-moderate
ptosis accentuated by upgaze, minimal weakness of bilateral eye closure,
and mild-moderate weakness of cheek puff. Extraocular muscles were
intact. She had no dysphagia or limb weakness.
After a long discussion, she agreed to discontinue mycophenolate
mofetil. Counseling regarding the natural history of MG during pregnancy
was provided. She was prescribed pyridostigmine 30 mg 3 times daily for
her symptoms. She had frequent follow-up assessments and remained
stable. She was enrolled in the mycophenolate mofetil pregnancy registry
and followed in a high-risk obstetric clinic.
She delivered a healthy son without complications, who had no
difficulties in the postnatal period. On no therapy, the mother had no
symptoms of MG until 1 year after delivery, when she began to develop
ptosis and diplopia. One consideration was to restart mycophenolate
mofetil at that time. However, she was not using contraception and had
no plans to do so. Given the unknown risks of fetal malformation
secondary to mycophenolate mofetil, corticosteroid therapy was initiated.
She also resumed pyridostigmine 60 mg 3 times daily. With good clinical
response, prednisone was gradually tapered to 5 mg/d.
Comment. This case typifies management decisions that arise in a
patient with known MG on immunosuppressive therapy who becomes
pregnant. Vast knowledge of medication side effects and potential
teratogenic effects is needed for appropriate therapeutic management in
patients with MG during childbearing age. Prepregnancy counseling is
important for the care of both mother and fetus. Education and
counseling may need reinforcement at subsequent visits.

nisone, and prednisolone compatible


THE POSTPARTUM PERIOD with lactation.32 Pyridostigmine is ex-
Symptoms may worsen in the puer- creted into human breast milk. Con-
perium, typically within 6 to 8 weeks clusions from very limited data have
after delivery. Close follow-up for po- estimated that infants would ingest
tential worsening is recommended. less than 0.1% of the maternal dose, so
Treatment selection during lactation adverse effects in the infant are unlikely.
may pose another challenge. Most of Manufacturers for prednisone recom-
the medications for the treatment of mend that caution be used when admin-
MG can be secreted through the milk istering prednisone to nursing women.
and therefore pose a potential risk to the Cyclosporine is excreted in human
newborn.32,33 The American Academy of breast milk. Because of potential
Pediatrics considers pyridostigmine, pred- effects in a nursing infant such as

122 www.ContinuumJournal.com February 2014

Copyright © American Academy of Neurology. Unauthorized reproduction of this article is prohibited.


KEY POINT

TABLE 6-2 Medications That May Exacerbate Myasthenia Gravis h All women of
childbearing potential
b D-Penicillamine and "-interferon should not be used in patients with (including pubertal girls
myasthenia gravis (can induce myasthenia gravis). and perimenopausal
b The following drugs produce worsening of weakness. Use with caution and women) who begin
monitor patients for exacerbation of myasthenic symptoms. or restart an
immunosuppressive
Succinylcholine, d-tubocurarine, vecuronium, and other neuromuscular
blocking agents including botulinum toxins regimen must receive
contraceptive
Quinine, quinidine, and procainamide
counseling and
Beta-blockers including propranolol, atenolol, and timolol maleate eye drops use effective
Calcium channel blockers contraception.
Iodinated contrast agents
Magnesium including milk of magnesia, antacids containing magnesium
hydroxide, and magnesium sulfate
Selected antibiotics including
Aminoglycosides (eg, tobramycin, gentamycin, kanamycin, neomycin, streptomycin)
Macrolides (eg, erythromycin, azithromycin, telithromycin)
Fluoroquinolones (eg, ciprofloxacin, moxifloxacin, norfloxacin, ofloxacin,
pefloxacin)
Colistin
b Many other drugs are reported to exacerbate weakness in some patients with
myasthenia gravis. All patients with myasthenia gravis should be observed for
increased weakness whenever a new medication is begun.
b Patients with myasthenia gravis or a history of thymoma should consider alternatives
to receiving yellow fever vaccine, shingles vaccine, or any other ‘‘live virus’’ vaccine.

immunosuppression, neutropenia, mother with MG. Patients may develop


growth retardation, and potential car- worsening of symptoms due to fatigue
cinogenesis, cyclosporine is considered induced by reduced sleep, frequent
contraindicated by the American feedings, and increased physical exertion
Academy of Pediatrics. Azathioprine related to caring for the baby. Symptoms
and methotrexate are unsafe dur- during this period can be transient and
ing breast-feeding. The safety of managed by conservative treatment, in-
mycophenolate mofetil, rituximab, or cluding engagement of a support system.
IVIg (human) during lactation is not All women of childbearing potential
known. No data have been published (including pubertal girls and perimen-
on the excretion of mycophenolic acid opausal women) who begin or restart
(the active metabolite of myco- an immunosuppressive regimen must
phenolate mofetil) in human breast receive contraceptive counseling and
milk.32,33 Rituximab is secreted in the use effective contraception. The pa-
milk of lactating cynomolgus monkeys, tient should begin using her chosen
and IgG is excreted in human breast contraceptive method 4 weeks before
milk. Table 6-3 summarizes the safety starting therapy for MG and continue
of the medications used in MG during contraceptive use during therapy. When
lactation. discontinuing immunosuppressive ther-
Even after a successful pregnancy, the apy, effective contraception should
care of the newborn poses new chal- be continued for 6 months before
lenges that may have an effect on the attempting pregnancy. Mycophenolate
Continuum (Minneap Minn) 2014;20(1):115–127 www.ContinuumJournal.com 123

Copyright © American Academy of Neurology. Unauthorized reproduction of this article is prohibited.


Pregnancy and Myasthenia Gravis

mofetil reduces blood levels of the


TABLE 6-3 Safety of hormones in oral contraceptive pills
Medications Used in
Myasthenia Gravis and could theoretically reduce its effec-
During Lactation tiveness. Two reliable forms of contra-
ception must be used simultaneously
b Safe: Considered Compatible for this particular medication un-
With Breast-Feeding less abstinence is the chosen method.
Pyridostigmine Case 6-3 demonstrates counseling and
Prednisone treatment decisions in a childbearing
Prednisolone female patient before pregnancy and
b Contraindicated: Likely to follow-up management after pregnancy.
Adversely Affect the Newborn
Azathioprine CONCLUSION
Cyclosporine MG can first present during pregnancy
Methotrexate or the postpartum period. Exacerba-
b Caution: No Significant Data tions may also occur in patients with
Available preexisting MG. Pregnancy may affect
Mycophenolate mofetil the course of MG in an unpredictable
Rituximab way, but worsening symptoms most
Immunoglobulin (human) frequently occur in the first trimester
or in the first 3 to 4 weeks postpartum.
The effect of one pregnancy on MG does
not predict the effect in subsequent

Case 6-3
A 23-year-old woman presented with diplopia, ptosis, then generalized
weakness over several months. The diagnosis of myasthenia gravis (MG)
was established by an abnormal single-fiber EMG. She underwent
thymectomy with partial improvement and had further benefit with
azathioprine. Prepregnancy counseling was provided.
She decided to become pregnant and presented to discuss
discontinuation of azathioprine. She reported some fatigue and difficulty
using her arms over her head. Her examination was normal. After
counseling, she agreed to discontinue azathioprine and continue birth
control pills for several months to provide time for azathioprine clearance.
She understood the possibility of her MG worsening and recognized the
risks associated with pregnancy. She was also informed regarding possible
intervention with prednisone, pyridostigmine, or plasma exchange during
the pregnancy, depending on her symptoms. The patient became
pregnant and was followed throughout her pregnancy with no significant
complications apart from minimal weakness of her upper extremities.
She also was followed by a high-risk pregnancy obstetric service. Her fetus
remained very active and was delivered without difficulty.
She did very well through the immediate postpartum period, and
therefore, no medications were reinstituted. Her baby had slight head lag
and was floppy. He had good suck, good grasp, and a robust cry and
showed no signs of difficulty breathing. He could bear weight on his legs
Continued on page 125

124 www.ContinuumJournal.com February 2014

Copyright © American Academy of Neurology. Unauthorized reproduction of this article is prohibited.


Continued from page 124
while supported. He had minimal manifestations of neonatal MG that
did not require treatment and resolved after a few weeks.
At 9 months postpartum, the patient experienced a recurrence of
symptoms and signs, and azathioprine was restarted. She was informed
about the risk of breast-feeding with this medication. Her symptoms
resolved within 3 months with no further complications.
Comment. This case highlights management in a patient before, during,
and after pregnancy. Patients should avoid pregnancy during the first
6 months after discontinuing immunosuppression. Close follow-up in this
special population is partnered with the guidance of a high-risk obstetric team.
The patient should be aware of potential treatment interventions with less
teratogenic potential for the fetus. During the postpartum period, the mother
and the baby are followed closely to determine whether additional treatment
is indicated. If immunosuppressive therapy needs to be initiated, adequate
counseling regarding breast-feeding and contraception must be provided.

pregnancies. Clinical status does not modification based on changes in sever-


reliably predict the course of MG during ity. Table 6-4 summarizes important
pregnancy. Close follow-up of women aspects that require attention and mon-
with MG in the childbearing age is itoring to ensure adequate manage-
essential. Frequent evaluation during ment in patients with MG during the
and before pregnancy allows therapy childbearing age.

TABLE 6-4 Issues for Women of Childbearing Age Who Have


Myasthenia Gravis

b Prepregnancy
Counseling about the effects of pregnancy on myasthenia gravis (MG)
Counseling about the effects of MG on pregnancy
Choice of therapy to optimize response may need to be altered in
anticipation of pregnancy
Consideration of various drugs on fetal health
Counseling of risk of arthrogryposis on the fetus
Monitor long-term effect of therapy, eg, prednisone, immunosuppression,
thymectomy
b Pregnancy
Need for close monitoring including high-risk obstetric clinic
Choice of therapy throughout pregnancy may need to be adjusted
Weighing the risk of immunosuppressive therapy
Thymectomy is not indicated during pregnancy
Monitor for worsening or onset of MG in the first trimester or postpartum
Patient may have improvement in the second and third trimesters
Physiologic changes of reduced diaphragm excursion may stress respiratory
reserve; greater body mass and blood volume increases fatigue
Continued on next page

Continuum (Minneap Minn) 2014;20(1):115–127 www.ContinuumJournal.com 125

Copyright © American Academy of Neurology. Unauthorized reproduction of this article is prohibited.


Pregnancy and Myasthenia Gravis

TABLE 6-4 Issues for Women of Childbearing Age Who Have


Myasthenia Gravis, (continued)

b Fetal Health
Neonatal monitoring needed because of risk of arthrogryposis or transient
neonatal MG
Monitor fetal effect of respiratory inadequacy in mother
Consideration of potential effects of therapy in the fetus, eg,
immunosuppression, prednisone, IVIg, plasma exchange
Consideration of supportive treatment for transient neonatal MG if indicated
b Postpartum
Mother and baby are at risk for weakness
Breast-feeding issues including medications, antibodies present in milk; late
feedings may excessively fatigue patient
When restarting immunosuppressive regimen, patient must receive
contraceptive counseling and use effective contraception

Immunosuppressive medications 6. Maggi L, Andreetta F, Antozzi C, et al. Two


cases of thymoma-associated myasthenia gravis
have potential teratogenic effects, and without antibodies to the acetylcholine receptor.
preferably their use should be dis- Neuromuscul Disord 2008;18(8):678Y680.
continued 4 to 6 months before con- 7. Choi DeCroos E, Hobson-Webb LD, Juel VC,
ceiving. Corticosteroids, plasma et al. Do acetylcholine receptor and striated
exchange, and IVIg have a lower poten- muscle antibodies predict the presence of
thymoma in patients with myasthenia
tial risk, have been used safely during gravis? [published online ahead of print
pregnancy, and therefore are more April 27, 2013]. Muscle Nerve 2013.
preferred choices for treatment of MG doi:10.1002/mus.23882.
exacerbation during pregnancy. An indi- 8. Batocchi A, Majolini L, Evoli A, et al. Course
vidualized and interdisciplinary ap- and treatment of myasthenia gravis during
pregnancy. Neurology 1999;52(3):447Y452.
proach to care is needed throughout
pregnancy and the postpartum period of 9. Ellison J, Thomson AJ, Walker ID, Greer IA.
Thrombocytopenia and leucopenia precipitated
patients with MG and their newborns. by pregnancy in a woman with myasthenia
gravis. BJOG 2000;107(8):1052Y1054.
REFERENCES 10. Igarashi S, Yamauchi T, Tsuji S, et al. A case
1. Myasthenia Gravis Foundation of America. of myasthenia gravis complicated by cyclic
www.myasthenia.org. Accessed October 16, thrombocytopenia [in Japanese]. Rinsho
2013. Shinkeigaku 1992;32(3):321Y323.

2. Ciafaloni E, Massey J. Myasthenia gravis 11. Duff GB. Preeclampsia and the patient with
and pregnancy. Neurol Clin 2004;22(4): myasthenia gravis. Obstet Gynecol 1979;54(3):
771Y782. 355Y358.
3. Schlezinger NS. Pregnancy in myasthenia 12. Mueksch JN, Stevens WA. Undiagnosed
gravis and neonatal myasthenia gravis. Am J myasthenia gravis masquerading as eclampsia.
Med 1955;19(5):718Y720. Int J Obstet Anesth 2007;16(4):379Y382.
4. Plauche WC. Myasthenia gravis in mothers 13. Polizzi A, Huson S, Vincent A. Teratogen
and their newborns. Clin Obstet Gynecol update: maternal myasthenia gravis as a
1991;34(1):82Y99. cause of congenital arthrogryposis.
Teratology 2000;62(5):332Y341.
5. Djelmis J, Sostarko M, Mayer D, Ivanisevic M.
Myasthenia gravis in pregnancy: report on 14. Guidon AC, Massey EW. Neuromuscular
69 cases. Eur J Obset Gynecol Reprod Biol disorders in pregnancy. Neurol Clin 2012;
2002;104(1):21Y25. 30(3):889Y911.

126 www.ContinuumJournal.com February 2014

Copyright © American Academy of Neurology. Unauthorized reproduction of this article is prohibited.


15. Hoff JM, Daltveit AK, Gilhus NE. Myasthenia 24. Watson WJ, Katz VL, Bowes WA Jr.
gravis in pregnancy and birth: identifying Plasmapheresis during pregnancy. Obstet
risk factors, optimising care. Eur J Neurol Gynecol 1984;76(3 pt 1):451Y457.
2007;14(1):38Y43.
25. Rodriguez-Pinilla E, Martinez-Frias ML.
16. O’Carroll P, Bertorini TE, Jacob G, Mitchell Corticosteroid during pregnancy and oral
CW, Graff J. Transient neonatal myasthenia clefts: a case-control study. Teratology
gravis in a baby born to a mother with 1998;58(1):2Y5.
new-onset anti-MuSK-mediated myasthenia
26. Kaaja R, Julkunen A, Ammala P, et al.
gravis. J Clin Neuromuscul Dis 2009;11(2):
Intravenous immunoglobulin treatment of
69Y71.
pregnant patients with recurrent pregnancy
17. Niks EH, Verrips A, Semmekrot BA, et al. A losses associated with antiphospholipid
transient neonatal myasthenic syndrome antibodies. Acta Obtet Gynecol Scand
with anti-MUSK antibodies. Neurology 1993;72(1):63Y66.
2008;70(14):1215Y1216.
27. Armenti VT, Radomski JS, Moritz MJ, et al.
18. Ahlsten G, Lefvert AK, Osterman PO, et al. Report from the National Transplantation
Follow-up study of muscle function in Pregnancy Registry (NTPR): outcomes of
children of mothers with myasthenia gravis pregnancy after transplantation. Clin
during pregnancy. J Child Neurol 1992; Transpl 2001;97Y105.
7(3):264Y269.
28. Kuczkowski KM. Labor analgesia for the
19. Oskoui M, Jacobson L, Chung WK, et al. parturient with neurological disease: what
Fetal acetylcholine receptor inactivation does an obstetrician need to know? Arch
syndrome and maternal myasthenia gravis. Gynecol Obstet 2006;274(1):41Y46.
Neurology 2008;71(24):2010Y2012.
29. Carr SR, Gilchrist JM, Abuelo DN, Clark D.
20. Hoff J, Daltveit A, Gilhus N. Myasthenia Treatment of antenatal myasthenia
gravis: consequences for pregnancy, gravis. Obstet Gynecol 1991;78(3 pt 2):
delivery, and the newborn. Neurology 485Y489.
2003;61(10):1362Y1366. 30. Zhang J, Xie F, Delzell E, et al. Association
21. Gardnerova M, Eymard B, Morel E, et al. between vaccination for herpes zoster and
The fetal/adult acetylcholine receptor risk of herpes zoster infection among older
ratio in mothers with myasthenia gravis patients with selected immune-mediated
as a marker for transfer of the disease to the diseases. JAMA 2012;308(1):43Y49.
newborn. Neurology 1997;48(1):50Y54. 31. Centers for Disease Control and Prevention.
22. Wen JC, Liu TC, Chen YH, et al. No increased Recommended adult immunization
scheduleVUnited States, 2012. JAMA
risk of adverse pregnancy outcomes for
women with myasthenia gravis: a 2012;308(1):22Y23.
nationwide population-based study. Eur J 32. American Academy of Pediatrics Committee
Neurol 2009;16(8):889Y894. on Drugs. Transfer of drugs and other
chemicals into human breast milk. Pediatrics
23. US Food and Drug Administration
2001;108(3):776Y789.
pregnancy categories, drug safety and
availability. www.accessdata.fda.gov/scripts/ 33. Drugs.com. Pregnancy and breastfeeding
cder/drugsatfda/index.cfm. Accessed warnings. www.drugs.com/pregnancy.
December 9, 2013. Accessed December 9, 2013.

Continuum (Minneap Minn) 2014;20(1):115–127 www.ContinuumJournal.com 127

Copyright © American Academy of Neurology. Unauthorized reproduction of this article is prohibited.


Review Article

Headache in Pregnancy
Address correspondence to
Professor E. Anne MacGregor,
Barts Sexual Health Centre,
St Bartholomew’s Hospital,
London EC1A 7BE, E. Anne MacGregor, MB BS, MD, FFSRH, MICR
United Kingdom,
e.macgregor@qmul.ac.uk.
Relationship Disclosure:
Professor MacGregor has ABSTRACT
acted as a paid consultant to Purpose of Review: This article provides an overview of the diagnosis and
and/or her department has
received research funding management of primary and secondary headaches that may occur during pregnancy
from Addex Therapeutics; and postpartum. Headache presenting in pregnancy is of significant concern to the
Allergan, Inc; AstraZeneca; affected woman. Quick and correct diagnosis leads to the optimal management,
Berlin-Chemie AG;
BTG International Ltd; minimizing risks to the pregnancy.
Endo Pharmaceuticals Inc; Recent Findings: Several strategies have been developed to distinguish secondary
GlaxoSmithKline; the headaches that need urgent assessment and management from benign primary and
Menarini Group; Merck & Co,
Inc; POZEN Inc; and UniPath. secondary headaches and to minimize the risk of misdiagnosis. Recent guidelines for
Unlabeled Use of the drug treatment of headaches are considered in the context of updated
Products/Investigational information on the safety of drugs in pregnancy and lactation.
Use Disclosure:
Professor MacGregor
Summary: Primary headaches are common and typically improve during pregnancy.
discusses the use of several Management during pregnancy and lactation is similar to management in the
drugs for the treatment of nonpregnant state, with a few exceptions. Secondary causes of headache that are
headaches, none of which are
labeled by the US Food and
more likely to occur during pregnancy include cerebral venous thrombosis, posterior
Drug Administration for use in reversible encephalopathy syndrome resulting from eclampsia, postYdural puncture
pregnancy. headache, stroke, and pituitary apoplexy.
* 2014, American Academy
of Neurology.
Continuum (Minneap Minn) 2014;20(1):128–147.

INTRODUCTION effective treatment of headaches dur-


Headaches are common during the ing pregnancy and lactation.
reproductive years and are just as
likely to occur in pregnancy as in CLASSIFICATION OF HEADACHES
nonpregnant women. Most headaches The International Classification of Head-
will be benign with no impact on the ache Disorders categorizes primary and
pregnancy. However, the potential secondary headaches.2 Primary head-
effects of drugs on the fetus and the aches account for the majority of head-
increased risk of certain secondary aches during pregnancy. Of the primary
headaches during pregnancy call for headaches, tension-type headache and
careful assessment and management migraine generally improve during
of the pregnant woman presenting pregnancy. The effect of pregnancy
with headaches. on cluster headache is limited because
Approximately half of the pregnan- of the rarity of the condition, and the
cies in the United States are unplanned; data are conflicting. A common cause of
of these, over 40% continue to birth. daily headache in a patient with a
Many women take medication during history of primary headache is medica-
pregnancy, with each woman taking an tion overuse headache. Secondary
average of four to five different medica- causes of headache that are more likely
tions.1 Drugs and other teratogens to occur during pregnancy include
exert their greatest effects on the fetus cerebral venous thrombosis, posterior
during the second and third months of reversible encephalopathy syndrome
gestation, so health care providers (PRES) resulting from eclampsia,
need to advise women on safe and postYdural puncture headache, stroke,

128 www.ContinuumJournal.com February 2014

Copyright © American Academy of Neurology. Unauthorized reproduction of this article is prohibited.


KEY POINTS
and pituitary apoplexy (Table 7-1). In secondary causes of headache that may h Of the primary
addition, headaches can be symptom- need urgent assessment (Table 7-2). headaches, tension-type
atic of emotional stress.3 Because secondary headaches can oc- headache and migraine
cur in a patient with a long-standing generally improve
ASSESSING THE PREGNANT history of primary headache, it is during pregnancy.
WOMAN WITH HEADACHE important to elicit new symptoms. This h A common cause of
The history should focus on estab- approach can separate those who need daily headache in a
lishing the likelihood that the headache further investigation from those with patient with a history
has some secondary cause or defining it benign secondary headaches or typical of primary headache is
among the primary headache disorders. histories of primary headaches. The medication overuse
Specific questions can help to evaluate latter can be reassured, treated, and headache.
h Because secondary
TABLE 7-1 International Classification of Headache Disorders, headaches can occur
3rd Edition (Beta Version)a in a patient with a
long-standing history of
Primary Headaches primary headache, it is
1. Migraine important to elicit new
2. Tension-type headache symptoms.

3. Trigeminal autonomic cephalalgias


4. Other primary headache disorders
Secondary Headaches
5. Headache attributed to trauma or injury to the head and/or neck
6. Headache attributed to cranial or cervical vascular disorder
6.1 Headache attributed to ischemic stroke or TIA
6.2.1 Headache attributed to nontraumatic intracerebral hemorrhage
6.2.2 Headache attributed to nontraumatic subarachnoid hemorrhage
6.3 Headache attributed to unruptured vascular malformation
6.6 Headache attributed to cerebral venous thrombosis
6.7.3 Headache attributed to reversible cerebral vasoconstriction syndrome
6.9 Headache attributed to pituitary apoplexy
7. Headache attributed to nonvascular intracranial disorder
7.1.1 Headache attributed to idiopathic intracranial hypertension
7.2.1 PostYdural puncture headache
8. Headache attributed to a substance or its withdrawal
8.2 Medication-overuse headache
9. Headache attributed to infection
10. Headache attributed to disorder of homeostasis
10.3.4 Headache attributed to preeclampsia or eclampsia
11. Headache or facial pain attributed to disorder of the cranium, neck, eyes,
ears, nose, sinuses, teeth, mouth, or other facial or cervical structures
12. Headache attributed to psychiatric disorder
Painful Cranial Neuropathies, Other Facial Pains, and Other Headaches
13. Painful cranial neuropathies and other facial pains
14. Other headache disorders
a
Data from Headache Classification Committee of the International Headache Society, Cephalalgia.2
cep.sagepub.com/content/33/9/629.full.

Continuum (Minneap Minn) 2014;20(1):128–147 www.ContinuumJournal.com 129

Copyright © American Academy of Neurology. Unauthorized reproduction of this article is prohibited.


Headache in Pregnancy

TABLE 7-2 Warning Signs and Symptoms for Secondary Headaches

Signs and Symptoms Possible Diagnosis


Thunderclap headache (intense headache Subarachnoid hemorrhage
with abrupt or ‘‘explosive’’ onset) Postpartum angiopathy
Pituitary apoplexy
Headache with atypical aura (duration TIA
91 hour, or including motor weakness) Ischemic stroke

Fever Meningitis
History of HIV or syphilis Meningoencephalitis
History of cancer Secondary brain metastases
Postural headache (avoids lying flat Subarachnoid hemorrhage
or standing up) Cerebral venous thrombosis
Intracranial hypotension
Progressive headache worsening over Intracranial space-occupying lesion
weeks or months; cognitive or personality Idiopathic intracranial hypertension
changes symptoms of raised intracranial
pressure (drowsiness, postural-related Cerebral venous thrombosis
headache, vomiting); new-onset seizures; Elampsia
progressive neurologic deficit (weakness,
sensory loss, dysphasia, ataxia)
Visual disturbance/scotoma Preeclampsia
Idiopathic intracranial hypertension
TIA = transient ischemic attack; HIV = human immunodeficiency virus.

followed by the neurologist or primary cluding papilledema or hemorrhages


care physician as appropriate. on funduscopy, neck stiffness, altered
Risk factors for pregnancy-related consciousness, or weakness.
stroke include diabetes mellitus,
migraine with aura, preexisting hyper- Symptoms and Signs of
tension, hypertensive disorders of Secondary Headaches
pregnancy, sickle cell disease, struc- in Pregnancy
tural heart disease, and thrombophilia. Although stroke can occur during
Complications of pregnancy and pregnancy, it is more likely to occur
the puerperium associated with in- in the 6 weeks postpartum. Focal
creased risk of stroke include anemia, neurologic symptoms and signs with
hyperemesis gravidarum, thrombocyto- or without alterations in consciousness
penia, postpartum hemorrhage, and are typical of stroke. Headache accom-
infection. Depressive disorders often panies ischemic stroke in 17% to 34%
present with frequent prepregnancy of cases and is usually nonspecific in
headache, which is a strong predictor quality and of moderate intensity.
of poor general and emotional health Subarachnoid hemorrhage typically
during pregnancy.3,4 presents as an abrupt-onset thunder-
Examination should focus on assess- clap headache that is intense and
ment for signs of concerning diagnoses incapacitating. It is often unilateral and
such as infection or hemorrhage; severe accompanied by nausea, vomiting, neck
hypertension; and neurologic signs in- stiffness, and fluctuating consciousness.

130 www.ContinuumJournal.com February 2014

Copyright © American Academy of Neurology. Unauthorized reproduction of this article is prohibited.


If neuroimaging is negative, lumbar it is associated with seizures, focal
puncture is imperative. Sentinel head- neurologic symptoms, and raised intra-
ache associated with a leaking cerebral cranial pressure.
aneurysm can precede subarachnoid The combination of hypertension and
hemorrhage in up to 50% of cases, proteinuria in a pregnant woman suggests
typically occurring a couple of weeks preeclampsia and needs urgent manage-
before the aneurysm ruptures. An acute ment to reduce blood pressure and
third nerve palsy, with retro-orbital pain prevent eclampsia. Headache is a senti-
and a dilated pupil, indicates impending nel symptom of eclamptic seizure in
rupture of a posterior communicating around three-quarters of women with
cerebral artery aneurysm. preeclampsia. The headache is progres-
A nonspecific unremitting headache sive and refractory to analgesia. Many
is a common symptom of cerebral patients with preeclampsia present with
venous thrombosis, as seen in Case 7-1. headache due to hypertensive enceph-
Cerebral venous thrombosis is esti- alopathy (ie, PRES) without seizures.
mated to occur in 1 per 2500 to 10,000 Blurred vision, scotoma, and flashing
pregnancies and is most likely in lights in such patients can be misdiag-
women with hypercoagulability. Head- nosed as migraine aura. Within the
ache may be the only finding, but often spectrum of preeclampsia/eclampsia,

Case 7-1
A 26-year-old woman who was 25 weeks pregnant with her first child
was admitted after having a generalized tonic-clonic seizure. During the
8 days before admission, she had a constant, nonthrobbing headache
associated with nausea and vomiting. She had taken the combined oral
contraceptive pill from 18 to 25 years of age and had no problems other
than headaches in the hormone-free interval, which resolved when she
took the pill continuously, without a break; she stopped taking the pill
when she was 25 years old because she wished to become pregnant and
conceived 6 months later. Apart from nausea and vomiting during early
pregnancy, she had been well.
On examination, blood pressure was normal, temperature was 36.9-C,
and body mass index was 28.4 kg/m2. Remaining physical and neurologic
assessments, including optic funduscopy, were normal.
Laboratory tests were all within normal ranges. A noncontrast brain
MRI scan and MR venogram revealed a superior sagittal sinus thrombosis.
She was treated with low-molecular-weight heparin for the remainder of
the pregnancy and for 6 weeks postpartum. Her headache resolved within
3 days of starting treatment, and the remainder of her pregnancy was
uneventful, with a spontaneous normal delivery of a healthy boy at 39 weeks.
Comment. This pregnant migraineur presented with a new headache.
While it is more common in the peripartum, cerebral venous thrombosis
should be excluded in patients with progressive headache with seizure
during pregnancy. Focal neurologic deficits and coma can also occur.
Funduscopy may show signs of raised intracranial pressure, but brain
imaging is indicated even if normal. This woman was fortunate to receive
an early diagnosis and management, which led to a full recovery without
compromising the pregnancy. Delayed diagnosis can result in permanent
dysfunction and death.

Continuum (Minneap Minn) 2014;20(1):128–147 www.ContinuumJournal.com 131

Copyright © American Academy of Neurology. Unauthorized reproduction of this article is prohibited.


Headache in Pregnancy

KEY POINTS
h Each primary headache while some patients will present with mass lesion) reveals increased CSF
has a specific pattern PRES, others will present with the rever- pressure with normal CSF chemistry.
of symptoms in the sible cerebral vasoconstriction syn- Headache improves with reduction in
absence of clinical signs. drome (RCVS), called postpartum CSF pressure.
h Headaches that lack angiopathy when it occurs in the post- PostYdural puncture headache af-
associated symptoms, partum period. This is a cerebral vaso- fects one-third of patients after lumbar
in an otherwise well constriction syndrome that can be puncture with onset typically within 5
person who is not complicated by ischemic stroke. It usu- days of the procedure. The headache
overusing medication, ally occurs in the week after an uncom- is postural, worsening on standing and
are likely to be plicated pregnancy and delivery. It can improving when lying flat. Associated
tension-type headaches. mimic subarachnoid hemorrhage, symptoms include neck stiffness, tin-
h Recurrent episodic presenting as a thunderclap headache nitus, hyperacusia, photophobia, and
headaches that last associated with fluctuating neurologic nausea. In the majority of cases, the
between 4 and deficits and sometimes seizures. Angi- headache resolves within a week with-
72 hours and are ography reveals a classic ‘‘string of out further intervention. If conserva-
associated with beads’’ appearance with areas of steno- tive methods fail, an epidural blood
photophobia, nausea, sis and dilation in multiple intracranial patch should be considered.
and disability in an vessels. There is strong evidence for the
otherwise well person Symptoms and Signs of Primary
benefit of magnesium sulfate for
are typical features of Headaches in Pregnancy
preeclampsia/eclampsia. The authors
migraine.
of the article ‘‘Cerebrovascular Disor- Each primary headache has a specific
ders Complicating Pregnancy’’ in this pattern of symptoms in the absence of
issue of recommend clinical signs (Figure 7-1). The history
this therapy for preeclampsia/eclampsia is diagnostic, and investigations are
in all of its forms, along with prompt only required to rule out a suspected
control of blood pressure. For further secondary headache.
details regarding PRES and RCVS, refer Tension-type headache. Head-
to ‘‘Cerebrovascular Disorders Compli- aches that lack associated symptoms,
cating Pregnancy’’ by Drs Steven K. in an otherwise well person who is not
Feske and Aneesh B. Singhal in this overusing medication, are likely to be
issue of . tension-type headaches.
Pituitary apoplexy, resulting from In the majority of women, tension-
spontaneous hemorrhagic infarction type headache will improve during
of the pituitary gland, is rare but life pregnancy.
threatening. It presents with a retro- Migraine. Recurrent episodic head-
orbital, frontal, or diffuse thunderclap aches that last between 4 and 72
headache associated with nausea and hours and are associated with photo-
vomiting, fluctuating consciousness, phobia, nausea, and disability in an
hypotension, and visual loss. If MRI is otherwise well person are typical fea-
not available, an urgent pituitary CT tures of migraine.
scan is indicated. Up to 60% to 70% of women with
Idiopathic intracranial hypertension preexisting migraine report fewer mi-
presents with a diffuse, nonthrobbing, graine attacks during pregnancy.
daily headache aggravated by cough- Women with a history of menstrual
ing and straining. Signs include or menstrually related migraine with-
papilledema, an enlarged blind spot, out aura are more likely to report
visual field defect, or sixth nerve palsy. improvement than for those with no
Lumbar puncture (performed after evidence of a menstrual association.6
brain imaging excludes an intracranial Migraine is likely to continue throughout
132 www.ContinuumJournal.com February 2014

Copyright © American Academy of Neurology. Unauthorized reproduction of this article is prohibited.


FIGURE 7-1 Algorithm for the diagnosis of primary headaches.
39
Adapted with permission from MacGregor A, Wiley Blackwell. B 2008, John Wiley and Sons.

pregnancy and postpartum if it does to continue to have attacks during


not improve by the end of the pregnancy. As seen in Case 7-2,
first trimester. Breast-feeding is en- women may develop aura for the first
couraged because it maintains the time during pregnancy, irrespective of
protective effect of pregnancy on whether they have a past history of
migraine headache during the post- migraine without aura.7
partum period. Women can be reassured that
Women who have preexisting mi- there is evidence that migraine does
graine with aura do not experience not adversely affect the outcome of
the same benefit and are more likely pregnancy in otherwise healthy women.8

Continuum (Minneap Minn) 2014;20(1):128–147 www.ContinuumJournal.com 133

Copyright © American Academy of Neurology. Unauthorized reproduction of this article is prohibited.


Headache in Pregnancy

KEY POINT
h Cluster headache is
frequently
Case 7-2
A 34-year-old woman was 32 weeks pregnant with her first child. While
misdiagnosed as
at work she developed blurred vision to the right, difficulty with speech,
migraine despite
and tingling, numbness, and weakness of her right arm lasting 20 minutes.
stereotypical symptoms
This was followed by nausea and a left temporal headache lasting 5 hours.
of strictly unilateral
By the time she was seen in the emergency department, all symptoms had
headache and
resolved apart from the headache. She had a history of migraine without
autonomic symptoms
aura since the age of 15 years but no other history of note. She took no
lasting up to 2 hours in
regular medication other than pregnancy supplements. On examination,
clusters typically lasting
her blood pressure was 110/70 mm Hg, temperature was 36.8-C, and
6 to 8 weeks.
body mass index was 23.6 kg/m2. Remaining physical and neurologic
assessments, including optic funduscopy, were normal.
Laboratory tests, thrombophilia screen, EEG, echocardiogram, and
noncontrast brain MRI were normal. By the time the investigations were
complete, the headache had resolved, and the patient felt ‘‘back to
normal.’’ Migraine with aura was considered the most likely diagnosis. She
was discharged with advice to return if her condition deteriorated at all
and with an analgesic/antiemetic combination to take at the onset of
symptoms if they recurred. She had three further attacks during the
remainder of the otherwise uneventful pregnancy, all of which responded
to medication.
Comment. Although migraine without aura typically improves during
pregnancy, migraine with aura can occur for the first time in women with
or without previous attacks of migraine without aura. While migraine
itself, with or without aura, poses no risk to the pregnancy, migraine in
pregnancy is a risk factor for hypertensive disorders of pregnancy, so these
women should be monitored carefully.

However, migraine during pregnancy graine despite stereotypical symptoms


is associated with increased risk of strictly unilateral headache and
of arterial and venous thrombosis, autonomic symptoms lasting up to 2
preeclampsia, and gestational hy- hours in clusters typically lasting 6 to
pertension. 9Y17 Pregnant women 8 weeks. Data on cluster headache
with migraine should be moni- during pregnancy are limited, but it
tored for these conditionsVfor ex- has been noted that women who
ample, rising systolic or diastolic blood have their first cluster headache before
pressure should prompt a check for their first pregnancy have fewer chil-
proteinuria. dren than those who already have
Disorders such as thrombocytope- children at the time of their first
nia, cerebral venous sinus thrombosis, attack. 18,19 While the possibility
or imminent eclampsia may mimic of hypofertility has been raised, the
aura and should be excluded in wom- more likely explanation is that women
en presenting with their first attack of choose not to conceive as they
aura during pregnancy. are concerned about the effects of
Cluster headache. Cluster head- medication for cluster headache on
ache affects fewer than 1 in 500 the outcome of pregnancy.19
people, and in contrast to migraine Medication overuse headache.
and tension-type headache, it is much Medication overuse headache affects
more common in men than in women. people with a history of a primary
It is frequently misdiagnosed as mi- headache that has become more frequent

134 www.ContinuumJournal.com February 2014

Copyright © American Academy of Neurology. Unauthorized reproduction of this article is prohibited.


(Case 7-3). Medication overuse head- during pregnancy, see the article ‘‘Neu-
ache should always be excluded in roradiology in Women of Childbearing
anyone using symptomatic treatments Age’’ by Drs Riley Bove and Joshua
for headache more often than 2 to 3 Klein in this issue of .
days a week or taking frequent caf-
feine in any form. It is an avoidable
cause of treatment failure, as head- MANAGEMENT OF PRIMARY
ache becomes resistant to all lines of HEADACHES IN PREGNANCY
management until symptomatic drugs Nonpharmacologic
are withdrawn. Drug withdrawal typi- Nonpharmacologic treatment should
cally resolves headache frequency, be considered as the initial step in the
and residual episodic headache can management of tension-type head-
be treated appropriately. ache or migraine but is ineffective for
cluster headache, which can only be
INVESTIGATIONS DURING controlled with medication.
PREGNANCY AND LACTATION The most frequently reported trig-
Investigations are indicated only to gers for tension-type headache and
exclude suspected secondary head- migraine are stress (mental or physi-
ache resulting from underlying pathol- cal), irregular or inappropriate meals,
ogy. For a discussion of brain imaging high intake or withdrawal of coffee

Case 7-3
A 27-year-old woman presented at 17 weeks of pregnancy with daily
headaches. She had had migraine without aura since she was 11 years old.
One year earlier, she had developed daily headaches and was diagnosed
with medication overuse headache as a consequence of frequent use of
triptans. She stopped this medication at that time and was started on
topiramate. She had a very severe headache for a week, after which
symptoms resolved. The pattern of headache settled to one migraine
attack every 4 to 6 weeks, which responded to a triptan, and she was free
of symptoms between attacks.
At the current presentation, her diaries showed a fluctuating pattern of
headache over the preceding months, with a gradual increase in headache
frequency beginning when she stopped topiramate 3 months earlier to
become pregnant. After she realized she was pregnant, she used only
acetaminophen with codeine, which she had been taking most days, as she
was worried that she would develop a severe migraine and be unable to
get to work. Vital signs were within normal range, and physical and
neurologic examination was unremarkable.
Medication overuse headache was diagnosed, and she chose to stop
analgesics without additional support. She was advised that a severe
migraine could be treated with IV magnesium sulfate, if necessary. At
follow-up 2 weeks later, she reported that she had been unwell for the
first week since stopping analgesics but felt much better in the last week
and had not had any further headaches.
Comment. Medication overuse headache is a common cause of daily
headaches in patients with a history of primary headaches. The key finding
is the use of symptomatic drugs for headache on more days than not in the
absence of any red flags in the history and in the presence of a normal
examination.

Continuum (Minneap Minn) 2014;20(1):128–147 www.ContinuumJournal.com 135

Copyright © American Academy of Neurology. Unauthorized reproduction of this article is prohibited.


Headache in Pregnancy

KEY POINTS
h Pregnant women with and other caffeine-containing drinks, women are often very concerned
tension-type headache dehydration, sleep disorders, too about the effects that medication
or migraine should be much or too little sleep, and reduced may have on the pregnancy. The
encouraged to avoid or excessive physical exercise.20,21 woman should be given sufficient
skipping meals, to take Pregnant women with tension-type information about any known risks to
regular exercise, to drink headache or migraine should be en- make her own decision about drug
plenty of fluids, and to couraged to avoid skipping meals, to use, with clear documentation of the
maintain a regular sleep take regular exercise, to drink plenty discussion.
pattern. of fluids, and to maintain a regular The US Food and Drug Adminis-
h Nondrug therapies sleep pattern. Alcohol and smoking tration (FDA) pregnancy labeling has
such as relaxation, are potentially harmful to the fetus and five categories: A, B, C, D, and X
biofeedback, and should be avoided during pregnancy. (Appendix A). These categories can
physical therapy are Nondrug therapies such as relaxa- be misleading because categories C,
safe and may be tion, biofeedback, and physical thera- D, and X are not only based on risk
effective in pregnancy.
py are safe and may be effective in but consider risk versus benefit, so
pregnancy.22,23 Acupuncture may also drugs in each of these three catego-
treat nausea and vomiting in preg- ries may pose similar risks. Several
nancy in addition to headache.24Y26 additional sources in the United States
Coenzyme Q10 daily is effective for provide information on the safety of
migraine prophylaxis and, when taken drugs during pregnancy and lactation,
during pregnancy, has been associated and these have been reviewed to
with a significant reduced risk of pre- produce the evidence for the recom-
eclampsia.27,28 Similarly, magnesium mendations below.
supplements, which can be used for Symptomatic treatment. The
migraine prophylaxis, can halve the symptomatic treatment of headaches
risk of eclampsia, with no evidence of during pregnancy and lactation is
adverse effects on pregnancy.29,30 See the same as for the nonpregnant state,
the section on complementary medi- with some exceptions (Table 7-3
cines during pregnancy and lactation and Table 7-4).
for further details about coenzyme Analgesics. Data from large-cohort
Q10 and magnesium during pregnancy and case-control studies confirm the
and lactation. safety of therapeutic doses (4 g or less
per day) of acetaminophen during
Drug Treatment of Headache in pregnancy and lactation. It is the
Pregnancy and Lactation analgesic of choice for the short-term
Drug use during pregnancy is com- relief of mild to moderate pain and
mon, and many women continue to pyrexia.
use their usual headache medication, While aspirin can be taken during
including triptans, throughout preg- the first and second trimesters of
nancy.31 The most comprehensive pregnancy, it is best avoided near term
guidelines for the drug treatment of because of increased risk of prolonged
primary headaches are those devel- labor, postpartum hemorrhage, and
oped by the European Federation of neonatal bleeding. Aspirin, in common
Neurological Societies. 5,29,32 Most with nonsteroidal anti-inflammatory
drugs are not licensed for use in drugs (NSAIDs), has been associated with
pregnancy so should only be consid- premature closure of the fetal ductus
ered if the potential benefits to the arteriosus. Aspirin is excreted in breast
woman and fetus outweigh the poten- milk, and although occasional use by the
tial risks. As discussed in Case 7-4, mother is unlikely to cause adverse effect,
136 www.ContinuumJournal.com February 2014

Copyright © American Academy of Neurology. Unauthorized reproduction of this article is prohibited.


Case 7-4
A 35-year-old woman presented when she was 13 weeks pregnant with
her second child. She had migraine without aura as a teenager but was
free of attacks during her twenties. Her first pregnancy, 7 years earlier,
had been uneventful; 8 months before this presentation, she had a
miscarriage at 9 weeks. Since the birth of her first child, she had only
occasional attacks, but over the previous year, she had migraine twice a
month that did not always respond to sumatriptan. She had started
amitriptyline 50 mg/d, which reduced the frequency to one attack every 4
to 6 weeks, which she could then control with sumatriptan. She continued
on this combination during the second pregnancy and was concerned that
this was the reason for her miscarriage. She stopped sumatriptan and
amitriptyline as soon as she realized that she was pregnant again. During
the past 12 weeks, she had had three migraine attacks, treated with
acetaminophen and codeine. This treatment was not effective, and she
had missed several days from work. She was concerned that any
medication may increase her chances of another miscarriage but was
equally worried that she would not be able to continue work. Other than
mild pregnancy-related nausea in the mornings, she was fit and well and
took only pregnancy supplements. It was explained to her that migraine
did not have any adverse effect on the outcome of pregnancy and neither
did most migraine medications, including amitriptyline. The importance
of regular snacks and hydration to help migraine and the morning
nausea was discussed. A combination of a nonsteroidal anti-inflammatory
drug and an antiemetic was considered for symptomatic treatment,
understanding that she could use this until the 30th week of pregnancy,
with the option of taking sumatriptan if necessary. Given that migraine
without aura often improves after the first trimester of pregnancy,
preventive treatment was deferred. At her review appointment 6 weeks
later, she reported only one further attack at 11 weeks of pregnancy
and none since.
Comment. Women are often concerned that they are responsible for a
miscarriage, so it is important to remind them that at least 10% of
pregnancies result in miscarriage, most occurring during the first trimester
as a result of a genetic abnormality. Balancing risk and benefit of
medication for migraine is an important consideration particularly since
migraine itself has no adverse effect on the pregnancy. However, an
untreated migraine can result in significant morbidity.

it should not be taken regularly during risk of premature closure of the ductus
breast-feeding because of the theoretical arteriosus and oligohydramnios. The
risk of Reye syndrome and impaired concentration of NSAIDs in breast
platelet function in susceptible infants. milk is very low so treatment during
Nonsteroidal anti-inflammatory breast-feeding is unlikely to affect the
drugs. Ibuprofen is the NSAID of infant.
choice during the first or second Opiates. Although safe for treatment
trimester. NSAIDs should be avoided of moderate to severe pain in preg-
during the third trimester because nancy, opiates are inappropriate for
chronic use or high doses after 30 migraine because they aggravate nausea
weeks are associated with an increased and reduce gastric motility. Chronic use

Continuum (Minneap Minn) 2014;20(1):128–147 www.ContinuumJournal.com 137

Copyright © American Academy of Neurology. Unauthorized reproduction of this article is prohibited.


Headache in Pregnancy

TABLE 7-3 Drugs Used for Acute Treatment of Headache During Pregnancy

FDA Pregnancy
Drug Categorya Notes
Analgesics
Acetaminophen B First-line; amounts in breast milk are much
less than doses usually given to infants
Aspirin C Second line; use in first and second trimesters
only; category D in third trimester
Diclofenac B Second line; use in first and second trimesters
only; category D in third trimester
Ibuprofen B Second line; nonsteroidal anti-inflammatory
drug of choice; use in first and second trimesters
only; category D in third trimester
Indomethacin C Use in first and second trimesters only; avoid
from 30 weeks; category D in third trimester
Mefenamic acid C Use in first and second trimesters; avoid from
30 weeks; category D in third trimester
Naproxen B Use in first and second trimesters only; avoid
from 30 weeks; category D in third trimester
Antiemetics
Metoclopramide B
Prochlorperazine C
Promethazine C
Barbiturates
Butalbital C Category D if used in high doses at term or
for prolonged periods
Ergots
Dihydroergotamine X
Ergotamine X
Opiates Opiates not recommended for migraine
Codeine C
Meperidine B Category D if used in high doses at term or
for prolonged periods
Morphine C
Tramadol C
Triptans
Almotriptan C
Eletriptan C
Frovatriptan C
Naratriptan C
Rizatriptan C
Sumatriptan C Third line; consider for severe unresponsive
attacks
Zolmitriptan C
Other drugs
Prednisone or prednisolone C
FDA = US Food and Drug Administration.
a
Please see Appendix A for the US Food and Drug Administration Pregnancy Category descriptions.

138 www.ContinuumJournal.com February 2014

Copyright © American Academy of Neurology. Unauthorized reproduction of this article is prohibited.


TABLE 7-4 Drugs for Acute Treatment of Headache During Lactation

Drug Level of Risk Notes


Analgesics
Acetaminophen Minimal risk First line; amounts in breast milk are much
less than doses usually given to infants
Aspirin Caution
Diclofenac Minimal risk Second line; short half-life
Ibuprofen Minimal risk Second line; nonsteroidal anti-inflammatory
drug of choice; short half-life; amounts in
breast milk are much less than doses usually
given to infants
Indomethacin Benefit likely to outweigh risk Used therapeutically in infants but drugs with
more data of use in lactation preferred
Mefenamic acid Caution Drugs with more data of use in lactation preferred
Naproxen Benefit likely to outweigh risk Drugs with short half-life preferred
Antiemetics
Cyclizine Minimal risk First line
Metoclopramide Minimal risk
Prochlorperazine Minimal risk
Promethazine Minimal risk First line
Barbiturates
Butalbital Caution
Ergots
Dihydroergotamine Contraindicated
Ergotamine Contraindicated
Opiates Opiates not recommended for migraine
Codeine Benefit likely to outweigh risk May cause infant sedation; non-narcotic
analgesic preferred
Morphine Benefit likely to outweigh risk May cause infant sedation; non-narcotic
analgesic preferred
Morphine Benefit likely to outweigh risk May cause infant sedation; non-narcotic
analgesic preferred
Tramadol Benefit likely to outweigh risk
Triptans
Almotriptan Insufficient data
Eletriptan Benefit likely to outweigh risk
Frovatriptan Insufficient data Long half-life; drugs with short half-life preferred
Naratriptan Insufficient data
Rizatriptan Insufficient data
Sumatriptan Minimal risk Third line; consider for severe unresponsive attacks
Zolmitriptan Insufficient data

of opiates in later pregnancy has been women metabolize opiates slowly,


associated with neonatal withdrawal which can cause higher levels of opiates
symptoms. Adverse effects during lac- to pass into their breast milk, resulting
tation are unlikely. However, some in infantile drowsiness and sedation.
Continuum (Minneap Minn) 2014;20(1):128–147 www.ContinuumJournal.com 139

Copyright © American Academy of Neurology. Unauthorized reproduction of this article is prohibited.


Headache in Pregnancy

Barbiturates. Only limited human Drugs used for prophylaxis. Re-


data are available for butalbital during striction on drugs used for prophylaxis
pregnancy. Phenobarbital has been for headache during pregnancy and
associated with teratogenicity and lactation is greater than on drugs used
can cause neonatal withdrawal and for symptomatic treatment, but a
hemorrhagic disease of the newborn. number of options are available
It is excreted into breast milk and that pose minimal risks to the unborn
accumulates in infants’ blood, causing or breast-feeding child (Table 7-5
sedation and withdrawal symptoms. and Table 7-6).
Antiemetics. Metoclopramide, pro- Low-dose aspirin. Low-dose aspirin
chlorperazine, and promethazine have (no more than 150 mg/d) has been
been used widely in pregnancy and extensively studied in preeclampsia
lactation without reports of adverse effects. during pregnancy, with no increase
Triptans. Women who have taken in bleeding complications and negligi-
triptans during pregnancy can be ble effects on the ductus arteriosus.
reassured that there is evidence to Beta-blockers. If prophylaxis is con-
support no adverse outcome. How- sidered necessary during pregnancy,
ever, this is not the same as evidence to the lowest effective doses of propra-
confirm safety, so continued use dur- nolol or metoprolol are the drugs of
ing pregnancy is not recommended choice. If beta-blockers are used in the
unless no other treatment is effective. third trimester, treatment should be
Data from the Sumatriptan/ stopped 2 to 3 days before delivery in
Naratriptan/Treximet Pregnancy Regis- order to reduce the likelihood of fetal
try (from 1996 to 2011) are reassuring bradycardia and a reduction in uterine
and confirm that inadvertent exposure contraction. Infants exposed to pro-
to sumatriptan during pregnancy has pranolol in utero should be monitored
not been associated with adverse out- for neonatal bradycardia, hypotension,
comes. Therefore, if clinically appro- and hypoglycemia. Propranolol or met-
priate, sumatriptan may be considered oprolol are the beta-blockers of choice
for the acute treatment of migraine during lactation.
during pregnancy if no other treat- Antidepressants. Low-dose amitrip-
ments are effective. However, a small tyline 10 mg/d to 25 mg/d is an option.
increased risk of specific birth defects While data are conflicting regarding
cannot be excluded. The low level of limb deformities associated with use
excretion of sumatriptan in breast of high doses of amitriptyline during
milk suggests that continued breast- pregnancy, no association has been
feeding following its use does not reported with low doses between 10
pose a significant risk to the infant. and 50 mg/d used for pain manage-
Data on other triptans are insufficient. ment. Where possible, it is recom-
Ergots. Ergotamine and dihydroer- mended that the dose be tapered 3 to
gotamine are contraindicated, because 4 weeks before delivery. Otherwise, the
uterine hypertonicity and vascular dis- physician should monitor the baby for
ruption increase the risk of miscar- adverse effects such as drowsiness,
riage. They should not be used during jitteriness, hyperexcitability, and suck-
breast-feeding because of reported ling problems. Amitriptyline or nortrip-
nausea, vomiting, diarrhea, and weak- tyline are barely detectable in breast
ness in the breast-feeding infant and milk and treatment of the mother with
suppression of prolactin secretion and these drugs is unlikely to affect the
lactation in the mother. infant adversely.
140 www.ContinuumJournal.com February 2014

Copyright © American Academy of Neurology. Unauthorized reproduction of this article is prohibited.


TABLE 7-5 Drugs Used for Prophylaxis of Headache During Pregnancy

FDA Pregnancy
Drug Categorya Notes
Angiotensin-converting
enzyme inhibitors
Lisinopril D Category C in first trimester
Angiotensin receptor blockers
Candesartan D Category C in first trimester
Antiepileptics
Gabapentin C
Topiramate D
Valproic acid X Teratogenic
Antiplatelet drugs
Aspirin C Doses e150 mg/d
Beta-blockers
Atenolol D Propranolol or metoprolol
preferred
Metoprolol C
Nadolol C Propranolol or metoprolol
preferred
Propranolol C Beta-blocker of choice
Timolol C Propranolol or metoprolol
preferred
Calcium channel blockers
Nifedipine C
Verapamil C No data on high doses in
pregnancy
SSRIs/SNRIs Amitriptyline or
nortriptyline preferred
Citalopram C
Escitalopram C
Fluoxetine C
Sertraline C
Venlafaxine C
Tricyclics
Amitriptyline C
Nortriptyline C
Other drugs
Botulinum toxin C
Lithium D Teratogenic
FDA = US Food and Drug Administration; SSRIs = selective serotonin reuptake inhibitors; SNRIs =
serotonin-norepinephrine reuptake inhibitors.
a
Please see Appendix A for the US Food and Drug Administration Pregnancy Category descriptions.

Data are conflicting on the risk of norepinephrine reuptake inhibitor


congenital malformations following use (SNRI). Although several studies have
in early pregnancy of selective serotonin demonstrated no statistically significant
reuptake inhibitor (SSRI) or serotonin- increase in risk, an increased risk of

Continuum (Minneap Minn) 2014;20(1):128–147 www.ContinuumJournal.com 141

Copyright © American Academy of Neurology. Unauthorized reproduction of this article is prohibited.


Headache in Pregnancy

TABLE 7-6 Drugs Used for Prophylaxis of Headache During Lactation

Drug Level of Risk Notes


Angiotensin II antagonists
Candesartan Insufficient data
Angiotensin-converting
enzyme inhibitors
Lisinopril Insufficient data
Antiepileptics
Gabapentin Benefit likely to Doses up to 2.1 g/d produce relatively low levels
outweigh risk in infant serum; monitor infant for side effects
Topiramate Benefit likely to Doses up to 200 mg/d produce relatively low levels
outweigh risk in infant serum; monitor infant for side effects
Valproic acid Benefit likely to Low levels in infant serum; theoretical risk of
outweigh risk infant hepatotoxicity; monitor for jaundice
Antiplatelet drugs
Aspirin Minimal risk Doses e150 mg/d
Beta-blockers Propranolol or metoprolol preferred
Atenolol Concern
Metoprolol Benefit likely to
outweigh risk
Nadolol Concern
Propranolol Minimal risk Beta-blocker of choice; low levels in breast milk;
no adverse reactions attributed
Timolol Concern Variable excretion into breast milk; limited data
Calcium channel blockers
Nifedipine Minimal risk
Verapamil Minimal risk Doses up to 360 mg/d produce low serum levels
in infant but no adverse reactions attributed
SSRIs/SNRIs Amitriptyline or nortriptyline preferred
Citalopram Concern Infant exposure lower with escitalopram
Escitalopram Benefit likely to
outweigh risk
Fluoxetine Concern Higher levels in breast milk than other SSRIs and
long-acting metabolite; other SSRIs preferred
unless patient established on treatment
Paroxetine Benefit likely to Low levels in breast milk
outweigh risk
Sertraline Benefit likely to Low levels in breast milk; no adverse reactions
outweigh risk attributed
Venlafaxine Benefit likely to Excreted into breast milk
outweigh risk
Tricyclics
Amitriptyline Minimal risk Low levels in breast milk
Nortriptyline Minimal risk Low or undetectable levels in breast milk
Other drugs
Botulinum toxin Insufficient data
Lithium Contraindicated Teratogenic
SSRIs = selective serotonin reuptake inhibitor; SNRIs = serotonin-norepinephrine reuptake inhibitor.

142 www.ContinuumJournal.com February 2014

Copyright © American Academy of Neurology. Unauthorized reproduction of this article is prohibited.


cardiovascular malformations was Angiotensin-converting enzyme
suggested in one large study. However, inhibitors. Lisinopril is used for mi-
the absolute risk appears to be small. graine prophylaxis. In common with
Use of any SSRI in late pregnancy may other angiotensin-converting enzyme
result in a mild, transient neonatal (ACE) inhibitors, it poses no signifi-
withdrawal syndrome, which includes cant fetal risk during the first trimes-
continuous crying, irritability, jitteriness, ter. During the second and third
or restlessness; shivering; fever; tremors; trimesters, it is toxic and teratogenic,
hypertonia or rigidity; tachypnea or causing prematurity, intrauterine
respiratory distress; feeding difficulty; growth retardation, renal tubular dys-
sleep disturbance; hypoglycemia; and plasia, severe oligohydramnios leading
seizures. Exposure to SSRIs beyond 20 to fetal distress, lung hypoplasia, skull
weeks of gestation has been associated hypoplasia, limb contractures, neona-
with persistent pulmonary hypertension tal hypotension, and anuria.
of the newborn, although the estimated Botulinum toxin. No human data
absolute risk is less than 0.5%. have been published regarding terato-
Antiepileptics. Risk of oral clefts is genicity of botulinum toxin, licensed
increased after exposure to topiramate for chronic migraine, in pregnancy
in pregnancy. Sodium valproate is and lactation.
associated with a high risk of fetal Lithium. Exposure to therapeutic
abnormalities and is contraindicated doses of lithium in pregnancy are
during pregnancy in the absence of associated with teratogenic and other
epilepsy; it can be used during lacta- toxic effects in the fetus and neonate.
tion. Currently available data do not Opinions are conflicting as to whether
suggest that gabapentin has any ad- lithium should be stopped during the
verse effect on pregnancy, but it is first trimester of pregnancy and
considered at best to be a third-line restarted in the second trimester. If
migraine prophylactic agent. Refer to treatment with lithium is indicated,
the article ‘‘Pregnancy and Epilepsy’’ combined specialist supervision by a
by Dr Cynthia Harden in this issue of neurologist and obstetrician is essen-
for a detailed discussion tial. Lithium is excreted in breast milk
of antiepileptic agents and pregnancy. and has been associated with adverse
Calcium channel blockers. Data effects in the infant.
for verapamil are limited. It has a
tocolytic effect on the uterus, so Approach to Drug Treatment of
should be avoided in late pregnancy. Tension-Type Headache During
No association with congenital anom- Pregnancy and Lactation
alies has been reported. Verapamil is Symptomatic treatment with nonopioid
excreted in human milk with potential analgesics is appropriate for episodic
for adverse reactions in infants, which attacks occurring on fewer than 2 days
is of particular concern with high per week.5 Prophylactic medications
doses necessary for cluster headache, are seldom necessary and are only
although no data are available. indicated for chronic tension-type
Angiotensin receptor blockers. headache or when headaches regularly
Case reports have described congenital occur more than 2 to 3 days a
malformations, fetal renal damage, week. Amitriptyline is the drug of
oligohydramnios, skull ossification de- first choice for prophylaxis of tension-
fects, and death in infants exposed to type headache during pregnancy
angiotensin receptor blockers in utero. and lactation.
Continuum (Minneap Minn) 2014;20(1):128–147 www.ContinuumJournal.com 143

Copyright © American Academy of Neurology. Unauthorized reproduction of this article is prohibited.


Headache in Pregnancy

Approach to Drug Treatment mil and are dependent on neither


of Migraine During Pregnancy duration nor dose. ECGs to assess PR
and Lactation interval prolongation should be un-
Symptomatic treatment of attacks in- dertaken at baseline, before each dose
cludes nonopioid analgesics and increment, and every 6 months during
prokinetic antiemetics, which should long-term treatment. Gabapentin is a
be limited to a maximum of 2 to 3 second-line prophylactic option dur-
treatment days per week (12 days per ing pregnancy and lactation. Lithium is
month). Sumatriptan may be indicated an additional option for prophylaxis
for severe attacks during pregnancy during lactation but should not be
that do not respond to first-line drugs used during pregnancy.
and can be used during lactation Acute treatment includes oxygen
without disruption to breast-feeding. (100% at 7 L/min for 10 to 15 minutes
Prophylaxis should be considered at onset of attack) or subcutaneous or
when attacks are frequent or fail to intranasal sumatriptan.33 No safety
respond to symptomatic management. concerns exist regarding use of oxy-
Analgesics can be used for symp- gen during pregnancy.34
tomatic treatment. Most antiemetics
can be continued throughout preg- Complementary Medicines
nancy and lactation. If prophylaxis is During Pregnancy and Lactation
indicated during pregnancy or lactation, Many pregnant women use dietary sup-
the lowest effective dose of propranolol plements during pregnancy, incorrectly
is the first-line recommendation. Ami- assuming that because they are not
triptyline is also an option. ‘‘drugs’’ they must be safe (Table 7-7).
Although some evidence suggests that
Approach to Drug Treatment supplementing deficient vitamins and
of Cluster Headache During minerals can be beneficial, megadose
Pregnancy and Lactation regimens should be avoided.
Prophylaxis is the mainstay of manage- The daily recommended dietary
ment of cluster headache. Preferred allowances of riboflavin (Vitamin B2)
treatments during pregnancy and lac- for pregnant women is 1.4 mg, and 1.6
tation are verapamil or prednisone/ mg for breast-feeding women.
prednisolone.33 Cardiac conduction Pregnant women are at risk of
problems affect about 20% of patients vitamin D deficiency due to increased
with cluster headache taking verapa- fetal and neonatal demands during

a
TABLE 7-7 Complementary Supplements and Pregnancy and Lactation

Supplements Notes
Coenzyme Q10 Insufficient data regarding safety for migraine prophylaxis
during pregnancy; compatible with breast-feeding
Magnesium sulfate Maximum of 350 mg/d
Vitamin B2 Recommended daily allowance doses only
Vitamin D 10 2g/d recommended for all pregnant and
breast-feeding women
a
Use of megadose vitamin regimens should be avoided.

144 www.ContinuumJournal.com February 2014

Copyright © American Academy of Neurology. Unauthorized reproduction of this article is prohibited.


KEY POINT
intrauterine growth and development. Corticosteroids successfully treat h During pregnancy and
Vitamin D deficiency has also been intractable nausea and vomiting in lactation, prochlorperazine
associated with increased risk of hyperemesis gravidarum. A 6-day reduc- 10 mg or chlorpromazine
migraine. Vitamin D supplements of ing course of prednisolone (60 mg/d for 25 mg to 50 mg by IM
10 2g/d (or equivalent) are currently 2 days, 40 mg/d for 2 days, and 20 mg/d injection are effective
recommended for all pregnant and for 2 days) can be considered to for emergency
breast-feeding women. Very high treat long-duration headache attacks, headache relief
doses of vitamin D can increase the particularly if evidence of medication even without
serum calcium concentration, which overuse is present.37 Chronic exposure additional analgesia.
could result in maternal, fetal, or to high doses of steroids in pregnancy
neonatal hypercalcemia. should be avoided as it may cause
Scientific evidence is insufficient to fetal/neonatal adrenal suppression, and
support the safe use of coenzyme Q10 increased incidence of congenital
during early pregnancy or breast- anomalies, neonatal cataracts, and still-
feeding. Supplementation with 100 mg birth have been reported when the
coenzyme Q10 twice daily from week drug is used throughout pregnancy.
20 of pregnancy has been shown to Prednisone/prednisolone is compatible
reduce the risk of preeclampsia in with breast-feeding.
women at risk for the condition.
Magnesium sulfate is considered to be CONCLUSIONS
compatible with pregnancy and breast- The majority of headaches occurring
feeding, but the FDA recommends an during pregnancy and lactation are
upper limit of 350 mg magnesium benign and likely to improve with
sulfate daily. minimal intervention. However, preg-
nancy can be a risk factor for certain
EMERGENCY TREATMENT potentially life-threatening headaches
OF HEADACHE that require urgent evaluation. The
During pregnancy and lactation, assessment of the pregnant patient
prochlorperazine 10 mg or chlorprom- with headache relies on a directed
azine 25 mg to 50 mg by IM injection history to elicit warning symptoms that
are effective for emergency headache require immediate investigation and
relief even without additional analge- management. Investigation of head-
sia and, together with IV fluids, are ache is the same as for nonpregnant
usually sufficient to abort an attack. IV women, although the threshold to
magnesium sulfate 1 g given intrave- consider certain disorders must be
nously over 15 minutes was well lower, and routine investigations
tolerated and effective in a random- should be deferred until postpartum.
ized, single-blind, placebo-controlled It is important to minimize drug
trial of 30 patients with migraine.35 A exposure in women who are planning
combination of IV prochlorperazine pregnancy or who are at high risk
10 mg every 8 hours and IV magne- of unplanned pregnancy. Many drugs
sium sulfate 1 g every 12 hours was and other teratogens exert their greatest
used successfully to abort two cases effects on the fetus during the second
of prolonged migraine aura during and third months of gestation. If a drug
pregnancy.36 Continuous administration is taken at any stage of pregnancy, it is
of magnesium sulfate injection beyond essential that the woman be provided
5 to 7 days in pregnancy should be with sufficient information about any
avoided because it can cause low calcium known risks in order for her to make
levels and bone changes in the baby. her own decision about its use.
Continuum (Minneap Minn) 2014;20(1):128–147 www.ContinuumJournal.com 145

Copyright © American Academy of Neurology. Unauthorized reproduction of this article is prohibited.


Headache in Pregnancy

Nonpharmacologic management 2. Headache Classification Committee of the


International Headache Society (IHS). The
and lifestyle changes can be effec- International Classification of Headache
tive without the need for additional Disorders, 3rd edition (beta version).
drug intervention and have the ad- Cephalalgia 2013;33(9):629Y808.
vantage of continued benefit beyond 3. Cripe SM, Frederick IO, Qiu C, Williams MA.
pregnancy. Risk of preterm delivery and hypertensive
disorders of pregnancy in relation to
maternal co-morbid mood and migraine
disorders during pregnancy. Paediatr Perinat
Epidemiol 2011;25(2):116Y123.
USEFUL WEBSITES
4. Aromaa M, Rautava P, Helenius H,
Sumatriptan/Naratriptan/Treximet Sillanpää ML. Prepregnancy headache and
Pregnancy Registry, Kendle International the well-being of mother and newborn.
pregnancyregistry.gsk.com/ Headache 1996;36(7):409Y415.
sumatriptan.html 5. Bendtsen L, Evers S, Linde M, et al. EFNS
guideline on the treatment of tension-type
Rizatriptan Pregnancy Registry, Merck headacheVreport of an EFNS task force. Eur
& Co, Inc J Neurol 2010;17(11):1318Y1325.

www.merckpregnancyregistries.com/ 6. Granella F, Sances G, Pucci E, et al. Migraine


with aura and reproductive life events: a case
maxalt.html control study. Cephalalgia 2000;20(8):701Y707.
Summary of Proposed Rule on 7. Cupini LM, Matteis M, Troisi E, et al.
Pregnancy and Lactation Labelling Sex-hormone-related events in migrainous
females. A clinical comparative study
www.fda.gov/Drugs/ between migraine with aura and migraine
DevelopmentApprovalProcess/ without aura. Cephalalgia 1995;15(2):140Y144.
DevelopmentResources/Labeling/ 8. Wainscott G, Sullivan M, Volans G, Wilkinson
ucm093310.htm M. The outcome of pregnancy in women
suffering from migraine. Postgrad Med J
National Library of Medicine Drugs 1978;54(628):98Y102.
and Lactation Database (LactMed) 9. Rotton WN, Sachtleben MR, Friedman EA.
toxnet.nlm.nih.gov/cgi-bin/sis/ Migraine and eclampsia. Obstet Gynecol
1959;14:322Y330.
htmlgen?LACT
10. Moore MP, Redman CW. Case-control study
Organization of Teratology Information of severe pre-eclampsia of early onset. Br
Med J (Clin Res Ed) 1983;287(6392):580Y583.
Specialists (OTIS)
www.OTISpregnancy.org 11. Marcoux S, Berube S, Brisson J, Fabia J. History
of migraine and risk of pregnancy-induced
hypertension. Epidemiology 1992;3(1):53Y56.
Teratogen Information Service (TERIS)
depts.washington.edu/terisweb/teris/ 12. Facchinetti F, Allais G, D’Amico R, et al.
The relationship between headache and
preeclampsia: a case-control study. Eur J Obstet
The International Classification of Head-
Gynecol Reprod Biol 2005;121(2):143Y148.
ache Disorders, 3rd edition (beta version)
13. Scher AI, Terwindt GM, Picavet HS, et al.
www.ihs-classification.org/_ Cardiovascular risk factors and migraine: the
downloads/mixed/International- GEM population-based study. Neurology
Headache-Classification-III-ICHD-III- 2005;64(4):614Y620.
2013-Beta.pdf 14. Banhidy F, Acs N, Horvath-Puho E, Czeizel
AE. Pregnancy complications and delivery
outcomes in pregnant women with severe
REFERENCES migraine. Eur J Obstet Gynecol Reprod Biol
2007;134(2):157Y163.
1. Bonati M, Bortolus R, Marchetti F, et al.
Drug use in pregnancy: an overview 15. Adeney KL, Williams MA, Miller RS, et al.
of epidemiological (drug utilization) Risk of preeclampsia in relation to maternal
studies. Eur J Clin Pharmacol 1990;38(4): history of migraine headaches. J Matern
325Y328. Fetal Neonatal Med 2005;18(3):167Y172.

146 www.ContinuumJournal.com February 2014

Copyright © American Academy of Neurology. Unauthorized reproduction of this article is prohibited.


16. Facchinetti F, Allais G, Nappi R, et al. pregnancy reduces the risk of pre-eclampsia.
Migraine is a risk factor for hypertensive Int J Gynaecol Obstet 2009;105(1):
disorders in pregnancy: a prospective cohort 43Y45.
study. Cephalalgia 2009;29(3):286Y292.
29. Evers S, Áfra J, Frese A, et al. EFNS guideline
17. Adeney KL, Williams MA. Migraine headaches on the drug treatment of migraineVrevised
and preeclampsia: an epidemiologic review. report of an EFNS task force. Eur J Neurol
Headache 2006;46(5):794Y803. 2009;16(9):968Y981.
18. Ekbom K, Waldenlind E. Cluster headache 30. Altman D, Carroli G, Duley L, et al., Magpie
in women: evidence of hypofertility(?) Trial Collaboration Group. Do women with
headaches in relation to menstruation and pre-eclampsia, and their babies, benefit
pregnancy. Cephalalgia 1981;1(3):167Y174. from magnesium sulphate? The Magpie
19. van Vliet JA, Favier I, Helmerhorst FM, et al. Trial: a randomised placebo-controlled trial.
Cluster headache in women: relation with Lancet 2002;359(9321):1877Y1890.
menstruation, use of oral contraceptives, 31. Nezvalová-Henriksen K, Spigset O, Nordeng
pregnancy, and menopause. J Neurol H. Triptan exposure during pregnancy and
Neurosurg Psychiatry 2006;77(5):690Y692. the risk of major congenital malformations
20. Rasmussen BK. Migraine and tension-type and adverse pregnancy outcomes: results
headache in a general population: psychosocial from the Norwegian Mother and Child
factors. Int J Epidemiol 1992;21(6):1138Y1143. Cohort Study. Headache 2010;50(4):
563Y575.
21. Rasmussen BK. Migraine and tension-type
headache in a general population: precipitating 32. May A, Leone M, Afra J, et al. EFNS
factors, female hormones, sleep pattern and guidelines on the treatment of cluster
relation to lifestyle. Pain 1993;53(1):65Y72. headache and other trigeminal-autonomic
22. Marcus DA, Scharff L, Turk DC. cephalalgias. Eur J Neurol 2006;13(10):
Nonpharmacological management of 1066Y1077.
headaches during pregnancy. Psychosom 33. Jüergens TP, Schaefer C, May A. Treatment
Med 1995;57(6):527Y535. of cluster headache in pregnancy and
23. Nestoriuc Y, Rief W, Martin A. Meta-analysis lactation. Cephalalgia 2009;29(4):391Y400.
of biofeedback for tension-type headache: 34. The National Teratology Information
efficacy, specificity, and treatment moderators. Service. Hyperbaric oxygen exposure in
J Consult Clin Psychol 2008;76(3):379Y396. pregnancy. 2006. www.toxbase.org/
24. Linde K, Allais G, Brinkhaus B, et al. upload/Pregnancy%20pdfs/
Acupuncture for tension-type headache. Carbon%20Monoxide%202012.pdf.
Cochrane Database Syst Rev 2009;(1):CD007587. Accessed November 23, 2013.
25. Linde K, Allais G, Brinkhaus B, et al. 35. Demirkaya S, Vural O, Dora B, Topcuoglu
Acupuncture for migraine prophylaxis. MA. Efficacy of intravenous magnesium
Cochrane Database Syst Rev 2009;(1):CD001218. sulfate in the treatment of acute migraine
attacks. Headache 2001;41(2):171Y177.
26. Neri I, Allais G, Schiapparelli P, et al.
Acupuncture versus pharmacological 36. Rozen TD. Aborting a prolonged migrainous
approach to reduce hyperemesis gravidarum aura with intravenous prochlorperazine and
discomfort. Minerva Ginecol 2005;57(4): magnesium sulfate. Headache 2003;43(8):
471Y475. 901Y903.

27. Sandor PS, Di Clemente L, Coppola G, et al. 37. Krymchantowski AV, Barbosa JS. Prednisone
Efficacy of coenzyme Q10 in migraine as initial treatment of analgesic-induced
prophylaxis: a randomized controlled trial. daily headache. Cephalalgia 2000;20(2):107Y113.
Neurology 2005;64(4):713Y715.
38. MacGregor A. Migraine. In: MacGregor EA,
28. Teran E, Hernandez I, Nieto B, et al. Frith A, eds. ABC of headache. Hoboken, NJ:
Coenzyme Q10 supplementation during BMJ/Wiley Blackwell, 2009.

Continuum (Minneap Minn) 2014;20(1):128–147 www.ContinuumJournal.com 147

Copyright © American Academy of Neurology. Unauthorized reproduction of this article is prohibited.


Review Article

Movement Disorders
Address correspondence to
Dr J. M. Miyasaki, 2E3.28 WMC,
8440 112th Street, Edmonton,
Alberta, Canada T6G 2B7,
miyasaki@ualberta.ca.
Relationship Disclosure:
Dr Miyasaki has served as a
in Pregnancy
speaker or on advisory boards Janis M. Miyasaki, MD, MEd, FRCP, FAAN; Amaal AlDakheel, MD
for Novartis Corporation and
Teva Pharmaceuticals.
Dr Miyasaki has received
research support from the ABSTRACT
Canadian Agency for Drugs
and Technologies in Health, Purpose of Review: This review discusses movement disorders that occur during
the Canadian Institute for pregnancy, the treatment of preexisting movement disorders, and the influence the
Health Research, the Michael pregnant state has on movement disorders symptoms, in order to guide clinicians in
J. Fox Foundation for
Parkinson’s Research, the providing better counseling for female patients who are pregnant or considering
National Center for pregnancy.
Complementary and Alternative Recent Findings: Unique considerations for movement disorders during pregnancy
Medicine, the National
Parkinson Foundation, the NIH, include investigations and their safety during pregnancy and the impact of
the Ontario Drug Benefits treatment on both the pregnant patient and her fetus.
Program, and the Ontario Summary: The most common movement disorders arising in pregnancy are restless
Ministry of Health and
Long-Term Care. The Movement leg syndrome and chorea gravidarum. Preexisting movement disorders in women
Disorders Centre at Toronto who become pregnant may also be seen.
Western Hospital has received
research support from
Teva Pharmaceuticals. Continuum (Minneap Minn) 2014;20(1):148–161.
Dr AlDakheel reports
no disclosure.
Unlabeled Use of
Products/Investigational INTRODUCTION may also be limited to the puerperium.
Use Disclosure:
Drs Miyasaki and AlDakheel Neurologists uncommonly encounter Patients with RLS report discomfort,
report no disclosures. creepy crawling sensations, or actual
patients with serious movement disor-
* 2014, American Academy
of Neurology. ders who are in the childbearing years. restlessness occurring in the legs, with an
Therefore, the published evidence for irresistible urge to move the legs. Once up
pregnancy and movement disorders and walking, the feelings are relieved. RLS
are limited.1,2 Furthermore, guidelines can involve the arms. Spread to the arms
do not exist for the treatment of typically occurs following treatment.3
movement disorders in pregnancy. RLS can occur for the first time
Nonetheless, clinicians require guid- during pregnancy, typically in the sec-
ance in this area, as several consider- ond and third trimesters (Case 8-1),
ations are unique to pregnancy, and usually resolves after delivery. RLS
including the safety of investigations prevalence in pregnancy is estimated
or medications during gestation, la- to be between 10% and 26%4 com-
bor, and delivery. This review summa- pared with a prevalence of 2.5% to 15%
rizes the published literature on of the general population. The in-
movement disorders and their medi- creased prevalence of RLS in pregnancy
cal management in pregnant women. may be linked to iron or folate deficiency,
known risk factors for RLS.5
RESTLESS LEGS SYNDROME Preexisting RLS may be affected by
Restless legs syndrome (RLS, also known pregnancy. The course of preexisting
as Ekbom syndrome) is the most com- idiopathic (unknown cause) RLS dur-
mon movement disorder of pregnancy. ing pregnancy was studied in a cohort
RLS usually is a chronic condition and is of 606 pregnant women; 59 had
believed to have an autosomal dominant preexisting RLS, of whom 36 reported
pattern of inheritance in most cases. RLS symptomatic worsening while seven

148 www.ContinuumJournal.com February 2014

Copyright © American Academy of Neurology. Unauthorized reproduction of this article is prohibited.


KEY POINTS

Case 8-1 h Restless legs syndrome


is the most common
A 30-year-old woman in her first pregnancy presented with leg cramping
movement disorder of
during her second trimester. Her only medication was a maternal vitamin.
pregnancy.
She did not drink caffeine and exercised modestly. No one in her family
had similar discomfort. In the past, she had been able to travel in cramped h Counseling should be
spaces without discomfort and sit through movies with ease. done for women with
At the time of the current presentation, she reported that by late preexisting restless legs
afternoon, sitting at her desk was uncomfortable and she often walked syndrome to let them
around the office to relieve her cramping. Further questioning revealed know that the majority
that the symptoms were worst at night while resting or right before bed of women experience
and could inhibit sleep onset. Her physical examination was normal and worsening during
her routine blood chemistry revealed a normal hemoglobin and normal pregnancy, although
iron saturation. 12% experience
Comment. This scenario is common in pregnancy-related restless legs improvement.
syndrome (RLS). The history confirmed that RLS was not present before the h Clinicians may opt to
patient’s pregnancy (eg, she had been able to sit in cramped spaces), and use levodopa as
she had no family history of RLS. This will be important in considering first-line therapy for
treatment because treatment can be expected to be limited to the safety and efficacy in
puerperium. women who have
restless legs syndrome
severe enough to
had symptomatic improvement during minergic agents such as levodopa.5 require treatment.
pregnancy. The remaining 17 reported Pergolide (no longer available in the
no change.4 Therefore, counseling United States) and ropinirole have been
should be done for women with associated with intrauterine growth re-
preexisting RLS to let them know that tardation, digit malformation, and fetal
the majority of women experience deaths in animals, while levodopa is
worsening during pregnancy, al- thought to be safe in pregnancy. There-
though 12% experience improvement. fore, in situations of RLS during preg-
Because of the association of RLS nancy, and given a theoretically lower
with iron deficiency, women with RLS risk of augmentation during the finite
should be tested for iron deficiency treatment period, clinicians may opt to
and receive replacement therapy if use levodopa as first-line therapy for
deficiency is confirmed. Unlike idio- safety and efficacy in women who have
pathic RLS, iron replacement therapy RLS severe enough to require treatment.
without iron deficiency has been ad-
vocated in pregnant women with RLS, DYSTONIA
but the utility and safety of this in Dystonia is the third most common
pregnancy are unknown. The main movement disorder in the general
pitfall in RLS therapy is augmentation. population, with a bimodal age of
This refers to escalating doses of onset peaking during childhood (early
medication required to control symp- onset) or the fifth decade of life.
toms, the use (often unsuccessful) of Typically, brain imaging for dystonia
polypharmacy, spread to other parts of is unremarkable. Its pathophysiology
the body, or symptoms present during has been proposed to be due to
the day in addition to nighttime onset. decreased inhibition in motor control,
Dopamine agonists are regarded as aberrant plasticity, and increased cor-
first-line agents in the treatment of tical excitability.6 Recent evidence has
idiopathic RLS due to lower risk of demonstrated that impairments in
augmentation compared to other dopa- basal ganglia function may lead to

Continuum (Minneap Minn) 2014;20(1):148–161 www.ContinuumJournal.com 149

Copyright © American Academy of Neurology. Unauthorized reproduction of this article is prohibited.


Movement Disorders in Pregnancy

KEY POINTS
h Drug-induced dystonia dystonia as well, probably due to or oculogyric crises consisting of neck
may also occur when abnormalities in synaptic function of extension with or without arching of
neuroleptic agents are striatopallidal or striatonigral terminals the back or extension of the arms can
used for the treatment in dystonia patients.6 Dystonia can occur. Most of these reactions resolve
of hyperemesis lead to abnormal posture secondary without treatment. In a few cases,
gravidarum. to abnormal sustained muscle contrac- anticholinergics such as benztropine
h Botulinum toxin B has tions. It can further be classified based 1 mg to 2 mg were given for rapid
not been linked to fetal on location into focal, multifocal, relief of dystonic reactions.
harm but is rated US segmental, or generalized. Also based Among women with dopa-
Food and Drug on etiology, dystonia can be idiopathic responsive dystonia, onset is typically
Administration or symptomatic. in childhood or early adulthood
pregnancy class C. The most common forms of dysto- consisting of initial foot dystonia with
nia affecting women in pregnancy are activity, typically later in the day. With
dystonia from Wilson disease, dopa- time, dystonia may generalize and also
responsive dystonia, and genetic forms occur throughout the day. The diurnal
of dystonia beginning in childhood or variation is typical of dopa-responsive
young adulthood (ie, DYT1).1,2 dystonia, also known as Segawa dis-
Some women develop dystonia ease. This is an autosomal dominant
during their reproductive years, be- illness. Patients may provide a history
cause the mean age of onset for all of family members with toe walking,
forms of dystonia is 27 years. Many scoliosis, or the need for orthopedic
reports have demonstrated the effect surgeries in childhood. Low doses of
of pregnancy on existing dystonia, levodopa (typically up to 300 mg/d)
which could be generalized or seg- are effective, and response to low
mental. In a survey of 62 women with doses confirms the diagnosis. Use of
dystonia, 27 had at least one pregnan- levodopa has not resulted in adverse
cy since dystonia onset, and four of events in pregnancy.10
these 27 women developed changes Another common treatment for
in their symptoms while pregnant.7 focal dystonia is botulinum toxin.
Another report described 10 women Botulinum toxin B has not been linked
who had been diagnosed with dysto- to fetal harm but is rated US Food and
nia before becoming pregnant. Two of Drug Administration (FDA) pregnancy
the 10 women had worsening of their class C. A survey of physicians’ prac-
dystonia, while three women had tice found 16 women received botuli-
complete or partial remission during num toxin A during pregnancy. Most
pregnancy. The remaining patients had of the pregnancies completed to term
no notable change. Only one of these with no apparent complication. One
10 women had generalized dystonia, of the women known to have previous
while the rest had focal or segmental history of spontaneous abortion had a
dystonia.8 Dystonia gravidarum is a miscarriage, while another woman
term that has been used for two cases had her pregnancy medically termi-
with cervical dystonia not having any nated.11 In another report, a woman
identifiable cause, which occurred had four uncomplicated full-term
during pregnancy and resolved before pregnancies while being treated with
delivery or shortly thereafter.9 botulinum toxin type A for idiopathic
Drug-induced dystonia may also cervical dystonia.12 At this point, the
occur when neuroleptic agents are product monograph for all forms of
used for the treatment of hyperemesis botulinum toxin recommends that
gravidarum. Acute dystonic reactions botulinum toxin not be administered
150 www.ContinuumJournal.com February 2014

Copyright © American Academy of Neurology. Unauthorized reproduction of this article is prohibited.


KEY POINTS
to pregnant women. Therefore, cau- majority noting onset during the first 2 h Chorea gravidarum is
tion is advised in this setting. trimesters.15 Chorea gravidarum typi- now most closely linked
Three women with early-onset dys- cally begins in the first trimester and to connective tissue
tonia previously treated with globus can spontaneously resolve before de- disorders.
pallidus internus deep brain stimula- livery in up to one-third of cases, with h The most common
tion (DBS) were able to conceive and the remainder resolving after delivery. cause of chorea is
successfully complete their pregnan- One in five women experience chorea probably drug-induced.
cies. DBS did not have any ill effect on gravidarum recurrence with subse-
h A careful medication
their offspring. All of them delivered quent pregnancies. Severe forms of history, including the
in the hospital, and there were no chorea gravidarum can cause hyper- use of over-the-counter
DBS-related contraindications to the thermia and rhabdomyolysis and, if agents and ‘‘natural‘‘
administration of general or regional not treated aggressively, can result in or herbal substances,
anesthesia.13 death.14 Proposed investigations (see should be taken, in
Table 8-1) for chorea arising in preg- conjunction with a toxin
CHOREA nancy should take into account the screen where
Chorea Arising During issues related to imaging during preg- appropriate.
Pregnancy: Chorea Gravidarum nancy discussed in the article ‘‘Neuro-
Chorea gravidarum was first described radiology in Women of Childbearing
approximately 4 centuries ago, by Age’’ by Drs Riley Bove and Joshua
Horatius, in 1661. The condition was Klein in this issue of .
originally associated with rheumatic As noted above, chorea can be self-
fever; however, chorea gravidarum is limited, and in general, treatment is
now most closely linked to connective safest after the peak period of fetal
tissue disorders, including systemic organogenesis. Treatment should be
lupus erythematosus, antiphos- directed to the etiology (rheumatic
pholipid antibody syndrome, syphilis, heart disease, lupus, Wilson disease,
McLeod neuroacanthocytosis syn- or other inherited dystonias), to con-
drome, and encephalitis.14 The most trol of the chorea itself, or to manage-
common cause of chorea is probably ment of the consequences of chorea
drug-induced (Table 8-1). Numerous (rhabdomyolysis, hyperthermia, pain,
medications cause chorea (outside weight loss). Therefore, symptomatic
of the setting of pregnancy) and treatment is reserved for situations
presumably can cause chorea during in which the health of either the
pregnancy. Opioid agents, sympatho- pregnant woman or her fetus would
mimetics such as amphetamine and be adversely affected if the chorea
cocaine, antiepileptic medications, continued. Dopamine receptorY
both typical and atypical neuroleptics, blocking agents (butyrophenones or
and over-the-counter medications phenothiazines) and dopamine-
such as antihistamines can all cause depleting agents (tetrabenazines) are
chorea. Therefore, a careful medication mainly the symptomatic treatment of
history, including the use of over- chorea of any cause. Treatment is
the-counter agents and ‘‘natural’’ or based on the proposed mechanism of
herbal substances, should be taken, in the chorea (heightened dopaminergic
conjunction with a toxin screen where activity in the striatum). However,
appropriate. dopamine receptor blockers and do-
In the case of rheumatic fever pamine depleting agents are generally
resulting in chorea, despite complete contraindicated during the first tri-
resolution, chorea occurs in up to 75% mester of pregnancy and are regarded
of patients during pregnancy, with the as FDA category C.
Continuum (Minneap Minn) 2014;20(1):148–161 www.ContinuumJournal.com 151

Copyright © American Academy of Neurology. Unauthorized reproduction of this article is prohibited.


Movement Disorders in Pregnancy

KEY POINT
h Haloperidol and TABLE 8-1 Chorea Arising in Pregnancy: Differential Diagnosis
chlorpromazine have and Suggested Investigations
been shown to be
helpful in the second Differential Diagnosis Investigations
and third trimesters and
Rheumatic fever Blood cultures (more than one)
appear to be safe in low
ECG
doses, such as
haloperidol 0.5 mg to Echocardiography
2 mg 2 to 3 times daily. Antistreptococcal antibody titer
(past infection if indicated)
Autoimmune disorders: Antiphospholipid antibody
antiphospholipid antibody For lupus, four of the following: malar
syndrome, lupus rash, discoid rash, photosensitivity,
oral ulcers, nonerosive arthritis,
pleuritis or pericarditis, renal disorder,
unprovoked seizures or psychosis,
hematologic disorder, anti-DNA antibody or
anti-Smith antibody or antiphospholipid
antibodies, lupus anticoagulant
assay, positive antinuclear antibody
Vascular disease: arteriovenous Neuroimaging
malformations, stroke, CNS
vasculitis
Wilson disease Ceruloplasmin (low)
24-hour urine copper collection
(elevated but can be G100 2g/d)
Liver biopsy: copper level 9250 2g/g dry weight
ATP7B genetic testing (gold standard)
Thyrotoxicosis Thyroid-stimulating hormone
Free T3, free T4
Drug-induced Cocaine, morphine, methadone,
amphetamine
ECG = electrocardiogram; DNA = deoxyribonucleic acid; CNS = central nervous system.

Haloperidol and chlorpromazine disorder characterizes Huntington dis-


have been shown to be helpful in the ease (HD). HD is caused by an
second and third trimesters and appear unstable polymorphic cytosine, ade-
to be safe in low doses, such as haloper- nine, and guanine (CAG) trinucleotide
idol 0.5 mg to 2 mg 2 to 3 times daily. repeat unit, coding for glutamine. HD
Haloperidol is recommended because is inherited in an autosomal dominant
it has fewer maternal anticholinergic, fashion, usually with complete pene-
hypotensive, and antihistaminergic trance when there are over 40 repeats.
effects than chlorpromazine.16 Intermediate repeat length as low as
27 can confer risk of gene expansion
Pregnancy in Women With and hence expression of the disease in
Chorea: Huntington Disease offspring. Individuals with a repeat
The triad of abnormal movements length of 26 are definitely not affected
(chorea, dystonia, or parkinsonism), and confer no risk of illness to their
cognitive impairment, and psychiatric offspring. The exact role of huntingtin,

152 www.ContinuumJournal.com February 2014

Copyright © American Academy of Neurology. Unauthorized reproduction of this article is prohibited.


KEY POINTS
the gene protein product, is not HD treatment is mainly symptom- h Genetic testing for the
known.17 atic as it involves treating the disabling Huntington disease
Because onset of HD is typically in chorea with dopamine receptorY gene has been done for
the fifth to sixth decade, except for blocking agents or dopamine- embryos at the
patients who have the Westphal vari- depleting agents (FDA category C).19 eight-cell stage.
ant (juvenile onset), most patients Clozapine has not been associated h Once structural causes
begin experiencing symptoms after with malformation, although the safety are eliminated, the most
childbearing has occurred. However, profile is still under examination common cause of ataxia
as the average age at which women (FDA category B). Benzodiazepines arising during pregnancy
give birth continues to rise, pregnancy may be helpful for chorea and some is Wernicke
with symptomatic HD may become of the behavioral aspects of HD.20 encephalopathy
more common. Therefore, counseling Psychiatric symptoms, especially de- secondary to hyperemesis
patients with HD regarding pregnancy pression, are common in HD. Treat- gravidarum causing
has been the current focus, especially ment of depression in pregnancy is vitamin B1 deficiency.
the issue of the ability of these complex and, given maternal and fetal
patients to care for their children. risk, should be undertaken with a
Since 1986, genetic testing has been psychiatrist or maternal health special-
available for people at risk for HD, but ist when possible. Similarly, treatment
as few as 5% of at-risk individuals of psychosis in HD pregnancy should
obtain testing, largely because no involve a psychiatrist or maternal
known cures or neuroprotective treat- health specialist.21
ments exist.17 In a French study of
patient choices for genetic testing of ATAXIA
HD and autosomal dominant cerebel- Ataxia Arising During
lar ataxia, of the 815 couples at risk of Pregnancy
having offspring with HD, only 4.6% Once structural causes are eliminated,
(n = 38) of those seeking pre- the most common cause of ataxia
symptomatic testing were pregnant arising during pregnancy is Wernicke
during the study. The majority of the encephalopathy secondary to hy-
HD couples (71%) decided to con- peremesis gravidarum causing vitamin
tinue the pregnancy and had pre- B 1 deficiency. Ninety percent of
symptomatic testing after delivery; Wernicke encephalopathy occurs in
another 21% had presymptomatic test- the setting of poor nutrition and
ing during the current pregnancy. An alcoholism, but it can occur in any
even smaller numberV11% (4/35)V setting where nutritional status is
eventually had prenatal testing.18 compromised. The classic triad of
Recently, genetic testing for the HD confusion, ataxia, and ophthal-
gene has been done for embryos at moparesis occurs in less than 10% of
the eight-cell stage, which have been all patients with Wernicke encepha-
formed using in vitro fertilization. lopathy. Therefore, a high degree of
Thereafter, the embryos that do not suspicion is required in the proper
have the HD gene are implanted. This clinical context, even with one sign.
gives the advantage of not having to Thiamine stores are depleted within 3
reveal the presymptomatic HD gene to 4 weeks of starvation. Neurologic
status to at-risk parents who do not symptoms occur as a result of depleted
wish to know their own HD gene CNS thiamine stores. This disorder
status. However, cost and the failure can be fatal. Therefore, early treat-
rate of pregnancy with in vitro fertili- ment with IV thiamine 200 mg 3 times
zation may be prohibitive. a day (given slowly over 30 minutes to
Continuum (Minneap Minn) 2014;20(1):148–161 www.ContinuumJournal.com 153

Copyright © American Academy of Neurology. Unauthorized reproduction of this article is prohibited.


Movement Disorders in Pregnancy

reduce the risk of anaphylaxis) until for preeclampsia underscore the im-
symptoms resolve is warranted in any portance of carefully monitoring treat-
pregnant patient presenting with any ments. Diabetes mellitus may begin
of the three signs in the context of during pregnancy with Friedreich ataxia.
hyperemesis. Guidelines do not exist Approximately one-third of patients
for thiamine deficiency treatment. were diagnosed after one pregnancy.24
However, undertreatment is common. Vaginal delivery is possible in
IM thiamine treatment with 250 mg/d women with Friedreich ataxia even if
for 3 to 5 days is an alternative treat- they are nonambulatory or experienc-
ment. Initial oral therapy is not ing reduced proprioception as a result
recommended. Either regimen (IV or of peripheral neuropathy.
IM) should be followed with 50 mg to
100 mg once daily until proper oral TREMOR
nutrition is maintained.22 Tremor may occur in pregnancy as a
result of enhanced physiologic tremor,
Pregnancy in Women drug-induced tremor from sympatho-
With Ataxia mimetic agents, or essential tremor
Many autosomal dominant forms of (Table 8-225). Essential tremor, one
spinocerebellar atrophy can be of the most common inherited move-
detected by routine genetic testing. ment disorders, is characterized by
For women with known spinocerebellar postural and kinetic tremor typically
atrophy confirmed with genetic testing, affecting the hands. Although it is
prenatal testing or in vitro fertilization bilateral, typically one hand is more
using tested embryos resulting in affected. Worsening with caffeine, im-
healthy births has been reported.23 provement with alcohol, and a positive
Friedreich ataxia is an autosomal family history are common in essential
recessive neurodegenerative dis- tremor. Essential tremor prevalence is
ease.24 A study of 31 women with estimated to be 1.7%. For those over 40
Friedreich ataxia with 65 pregnancies years of age, prevalence is 5.5%, affecting
resulting in 56 live offspring found women and men equally.26 It is inherited
that spontaneous abortion, pre- in an autosomal dominant pattern,
eclampsia, and preterm birth were although genetic testing is currently
not increased. The majority of women only conducted for research purposes.
had normal vaginal deliveries. Women Anticipation can occur, with chil-
reported unchanged, worse, or better dren displaying tremor before onset
ataxia with pregnancy in equal num- in their parents. Thus, a young woman
bers. Other features of Friedreich may note tremor before her parent is
ataxia (cardiac conduction block, re- diagnosed. Essential tremor can begin
duced mobility due to weakness, and at any stage of life, with slow progres-
diabetes mellitus) should be moni- sion. Typically, disability will only occur
tored by the obstetric team or mater- after a prolonged duration of decades.
nal health specialist, and care should Table 8-3 lists the medications with
be given to prevent deep vein throm- best evidence for efficacy in essential
bosis and pulmonary embolism. In tremor with FDA-assigned risks dur-
fact, case reports of deep vein throm- ing pregnancy where known. Given the
bosis occurring despite anticoagu- evidence of risk during the first trimes-
lation and of profound weakness ter or throughout pregnancy for all
occurring with ventilatory compromise agents, clinicians should counsel their
after the administration of magnesium patients appropriately. Only patients
154 www.ContinuumJournal.com February 2014

Copyright © American Academy of Neurology. Unauthorized reproduction of this article is prohibited.


a
TABLE 8-2 Medications Causing Tremor

Intention
Medication Action or Postural Tremor Tremor Resting Tremor
Antiarrhythmics Amiodarone, mexiletine, Y Y
procainamide
Antibiotics, antivirals, Y Vidarabine Co-trimoxazole,
and antimycotics amphotericin B
Antidepressants and Amitriptyline, lithium, selective Lithium SSRIs
mood stabilizers serotonin reuptake inhibitors
(SSRIs)
Antiepileptics Valproic acid Y Valproic acid
Bronchodilators Salbutamol, salmeterol Salbutamol, Y
salmeterol
Chemotherapeutics Tamoxifen, cytarabine, ifosfamide Cytarabine, Thalidomide
ifosfamide
Drugs of misuse Cocaine, ethanol, 3,4- Ethanol Cocaine, ethanol, MDMA,
methylenedioxymethamphetamine 1-methyl-4-pheynyl-
(MDMA) (ecstasy), nicotine 1,2,5,6-tetrahydropyridine
Gastrointestinal drugs Metoclopramide, cimetidine Y Metoclopramide
Hormones Thyroxine, calcitonin, Epinephrine Medroxyprogesterone
medroxyprogesterone
Immunosuppressants Tacrolimus, cyclosporine, Tacrolimus, Y
interferon-alfa cyclosporine
Methylxanthines Theophylline, caffeine Y Y
Neuroleptics and Haloperidol, thioridazine, Y Haloperidol, thioridazine,
dopamine depleters cinnarizine, reserpine, cinnarizine, reserpine,
tetrabenazine tetrabenazine
a
Reprinted from Morgan JC, Sethi KD, Lancet Neurol.25 B 2005, with permission from Elsevier. www.thelancet.com/journals/laneur/article/
PIIS1474-4422(05)70250-7/abstract.

TABLE 8-3 Treatment for Essential Tremor

Drug Level of Evidencea Dose Teratogenicityb


Propranolol A 60Y800 mg/d Category C
Primidone A Up to 750 mg/d Category D (consider vitamin K therapy 1 month
before delivery to avoid fetal hemorrhage)
Alprazolam B 0.125Y3 mg/d Category D
Atenolol B 50Y150 mg/d Category D
Gabapentin B 1200Y1800 mg/d Category C
Sotalol B 75Y200 mg/d Category B
Topiramate B Up to 400 mg/d Category D
Nadolol C 120Y240 mg/d Category C
Nimodipine C 120 mg/d Category C
Clonazepam C 0.5Y6 mg/d Category D
a
Levels of evidence based on the AAN’s Clinical Practice Guideline Process Manual, 2011 ed.27
b
Please see Appendix A for the US Food and Drug Administration Pregnancy Category descriptions.

Continuum (Minneap Minn) 2014;20(1):148–161 www.ContinuumJournal.com 155

Copyright © American Academy of Neurology. Unauthorized reproduction of this article is prohibited.


Movement Disorders in Pregnancy

KEY POINT
h Patients with Parkinson with disabling tremor should consider of PD in pregnant patients has been
disease may have treatment.28 obtained from small retrospective series
worsening of their and case reports (Table 8-4). Patients
motor symptoms during PARKINSONISM with PD may have worsening of their
pregnancy, although Parkinson disease (PD) is a neurode- motor symptoms during pregnancy,
women report return to generative disease characterized by although women report return to base-
baseline in follow-up. cogwheel rigidity, bradykinesia, rest- line in follow-up. One report of twin
ing tremor, and later imbalance. The pregnancy in an inherited form of parkin-
occurrence of pregnancy in patients sonism (DJ1) using levodopa resulted in
with PD is rare. Evidence regarding normal offspring at 2-year follow-up.
the effects of estrogen on PD is Only one case has been reported of
conflicting. Improvement, no change, a woman who developed signs of new-
or worsening motor symptoms during onset parkinsonism of unknown rea-
pregnancy have been reported. son at 11 weeks’ gestational age. She
Combining reports in the literature, had a miscarriage even though she
of a total of 36 pregnancies in 27 had received depot progesterone for a
women diagnosed with PD, 47% had threatened abortion. Her symptoms
worsening or the appearance of new PD resolved completely without requiring
symptoms during pregnancy or shortly any antiparkinsonian treatment.30
after delivery.29 Potential explanations
for worsening motor symptoms include TICS OR TOURETTE SYNDROME
changes in the volume of distribution Tics are sudden, repetitive, non-
for antiparkinsonian medications, meta- rhythmic motor movements or vocali-
bolic changes of pregnancy, or the zations, often associated with a
effects of changing levels of estrogen premonitory sensation that is relieved
particularly in women noting pre- by completing the movement. Tics
pregnancy effects between ovulation are classified as either simple or com-
and menstruation (when estrogen in- plex, and motor or phonic. Tourette
creases). The data regarding treatment syndrome is a movement disorder

TABLE 8-4 Medications for Treatment of Parkinson Disease in Pregnant


and Breast-Feeding Women

FDA Pregnancy
Drug Categorya Breast-Feeding
Levodopa-carbidopa C Do not useVno data on
excretion in breast milk
Rotigotine C No human data
Amantadine C Do not use
Trihexyphenidyl C No data on excretion
Bromocriptine B Do not use
Cabergoline B Do not use
Pramipexole C Do not use
Ropinirole C Do not useVexcreted
in breast milk
FDA = US Food and Drug Administration.
a
Please see Appendix A for the US Food and Drug Administration Pregnancy Category descriptions.

156 www.ContinuumJournal.com February 2014

Copyright © American Academy of Neurology. Unauthorized reproduction of this article is prohibited.


KEY POINT
characterized by chronic motor and the disease by 5 to 10 years in h Wilson disease
vocal tics beginning before 18 years of adolescents, while young adults typi- treatment with chelating
age. The condition is often lifelong, but cally present with neuropsychiatric agents (D-penicillamine,
tic severity frequently diminishes with illness.33 zinc, trientine, and
age. Tourette syndrome is often asso- Wilson disease predisposes pregnant tetrathiomolybdate) is
ciated with obsessive-compulsive dis- patients to miscarriages, while pregnan- recommended throughout
order and attention deficit disorder. cy does not change the course of the pregnancy because
Family history of tics, obsessive com- disease. In a series of 16 patients with 59 untreated pregnant
pulsive disorder, or learning disorders pregnancies, three stillbirths and 24 patients would
are common, although a definite ge- miscarriages occurred.34 One case has potentially develop fatal
liver disease or hemolytic
netic cause has not been identified. been reported of a 23-year-old preg-
anemia, while the fetus
Males are 3 to 4 times more commonly nant woman who at 23 weeks’ ges-
would develop liver
affected than females.31 tation developed pedal edema, damage and copper
Most tics do not result in disability. thrombocytopenia, coagulopathy, and accumulation.
In patients with nondisabling tics, med- liver cirrhosis. At 35 weeks’ gestation,
ication withdrawal should be attempted the diagnosis of Wilson disease was
before pregnancy. In a series of eight made. She underwent elective cesarean
women with 11 total pregnancies, five delivery at 36 weeks’ gestation. The
of the pregnancies had symptomatic ceruloplasmin level in the umbilical
improvement, while three had symp- serum was decreased by half compared
tomatic worsening. The remainder had with normal levels, while the copper
no change.32 Medications used to treat concentration was twice the normal
tics or Tourette syndrome consist of value. The neonate demonstrated hepa-
neuroleptics (category C), dopamine tomegaly and elevated liver enzymes,
depleting agents (tetrabenazine, cate- which persisted even after a year, de-
gory C), and benzodiazepines (category spite normal development. This report
B). Clonidine (category C) is also used predated genetic testing, and therefore
to treat tics. One case of pimozide use the child’s genetic status was not
(with fluoxetine) during pregnancy known.35
reported no untoward effects on the Wilson disease treatment with che-
fetus. lating agents (D-penicillamine, zinc,
trientine, and tetrathiomolybdate) is
WILSON DISEASE recommended throughout pregnancy
Wilson disease, also known as because untreated pregnant patients
hepatolenticular degeneration, is an would potentially develop fatal liver
autosomal recessive neurodegenera- disease or hemolytic anemia, while the
tive disorder secondary to abnormal fetus would develop liver damage and
copper metabolism leading to copper copper accumulation. D-penicillamine
accumulation in the liver, brain, and has been considered teratogenic as
cornea. The gene locus is on chromo- there are case reports that associate
some 13 and codes for P-type copper- D-penicillamine use with diffuse cutis
transporting adenosine triphosphatase laxa, severe micrognathia, contractures
(ATPase). The age of onset of Wilson of all limbs, CNS abnormalities (includ-
disease ranges from 5 to 50 years. The ing agenesis of the corpus callosum),
clinical picture consists of liver cirrho- and transient myelosuppression. Hy-
sis, chorea, tremor, psychiatric distur- pothyroidism and dyshormonogenesis
bances, and Kayser-Fleischer rings. In has been reported as well in a woman
most cases, hepatic disease predates taking penicillamine.36 Healthy in-
the neuropsychiatric component of fants may be born to women with
Continuum (Minneap Minn) 2014;20(1):148–161 www.ContinuumJournal.com 157

Copyright © American Academy of Neurology. Unauthorized reproduction of this article is prohibited.


Movement Disorders in Pregnancy

treated Wilson disease (penicillamine cally result in impaired inhibition of


500 mg/d), as evidenced by a meta- motor neurons and thus the presence
analysis of 18 Wilson disease patients of continuous motor unit activity.
who had 29 pregnancies while using Patients may have antiamphiphysin
D-penicillamine that did not show antibodies, antigephyrin and antiY,-
adverse outcomes.37 aminobutyric acid type A (GABA[A])
While chelation is necessary for receptor-associated protein antibody,
patients with Wilson disease during diabetes mellitus, thyroiditis, vitiligo,
pregnancy, given these concerns with or pernicious anemia. Anti-GABA(A)
D-penicillamine, chelation may be receptor-associated protein occurs in
most safely achieved using zinc ace- approximately 70% of patients and can
tate therapy alone (maintenance dose disrupt the synaptic stability and sur-
50 mg 3 times per day). Zinc acetate is face expression of GABA(A) receptors.
FDA category A during pregnancy; Antigephyrin antibodies are directed
however, it should be avoided during at the postsynaptic cellular protein
lactation, as taking zinc acetate may that clusters glycine receptors in the
result in copper deficiency in the spinal cord and GABA(A) receptors in
infant. Trientine is FDA category C, the brain. Amphiphysin is involved in
and therefore caution should be used. endocytosis of the vesicle membrane
after exocytosis of GABA from axonal
STIFF PERSON SYNDROME terminals and will therefore result in
Stiff person syndrome, also known as decreased inhibition. This particular
Moersche-Woltman syndrome or stiff antibody is characteristic of the
man syndrome, is a rare progressive paraneoplastic form of stiff person
disorder of fluctuating trunk and limb syndrome. Diagnosis according to the
stiffness, painful muscle spasms, and Dalakas criteria requires ‘‘stiffness of
exaggerated startle response that can axial muscles leading to fixed deformity
also be triggered by voluntary move- (hyperlordosis); superimposed pain-
ment or tactile or emotional stimuli. ful spasms precipitated by unex-
Symptoms occur predominantly in pected noises, emotional stress, tactile
axial musculature. Disability and re- stimuli; confirmation of continuous
duced quality of life are common; motor unit activity in agonist and
therefore, the majority of patients antagonist muscles by EMG; absence
require treatment. The prevalence is of neurological or cognitive impair-
estimated to be 1 in 1,000,000, and ments that could explain the stiffness;
two-thirds of patients are female. Age positive serology for GAD65 or
of onset peaks around 45 years, and amphiphysin antibodies by immuno-
symptoms develop over months or cytochemistry, western blot, or radio-
years. immunoassay; and response to
Stiff person syndrome is thought to diazepam.’’38,39 Treatment outside of
be an autoimmune disorder because pregnancy includes diazepam, 5 mg/d
more than two-thirds of patients have to 100 mg/d in divided doses (FDA
glutamic acid decarboxylase anti- category D); clonazepam, 1 mg/d to
bodies. A paraneoplastic variant is also 20 mg/d in divided doses (category D);
recognized. High titers of antibodies baclofen, 5 mg/d to 60 mg/d in divided
directed against a 65-kD isoform of doses (category C); levetiracetam, up
glutamic acid decarboxylase (GAD65) to 2000 mg/d in divided doses (cate-
are found in serum or CSF in 85% of gory C); plasma exchange, 5 to 6
patients. These antibodies theoreti- exchanges (safe in case reports and
158 www.ContinuumJournal.com February 2014

Copyright © American Academy of Neurology. Unauthorized reproduction of this article is prohibited.


given to pregnant women with other SUMMARY
medical conditions); gabapentin, up to The most common movement disor-
3600 mg/d in divided doses (cate- der in pregnancy is RLS followed by
gory B); or IV immunoglobulin, 2 g/kg chorea gravidarum. All movement dis-
body weight divided over 2 to 5 days orders may occur before pregnancy,
(safe in case reports and given to requiring a review of medications for
pregnant women with other medical safety during pregnancy and with
conditions).40 breast-feeding. Movement disorders
The occurrence of stiff person may also occur de novo in pregnancy.
syndrome in pregnancy is rare. The Chorea, dystonia, parkinsonism, and
following cases have been reported in stiff person syndrome require investi-
the literature: gation to rule out underlying causes.
The potential risks to mother and
& A 41-year-old patient with stiff
limb syndrome had a full-term child of various pharmacologic agents
pregnancy and delivered by are often unknown in pregnancy, and
forceps-assisted vaginal delivery. this should be discussed with patients.
During episiotomy, she had a Treatment following delivery may not
generalized spasm. During always be necessary (as in the case of
pregnancy, her symptoms RLS, which typically resolves).
improved in the second
trimester as she required less REFERENCES
1. Bordelon Y, Smith M. Movement disorders
baclofen and diazepam, although in pregnancy. Semin Neurol 2007;27(5):
her symptoms worsened 467Y475.
postpartum.41 2. Kranick S, Mowry E, Colcher A, et al.
& A 36-year-old primigravida with Movement disorders and pregnancy: a
stiff person syndrome was treated review of the literature. Mov Disord 2010;
25(6):665Y671.
with diazepam from the third
trimester onward. She delivered 3. Earley CJ. Clinical practice. Restless legs
syndrome. N Engl J Med
a healthy neonate by scheduled 2003;348(21):2103Y2109.
cesarean delivery and reported
4. Manconi M, Govoni V, De Vito A, et al.
remission of her symptoms by Restless legs syndrome and pregnancy.
2 weeks postpartum.42 Neurology 2004;63(6):1065Y1069.
& Finally, a 27-year-old woman 5. Chesson AL Jr, Wise M, Davila D, et al.
was diagnosed with stiff person Practice parameters for the treatment of
syndrome 2 months before restless legs syndrome and periodic limb
movement disorder. An American Academy
pregnancy. During the pregnancy, of Sleep Medicine Report. Standards of
she continued on prednisone Practice Committee of the American
and gabapentin, while diazepam Academy of Sleep Medicine. Sleep 1999;
22(7):961Y968.
was discontinued. Her muscle
spasms increased without the 6. Prescott IA, Dostrovsky JO, Moro E, et al.
Reduced paired pulse depression in the
diazepam; therefore, she received basal ganglia of dystonia patients.
baclofen 10 mg 3 times daily. In Neurobiol Dis 2013;51:214Y221.
the 40th week of gestation, she 7. Gwinn-Hardy KA, Adler CH, Weaver AL,
began labor, and a healthy et al. Effect of hormone variations and other
neonate was delivered via urgent factors on symptom severity in women
with dystonia. Mayo Clin Proc 2000;75(3):
cesarean delivery. On postpartum 235Y240.
day 1, diazepam was reintroduced
8. Rogers JD, Fahn S. Movement disorders and
and led to resolution of pregnancy. In: Devinsky O, ed. Advances in
symptoms.43 neurology: Neurological complications of

Continuum (Minneap Minn) 2014;20(1):148–161 www.ContinuumJournal.com 159

Copyright © American Academy of Neurology. Unauthorized reproduction of this article is prohibited.


Movement Disorders in Pregnancy

pregnancy. Vol 64, New York: Raven Press, (PGD) of spinocerebellar ataxia 2 (Sca2).
1994:163Y178. Prenat Diagn 2008;28(2):126Y130.
9. Lim EC, Seet RC, Wilder-Smith EP, Ong BK. 24. Friedman LS, Paulsen EK, Schadt KA, et al.
Dystonia gravidarum: a new entity? Mov Pregnancy with Friedreich ataxia: a
Disord 2006;21(1):69Y70. retrospective review of medical risks and
psychosocial implications. Am J Obstet
10. Nomoto M, Kaseda S, Iwata S, et al.
Gynecol 2010;203(3):224.e1Ye5.
Levodopa in pregnancy. Mov Disord 1997;
12(2):261. 25. Morgan JC, Sethi KD. Drug-induced tremors.
Lancet Neurol 2005;4(12):866Y876.
11. Morgan JC, Iyer SS, Moser ET, et al.
Botulinum toxin A during pregnancy: a 26. Tanner CM, Goldman SM. Epidemiology of
survey of treating physicians. J Neurol movement disorders. Curr Opin Neurol
Neurosurg Psychiatry 2006;77(1):117Y119. 1994;7(4):340Y345.
12. Newman WJ, Davis TL, Padaliya BB, et al. 27. American Academy of Neurology. Clinical
Botulinum toxin type A therapy during practice guideline process manual, 2011 ed.
pregnancy. Mov Disord 2004;19(11): St Paul, MN: American Academy of
1384Y1385. Neurology, 2011.
13. Paluzzi A, Bain PG, Liu X, et al. Pregnancy in 28. Zesiewicz TA, Elble RJ, Louis ED, et al.
dystonic women with in situ deep brain Evidence-based guideline update: treatment
stimulators. Mov Disord 2006;21(5):695Y698. of essential tremor. Neurology
2011;77(19):1752Y1755.
14. Robottom BJ, Weiner WJ. Chorea
gravidarum. Handb Clin Neurol 2011;100: 29. Scott M, Chowdhury M. Pregnancy in
231Y235. Parkinson’s disease: unique case report and
15. Maia DP, Fonseca PG, Camargos ST, et al. review of the literature. Mov Disord
Pregnancy in patients with Sydenham’s 2005;20(8):1078Y1079.
Chorea. Parkinsonism Relat Disord 2012; 30. Demirkiran M, Aslan K, Bicakci S, et al.
18(5):458Y461. Transient parkinsonism: induced by
16. American Academy of Pediatrics Committee progesterone or pregnancy? Mov Disord
on Drugs. Use of psychoactive medication 2004;19(11):1382Y1384.
during pregnancy and possible effects on 31. Bruun RD, Budman CL. The natural history of
the fetus and the newborn. Pediatrics Tourette syndrome. Adv Neurol 1992;58:1Y6.
2000;105(4 pt 1):880Y887.
32. Stern JS, Orth M, Robertson MM. Gilles
17. Walker FO. Huntington’s disease. Lancet de la Tourette syndrome in pregnancy: a
2007;369(9557):218Y228. retrospective series. Obstet Med 2009;2:
18. Lesca G, Goizet C, Durr A. Predictive testing 128Y129.
in the context of pregnancy: experience 33. Rosencrantz R, Schilsky M. Wilson disease:
in Huntington’s disease and autosomal pathogenesis and clinical considerations in
dominant cerebellar ataxia. J Med Genet diagnosis and treatment. Semin Liver Dis
2002;39(7):522Y525. 2011;31(3):245Y259.
19. Armstrong MJ, Miyasaki JM. Evidence-based 34. Sinha S, Taly AB, Prashanth LK, et al.
guideline: the treatment of chorea in Successful pregnancies and abortions in
Huntington disease. Neurology 2012;79(6): symptomatic and asymptomatic Wilson’s
597Y603. disease. J Neurol Sci 2004;217(1):37Y40.
20. Oyebode F, Rastogi A, Berrisford G, Coccia F.
35. Oga M, Matsui N, Anai T, et al. Copper
Psychotropics in pregnancy: safety and other deposition of the fetus and placenta in a
considerations. Pharmacol Ther 2012;135(1): patient with untreated Wilson’s disease. Am
71Y77. J Obstet Gynecol 1993;169(1):196Y198.
21. Byatt N, Deligiannidis KM, Freeman MP.
36. Hanukoglu A, Curiel B, Berkowitz D, et al.
Antidepressant use in pregnancy: a critical
Hypothyroidism and dyshormonogenesis
review focused on risks and controversies.
induced by D-penicillamine in children with
Acta Psychiatr Scand 2013;127(2):94Y114.
Wilson’s disease and healthy infants born to
22. Isenberg-Grzeda E, Kutner HE, Nicolson SE. a mother with Wilson’s disease. J Pediatr
Wernicke-Korsakoff-syndrome: 2008;153(6):864Y866.
under-recognized and under-treated.
37. Scheinberg IH, Sternlieb I. Pregnancy in
Psychosomatics 2012;53(6):507Y516.
penicillamine-treated patients with Wilson’s
23. Moutou C, Nicod JC, Gardes N, Viville S. Birth disease. N Engl J Med
after pre-implantation genetic diagnosis 1975;293(25):1300Y1302.

160 www.ContinuumJournal.com February 2014

Copyright © American Academy of Neurology. Unauthorized reproduction of this article is prohibited.


38. Hadavi S, Noyce AJ, Leslie RD, Giovannoni G. of neurological diseases. Eur J Neurol 2008;
Stiff person syndrome. Pract Neurol 15(9):893Y908.
2011;11(5):272Y282.
41. Weatherby SJ, Woolner P, Clarke CE.
39. Dalakas MC. Stiff person syndrome: Pregnancy in stiff-limb syndrome. Mov
advances in pathogenesis and therapeutic Disord 2004;19(7):852Y854.
interventions. Curr Treat Options Neurol
2009;11(2):102Y110. 42. Cerimagic C, Bilic E. Stiff-person syndrome
first manifesting in pregnancy. Gynecol
40. Elovaara I, Apostolski S, van Doorn P, et al. Obstet Invest 2009;67(2):134Y136.
EFNS guidelines for the use of intravenous
immunoglobulin in treatment of neurological 43. Goldkamp J, Blaskiewicz R, Myles T.
diseases: EFNS task force on the use of Stiff person syndrome and pregnancy.
intravenous immunoglobulin in treatment Obstet Gynecol 2011;118(2 pt 2):454Y457.

Continuum (Minneap Minn) 2014;20(1):148–161 www.ContinuumJournal.com 161

Copyright © American Academy of Neurology. Unauthorized reproduction of this article is prohibited.


Review Article

Neuro-ophthalmic
Address correspondence to
Dr Kathleen Digre, Moran Eye
Center, University of Utah, 65
Mario Capecchi Drive, Salt
Lake City, UT 84132,
Kathleen.digre@hsc.utah.edu.
Relationship Disclosure:
Disorders in Pregnancy
Dr Digre has received a grant Kathleen B. Digre, MD, FAAN; Krista Kinard, MD
from the National Eye Institute
and salary support from the
Neuro-ophthalmology
Research/Disease Investigator ABSTRACT
Consortium. Dr Kinard reports
no disclosure. Purpose of Review: This review discusses evaluation and treatment of neuro-
Unlabeled Use of ophthalmic disorders in the pregnant patient.
Products/Investigational Recent Findings: Any neuro-ophthalmic abnormality seen in nonpregnant women
Use Disclosure:
Drs Digre and Kinard
can be seen in pregnant women. Pregnancy-specific complications (preeclampsia
discuss the use of several and eclampsia) cause visual symptoms and can affect the entire visual axis.
drugs for the treatment of Summary: Appropriate evaluation and examination is important to preserve the
neuro-ophthalmic disorders,
none of which are labeled
health and vision of the mother and prevent complications in the fetus. Evaluation
by the US Food and Drug should proceed in the same way for a pregnant patient as it would for a
Administration for use nonpregnant patient, with few exceptions. Treatment decisions may be influenced
in pregnancy.
by stage of pregnancy.
* 2014, American Academy
of Neurology.
Continuum (Minneap Minn) 2014;20(1):162–176.

INTRODUCTION distortion or waviness), or diplopia


Almost every neurologist will see (double vision).
pregnant women who have visual Blurred vision due to a refractive
concerns. Neurologists should be able error is a common problem in other-
to evaluate these concerns and deter- wise healthy pregnant women. The
mine whether these are neurologically refractive error occurs due to a myo-
based, and should be aware of com- pic shift and/or decreased accommo-
mon neuro-ophthalmic disorders that dation. Pregnancy-specific hormonal
occur in pregnancy. changes, specifically the increase in
As with all neurologic disorders, a progesterone, result in edema of the
detailed history and neurologic exam- cornea and lens. The increase in corneal
ination contribute the most to accu- thickness can lead to a myopic shift
rate diagnosis. which also causes nyctalopia (difficulty
seeing at night). Corneal changes cause
COMMON VISUAL CONCERNS problems with contact lenses, since a
Decreased visual acuity or blurred transient myopic shift occurs, and swell-
vision is a common and nonspecific ing of the cornea can result in a less
concern (Table 9-1). To determine than perfect fit of the lenses on the
the etiology, the neurologist should cornea. Fluid shifts in the lens can
start by getting visual acuity using a decrease accommodation, making near
near card and then discuss with the vision blurry. These changes are con-
patient how the visual symptoms vary sidered normal in pregnancy.1Y3 A way
from baseline. Since blurred vision is a to determine if the blurred vision is
nonspecific concern, the examiner related to a refractive error is to ask the
must discern whether the blurring is woman to look at a near card through
really a refractive error, a scotoma a pinhole and see if the blurred vision
(blind spot), metamorphopsia (visual is correctable. The remainder of the

162 www.ContinuumJournal.com February 2014

Copyright © American Academy of Neurology. Unauthorized reproduction of this article is prohibited.


KEY POINTS

TABLE 9-1 Etiologies of Common Visual Symptoms h Differentiation of visual


symptoms can help
Symptom Etiology with diagnosis. The
neurologist must
Blurred vision Refractive error, dry eye, cataracts, medications,
determine whether
migraines, age-related macular degeneration,
central serous chorioretinopathy, corneal ectasia, the symptom is mild or
corneal edema, uveitis, preeclampsia severe blurring of vision,
diplopia, scotoma (visual
Visual loss Idiopathic intracranial hypertension, retinal vascular
occlusions, ocular ischemic syndrome, increased loss), or ocular pain.
intracranial pressure, TIA, optic pathway abnormalities h Vascular occlusions
(mass, ischemia), optic neuritis, glaucoma, orthostatic can occur with more
hypotension, migraine, vitreous hemorrhage, preeclampsia frequency during
Diplopia Cranial nerve palsies, venous sinus thrombosis, pregnancy due to the
monocular (cataract, dislocated lens, refractive error), hypercoagulable state
myasthenia, thyroid eye disease, idiopathic orbital of pregnancy.
inflammatory disease, cavernous sinus thrombosis,
preeclampsia h Blurred vision can also
be migraine aura, which
Anisocoria Congenital or acquired Horner syndrome, Adie pupil,
medications, third nerve palsy, iris sphincter trauma, can be first experienced
physiologic in the first trimester of
pregnancy.
TIA = transient ischemic attack.

screening examination should be nor- More serious causes of decreased


mal. In addition, pregnancy induces visual acuity in pregnancy can include
changes in the lacrimal gland that can vascular disease (such as diabetic reti-
result in dry eye.4 nopathy), ocular vascular events, cere-
Migraine is common in women of bral vascular events, and preeclampsia
childbearing age and may first present and eclampsia. Pregnancy is a mildly
during pregnancy. A common cause hypercoagulable state, leading to the
of visual blurring in pregnancy is unmasking of previously unknown
migraine aura. The visual symptoms clotting disorders. During pregnancy,
can include fortification spectra, de- the clotting factors plasminogen, fibrin-
creased accommodative ability, or sco- ogen, and factors I, VII, IX, X are all
toma affecting both eyes. Typically, increased. In addition, fibrinolysis, pro-
fortification spectra symptoms start tein S, and antithrombin levels decrease.
centrally and move outward or vice Although some controversy about this
versa. In contrast, visual blurring re- exists, protein C levels probably remain
lated to ischemia or other vascular unchanged, but many women have
etiologies are typically abrupt and activated protein C resistance during
maximal at onset, and ischemia to pregnancy. Smooth muscle hyperplasia
the retina can cause a visual symptom occurs as well as fragmentation of retic-
described as a shade being pulled ular fibers in blood vessels. All of these
down. Migraine often starts in the first changes are a setup for the Virchow triad
trimester and may or may not im- of hypercoagulability, hemodynamic
prove during the second and third changes, and endothelial dysfunction/
trimester.5Y7 For more on this topic, injury.8Y10 Consequently, a variety of
refer to ‘‘Headache in Pregnancy’’ by vascular events can occur, resulting in
Professor E. Anne MacGregor in this decreased visual acuity, blurred vision,
issue of . or visual loss.

Continuum (Minneap Minn) 2014;20(1):162–176 www.ContinuumJournal.com 163

Copyright © American Academy of Neurology. Unauthorized reproduction of this article is prohibited.


Neuro-ophthalmic Disorders in Pregnancy

KEY POINTS
h Refractive error and Diabetic women are at risk for non- pupillary defect (RAPD), dilated exami-
corneal or lens changes proliferative and proliferative diabetic nation, fluorescein angiography, and
are the most common retinopathy in pregnancy. Progression Humphrey visual fields (HVFs) can be
reason for mildly blurred during pregnancy can occur and then used to detect and monitor these
vision in pregnant regress after delivery. Rate of change abnormalities. Arteriolar occlusion leads
women. is often dependent on the level of to ischemia and whitening of the retina
h Diabetic pregnant glucose and blood pressure control, seen on dilated fundus examination.
women are at risk for amount of retinopathy before concep- The majority of arteriolar occlusions in
development or tion, and number of years with diabetes pregnancy are from a cardiac
progression of mellitus. The most common symptom source.13Y15 However, vascular occlu-
retinopathy; each may be visual blurring. Changes seen in sion can result from a hematologic
diabetic woman should diabetic retinopathy include micro- disorder, inflammatory vasculitic disor-
have an eye aneurysms, nerve fiber layer and intra- der, or emboli from other sources, such
examination during retinal hemorrhages, macular edema, as amniotic fluid. Venous occlusions can
pregnancy.
cotton wool spots, exudates, retinal also cause monocular blindness and
h If a retinal artery vascular changes, and disc edema.11,12 may cause hemorrhages of the retina
occlusion is present in Diabetic women with visual symptoms along with mild or severe vision loss,
pregnancy, consider should be assessed by an ophthalmol- depending on the amount of ischemia
echocardiogram, since
ogist for diabetic retinopathy. present. Central or branch retinal arterial
most occlusions are
If visual loss is abrupt and monocular occlusions are usually treated with 81
cardiac related.
and not transient, a careful look for a mg/d aspirin and referred for evalua-
retinal vascular occlusion should ensue. tion and control of atherosclerosis or
Visual acuity, confrontation visual fields, possible emboli (Figure 9-2).13,16Y19
Amsler grid (Figure 9-1), swinging flash- See Table 9-220 for a summary of
light test looking for a relative afferent pregnancy-related issues regarding
ophthalmic testing for pregnant women
with ocular symptoms.
Amniotic fluid embolism is a rare, life-
threatening complication of pregnancy
that affects multiple systems, including
the eye. Breakdown of the maternal-fetal
barrier occurs with release of amniotic
particulate in the maternal circulation;
this can result in coagulopathy and
cardiovascular collapse. Two case re-
ports link amniotic fluid embolism
with retinal arteriolar occlusions.16
Central serous chorioretinopathy is
accumulation of fluid underneath the
retinal pigmented epithelium. This con-
dition results in metamorphopsia and
mildly decreased visual acuity without a
RAPD. Its funduscopic appearance is like
a blister under the macula. It is most
common in young to middle-aged male
FIGURE 9-1 Normal Amsler grid. The Amsler grid is a patients; however, pregnancy, steroid
great asset to have in the neurologist’s tool use, stress, and hypertension are known
kit for evaluating metamorphopsia and visual
loss at the pregnant woman’s bedside. to increase the risk.21 Patients are typi-
cally observed without treatment, and
164 www.ContinuumJournal.com February 2014

Copyright © American Academy of Neurology. Unauthorized reproduction of this article is prohibited.


FIGURE 9-2 Retinal vascular occlusion. A, Color fundus photo example of a small branch retinal embolus that was found
near the left macula (arrow). A small corresponding central visual field defect was detected on automated
visual field testing (B) and an extensive evaluation showed a patent foramen ovale as a possible etiology for
the embolus.
Reprinted with permission from Digre KB, Corbett JJ. Butterworth-Heinemann.19

the fluid resolves spontaneously. The lobe. In pregnant women with any visual KEY POINT
patient should be seen by an ophthal- concern, preeclampsia should be con- h Preeclampsia and
mologist or neuro-ophthalmologist to sidered as an underlying etiology. eclampsia can cause a
diagnose and follow (see Table 9-1). Funduscopic changes in preeclampsia wide variety of visual
abnormalities and must
Preeclampsia and eclampsia cause a and eclampsia are from acute or chronic
be considered in any
variety of visual symptoms, including systemic hypertension. The funduscopic
pregnant woman with
blurred or decreased vision, spots in the signs include vascular narrowing, seg- visual concerns.
vision, or color defects. Preeclampsia mental vasospasm, cotton wool spots,
and eclampsia can affect the visual hemorrhages, disc edema, or emboli.
system from the retina to the occipital Rarely, bilateral retinal infarction can

a
TABLE 9-2 Ophthalmic Testing for Pregnant Women With Ocular Symptoms

US FDA Risk to Risk to


Test Classification Mother Fetus Contraindication Notes
Fluorescein C None None Allergy Crosses placenta and
angiogram known also into breast milk
Indocyanine green C None None Dye allergy Does not cross placenta;
angiography known used in pregnancy
Orbital ultrasound n/a None None None
Optical coherence n/a None None None
tomography known
Dilating drops C None None Angle-closure Occlude puncta when
tropicamide known glaucoma administering
a
Modified with permission from Digre K, J Neuro-Ophthalmol. B 2011 Lippincott Williams & Wilkins, Inc.20 journals.lww.com/jneuro-ophthalmology/
Fulltext/2011/12000/Neuro_Ophthalmology_and_Pregnancy___What_Does_a.17.aspx.

Continuum (Minneap Minn) 2014;20(1):162–176 www.ContinuumJournal.com 165

Copyright © American Academy of Neurology. Unauthorized reproduction of this article is prohibited.


Neuro-ophthalmic Disorders in Pregnancy

occur.22 Occlusions of the choroidal present, pupillary light reflexes will


vascular system can lead to asymptom- be intact. The well-known Cookie
atic overlying retinal pigment epithelial Theft Picture from the Boston Naming
changes known as Elschnig spots, or to Test can reveal simultanagnosia (af-
effusion and serous retinal detachments. fected patients will be unable to
Elschnig spots are red or yellow spots integrate the picture). Hoffmann
with dark centers that result from reported that 97% of women with
microvascular disease and infarction of eclampsia could not correctly describe
the choroid (Figure 9-3 shows typical the photo.23 Balint syndrome is char-
findings in severe preeclampsia and acterized by spasm of fixation, optic
eclampsia). These may be subtle and ataxia, and simultanagnosia and can
are often best seen on fluorescein and be seen if ischemia to the bilateral
indocyanine green (ICG) angiography. inferior parietal lobes also occurs.24
The examination should focus on visual As part of the evaluation of a
acuity and visual fields. If retinal changes patient with preeclampsia or eclamp-
are suspected, the patient should sia, the neurologic examination may
be referred to an ophthalmologist reveal hyperreflexia and other focal
for further evaluation and a dilated neurologic abnormalities. MRI findings
fundus examination. Case 9-1 demon- include disruption of the gray-white
strates the classical visual symptoms junction, especially in the parieto-
of scotoma, with funduscopy revealing occipital regions, and other features
disc and retinal edema with associated of the posterior reversible encephalop-
visual field abnormalities on automated athy syndrome, as discussed in the
perimetry. articles ‘‘Neuroradiology in Women of
Visual blurring or visual loss occur- Childbearing Age’’ by Drs Riley Bove
ring with eclampsia can also reflect and Joshua Klein and ‘‘Cerebrovascular
higher-order cerebral visual function Disorders Complicating Pregnancy’’ by
disorders, especially of the occipital Drs Steven Feske and Aneesh Singhal
lobe, causing alexia with or without in this issue of . Neuro-
agraphia and simultanagnosia. Careful ophthalmic complications of eclampsia
evaluation of visual acuity, pupils, and include cerebral blindness in 15%
visual fields is helpful in the setting of of patients.25 Spontaneous recovery
eclampsia. If cerebral blindness is from cerebral blindness occurs most

FIGURE 9-3 A color fundus photo of a 28-year-old primigravida with severe preeclampsia
and visual blurring. Dilated examination revealed subtle small cream-colored
lesions in the choroidVsometimes referred to as Elschnig spots (A, arrows; B).
Fluorescein angiogram revealed choroidal infarction and staining (C, arrows).
Reprinted with permission from Digre KB, Corbett JJ. Butterworth-Heinemann.19

166 www.ContinuumJournal.com February 2014

Copyright © American Academy of Neurology. Unauthorized reproduction of this article is prohibited.


KEY POINT

Case 9-1 h Multiple sclerosis has


an increased risk of
A 29-year-old woman at 25 weeks’ gestation was admitted to the hospital
symptom exacerbation
with 3 to 4 days of intermittent epigastric pain. She also reported
during the postpartum
intermittent headache and scotomas. Laboratory results confirmed
period, while
preeclampsia. She was started on IV magnesium and steroids, but cesarean
neuromyelitis optica
delivery had to be performed because of worsening blood pressure. After
has an increased risk of
delivery, she had an eclamptic seizure and continued to have problems
symptom exacerbation
with persistent preeclampsia and scotomas, so she was referred to the
during the last trimester
neuro-ophthalmology department. Ocular examination was normal
of pregnancy.
except for grade 3 bilateral disc edema with retinal edema on the left.
Humphrey visual field testing showed bilaterally enlarged blind spots. MRI
and magnetic resonance angiography confirmed thrombosis of the left
transverse and sigmoid sinus. Lumbar puncture revealed an opening
pressure of 333 mm CSF. She was treated with acetazolamide for the
secondarily induced intracranial hypertension and anticoagulation with
enoxaparin sodium for her thrombosis. Hypercoagulability workup
was negative.
Comment. This case demonstrates three reasons for abnormal vision
in the pregnant woman, including initial diagnosis of eclampsia, followed
by a complication of sinus thrombosis, and increased intracranial pressure.
It is important to consider all complications in pregnancy and explain all
of the visual phenomena.

commonly, but vision loss can be field to confrontation, and either a


permanent if progression to ischemic normal disc or an edematous disc.
or hemorrhagic stroke occurs. Infarc- The MRI may show T2 signal in a
tions of the lateral geniculate have noncontrasted dedicated orbit study,
been observed, which lead to perma- and periventricular fluid-attenuated
nent homonymous hemianopias. 26 inversion recovery (FLAIR) intensities
Treatment for preeclampsia/eclampsia may be evident if this is a presenting
is discussed in ‘‘Cerebrovascular Dis- sign of multiple sclerosis (MS). Evalu-
orders Complicating Pregnancy’’ by ation in the pregnant woman should
Drs Steven Feske and Aneesh Singhal proceed in a similar way to evaluation
in this issue of . Mag- of a nonpregnant patient.28 Optic
nesium, used in treatment of this neuritis can be seen in MS and
syndrome, can also cause visual blur- neuromyelitis optica (NMO). During
ring because it causes relaxation of pregnancy, immunoregulatory changes
accommodation, decreased conver- typically lead to a decrease in MS
gence, and ptosis of the eyelids. activity, and relapses are lowest during
However, the Amsler grid will be the last trimester. After delivery, relapse
normal if magnesium is the reason risk in MS is increased for 12 weeks.
for visual blurring.27 NMO differs in that risk of demyelin-
ation is increased during the last trimes-
OPTIC NEURITIS ter.29 Look for the NMO antibody in
Another cause of visual loss is optic anyone with optic neuritis, especially in
neuritis. Symptoms of optic neuritis the third trimester. Optic neuritis oc-
include visual loss, eye pain (especially curring in lactating women has been
with movement), and central visual reported but is rare, and the exact
field changes. Signs of optic neuritis relationship between demyelination
may include RAPD, abnormal visual and lactation is not understood.30

Continuum (Minneap Minn) 2014;20(1):162–176 www.ContinuumJournal.com 167

Copyright © American Academy of Neurology. Unauthorized reproduction of this article is prohibited.


Neuro-ophthalmic Disorders in Pregnancy

KEY POINTS
h The cause of disc edema Treatment of severe optic neuritis in hypertension, optic neuritis, and other
or papilledema must be pregnancy can be with IV steroids, as is entities. Papilledema, by definition, is
found and evaluated. the case in the nonpregnant state. optic disc edema from increased intra-
There have been no trials of the cranial pressure, which can be primary
h When papilledema is
present in pregnancy,
treatment of optic neuritis in preg- or secondary to another lesion, such as
look carefully for nancy; each case should be discussed venous sinus thrombosis. Symptoms of
secondary causes with the treating obstetrician.29,31 In intracranial hypertension include dip-
including venous sinus general, the authors usually wait to lopia, headache, nausea, visual field
thrombosis. Idiopathic treat the associated MS with disease- defects, pulse-synchronous tinnitus,
intracranial hypertension modifying therapy until after the preg- and decreased visual acuity.20,34
can first occur in nancy ends. For more on this topic, Primary intracranial hypertension,
pregnancy in obese refer to ‘‘Multiple Sclerosis in Pregnancy’’ now known as idiopathic intracranial
women. by Dr Patricia K. Coyle in this issue hypertension (IIH), is a diagnosis in
of . women of childbearing age, so this
disorder is seen in pregnancy. It is the
OTHER OPTIC NEUROPATHIES most frequent cause of papilledema in
Optic neuropathy refers to damage pregnant women and in nonpregnant
of the optic nerve for any reason. obese women (Figure 9-5). However,
Damage can be infiltrative, inflamma- pregnancy does not precipitate or ex-
tory, demyelinating, ischemic, or the acerbate IIH.35 Obesity and recent
result of mass effect. The same enti- weight gain are the major risk factors
ties that can cause optic neuropathy in for this condition. Criteria for the
nonpregnant patients, such as such as diagnosis include signs and symptoms
malignancy, diabetes mellitus, and of increased intracranial pressure,
glaucoma, can occur in pregnant such as headache, papilledema, diplo-
women. Pituitary tumors can enlarge pia (often from cranial nerve 6 palsy),
during pregnancy and may manifest as nausea, vomiting, transient visual ob-
visual loss (Case 9-2). For mass effect, scurations, and pulsatile tinnitus.35Y37
the physician should look carefully at Patients must have no localizing findings
visual acuity, visual fields to confron- on neurologic examination, as well as
tation, and funduscopy. A formal vi- normal brain imaging with no evidence
sual field is helpful when assessing for of venous thrombosis, an opening CSF
visual field dysfunction from a mass pressure of greater than 250 mm CSF
lesion such as a pituitary tumor. while the patient is in the lateral
Imaging, usually MRI, should be di- decubitus position with legs straight,
rected to the orbit and brain. Tumors normal CSF studies, and no other iden-
such as meningiomas may become tified cause of increased intracranial
symptomatic in pregnancy because pressure.20,34 Delivery can proceed
some of these tumors have estrogen based on normal obstetric practice. If
and progesterone receptors.32,33 concern exists for visual loss with
Transient vision loss can commonly Valsalva maneuver induced by labor,
occur from interrupted blood flow, as low outlet forceps may be used or
discussed above, but can also occur in cesarean delivery may be done.35
papilledema. Optic disc edema and Causes of secondary intracranial
papilledema are not synonymous, al- hypertension are numerous. Most im-
though papilledema is one of the portantly, a careful search for cereb-
causes of optic disc edema. Optic disc ral venous sinus thrombosis must
edema can be seen with diabetes be done in any pregnant woman
mellitus, infections, infiltration, acute with papilledema. Cerebral/dural
168 www.ContinuumJournal.com February 2014

Copyright © American Academy of Neurology. Unauthorized reproduction of this article is prohibited.


Case 9-2
A 34-year-old right-handed woman was first evaluated in the neuro-ophthalmology clinic for
decreased vision. Examination showed stage 5 disc edema of the left eye (Figure 9-4A) but was
otherwise normal. Clinical testing showed an enlarged blind spot on Humphrey visual field (HVF)

FIGURE 9-4 Color fundus photo of left optic disc edema from an optic nerve meningioma in the patient in Case 9-2 (A)
with a normal visual field on the right and a slightly enlarged physiologic blind spot on the left (B)
during pregnancy. MRI shows a meningioma of the left optic nerve (C).

Continued on page 170

Continuum (Minneap Minn) 2014;20(1):162–176 www.ContinuumJournal.com 169

Copyright © American Academy of Neurology. Unauthorized reproduction of this article is prohibited.


Neuro-ophthalmic Disorders in Pregnancy

Continued from page 169


testing (Figure 9-4B), and imaging showed a mass of the optic nerve sheath compressing the left
optic nerve (Figure 9-4C); it had typical features of an optic nerve sheath meningioma. Because of her
episodes of vision loss, she underwent 6 weeks of low-dose orbital radiation. Within a year after
radiation therapy, she had stable vision, decreased frequency of episodic vision loss, marked
improvement of her left optic nerve edema, and improvement of the enlarged physiologic blind spot
on HVF testing. After 6 months of pregnancy, 2 years after the radiation, she had increased frequency
of episodic vision loss as well as return of stage 5 disc edema of the left eye. HVF testing continued
to show an enlarged physiologic blind spot of the left eye while remaining normal on the right.
She was followed carefully during the remainder of her gestation, and her optic neuropathy
remained stable. After successful delivery, her vision was stable on follow-up, and no significant
changes were seen on her MRI.
Comment. This case demonstrates the enlargement that can occur with meningiomas during
pregnancy. Close observation is the typical course of action unless there is significant progressive loss
of vision or spread into the chiasm.

KEY POINT venous thrombosis occurs in 1/1000 to Leiden and protein C/S deficiencies
h Meningiomas may 1/10,000 pregnant women.38 Papilledema can manifest. Evaluation of papille-
increase in size during is seen in cerebral venous sinus throm- dema, whether primary or secondary,
pregnancy; optic nerve bosis along with headache. Patients includes appropriate imaging of the
meningioma may cause may have seizures and other neuro- brain and venous system (see ‘‘Neuro-
visual field changes.
logic findings. While thrombosis is radiology in Women of Childbearing
most common in the puerperium, it Age’’ by Drs Riley Bove and Joshua
can occur at any time during preg- Klein in this issue of ).
nancy. Since pregnancy is already a All women with papilledema should
slightly hypercoagulable state, familial also have a dilated examination and a
clotting disorders such as factor V formal visual field examination, since

FIGURE 9-5 A color fundus photo of bilateral optic discs in a woman with idiopathic
intracranial hypertension discovered at about 24 weeks‘ gestation. Her discs
remained swollen during pregnancy (A, B), with enlarged blind spots on
automated visual field testing. She was not treated since her vision remained stable on repeat
examinations. She delivered a healthy baby vaginally and uneventfully. Later, after weight loss,
her discs became completely normal.
Reprinted with permission from Digre KB, Corbett JJ. Butterworth-Heinemann.19

170 www.ContinuumJournal.com February 2014

Copyright © American Academy of Neurology. Unauthorized reproduction of this article is prohibited.


KEY POINT
visual acuity is affected later than visual medication must be stopped later in h All women with
fields in papilledema.20,34 Case 9-1 pregnancy because of risk of prema- papilledema, regardless
demonstrates classical visual symptoms ture closure of the patent ductus of its cause, need to be
and examination findings consistent arteriosus. For uncontrolled head- followed with visual
with secondary intracranial hyperten- aches, a CSF diversion procedure can fields.
sion from venous thrombosis during be considered.35,41 If venous sinus
the puerperium. thrombosis is causing secondary intra-
Underlying causes of secondary cranial hypertension, anticoagulation
intracranial hypertension should be is indicated. Heparin and heparinoids
treated directly, when possible. If the are usually chosen because these do
diagnosis is primary IIH and there are not cross the placenta in pregnancy,
no, or only mild, vision changes, then and warfarin is avoided because it
limit weight gain to 9.07 kg (20 causes fetal malformations.42
pounds) and treat with diuretics. Ac-
etazolamide is the medication of DIPLOPIA
choice because it allegedly reduces Diplopia (double vision) has many
CSF production by inhibiting carbonic etiologies. When a patient reports
anhydrase. While acetazolamide is a diplopia, the examiner first has to
US Food and Drug Administration determine whether it is binocular or
(FDA) class C drug, it has been used monocular, constant or intermittent,
safely in pregnancy. In the largest vertical or horizontal, and comitant
study, Falardeau and colleagues (the same in all directions) or in-
contacted pediatricians of children comitant (worse in one direction).
whose mothers took acetazolamide Monocular diplopia is usually the
in the first trimester, and no major result of structural ocular abnormality
defects or complications were re- such as dry eye or cataract. Binocular
ported.39 Chlorthalidone (FDA class diplopia suggests an orbital, cranial
B) or hydrochlorothiazide (FDA class nerve, or brainstem abnormality. In
B) can be considered as an alternative pregnancy, risk of cranial nerve
for diuretic therapy. No systematic palsies, growth of tumors, and nutri-
studies of these drugs in pregnancy tional deficiencies is increased.20,34
have been done.20,40 The most common cranial neurop-
The major threat of papilledema athy in pregnancy involves cranial
from primary or secondary intracranial nerve 7, followed by cranial nerves 6
pressure is visual loss. All pregnant and then 4. Cranial nerve 7 palsy may
women with IIH need to be followed cause a monocular diplopia secondary
with formal visual fields. The neu- to exposure and dry eye. Supportive
rologist should work closely with measures for dry eye include ocular
an ophthalmologist or a neuro- lubrication or tarsorrhaphy (a surgical
ophthalmologist to follow the vision. procedure in which the eyelids are
Repeat lumbar punctures with CSF temporarily or permanently partially
drainage can be used for temporary sutured together to limit corneal ex-
treatment. If vision is threatened de- posure). Cranial nerve 6 palsy can be
spite medical therapy, optic nerve due to increased intracranial pressure
sheath fenestration can be considered. or other causes and results in a bino-
Headache management includes non- cular horizontal diplopia. Diplopia will
steroidal anti-inflammatory drugs be worse in the affected direction of
(class B early in pregnancy and class gaze. Esotropia, nasal deviation of the
D later in pregnancy) early on, but this affected eye, may be seen on clinical
Continuum (Minneap Minn) 2014;20(1):162–176 www.ContinuumJournal.com 171

Copyright © American Academy of Neurology. Unauthorized reproduction of this article is prohibited.


Neuro-ophthalmic Disorders in Pregnancy

KEY POINT
h Sudden onset of a examination. The etiologies of sixth While an Arnold-Chiari type 1 mal-
headache in a patient nerve palsy in pregnancy include hy- formation may become symptomatic
known to have a pertension (which seems to be the during pregnancy, it must be differen-
pituitary tumor calls for most commonly associated factor and tiated from the more common ac-
imaging to look for usually resolves with delivery),43 de- quired tonsillar herniation due to
apoplexy. myelinating disease (eg, MS), and intracranial hypotension. Leakage
increased intracranial pressure. Some- from a lumbar puncture or inadver-
times the cause is not found. Cranial tent lumbar puncture during epidural
nerve 4 palsy causes a vertical or anesthesia can lead to intracranial
oblique binocular diplopia. Extraocular hypotension. Besides headache, dip-
movement examination will show lopia is a common symptom of intra-
weakness of the superior oblique mus- cranial hypotension; it is usually due
cle (trouble moving the eye in and to subtle bilateral sixth nerve palsies
down). Cover/cross-cover testing will or comitant esotropia. Diagnosis of
show the affected eye to be higher intracranial hypotension is by imaging
(hypertropia). Cranial nerve palsies can that shows low-lying tonsils, flattening
also be caused by excess interstitial of the pons against the clivus, pitui-
fluid in pregnancy,44 which will resolve tary gland enlargement, low-lying cer-
with delivery. Other causes of fourth ebellum, and enhancement of the
nerve palsy include congenital, inflam- pachymeninges with gadolinium.20,47
matory orbital disease, idiopathic, Finding and repairing the source of
trauma, and demyelination.20,34 the leak or doing a blood patch will
Pituitary microadenomas and typically resolve the problem. Asking
asymptomatic macroadenomas may the patient about a previous epidural
grow during pregnancy and become for delivery may help to identify the
symptomatic macroadenomas. Presen- source of the leak.
tation is variable from asymptomatic Wernicke encephalopathy presents
to diplopia, photophobia, ophthal- with diplopia, ptosis, or nystagmus,
moplegia, and typically bitemporal with or without retinal hemorrhages,
visual field defects, but junctional as well as ataxia and confusion (typi-
scotomas also occur. Any woman cally memory disturbance). This entity
with a known pituitary tumor should is most typically seen during the first
have formal visual field testing during trimester with nausea and vomiting.
pregnancy. Sudden onset of headache Demand for thiamine increases during
and diplopia is concerning for pregnancy; consequently, if hyper-
apoplexy, and patients should have emesis gravidarum or poor diet are
immediate imaging. Apoplexy is most present, a deficiency may result.48
frequently seen postpartum after Clinicians should have a high index
hemorrhage or shock, sometimes of suspicion for Wernicke encephalop-
known as Sheehan syndrome. Hemor- athy even if only portions of the
rhagic necrosis of a tumor can clinical syndrome are present, and
also result in apoplexy or infarction immediate treatment with IV thiamine
into a pituitary tumor. Sometimes is critical in order to prevent devastat-
the only manifestation of apoplexy ing complications, including a pro-
is the lack of menstrual periods found and potentially irreversible
postpartum. 32,45,46 A rare entity amnestic state.
known as lymphocytic hypophysitis Ocular myasthenia can occur in
can occur postpartum and mimics a pregnancy and cause diplopia, oph-
pituitary tumor. thalmoparesis, or ptosis just as in a
172 www.ContinuumJournal.com February 2014

Copyright © American Academy of Neurology. Unauthorized reproduction of this article is prohibited.


KEY POINTS
nonpregnant woman. Ocular myasthe- Spontaneous orbital hemorrhage can h Thyroid-associated
nia during pregnancy can worsen in occur with the Valsalva maneuver orbitopathy is the most
some patients but be unchanged or (vomiting or during labor). Symptoms common etiology for
improve in others. If symptom exac- include sudden pain, proptosis, and proptosis in pregnancy.
erbation occurs, it is more likely to diplopia. Anticoagulation is a risk factor h With anisocoria,
occur in the first trimester or after for this spontaneous event. The diagnosis onset and associated
delivery (see ‘‘Pregnancy and Myasthe- is made with orbital ultrasound, orbital symptoms must be
nia Gravis’’ by Drs Janice Massey and CT scan, or orbital magnetic resonance obtained. A careful
Carolina De Jesus-Acosta in this issue of scanning. Generally, these orbital hemor- search of the patient’s
for more on this topic).49 rhages resolve without treatment.52,53 recent medications also
Orbital varices are not caused by needs to be evaluated.
OCULAR AND ORBITAL PAIN pregnancy but can become manifest
Diseases of the orbit will usually cause during pregnancy. A varix is an
proptosis, possibly diplopia, and fre- intraorbital mass consisting of abnor-
quently pain. The most common or- mally large or dilated veins or a plexus
bital condition causing proptosis is of venous channels. Orbital varices
thyroid eye disease. Approximately present as a slowly increasing fullness
5% of pregnant women have thyroid of the orbit that may increase with
abnormalities. Thyroid-associated eye bending over, sinus pressure, or other
disease is more common in hyperthy- Valsalva maneuvers. The reason these
roid than in euthyroid or hypothyroid manifest in pregnancy is related in
states. Hypothyroid states must be part to increased blood volume that
treated pharmacologically for normal occurs in pregnancy. Diagnosis is made
mental development of the fetus. by orbital ultrasound, CT, or MRI, and
Regardless of the etiology of thyroid- no special management is usually
associated orbitopathy, the ocular re- needed for orbital varices because they
sults are the same.50 Ocular symptoms usually do not cause problems.54
of thyroid eye disease include diplopia,
conjunctival injection, photophobia, ABNORMAL PUPILS AND EYELIDS
gritty sensation, and exophthalmos. Anisocoria or other pupil abnormalities
Normal pregnancy can result in physi- should be approached in a pregnant
ologic thyroid changes that mimic woman the same way a nonpregnant
hyperthyroidism. Graves hyperthyroid- patient would be evaluated. The neurolo-
ism and gestational hyperthyroidism gist needs to determine onset and chro-
are the two most common etiologies nicity of the anisocoria. It can be helpful
for hyperthyroidism seen in pregnancy. to look at old photographs, determine
Gestational hyperthyroidism can be whether the patient has had prior ocular
observed. Graves hyperthyroidism trauma or surgeries, and do a detailed
should be treated with propylthiouracil review of all medicationsVeven topical
(FDA grade D) during the first trimes- medications can inadvertently reach the
ter and methimazole (FDA grade D) eye. On examination, the neurologist
during the second and third trimester, should evaluate the patient’s direct and
since methimazole has a known terato- consensual constrictive response to light,
genic effect in the first trimester. rate of redilation, and near response. If
Additionally, painless thyroiditis is the the anisocoria is greater in dim light, the
cause of thyrotoxicosis in 10% of pa- smaller pupil is the abnormal pupil, and
tients overall and can occur in postpar- Horner syndrome should be considered.
tum women, referred to as postpartum If the anisocoria is greater in bright light,
thyroiditis.50,51 the larger pupil is the abnormal pupil,
Continuum (Minneap Minn) 2014;20(1):162–176 www.ContinuumJournal.com 173

Copyright © American Academy of Neurology. Unauthorized reproduction of this article is prohibited.


Neuro-ophthalmic Disorders in Pregnancy

and Adie syndrome or an iris sphincter the distance of travel is the amount
problem should be considered. Finally, of levator function, which is normally
evaluation should include the position of 12 mm or greater. The neurologist
the eyelids and whether or not any pain should then check for fatigability of
or headache is present with the onset of eyelid elevation without blinking
the abnormal pupils, suggesting carotid for 30 or more seconds. Progressive
artery dissection. ptosis with upgaze is indicative of
A painful Horner syndrome can myasthenia. Associated miosis sug-
occur as the presenting finding in gests Horner syndrome. A dilated
carotid dissection, aneurysm, or tumor/ pupil and associated adduction and
mass effect. Self-limited acquired vertical motility defects suggest a third
Horner syndrome can occur due to cranial nerve palsy. Pupils are normal
cephalad spread of anesthesia and in myasthenia gravis.
because of a high epidural block.55 If
other findings of a third nerve palsy, CONCLUSION AND TRENDS
including a dilated pupil, are present, Pregnancy and the puerperium are a
magnetic resonance or CT angiographic unique time period. However, workup
imaging should be done, because vas- for most eye concerns should proceed
cular causes, such as expansion and as if the patient were not in the
rupture of a posterior communicating pregnant state. Treatment should en-
artery aneurysm, may occur during sue, following the information avail-
pregnancy. The physician should look able with regard to medication safety
carefully for other associated signs to during pregnancy. Diagnosis of the
direct imaging: proptosis, ptosis, and problem causing the neuro-ophthalmic
any neurologic findings.53 symptom is often crucial for the health
Ptosis can be congenital or ac- of the patient and fetus.
quired. Etiologies include neurogenic
(Horner syndrome, third nerve palsy), ACKNOWLEDGMENTS
myogenic (myasthenia gravis), apo- This review was supported in part by
neurotic/levator dehiscence (most an unrestricted grant from Research to
common), or mechanical. Mechanical Prevent Blindness, Inc, New York,
ptosis is usually caused by an eyelid New York, USA, to the Department
mass such as chalazion, eyelid edema, of Ophthalmology and Visual Sci-
or dermatochalasis (excess upper eye- ences, Moran Eye Center, University
lid skin). The neurologist can evaluate of Utah. Michael Varner, MD, provided
by looking at the margin reflex dis- suggestions about the content of this
tance, which is the distance between article. Susan Schulman assisted with
the light reflex on the cornea and the preparation of the manuscript.
upper and lower eyelids, and measur-
ing palpebral fissure height and dis-
tance of upper eyelid crease from REFERENCES
upper eyelid margin, as well as levator 1. Martin C, Varner MW. Physiologic changes
in pregnancy: surgical implications. Clin
function (upper eyelid excursion). To Obstet Gynecol 1994;37(2):241Y255.
evaluate eyelid excursion, the neurol-
2. Pizzarello LD. Refractive changes in
ogist can place a thumb on the brow pregnancy. Graefes Arch Clin Exp
to eliminate assistance from the Ophthalmol 2003;241(6):484Y488.
frontalis muscle, then ask the patient 3. Barbazetto IA, Pizzarello LD. Ocular changes
to look down toward his or her toes during pregnancy. Compr Ophthalmol
and then all the way up. Measuring Update 2007;8(3):155Y167.

174 www.ContinuumJournal.com February 2014

Copyright © American Academy of Neurology. Unauthorized reproduction of this article is prohibited.


4. Omoti AE, Waziri-Erameh JM, Okeigbemen 19. Digre K, Corbett J. Practical Viewing of the
VW. A review of the changes in the Optic Disc. Boston: Butterworth Heinemann,
ophthalmic and visual system in pregnancy. 2003.
Afr J Reprod Health 2008;12(3):185Y196.
20. Digre KB. Neuro-ophthalmology and
5. Liberman A, Karussis D, Ben-Hur T, et al. pregnancy: what does a neuro-ophthalmologist
Natural course and pathogenesis of need to know? J Neuroophthalmol 2011;31(4):
transient focal neurologic symptoms 381Y387.
during pregnancy. Arch Neurol 2008;
65(2):218Y220. 21. Al-Mujaini A, Wali U, Ganesh A, Montana C.
Natural course of central serous
6. Goadsby PJ, Goldberg J, Silberstein SD. chorioretinopathy without subretinal
Migraine in pregnancy. BMJ 2008;336(7659): exudates in normal pregnancy. Can J
1502Y1504. Ophthalmol 2008;43(5):588Y590.
7. Marcus DA, Scharff L, Turk D. Longitudinal 22. Lara-Torre E, Lee MS, Wolf MA, Shah DM.
prospective study of headache during Bilateral retinal occlusion progressing to
pregnancy and postpartum. Headache long-lasting blindness in severe preeclampsia.
1999;39(9):625Y632. Obstet Gynecol 2002;100(5 pt 1):940Y942.
8. Brenner B. Haemostatic changes in
23. Hoffmann M, Keiseb J, Moodley J, Corr P.
pregnancy. Thromb Res 2004;114(5Y6): Appropriate neurological evaluation and
409Y414. multimodality magnetic resonance imaging
9. Miller CH. Laboratory tests for the in eclampsia. Acta Neurol Scand 2002;106(3):
diagnosis of thrombotic disorders. Clin Obstet 159Y167.
Gynecol 2006;49(4):844Y849.
24. Gurjinder PS, Dhand UK, Chopra JS. Balint’s
10. Johnson CM, Mureebe L, Silver D. syndrome following eclampsia. Clin Neurol
Hypercoagulable states: a review. Vasc Neurosurg 1989;91(2):161Y165.
Endovascular Surg 2005;39(2):123Y133.
25. Sabet HY, Blake P, Nguyen D. Alexia without
11. Rahman W, Rahman FZ, Yassin S, et al. agraphia in a postpartum eclamptic patient
Progression of retinopathy during with factor V Leiden deficiency. AJNR Am J
pregnancy in type 1 diabetes mellitus. Clin Neuroradiol 2004;25(3):419Y420.
Experiment Ophthalmol 2007;35(3):231Y236.
26. Imes RK, Kutzscher E, Gardner R. Binasal
12. Sheth BP. Does pregnancy accelerate the hemianopias from presumed intrageniculate
rate of progression of diabetic retinopathy?: myelinolysis: report of a case with MR
an update. Curr Diab Rep 2008;8(4):270Y273. images of bilateral gneiculate involvement
after emergency cesarean section and
13. Gull S, Prentice A. Branch retinal artery
hysterectomy. Neuro-Ophthalmology
occlusion in pregnancy. Br J Obstet Gynaecol
2002;28:45Y50.
1994;101(1):77Y78.
27. Digre KB, Varner MW, Schiffman JS.
14. Bodur H, Caliskan E, Anik Y, et al. Cranial
thromboembolism secondary to patent Neuroophthalmologic effects of intravenous
foramen ovale and deep venous thrombosis magnesium sulfate. Am J Obstet Gynecol
after cesarean section. Gynecol Obstet 1990;163(6 pt 1):1848Y1852.
Invest 2008;65(4):258Y261. 28. Confavreux C, Hutchinson M, Hours MM,
15. Futterman LG, Lemberg L. Peripartum et al. Rate of pregnancy-related relapse in
cardiomyopathy: an ominous complication multiple sclerosis. Pregnancy in Multiple
of pregnancy. Am J Crit Care 2000;9(5): Sclerosis Group. N Engl J Med 1998;339(5):
362Y366. 285Y291.

16. Kim IT, Choi JB. Occlusions of branch 29. Coyle PK. Pregnancy and multiple sclerosis.
retinal arterioles following amniotic fluid Neurol Clin 2012;30(3):877Y888.
embolism. Ophthalmologica 2000;214(4): 30. Retzloff MG, Kobylarz EJ, Eaton C. Optic
305Y308. neuritis with transient total blindness
17. Vela JI, Diaz-Cascajosa J, Crespi J, et al. during lactation(1). Obstet Gynecol 2001;
Protein S deficiency and retinal arteriolar 98(5 pt 2):902Y904.
occlusion in pregnancy. Eur J Ophthalmol 31. Pula GH, Macdonald CJ. Current options for
2007;17(6):1004Y1006. the treatment of optic neuritis. Clin
Ophthalmol 2012;6:1211Y1223.
18. Branch DW, Andres R, Digre KB, et al. The
association of antiphospholipid antibodies 32. Molitch ME. Pituitary disorders during
with severe preeclampsia. Obstet Gynecol pregnancy. Endocrinol Metab Clin North
1989;73(4):541Y545. Am 2006;35(1):99Y116, vi.

Continuum (Minneap Minn) 2014;20(1):162–176 www.ContinuumJournal.com 175

Copyright © American Academy of Neurology. Unauthorized reproduction of this article is prohibited.


Neuro-ophthalmic Disorders in Pregnancy

33. Vaphiades MS, Simmons D, Archer RL, 46. Digre K, Varner MW. Neurologic diseases
Stringer W. Sheehan syndrome: a splinter of in pregnancy. In: Biller J, ed. Practical
the mind. Surv Ophthalmol 2003;48(2):230Y233. neurology. Philadelphia, PA: Lippincott
34. Digre KB, Varner MW. Neuro-ophthalmic Williams & Wilkins, 2002:770Y791.
complications in pregnancy. Expert Rev 47. Corbonnois G, O’Neill T, Brabis-Henner A,
Ophthalmol 2009;4:413Y428. et al. Unrecognized dural puncture during
35. Digre KB, Varner MW, Corbett JJ. epidural analgesia in obstetrics later
Pseudotumor cerebri and pregnancy. confirmed by brain imaging. Ann Fr Anesth
Neurology 1984;34(6):721Y729. Reanim 2010;29(7Y8):584Y588.
36. Durcan FJ, Corbett JJ, Wall M. The incidence 48. Chiossi G, Neri I, Cavazzuti M, et al.
of pseudotumor cerebri. Population studies Hyperemesis gravidarum complicated by
in Iowa and Louisiana. Arch Neurol 1988; Wernicke encephalopathy: background,
45(8):875Y877. case report, and review of the literature.
37. Digre K, Corbett JJ. Idiopathic intracranial Obstet Gynecol Surv 2006;61(4):255Y268.
hypertension (pseudotumor cerebri): a 49. Chaudhry SA, Vignarajah B, Koren G.
reappraisal. Neurologist 2001;7:2Y67. Myasthenia gravis during pregnancy.
38. Biousse V, Ameri A, Bousser MG. Isolated Can Fam Physician 2012;58(12):
intracranial hypertension as the only sign of 1346Y1349.
cerebral venous thrombosis. Neurology
50. Garber JR, Cobin RH, Gharib H, et al. Clinical
1999;53(7):1537Y1542.
practice guidelines for hypothyroidism
39. Falardeau J, Lobb BM, Golden S, et al. The in adults: cosponsored by the American
use of acetazolamide during pregnancy Association of Clinical Endocrinologists and
in intracranial hypertension patients. the American Thyroid Association. Endocr
J Neuroophthalmol 2013;33(1):9Y12. Pract 2012;18(6):988Y1028.
40. Lee AG, Pless M, Falardeau J, et al. The
51. Bahn Chair RS, Burch HB, Cooper DS, et al.
use of acetazolamide in idiopathic
Hyperthyroidism and other causes of
intracranial hypertension during pregnancy.
thyrotoxicosis: management guidelines
Am J Ophthalmol 2005;139(5):855Y859.
of the American Thyroid Association
41. Tang RA, Dorotheo EU, Schiffman JS, and American Association of Clinical
Bahrani HM. Medical and surgical Endocrinologists. Thyroid 2011;21(6):
management of idiopathic intracranial 593Y646.
hypertension in pregnancy. Curr Neurol
Neurosci Rep 2004;4(5):398Y409. 52. Jacobson DM, Itani K, Digre KB, et al.
Maternal orbital hematoma associated with
42. Srinivasan K. Cerebral venous and arterial labor. Am J Ophthalmol 1988;105(5):
thrombosis in pregnancy and puerperium. A 547Y553.
study of 135 patients. Angiology 1983;
34(11):731Y746. 53. Chang WJ, Nowinski TS, Repke CS, Buerger
DG. Spontaneous orbital hemorrhage
43. Fung TY, Chung TK. Abducens nerve palsy
in pregnant women treated with
complicating pregnancy: a case report. Eur J
subcutaneous heparin. Am J Ophthalmol
Obstet Gynecol Reprod Biol 1999;83(2):223Y224.
1996;122(6):907Y908.
44. Jacobson DM. Superior oblique palsy
manifested during pregnancy. 54. Abboud IA, Hanna LS. Intermittent
Ophthalmology 1991;98(12):1874Y1876. exophthalmos. Br J Ophthalmol 1971;
55(9):628Y632.
45. Murad-Kejbou S, Eggenberger E. Pituitary
apoplexy: evaluation, management, and 55. Rohrer JD, Schapira AH. Transient Horner’s
prognosis. Curr Opin Ophthalmol syndrome during lumbar epidural anaesthesia.
2009;20(6):456Y461. Eur J Neurol 2008;15(5):530Y531.

176 www.ContinuumJournal.com February 2014

Copyright © American Academy of Neurology. Unauthorized reproduction of this article is prohibited.


LIFELONG LEARNING IN NEUROLOGY ®
\

Neuroradiology in Women of
Childbearing Age
Bove, Riley M. MD; Klein, Joshua P. MD, PhD. Continuum (Minneap Minn). February 2014;
20(1 Neurology of Pregnancy): 23Y41.

Abstract
Purpose of Review:
This review summarizes safety concerns associated with diagnostic neuroimaging in patients
who are of childbearing age, focusing on diagnostic modalities and radiologic features of
neurologic conditions encountered by pregnant women.
Recent Findings:
During pregnancy, women experience a range of physiologic changes that can affect neurologic
function. These include endocrine, hemodynamic, endothelial, immunologic, and coagulopathic
changes that can alter susceptibility to stroke, subarachnoid hemorrhage, demyelination, venous
thrombosis, and other neurologic conditions. Unique safety concerns are associated with imaging
procedures performed to diagnose neurologic conditions that occur during pregnancy.
Summary:
This review discusses the use of diagnostic neuroimaging, including administration of
IV contrast, in pregnant women and in nonpregnant women of childbearing age.

Key Points
& Physiologic changes during pregnancy modulate the incidence and presentation of a
number of neurologic conditions.
& Elective imaging should, when possible, be deferred to the postpartum period. When
imaging is essential for the evaluation and treatment of a pregnant patient, careful review
of the indications for, risks and benefits of, and alternatives to neuroimaging should
be documented.
& During head CT examination of the mother, the fetus is exposed only to radiation that
is scattered through the mother’s body. Therefore, shielding of the abdomen, such as
with a lead vest, does not significantly reduce the minimal fetal radiation exposure
but may help to alleviate maternal anxiety.

* 2014, American Academy of Neurology.

Copyright © American Academy of Neurology. Unauthorized reproduction of this article is prohibited.


& The radiologist or a radiation dosimetry expert can assist the neurologist and obstetrician
in deciding on CT or MRI in the pregnant patient. Some CT and MRI examinations
can be modified to provide diagnostically critical information while exposing the
conceptus to as little risk as possible.
& Iodinated contrast is US Food and Drug Administration class B. If use of iodinated
contrast cannot be avoided during pregnancy, neonatal thyroid testing should be
performed during the first week. During breast-feeding, administration of iodinated
contrast is not contraindicated.
& MRI is the diagnostic modality of choice during pregnancy. Nonetheless, whenever
possible it should be delayed to the postpartum period.
& Gadolinium contrast is US Food and Drug Administration class C and should be
avoided during pregnancy when possible. As an alternative to contrast injection, imaging
of the arterial and venous circulation can often be performed using time-of-flight
sequences. When gadolinium is essential, no specific monitoring tests are required.
& During breast-feeding, gadolinium administration is not contraindicated.
& Pregnancy-specific causes of ischemic stroke may include preeclampsia/eclampsia,
paradoxical embolism from deep vein thrombosis through a patent foramen ovale,
trophoblastic embolism, amniotic fluid embolism,air embolism, and cardioembolism
from postpartum cardiomyopathy.
& Contributing factors to cerebral venous thrombosis include dehydration, cesarean
delivery or traumatic delivery, intracranial hypotension from dural puncture during
neuraxial anesthesia, anemia, raised homocysteine levels, or other etiologies.
& Recent reviews have suggested that, contrary to conclusions from prior studies, risk
of subarachnoid hemorrhage is not increased during pregnancy.
& Preeclampsia and eclampsia refer to a multisystem condition that arises in 2% to 8%
of pregnancies.
& Most women with typical preeclampsia or eclampsia do not require neuroimaging,
but diagnostic imaging should be performed in the following circumstances: seizures
arising before week 20 of pregnancy, postpartum eclampsia, focal neurologic deficits,
persistent visual symptoms, and symptoms that are refractory to magnesium infusion
and antihypertensive therapy.
& The radiologic appearance of posterior reversible encephalopathy syndrome, probably
due to a shared underlying pathophysiology, is similar to eclampsia, hypertensive
encephalopathy, and thrombotic thrombocytopenic purpura.
& Postpartum angiopathy/reversible cerebral vasoconstriction syndrome in pregnancy
is likely to be a manifestation of the preeclampsia/eclampsia syndrome, even though
patients may be normotensive and without proteinuria or other features of eclampsia.
& Pregnancy-related hormonal changes have trophic effects on intracranial neoplasms,
most commonly meningiomas and pituitary adenomas, but also ependymomas,
hemangioblastomas, and schwannomas, as well as on metastases from breast cancer
and melanoma.
& Meningiomas arise from the arachnoid cap cells in the meningeal arachnoid villi,
and the presence of hormonal receptors causes them to increase in size during
pregnancy.
& It has been estimated that the pituitary gland enlarges by 136% during pregnancy,
exceeding 10 mm in height and regressing by 6 months postpartum. Other, more serious
changes to the pituitary include apoplexy, pituitary tumor growth, and lymphocytic
hypophysitis.
& MRI is the preferred imaging modality for pregnancy-related back pain, except in cases of
acute trauma.

* 2014, American Academy of Neurology.

Copyright © American Academy of Neurology. Unauthorized reproduction of this article is prohibited.


& Pregnancy engenders a series of physiologic changes that increase the risk of certain
neurologic conditions, such as preeclampsia, stroke, hemorrhage, and back pain. These
changes should be considered, along with the typical range of neurologic conditions
affecting women of reproductive age, in any pregnant woman presenting with neurologic
symptoms.

Multiple Sclerosis in Pregnancy


Coyle, Patricia K. MD, FAAN. Continuum (Minneap Minn). February 2014;
20(1 Neurology of Pregnancy): 42Y59.

Abstract
Purpose of Review:
This article reviews the current understanding of the interactions between multiple sclerosis (MS)
and pregnancy, and implications for reproductive counseling. This is a key topic in MS because
the typical patient is a young woman of childbearing age.
Recent Findings:
It has been known for some time that MS disease activity markedly reduces during the last
trimester of pregnancy, then markedly increases in the 3 months postpartum before returning to
the prepregnancy baseline. High relapse rate or disability before pregnancy, as well as relapse
during pregnancy, have been associated with increased risk for postpartum attacks. Recent
data continue to support the conclusion that long-term disease progression is not worsened
(and may actually be lessened) with pregnancy in patients with relapsing MS; the data are not
so clear for those with progressive MS. Among the MS disease-modifying therapies, the only
one that requires contraception use by men is the new oral agent teriflunomide, because the drug
is present in semen. It is reassuring that, to date, no human teratogenic effects have been
documented for any of the MS disease-modifying therapies.
Summary:
Pregnancy has a profound effect on MS disease activity. Identification of the responsible
mechanisms for this effect should lead to new disease insights and therapeutic strategies.

Key Points
& Multiple sclerosis is on the rise among young women.
& The prototypical multiple sclerosis patient remains the young woman with relapsing
disease.
& Pregnancy is considered not an immunosuppressive state but an immune-tolerant state.
& Insights into how the immune system operates during pregnancy may hopefully
provide ideas about novel treatment strategies for multiple sclerosis.
& Multiple sclerosis has little to no direct effect on pregnancy.
& Approximately 97% of pregnant patients with multiple sclerosis have the relapsing
subtype.

* 2014, American Academy of Neurology.

Copyright © American Academy of Neurology. Unauthorized reproduction of this article is prohibited.


& The most current data support that pregnancy is not detrimental and may actually be
a favorable prognostic feature for multiple sclerosis.
& Reliable information is available about genetic risk of multiple sclerosis, and it is very
reassuring.
& When a parent has multiple sclerosis, risk to a child is about 2% to 2.5%.
& The latter part of pregnancy confers profound reduction in multiple sclerosis disease
activity.
& The most consistently identified markers associated with postpartum activity are high
relapse rate in the year before pregnancy, higher disability level before pregnancy,
and relapse during pregnancy.
& Birth month appears to be a risk factor for multiple sclerosis.
& None of the published reports or ongoing pregnancy registries for any of the
disease-modifying therapies has ever shown teratogenic effects.
& No uniform guidelines exist to direct the stopping of a disease-modifying therapy before
trying to become pregnant.
& Steroids are used fairly broadly in obstetrics and are probably safe in the second and
third trimesters.
& The fact that a woman has multiple sclerosis should have little to no impact on delivery
issues.
& Multiple sclerosis should not affect choice of anesthetic.
& Conflicting data indicate either no effect from breast-feeding or a positive benefit in
reducing multiple sclerosis activity.
& Patients with multiple sclerosis should be told there may well be a benefit in suppressing
postpartum disease activity by breast-feeding, but it is not certain.
& It is probably best to try exclusive breast-feeding.
& Multiple sclerosis disease severity and prognosis should be a factor in deciding how
rapidly to institute therapy postpartum.
& Breast-feeding while taking interferon A or glatiramer acetate is likely to be safe.
& Use of gonadotropin-releasing hormone agonists during the in vitro fertilization/assisted
reproductive technology procedure puts women with multiple sclerosis at risk for
relapses and new MRI lesion activity.

Pregnancy and Epilepsy


Harden, Cynthia L. MD. Continuum (Minneap Minn). February 2014;
20(1 Neurology of Pregnancy): 60Y79.

Abstract
Purpose of Review:
This review encompasses issues regarding the management of women with epilepsy and
pregnancy, including preconception planning, antiepileptic drug (AED) effects on the exposed
offspring, and consequences of seizures during pregnancy, with an emphasis on counseling
points and risk management.

* 2014, American Academy of Neurology.

Copyright © American Academy of Neurology. Unauthorized reproduction of this article is prohibited.


Recent Findings:
In utero valproate exposure continues to show the highest risk of congenital malformations
and of adverse cognitive outcomes, including autism, compared to other AEDs. In utero
topiramate exposure is associated with facial clefts. In utero exposure to lamotrigine, carbamazepine,
phenytoin, and levetiracetam has been evaluated in large numbers of offspring, and all of these
AEDs have a low risk of major congenital malformations, near 2.5%. The risk of major congenital
malformations due to in utero AED exposure is dose associated. Folic acid taken at the time of
conception decreases the risk of adverse cognitive outcomes. AED polytherapy may adversely
affect fertility, and in utero exposure of AED polytherapy is associated with infants who are
small for their gestational age. Seizure freedom before pregnancy is a good predictor of
remaining seizure free during pregnancy.
Summary:
Counseling points are complex for managing women with epilepsy contemplating pregnancy,
including evaluation of AED choice and dose. The physician must have knowledge of the issues
to enable an honest discussion and appropriate decision making in partnership with the patient.

Key Points
& Low birth rates are not the same as infertility.
& Psychosocial factors play a role in lower birth rates to women with epilepsy.
& Medical factors contributing to lower birth rates to women with epilepsy include severe
neurologic disability and antiepileptic drug polytherapy.
& The assessment of structural teratogenesis is much different than the assessment for
cognitive teratogenesis.
& Structural teratogenesis is due to first trimester exposure, while cognitive teratogenesis is
a likely risk with exposure throughout pregnancy.
& Dose-related risks are likely to be present with most antiepileptic drug intrauterine
exposure.
& The most clearly demonstrated dose-related risk is a marked increase in risk with maternal
use of valproate at more than 700 mg/d.
& For antiepileptic drugs with large numbers of exposures, such as carbamazepine,
lamotrigine, phenytoin, and levetiracetam, the estimates of risks for major congenital
malformations is precise. For less frequently used or newer antiepileptic drugs, such as
zonisamide, oxcarbazepine, clonazepam, topiramate, and phenobarbital, the estimates are
less precise, and larger numbers of exposure must be evaluated in order to determine the
actual risk.
& Little information regarding lacosamide is available at this time.
& The risk of midline defects, such as facial clefts, spina bifida, and hypospadias, may be
increased with in utero exposure to several antiepileptic drugs. However, the risk of these
defects is much greater with valproate exposure.
& In the Neurodevelopmental Effects of Antiepileptic Drugs study, in utero valproate
exposure was associated with impaired cognitive development; IQ at 6 years of age was
lower after exposure to valproate than to carbamazepine, lamotrigine, or phenytoin.
& Adverse effects of seizures in pregnancy include (1) maternal death from seizures when
antiepileptic drugs are abruptly stopped; (2) infants who are small for their gestational
age; (3) decreased verbal IQ in offspring when five or more convulsive seizures occur
during pregnancy; and (4) fetal heart rate alterations during convulsive seizures.

* 2014, American Academy of Neurology.

Copyright © American Academy of Neurology. Unauthorized reproduction of this article is prohibited.


& Women with epilepsy should be advised that folic acid use does not eliminate the risk of
major congenital malformations associated with antiepileptic drug use.
& Folic acid use during preconception and pregnancy in women with epilepsy decreases the
risk of lowered verbal IQ caused by antiepileptic drug exposure. It also decreases the risk
of spontaneous miscarriage.

Cerebrovascular Disorders
Complicating Pregnancy
Feske, Steven K. MD; Singhal, Aneesh B. MD. Continuum (Minneap Minn). February 2014;
20(1 Neurology of Pregnancy): 80Y99.

Abstract
Purpose of Review:
This article discusses the physiologic changes of pregnancy and how they affect risk of
ischemic and hemorrhagic stroke and then reviews epidemiology, diagnosis, and treatment
of ischemic and hemorrhagic stroke in pregnancy and the puerperium.
Recent Findings:
This article updates our understanding of the relationship of preeclampsia/eclampsia to the
posterior reversible encephalopathy syndrome and the reversible cerebral vasoconstriction
syndrome, emphasizing their shared pathogenesis. It reviews the most recent data and offers
recommendations concerning the use of thrombolytic and other revascularization therapies
for pregnancy-related strokes.
Summary:
Although cerebrovascular complications are uncommon occurrences during pregnancy and
the puerperium, stroke is still the most common seriously disabling complication of pregnancy.
Therefore, stroke and other vascular issues raise questions about the best evaluation and
management that is safe for mother and child.

Key Points
& Pregnancy results in increased metabolic demand, sodium and water retention, and
decrease in systemic vascular resistance, leading to expansion of plasma volume;
mild anemia; increased stroke volume, heart rate, and cardiac output; and decreased
systolic and diastolic blood pressures.
& Changes in vascular structure and the coagulation system, although adaptive, also lead
to a relative vulnerability to hemorrhage and ischemic stroke, especially during the
postpartum period.
& Preeclampsia/eclampsia is a state of hypertension, endothelial and platelet dysfunction,
and enhanced coagulability with many pathologic consequences.
& The risk of ischemic stroke is increased during the postpartum period.

* 2014, American Academy of Neurology.

Copyright © American Academy of Neurology. Unauthorized reproduction of this article is prohibited.


& Although pregnancy-associated ischemic stroke is rare, it is a major contributor to
long-term disability resulting from pregnancy.
& Cardioembolism, preeclampsia/eclampsia, and cerebral venous sinus thrombosis
account for most pregnancy-related ischemic stroke.
& Although data on the use of thrombolytic therapies during pregnancy are scarce,
limited experience suggests that these agents can be given with safety comparable
to that in nonpregnant patients.
& Preeclampsia/eclampsia can lead to several cerebrovascular syndromes, including
posterior reversible encephalopathy syndrome, reversible cerebral vasoconstriction
syndrome, and ischemic and hemorrhagic strokes.
& The major CNS complications of preeclampsia/eclampsia are a form of hypertensive
encephalopathy. They should be treated aggressively with rapid control of blood
pressure and IV magnesium sulfate.
& Cerebral venous thrombosis, especially postpartum, is one of the most common
cerebrovascular complications of pregnancy.
& Patients with cerebral venous sinus thrombosis benefit from anticoagulation.
& Pregnancy increases the risk of hemorrhagic stroke. This increased risk is greatest in
the postpartum period.
& The major causes of pregnancy-associated hemorrhage are preeclampsia/eclampsia and
cerebral vascular malformations, such as aneurysms and arteriovenous malformations.
& Aneurysmal subarachnoid hemorrhage during pregnancy confers a high risk of death
to both mother and baby.
& Women with subarachnoid hemorrhage should be seen by a neurosurgeon and undergo
vascular imaging to look for aneurysm, arteriovenous malformation, or other vascular lesions.
& Aneurysmal subarachnoid hemorrhage should be treated with early surgery or endovascular
techniques to secure the ruptured aneurysm and minimize the risk of recurrent hemorrhage.
& Women with arteriovenous malformations should be managed in consultation with a
neurosurgeon. In most cases, they can be treated with conservative observation until
after delivery. However, when hemorrhage occurs during pregnancy, patients with
low-grade arteriovenous malformations may benefit from early definitive surgery or
endovascular embolization.

Peripheral Neuropathies in Pregnancy


Massey, E. Wayne MD, FAAN; Guidon, Amanda C. MD. Continuum (Minneap Minn).
February 2014; 20(1 Neurology of Pregnancy): 100Y114.

Abstract
Purpose of Review:
This article provides an overview of the most common peripheral neuropathic disorders in
pregnancy with a focus on clinical recognition, diagnosis, and treatment.
Recent Findings:
The literature on this topic consists primarily of case reports, case series, and retrospective
reviews. Recent work, particularly in carpal tunnel syndrome, brachial neuritis, and inherited

* 2014, American Academy of Neurology.

Copyright © American Academy of Neurology. Unauthorized reproduction of this article is prohibited.


neuropathies in pregnancy, has added to our knowledge of this field. Awareness of diabetic
polyneuropathy with associated autonomic dysfunction in pregnancy has grown as the
incidence of diabetes mellitus increases in women of childbearing age.
Summary:
Women may develop mononeuropathy, plexopathy, radiculopathy, or polyneuropathy
during pregnancy or postpartum. Pregnancy often influences consideration of etiology,
treatment, and prognosis. In women of childbearing age with known acquired or genetic
neuromuscular disorders, pregnancy should be anticipated and appropriate counseling
provided. An interdisciplinary approach with other medical specialties is often necessary.

Key Points
& Carpal tunnel syndrome is common in pregnancy. Treatment is typically conservative,
particularly when symptoms start during the third trimester.
& While carpal tunnel syndrome in pregnancy is often less severe than nonYpregnancy-related
carpal tunnel syndrome, a significant percentage of women may have ongoing symptoms
postpartum. Electrodiagnostic studies are recommended to support the diagnosis in cases
of severe, atypical, or persistent symptoms.
& Carpal tunnel syndrome may present or worsen postpartum.
& Neuralgic amyotrophy has both idiopathic and autosomal dominant hereditary forms.
Both forms may occur postpartum. The inherited variety can recur with subsequent
pregnancies, and no known method exists to prevent attacks.
& Attention to positioning in labor helps prevent lower extremity mononeuropathies.
Femoral, fibular, and (less commonly) obturator neuropathies may occur.
& The etiology of compression or stretch should be evaluated when women develop
lower extremity mononeuropathies or plexopathy postpartum. Electrodiagnostic studies
confirm localization and assist with assessment of severity and prognosis, which is
typically favorable.
& Meralgia paresthetica involves sensory loss and paresthesia of the anterior and lateral
thigh. While symptoms are bothersome to the patient, it is typically a benign disorder,
which is diagnosed clinically. Electrodiagnostic studies can be performed if the
localization is in question. Normal strength and deep tendon reflexes help distinguish
this from other causes of sensory disturbance of the thigh.
& Back pain with focal neurologic deficits should prompt further investigation for
radiculopathy, which occurs only rarely in pregnancy.
& Epidural anesthesia very uncommonly results in neurologic complications, including
radiculitis, arachnoiditis, or epidural hematoma.
& Women have an increased risk of Bell palsy in the third trimester and postpartum.
Pregnancy-related Bell palsy may be more severe than nonYpregnancy-related Bell palsy.
The risks versus benefits of treatment with steroids should be considered for each patient.
& Intercostal neuralgia results in pain and sensory change in the distribution of one or
two thoracic roots. The etiology in pregnancy is unknown, and symptoms typically
resolve within hours of delivery.
& The approach to evaluation of polyneuropathy is not significantly changed by pregnancy.
& Acute inflammatory demyelinating polyradiculoneuropathycan be treated with plasma
exchange or IV immunoglobulin in pregnancy and does not appear to have adverse
perinatal or neonatal outcomes. Testing for cytomegalovirus should be performed
because cytomegalovirus can be transmitted to the fetus.

* 2014, American Academy of Neurology.

Copyright © American Academy of Neurology. Unauthorized reproduction of this article is prohibited.


& Communication with the obstetrician and anesthesiologist regarding the method of
delivery in women with acute inflammatory demyelinating polyradiculoneuropathy is
recommended.
& Autonomic dysfunction in diabetic polyneuropathy may cause gastroparesis or cardiac
complications during pregnancy and delivery. Diabetes also predisposes pregnant women
to focal compressive neuropathies.
& Deficiency of B vitamins should be considered as a cause of polyneuropathy in
pregnancy, particularly in women with hyperemesis gravidarum.
& In women presenting with polyneuropathy postpartum, a detailed history of emesis, diet,
and supplementation during pregnancy is essential.
& Treatment of Charcot-Marie-Tooth disease is heavily focused on adaptive equipment.
Functional needs or abilities may change during pregnancy or postpartum, and
reassessment is warranted.
& Pregnancy outcomes are typically favorable in Charcot-Marie-Tooth disease.
Transient worsening of Charcot-Marie-Tooth disease occurs in approximately
30% of pregnancies, while persistent worsening is seen in an additional 22% of
pregnancies.

Pregnancy and Myasthenia Gravis


Massey, Janice M. MD, FAAN; De Jesus-Acosta, Carolina MD. Continuum (Minneap Minn).
February 2014; 20(1 Neurology of Pregnancy): 115Y127.

Abstract
Purpose of Review:
Myasthenia gravis (MG) is an acquired autoimmune disorder characterized by fluctuating
ocular, limb, or oropharyngeal muscle weakness due to an antibody-mediated attack at the
neuromuscular junction. The female incidence of MG peaks in the third decade during the
childbearing years. A number of exacerbating factors may worsen MG, including pregnancy.
When treatment is needed, it must be carefully chosen with consideration of possible effects
on the mother with MG, the pregnancy, and the fetus.
Recent Findings:
Decisions are complex in the treatment of women with MG contemplating pregnancy or with
presentation during pregnancy. While data is largely observational, a number of characteristic
patterns and issues related to risk to the patient, integrity of the pregnancy, and risks to the fetus
are recognized. Familiarity with these special considerations when contemplating pregnancy
is essential to avoid potential hazards in both the patient and the fetus. Use of immunosuppressive
agents incurs risk to the fetus. Deteriorating MG with respiratory insufficiency poses risk to
both the mother and the fetus.
Summary:
This article reviews available information regarding expectations and management for
patients with MG in the childbearing age. Treatment decisions must be individualized based

* 2014, American Academy of Neurology.

Copyright © American Academy of Neurology. Unauthorized reproduction of this article is prohibited.


on MG severity, distribution of weakness, coexisting diseases, and welfare of the fetus.
Patient participation in these decisions is essential for successful management.

Key Points
& Women with myasthenia gravis benefit from a personalized interdisciplinary approach
to care during pregnancy and the postpartum period, including neuromuscular, high-risk
obstetric, and neonatal pediatric specialists.
& Myasthenia gravis is unmasked or worsened in approximately one-third of patients
during their pregnancy.
& Elevated serum levels of AntiYacetylcholine receptor-binding antibodies or
antiYmuscle-specific kinase (MuSK) antibodies in patients with clinical signs and
symptoms of myasthenia gravis confirm the diagnosis. In the seronegative patient,
electrophysiologic demonstration of an abnormality of neuromuscular transmission
establishes the diagnosis.
& Patients may undergo chest CT imaging without contrast to assess the thymus gland;
however, postponement until after delivery is preferable, particularly in
antibody-negative patients.
& The severity of weakness at the beginning of pregnancy does not predict either
remission or exacerbation, and, in fact, disease exacerbations, myasthenic crisis, or
even disease remission may each occur during pregnancy.
& Close monitoring for respiratory difficulties is essential throughout pregnancy to maintain
the welfare of both mother and fetus.
& No evidence has been published that babies born to mothers with myasthenia gravis have
any increased risk of developing autoimmune-mediated myasthenia gravis.
& The risk of generalized myasthenia gravis is highest in the first 2 to 3 years after onset.
During these years, it is advisable for a patient to delay pregnancy, thereby reducing
potential worsening provoked by pregnancy and clarifying her severity and response to
treatment.
& Treatment is stepwise and depends on the clinical scenario. With only minimal
manifestations of the disease, pyridostigmine for symptomatic treatment before
contemplating pregnancy may be considered.
& Corticosteroids, plasma exchange, and IV immunoglobulin have been used safely during
pregnancy and are agents often chosen for treatment of exacerbation of weakness.
& Azathioprine and mycophenolate mofetil are US Food and Drug Administration
pregnancy category D, and methotrexate is category X. The use of these medications is
not recommended in pregnancy because they pose significant risk to the fetus.
& Myasthenia gravis typically does not hinder the early stages of labor, as smooth muscle
contraction is involved in the first stage of labor.
& During labor, regional anesthesia can be used safely and lessens the risk of
medication-induced neuromuscular blockade from nondepolarizing anesthetic or
curare-like agents.
& Infections, electrolyte disturbances, and numerous drugs have been found to unmask
latent myasthenia gravis or trigger a myasthenic crisis.
& The American Academy of Pediatrics considers pyridostigmine, prednisone, and
prednisolone compatible with lactation.
& All women of childbearing potential (including pubertal girls and perimenopausal
women) who begin or restart an immunosuppressive regimen must receive contraceptive
counseling and use effective contraception.

* 2014, American Academy of Neurology.

Copyright © American Academy of Neurology. Unauthorized reproduction of this article is prohibited.


Headache in Pregnancy
MacGregor, E. Anne MB BS, MD, FFSRH, MICR. Continuum (Minneap Minn).
February 2014; 20(1 Neurology of Pregnancy): 128Y147.

Abstract
Purpose of Review:
This article provides an overview of the diagnosis and management of primary and secondary
headaches that may occur during pregnancy and postpartum. Headache presenting in pregnancy
is of significant concern to the affected woman. Quick and correct diagnosis leads to the optimal
management, minimizing risks to the pregnancy.
Recent Findings:
Several strategies have been developed to distinguish secondary headaches that need urgent
assessment and management from benign primary and secondary headaches and to minimize
the risk of misdiagnosis. Recent guidelines for the drug treatment of headaches are considered
in the context of updated information on the safety of drugs in pregnancy and lactation.
Summary:
Primary headaches are common and typically improve during pregnancy. Management
during pregnancy and lactation is similar to management in the nonpregnant state, with a few
exceptions. Secondary causes of headache that are more likely to occur during pregnancy include
cerebral venous 1thrombosis, posterior reversible encephalopathy syndrome resulting from
eclampsia, postYdural puncture headache, stroke, and pituitary apoplexy.

Key Points
& Of the primary headaches, tension-type headache and migraine generally improve
during pregnancy.
& A common cause of daily headache in a patient with a history of primary headache is
medication overuse headache.
& Because secondary headaches can occur in a patient with a long-standing history of
primary headache, it is important to elicit new symptoms.
& Each primary headache has a specific pattern of symptoms in the absence of clinical signs.
& Headaches that lack associated symptoms, in an otherwise well person who is not
overusing medication, are likely to be tension-type headaches.
& Recurrent episodic headaches that last between 4 and 72 hours and are associated
with photophobia, nausea, and disability in an otherwise well person are typical
features of migraine.
& Cluster headache is frequently misdiagnosed as migraine despite stereotypical symptoms
of strictly unilateral headache and autonomic symptoms lasting up to 2 hours in clusters
typically lasting 6 to 8 weeks.
& Pregnant women with tension-type headache or migraine should be encouraged to
avoid skipping meals, to take regular exercise, to drink plenty of fluids, and to maintain
a regular sleep pattern.

* 2014, American Academy of Neurology.

Copyright © American Academy of Neurology. Unauthorized reproduction of this article is prohibited.


& Nondrug therapies such as relaxation, biofeedback, and physical therapy are safe and
may be effective in pregnancy.
& During pregnancy and lactation, prochlorperazine 10 mg or chlorpromazine 25 mg to
50 mg by IM injection are effective for emergency headache relief even without
additional analgesia.

Headache in Pregnancy
Miyasaki, Janis M. MD, MEd, FRCP, FAAN; AlDakheel, Amaal MD. Continuum (Minneap Minn).
February 2014; 20(1 Neurology of Pregnancy): 148Y161.

Abstract
Purpose of Review:
This review discusses movement disorders that occur during pregnancy, the treatment of
preexisting movement disorders, and the influence the pregnant state has on movement disorders
symptoms, in order to guide clinicians in providing better counseling for female patients who
are pregnant or considering pregnancy.
Recent Findings:
Unique considerations for movement disorders during pregnancy include investigations and their
safety during pregnancy and the impact of treatment on both the pregnant patient and her fetus.
Summary:
The most common movement disorders arising in pregnancy are restless leg syndrome and chorea
gravidarum. Preexisting movement disorders in women who become pregnant may also be seen.

Key Points
& Restless legs syndrome is the most common movement disorder of pregnancy.
& Counseling should be done for women with preexisting restless legs syndrome to let them
know that the majority of women experience worsening during pregnancy, although 12%
experience improvement.
& Clinicians may opt to use levodopa as first-line therapy for safety and efficacy in women
who have restless legs syndrome severe enough to require treatment.
& Drug-induced dystonia may also occur when neuroleptic agents are used for the treatment
of hyperemesis gravidarum.
& Botulinum toxin B has not been linked to fetal harm but is rated US Food and Drug
Administration pregnancy class C.
& Chorea gravidarum is now most closely linked to connective tissue disorders.
& The most common cause of chorea is probably drug-induced.
& A careful medication history, including the use of over-the-counter agents and ‘‘natural’’
or herbal substances, should be taken, in conjunction with a toxin screen where
appropriate.

* 2014, American Academy of Neurology.

Copyright © American Academy of Neurology. Unauthorized reproduction of this article is prohibited.


& Haloperidol and chlorpromazine have been shown to be helpful in the second and
third trimesters and appear to be safe in low doses, such as haloperidol 0.5 mg to 2 mg
2 to 3 times daily.
& Genetic testing for the Huntington disease gene has been done for embryos at the
eight-cell stage.
& Once structural causes are eliminated, the most common cause of ataxia arising during
pregnancy is Wernicke encephalopathy secondary to hyperemesis gravidarum causing
vitamin B1 deficiency.
& Patients with Parkinson disease may have worsening of their motor symptoms during
pregnancy, although women report return to baseline in follow-up.
& Wilson disease treatment with chelating agents (D-penicillamine, zinc, trientine, and
tetrathiomolybdate) is recommended throughout pregnancy because untreated pregnant
patients would potentially develop fatal liver disease or hemolytic anemia, while the
fetus would develop liver damage and copper accumulation.

Neuro-ophthalmic Disorders
in Pregnancy
Digre, Kathleen B. MD, FAAN; Kinard, Krista MD. Continuum (Minneap Minn).
February 2014; 20(1 Neurology of Pregnancy): 162Y176.

Abstract
Purpose of Review:
This review discusses evaluation and treatment of neuroophthalmic disorders in the pregnant
patient.
Recent Findings:
Any neuro-ophthalmic abnormality seen in nonpregnant women can be seen in pregnant women.
Pregnancy-specific complications (preeclampsia and eclampsia) cause visual symptoms and
can affect the entire visual axis.
Summary:
Appropriate evaluation and examination is important to preserve the health and vision of the
mother and prevent complications in the fetus. Evaluation should proceed in the same way
for a pregnant patient as it would for a nonpregnant patient, with few exceptions. Treatment
decisions may be influenced by stage of pregnancy.

Key Points
& Differentiation of visual symptoms can help with diagnosis. The neurologist must
determine whether the symptom is mild or severe blurring of vision, diplopia, scotoma
(visual loss), or ocular pain.
& Vascular occlusions can occur with more frequency during pregnancy due to the
hypercoagulable state of pregnancy.

* 2014, American Academy of Neurology.

Copyright © American Academy of Neurology. Unauthorized reproduction of this article is prohibited.


& Blurred vision can also be migraine aura, which can be first experienced in the first
trimester of pregnancy.
& Refractive error and corneal or lens changes are the most common reason for mildly
blurred vision in pregnant women.
& Diabetic pregnant women are at risk for development or progression of retinopathy;
each diabetic woman should have an eye examination during pregnancy.
& If a retinal artery occlusion is present in pregnancy, consider echocardiogram, since
most occlusions are cardiac related.
& Preeclampsia and eclampsia can cause a wide variety of visual abnormalities and must
be considered in any pregnant woman with visual concerns.
& Multiple sclerosis has an increased risk of symptom exacerbation during the
postpartum period, while neuromyelitis optica has an increased risk of symptom
exacerbation during the last trimester of pregnancy.
& The cause of disc edema or papilledema must be found and evaluated.
& When papilledema is present in pregnancy, look carefully for secondary causes including
venous sinus thrombosis. Idiopathic intracranial hypertension can first occur in
pregnancy in obese women.
& Meningiomas may increase in size during pregnancy; optic nerve meningioma may cause
visual field changes.
& All women with papilledema, regardless of its cause, need to be followed with visual
fields.
& Sudden onset of a headache in a patient known to have a pituitary tumor calls for imaging
to look for apoplexy.
& Thyroid-associated orbitopathy is the most common etiology for proptosis in pregnancy.
& With anisocoria, onset and associated symptoms must be obtained. A careful search of
the patient’s recent medications also needs to be evaluated.

* 2014, American Academy of Neurology.

Copyright © American Academy of Neurology. Unauthorized reproduction of this article is prohibited.


Ethical Perspective

Reproductive Issues in
Address correspondence to
Dr Bethanie Morgan-Followell,
700 Children’s Drive, Columbus,
OH 43205, Bethanie.morgan@

Women With Multiple nationwidechildrens.org.


Relationship Disclosure:
Dr Morgan-Followell reports

Sclerosis: Ethical no disclosure. Dr Nicholas’


fellowship is funded through a
Sylvia Lawry Physician

Considerations Fellowship grant from the


National Multiple Sclerosis
Society, and she receives
additional funding for clinical
Bethanie N. Morgan-Followell, MD; Jacqueline A. Nicholas, MD, MPH; research from the National
Multiple Sclerosis Society as
Pedro Weisleder, MD, PhD an assistant professor.
Dr Weisleder serves as a
consultant for the Medical
Review Institute of America
ABSTRACT and as associate editor of the
Journal of Child Neurology.
Addressing the reproductive concerns of women with multiple sclerosis (MS) is vital
Unlabeled Use of
for comprehensive care. Contraception, conception, pregnancy, and breast-feeding Products/Investigational
present many vexing questions to the woman with MS. The risks and benefits of Use Disclosure:
using disease-modifying therapy during the various stages of a woman’s Drs Morgan-Followell,
Nicholas, and Weisleder
reproductive life are topics that need to be discussed. The physician’s primary duty discuss the unlabeled use of
is to the patient; however, the physician must also consider the fetus and later the disease-modifying therapies
child. In helping guide the patient in making medical decisions, the physician must during attempts at conception
and during pregnancy.
take into account the patient’s motivation for those decisions, including family
* 2014, American Academy
obligations, cultural norms, and patient values. The physician is instrumental in of Neurology.
providing the patient with sound, nonjudgmental information and advice so that
she may make a well-informed, autonomous decision about her health and her
disease.

Continuum (Minneap Minn) 2014;20(1):177–180.

Case
A 27-year-old woman with relapsing-remitting multiple sclerosis (MS)
presented to her neurologist because of a several-day history of
progressive, painful loss of vision in the left eye. She was 27 weeks
pregnant. Before her pregnancy, she used glatiramer acetate (GA) as the
disease-modifying therapy (DMT) for her MS. The medication had been
discontinued at 12 weeks of gestation when she discovered that she was
pregnant. Physical examination was consistent with a severe acute left
optic neuritis. The patient was treated with methylprednisolone 1000 mg
IV daily for 3 days. Symptoms resolved within a month. At 38 weeks of
pregnancy, the patient delivered a healthy baby boy. At that time, the
patient elected not to resume GA while breast-feeding. The patient agreed
to reinitiate DMT once she was finished breast-feeding her child. At
5 months postpartum, she developed vertigo and diplopia. The physician
recommended that the patient discontinue breast-feeding and start DMT
to protect her from further relapses. She wanted to defer treatment
and continue breast-feeding her child. The neurologist was concerned that
this decision would adversely affect her health and well-being, as well as
that of her child.

Continuum (Minneap Minn) 2014;20(1):177–180 www.ContinuumJournal.com 177

Copyright © American Academy of Neurology. Unauthorized reproduction of this article is prohibited.


Reproductive Issues With MS

DISCUSSION
Currently, there are no evidence-based guidelines to aid the physician in making
recommendations regarding conception, pregnancy, and breast-feeding. This
patient’s case raises the following questions:
1. What is the physician’s duty to the patient?
2. What is the physician’s duty to the patient’s child?
3. How does the physician help the patient make medical decisions that are in
her, her child’s, and the family’s best interests?
MS is an autoimmune disease characterized by inflammation within the brain
and spinal cord that ultimately leads to demyelination. MS is the leading cause
of nontraumatic disability in young adults.1 Women are twice as likely as men to
develop MS, and the first presentation of symptoms may occur during the years
of childbearing potential. Therefore, counseling women on the potential impact
that MS and its disease-modifying therapy has on pregnancy and motherhood
is important for comprehensive patient care. For an extensive review of
the medical issues and evidence, see ‘‘Multiple Sclerosis in Pregnancy’’ by
Dr Patricia Coyle in this issue of .

The Physician’s Duty to the Patient, the Fetus, and Later the Child
When caring for pregnant women with MS, neurologists should determine
whether their duty is to the patient, the fetus, or both. The answer to this
conundrum depends on whether the fetus is conferred the status of
patienthood (ie, whether the physician has fiduciary duties toward the fetus).
Only the mother can confer such status to the fetus. The ethical concept of the
fetus as a patient has considerable clinical significance.2 If the fetus is also a
patient, then the physician’s recommendations must also include the fetus’ best
interest. If the fetus is not a patient, then the physician’s responsibility to act in
the fetus’ best interest is no longer obligatory.
Obstetricians have long wrestled with this issue. In most instances, the
conundrum arises when the mother’s actions or intended actions are potentially
harmful to the fetus. It is worth considering models used to evaluate perinatal
ethical dilemmas. The maternal-fetal relationship can be viewed using one of
the following three models.3 In the first, mother and child are considered as a
single entity, where the fetus is a part of the woman’s body. As such, the
physician only has one patientVthe mother. Critics point out that a fetus is not a
body part and cannot be treated as such.2 In the second model, mother and fetus
are viewed as separate entities. In this so-called reductionistic or zero-sum-game
model, the physician has two patientsVthe mother and the fetus, each with a
set of rights. The risk is that should rights-based tensions between the mother
and the fetus arise, rights afforded to one will have to take precedence over
the rights of the other. The third model acknowledges that mother and fetus
have a unique and indivisible relationship and should be viewed as
a maternal-fetal dyad. The expectation is that the mother will act in the best
interest of the dyad.4 This model, dubbed ‘‘the professional responsibility’’
model by Chervenak and colleagues,5 is autonomy and beneficence based.
The American College of Obstetricians and Gynecologists (ACOG) endorses the
professional responsibility model6 and maintains that as a competent adult, the
pregnant woman has a claim to autonomy. Because the fetus lacks capacity and

178 www.ContinuumJournal.com February 2014

Copyright © American Academy of Neurology. Unauthorized reproduction of this article is prohibited.


cannot possess values and beliefs, the fetus has no autonomy claim, and the
physician has no autonomy-based obligations to the fetus.7 It is only when the
pregnant woman confers the status of patienthood to the fetus (ie, an
autonomous decision of the mother expressed to the treating physician or
physicians) that the fetus gains beneficence-based rights.
Helping the Patient Make Medical Decisions in Her, Her Child’s,
and the Family’s Best Interests
An interpretation of autonomy in line with the concept of the mother-child dyad
is one in which the neurologist provides the patient, in nonjudgmental fashion,
with all the information necessary to help her make informed consent or refusal
decisions. According to this interpretation, the mother has the right to make
decisions about her health with the knowledge that those decisions may have
consequences for the fetus and, after birth, for the child. The information
provided to the mother must be honest and unbiased. Respecting the mother’s
autonomy and decisional capacity recognizes that she is better positioned than
anyone else to make decisions that take into account her own values,
circumstances, and concerns, as with any other health care decision made by
autonomous adults.6
When counseling a pregnant patient, the neurologist will have to explain the
direct and indirect effects that available treatment options (including no
treatment) will have on her and the fetus. In circumstances when the mother’s
decision could affect her future health status, the discussion should also broach
the future impact of her decision on the child and the family unit. The
conversation will need to start with a delineation of risks and benefits. For
example, if resuming DMT in the early postpartum period precludes breast-
feeding because exposure of the child to DMT transmitted via breast milk
carries risk, then the child would not receive the immunologic and emotional
benefits of breast-feeding. Conversely, potential benefits of DMT may include a
decreased number of MS relapses and slower accrual of disability. This direct
benefit to the mother is associated with indirect benefits to the child. On one
hand, prevention of relapse or accrued neurologic disability through the use of
DMT may allow the mother to effectively care for her child. On the other hand,
avoiding DMT in the postpartum period creates a risk of relapse and disability,
which can impair the mother’s ability to care for the child and may also take a
financial toll through loss of income or hospitalizations.8
At this time, no evidence-based guidelines exist concerning use of DMT
during attempts at conception, pregnancy, or breast-feeding. However, based
on the literature available, some DMTs may be considered in situations in which
the therapy is clearly needed and the potential benefits outweigh the presumed
risk. The neurologist has autonomy- and beneficence-based obligations to the
patient, as she has presented herself for treatment. The neurologist’s duty to a
pregnant patient’s fetus may be less clear; however, in the great majority of
cases, the woman confers the status of patienthood to the fetus, which means
the neurologist must consider the fetus’ beneficence and therefore has a duty
to the fetus. The rights of the fetus are upheld by the mother’s autonomous
decisions to maximize her health and her fetus’ health.
Comprehensive MS care takes into account the fact that pregnancy is unique
to women and that MS influences women’s decisions about reproduction not

Continuum (Minneap Minn) 2014;20(1):177–180 www.ContinuumJournal.com 179

Copyright © American Academy of Neurology. Unauthorized reproduction of this article is prohibited.


Reproductive Issues With MS

only physically (ie, in terms of risks of recurrence of disease activity or disability)


but also emotionally and psychologically (ie, by provoking anxiety, depression,
or guilt about decisions that may adversely affect a woman’s health and well-
being or that of her fetus or child). Ultimately, it is prudent to discuss
reproduction, including planning, conception, pregnancy, and breast-feeding, at
the appropriate times with each woman with MS of childbearing potential
regardless of disease status. Holding such conversations may be considered
‘‘preventive ethics.’’ And while the concept ‘‘identify, prioritize, and address
systemic ethics issues’’9 was conceived with healthcare systems in mind, the
notions can be extrapolated to individual patients. Applying the model to a
patient such as the one we are discussing would begin by identifying and
prioritizing her wishes about family planning and, with those in mind,
establishing treatment plans to address contingencies such as disease exacer-
bation or accrual of disability.

CONCLUSION
Following a frank discussion of the risks and benefits of resuming DMT and
discontinuing breast-feeding versus deferring DMT and continuing to breast-
feed, the physician and patient in this case agreed that when her son reached
12 months of age she would stop breast-feeding and resume GA. The patient
and physician agreed that setting a time limit for breast-feeding would be in her
son’s best interest, as he would receive the benefits of breast-feeding, and in
her best interest, as she would be off DMT only for a limited time period, which
hopefully would not increase her risk of MS relapse. The patient also agreed to
reconsider starting DMT earlier than her son’s first birthday if she had recurrent
relapses during this time.
The patient declined high-dose corticosteroids to treat her current attack of
vertigo and diplopia because she would have needed to discard breast milk and
feed her son infant formula.

REFERENCES
1. Noseworthy JH, Lucchinetti C, Rodriguez M, Weinshenker BG. Multiple sclerosis. N Engl J Med
2000;343(13):938Y952.
2. Chervenak FA, McCullough LB. Ethical dimensions of fetal neurology. Semin Fetal Neonatal Med
2012;17(5):252Y255.
3. Newnham H. Mother v fetus: who wins? Aust J Midwifery 2003;16(1):23Y26.
4. Seymour J. A pregnant woman’s decision to decline treatment: how should the law respond?
J Law Med 1994;2(1):27Y37.
5. Chervenak FA, McCullough LB, Brent RL. The professional responsibility model of obstetrical
ethics: avoiding the perils of clashing rights. Am J Obstet Gynecol 2011;205(4):315.e1Y315.e5.
6. American College of Obstetricians and Gynecologists Committee on Ethics. ACOG Committee
Opinion #321: maternal decision making, ethics, and the law. Obstet Gynecol 2005;106(5 pt 1):
1127Y1137.
7. Chervenak FA, McCullough LB. Ethics in obstetrics and gynecology. An overview. Eur J Obstet
Gynecol Reprod Biol 1997;75(1):91Y94.
8. De Judicibus MA, McCabe MP. The impact of the financial costs of multiple sclerosis on quality of
life. Int J Behav Med 2007;14(1):3Y11.
9. Fox E, Bottrell M, Foglia MB, et al. IntegratedEthics: an innovative program to improve ethics
quality in health care. Innovation J Public Sector Innovation J 2010;(15)2:article 8.
www.ethics.va.gov/IEoverview.pdf. Accessed Dec 3, 2013.

180 www.ContinuumJournal.com February 2014

Copyright © American Academy of Neurology. Unauthorized reproduction of this article is prohibited.


Practice

Epilepsy and Pregnancy:


Address correspondence to
Dr Joseph S. Kass, Baylor
College of Medicine,
Department of Neurology,

A Practical Approach to 1 Baylor Plaza NB-302,


Houston, Texas 77030,
kass@bcm.edu.

Mitigating Legal Risk Relationship Disclosure:


Dr Kass reports no disclosure.
Unlabeled Use of
Products/Investigational
Joseph S. Kass, MD, JD Use Disclosure:
Dr Kass reports no disclosure.
* 2014, American Academy
of Neurology.
ABSTRACT
Women with epilepsy are at increased risk of obstetric complications and poorer
cognitive outcomes for their offspring. This article aims to provide neurologists with
a practical framework for understanding, identifying, and managing legal risk when
treating pregnant women with epilepsy.

Continuum (Minneap Minn) 2014;20(1):181–185.

Seizures may injure both the mother and her fetus through several mecha-
nisms, including trauma, hypoxia, and complications of status epilepticus.
Minimizing seizures in pregnant patients improves outcomes for both the
pregnant woman and her unborn offspring. Antiepileptic drugs (AEDs),
however, carry risks of causing major congenital malformations, and the US
Food and Drug Administration (FDA) has rated AEDs no safer than pregnancy
class C. AEDs also can cause adverse reactions, which, even in the absence of
teratogenesis, may negatively impact the pregnancy.
Recognizing the challenges of managing epilepsy safely in pregnant women,
the American Academy of Neurology (AAN) issued three practice parameter
updates in 2009 specific to pregnant women with epilepsy. These focused on
obstetric complications and changes in seizure frequency;1 teratogenesis and
perinatal outcomes;2 and vitamin K, folic acid, AED blood levels, and breast-
feeding.3 As reflected in the practice parameters, considerable uncertainty remains
in the high-stakes environment of the obstetric care of women with epilepsy.
Although the vast majority of medical injuries are not due to medical
malpractice,4 physicians understandably perceive themselves as vulnerable to a
lawsuit whenever a patient experiences an adverse outcome. As the AAN
practice parameters underscore, women with epilepsy are at increased risk of
obstetric complications and poorer cognitive outcomes for offspring. This
article aims to provide neurologists with a practical framework for understand-
ing, identifying, and managing legal risk when treating pregnant women with
epilepsy.
The four hypothetical clinical scenarios below are ones in which the treating
neurologist may be exposed to legal liability if medical outcomes are poor. All
presume that the woman has chosen not to terminate her pregnancy and
considers herself and her fetus as the neurologist’s patients. The discussion that
follows is not intended to substitute for appropriate legal or institutional risk-
management counsel and does not guarantee mitigation of legal risk.

Continuum (Minneap Minn) 2014;20(1):181–185 www.ContinuumJournal.com 181

Copyright © American Academy of Neurology. Unauthorized reproduction of this article is prohibited.


A Practical Approach to Mitigating Legal Risk

Practice Case 1
A 28-year-old primiparous woman who is 16 weeks pregnant is transported
by ambulance to the hospital in status epilepticus. She stopped divalproex
sodium (which she has been taking for 12 years because of her juvenile
myoclonic epilepsy) without consulting her physician when she and her
husband began trying to conceive 5 months ago. She was given IM
diazepam and intubated in the field but continues to experience
convulsive seizures. The neurologist on call implements a standard
status epilepticus treatment protocol, administering IV fosphenytoin and
midazolam to achieve seizure control in a medically reasonable period of
time. The patient awakens without obvious sequelae but is concerned
about the impact of the treatment on her pregnancy.

Practice Case 2
A 28-year-old woman with juvenile myoclonic epilepsy has been treated
for many years with divalproex sodium. She has been seizure free for more
than 5 years and now wishes to become pregnant. She consults with her
neurologist before attempting conception because she is concerned about
the effects of divalproex sodium on her ability to get pregnant and on
the fetus. She is also worried about losing seizure control and the
implications that such loss of control may have for her quality of life
and the outcome of her pregnancy.

Practice Case 3
A 28-year-old woman with juvenile myoclonic epilepsy successfully
managed on divalproex sodium for the past 5 years discovers that she
is approximately 8 weeks pregnant. She seeks information from her
long-standing neurologist about the effects of this medication on her
pregnancy and developing fetus.

Practice Case 4
A 28-year-old woman presents for follow-up of her symptomatic
localization-related epilepsy with increased seizure frequency despite
being on high doses of levetiracetam. Oxcarbazepine is added to her
seizure regimen to improve seizure control. Two months later, she informs
her neurologist that she is pregnant. The patient does not understand
how she became pregnant while taking oral contraceptive pills with 100%
compliance. She is worried about the potential teratogenicity of her AEDs.

DISCUSSION
In a medical malpractice claim, the plaintiff alleges that the provider’s care
deviated from accepted norms, resulting in injury. Since medical malpractice
claims are a type of negligence claim, the plaintiff must demonstrate by the
preponderance of the evidence (51% certainty) each of four elements of a
negligence claim in order to prevail.5

182 www.ContinuumJournal.com February 2014

Copyright © American Academy of Neurology. Unauthorized reproduction of this article is prohibited.


First, the plaintiff must show that a physician-patient relationship existed,
creating in the neurologist a duty to care for the plaintiff at the nationally
accepted standard. The physician-patient relationship inherently imposes this
duty on the neurologist. Second, the plaintiff must establish that the
neurologist’s actions deviated from the acceptable professional standard. The
plaintiff must generally provide expert testimony to demonstrate that a
reasonably prudent neurologist would have acted differently under the same
circumstances. The deviation from the standard of care is specialty specific and
based on a national, not a local, standard in most jurisdictions. The alleged
deviation from professional standards can take the form of poor-quality medical
care, unprofessional behavior, or failure to provide the patient (or surrogate)
with enough information to make an informed decision about whether to
accept or reject the proposed intervention. Cases in which the quality of
informed consent is in dispute may rely on either a ‘‘reasonably prudent
medical professional’’ or a ‘‘reasonably prudent layperson’’ standard, depending
on the jurisdiction.6 The plaintiff in a jurisdiction operating under the first
standard must demonstrate that the defendant neurologist failed to disclose
what reasonably prudent neurologists typically disclose to patients under similar
circumstances. Plaintiffs in jurisdictions employing the second, more common
standard must demonstrate that the defendant failed to disclose what a
reasonably prudent patient would want to know in order to assess the risks and
benefits adequately. Third, the plaintiff must demonstrate injuryVphysical,
emotional, or financial. Fourth, the plaintiff must establish that the negligent
actions were the proximate cause of the injury (ie, but for the provider’s behavior,
the patient would not have been injured). Neurologists who care for adult
patients should also keep in mind an important technical issue regarding the
statute of limitations for medical malpractice lawsuits that may be different in
obstetric cases, in which injury to a minor is involved: in most states, the statute of
limitations for injuries to minors may extend the statute of limitations for bringing
a medical malpractice lawsuit well beyond what is allowed with adult plaintiffs.
In Practice Case 1, although the neurologist exposed the developing fetus to a
potential teratogen, the neurologist properly followed the standard of care for status
epilepticus, a life-threatening emergency. In following a generally accepted status
epilepticus protocol, the neurologist used best evidence to manage the emergency
and minimize harm to the patient. Without achieving rapid seizure control, both the
mother and the fetus may have been irreversibly injured. This case highlights that in
emergency situations, what is best for the pregnant woman is generally accepted to
be best for the fetus. Although fosphenytoin is a category D medication and may
not be ideal for long-term management of epilepsy during pregnancy,2 it met the
standard of care in the treatment of this neurologic emergency. This case also
highlights that informed consent for treatment during a medical emergency, when
the patient’s decision-making ability is impaired, is implied consent. Therefore, the
neurologist’s first duty to the patient was to manage the status epilepticus properly.
The neurologist was not obligated to obtain explicit consent for the use of
fosphenytoin, despite its category D status, because it was used for treatment of
the patient’s time-sensitive medical emergency.
Neurologists recognize that no AED is perfect for use during pregnancy. Certain
AEDs, especially valproate-containing medications, present a particularly heightened
risk of adverse effects to the developing fetus.2 Because the patient in Practice Case 2
Continuum (Minneap Minn) 2014;20(1):181–185 www.ContinuumJournal.com 183

Copyright © American Academy of Neurology. Unauthorized reproduction of this article is prohibited.


A Practical Approach to Mitigating Legal Risk

is consulting with her neurologist before conception, the neurologist and the
patient have time to explore the therapeutic options thoroughly before exposing
the fetus to risk. The neurologist can fulfill the ethical and legal obligations of
informed consent thoroughly because of the patient’s prudent choice to plan in
advance. The neurologist must discuss with the pregnant patient the risks and
benefits of treating versus not treating epilepsy. Furthermore, the discussion must
include a disclosure of medication-specific risks to the health of both the woman
and the fetus, as well as an explanation of why a particular medication is being
recommended over other medically appropriate choices. The neurologist must also
realize that the degree of detail a reasonable patient would want to know in
weighing treatment options may be greater than usual when pregnancy outcome is
at stake. The neurologist must also ensure that the patient truly understands the
risks and benefits of the proposed treatment plans. The neurologist should use one
of the standard communication techniques for assessing a patient’s comprehension,
such as asking the patient to explain back what she understands about the
treatment options. The patient should also be asked to explain why she is choosing
or rejecting the particular treatment options. The neurologist must then document
thoroughly the entire informed consent process in the medical record, including
this verification of understanding based on the patient’s clearly articulated
understanding of the risks and benefits of the treatment options. This thorough
weighing of options, disclosing of risks and benefits, ensuring a meaningful
informed consent process, and properly documenting the informed consent
process in the medical record should satisfy the neurologist’s ethical obligations
and mitigate the risk of a successful negligence claim based on failure to obtain
meaningful informed consent.
In Practice Case 3, if the fetus suffers an adverse outcome, the neurologist’s
liability exposure will depend on the thoroughness of the prepregnancy informed
consent process. When initially prescribing potentially teratogenic medications to a
woman of childbearing potential, the neurologist should disclose this teratogenicity
in the process of reviewing the risks and benefits of the medication. The neurologist
may believe that divalproex sodium is the most appropriate medication to treat this
patient’s epilepsy; however, just as a reasonable patient would want to know about
the potential metabolic, cosmetic, and hepatic risks with valproate-containing
medications, she would also want to consider the potentially deleterious effects of
this medication on a developing fetus when deliberating the appropriateness of this
medication for her situation. As part of the informed consent process, the
neurologist should recommend effective contraception and consider prescribing
folic acid.3 Although not responsible for birth control failure in this case, the
neurologist may be liable depending on the quality of the initial informed consent.
In Practice Case 4, the neurologist’s liability exposure depends on how the
neurologist counseled the patient about the potential effect of oxcarbazepine
on the efficacy of oral contraceptive pills. If the neurologist either (1) performed
proper medication reconciliation and failed to realize that oxcarbazepine
reduces oral contraceptive efficacy or (2) failed outright to perform adequate
medication reconciliation, the neurologist may be found negligent for having
provided substandard care. This case, and to some extent Practice Case 3, also
raises the issue of how and when to apologize for a medical error.
Although a thorough review of the topic of apologies for medical error is
beyond the scope of this article, neurologists should be aware that the
184 www.ContinuumJournal.com February 2014

Copyright © American Academy of Neurology. Unauthorized reproduction of this article is prohibited.


admissibility of apologies during a medical malpractice lawsuit varies from state
to state; while some have no law in this area, the majority of states have apology
laws. Apology laws typically protect only the actual expression of sympathy but
not the expression of fault that may accompany the expression of sympathy. A
minority of states have disclosure laws. These laws mandate disclosure of
certain unanticipated outcomes and may protect the communication from
being used against the physician in a legal or administrative action.7 Knowing
the law and working with local risk management personnel is advisable in these
difficult situations when a medical error has been made.

CONCLUSION
Neurologists managing pregnant women with epilepsy face a number of challenges
in their attempt to optimize pregnancy outcome and fetal wellbeing. Minimizing
legal liability requires communicating and documenting the risks and benefits of
treatment clearly to the patient and acknowledging the imperfect state of evidence-
based medicine in this field. Neurologists can mitigate risk by discussingVand
documenting the discussion ofV pregnancy-related issues with their nonpregnant
epileptic female patients of childbearing age. Furthermore, neurologists should keep
informed of their local informed consent and apology laws and seek the advice of
legal counsel and risk management professionals in appropriate circumstances.

REFERENCES
1. Harden CL, Hopp J, Ting TY, et al. Practice parameter update: management issues for women with
epilepsyVfocus on pregnancy (an evidence-based review): obstetrical complications and change
in seizure frequency : report of the Quality Standards Subcommittee and Therapeutics and
Technology Assessment Subcommittee of the American Academy of Neurology and American
Epilepsy Society. Neurology 2009;73(2):126Y132.
2. Harden CL, Meador KJ, Pennell PB, et al. Practice parameter update: management issues for
women with epilepsyVfocus on pregnancy (an evidence-based review): teratogenesis and
perinatal outcomes: report of the Quality Standards Subcommittee and Therapeutics and
Technology Assessment Subcommittee of the American Academy of Neurology and American
Epilepsy Society. Neurology 2009;73(2):133Y141.
3. Harden CL, Pennell PB, Koppel BS, et al. Practice parameter update: management issues for
women with epilepsyVfocus on pregnancy (an evidence-based review): vitamin K, folic acid,
blood levels, and breastfeeding: report of the Quality Standards Subcommittee and Therapeutics
and Technology Assessment Subcommittee of the American Academy of Neurology and American
Epilepsy Society. Neurology 2009;73(2):142Y149.
4. Brennan TA, Leape LL, Laird NM, et al. Incidence of adverse events and negligence in hospitalized
patients results of the Harvard Medical Practice Study 1. 1991. Qual Saf Health Care
2004;13(2):145Y151.
5. Kapp MB. The interface of law and medical ethics in medical intensive care. Chest 2009;136(3):
904Y909.
6. Nepps ME. The basics of medical malpractice: a primer on navigating the system. Chest 2008;
134(5):1501Y1555.
7. Mastroianni, Mello MM, Sommer S, et al. The flaws in state ‘apology’ and ‘disclosure’ laws dilute
their intended impact on malpractice suits. Health Affai (Millwood) 2010;29(9):1611Y1619.

Continuum (Minneap Minn) 2014;20(1):181–185 www.ContinuumJournal.com 185

Copyright © American Academy of Neurology. Unauthorized reproduction of this article is prohibited.


Practice

Coding in Pregnancy
Address correspondence to
Dr Mark S. Yerby, North
Pacific Epilepsy Research,
2311 Northwest Northrup
Street, Suite 202, Portland,
OR 7210-2955,
yerby@seizures.net.
With a Focus on Epilepsy
Relationship Disclosure: Mark S. Yerby, MD, MPH, FAAN; Laura B. Powers, MD, FAAN
Dr Yerby serves on the
speakers bureaus for
Lundbeck and Supernus
Pharmaceuticals, Inc. Dr
Powers serves as ICD-9-CM Accurate coding is an important function of neurologic practice. This contribution
Advisor for the Coding
Subcommittee of the AAN
to is part of an ongoing series that presents helpful coding in-
Medical Economics and formation along with examples related to the issue topic. Tips for diagnosis coding,
Management Committee Evaluation and Management coding, procedure coding, or a combination are
and serves in an editorial
capacity for Neurology: Clinical
presented, depending on which is most useful for the subject area of the issue.
Practice.
Unlabeled Use of
Products/Investigational INTRODUCTION
Use Disclosure:
Drs Yerby and Powers report The evaluation and management of epilepsy during pregnancy is complex,
no disclosures. requiring coordinated care between the neurologist and obstetrician.1,2 Given
* 2014, American Academy the complexity of these conditions, the American Academy of Neurology has
of Neurology.
developed guidelines to assist neurologists in developing comprehensive plans
for such patients. The potential for liability is significant when managing women
with epilepsy immediately before and during pregnancy; therefore, the
following issues will need to be covered in the medical record.
& Education of the patient with a clear statement of the risks of seizures and
antiepileptic medication, and the use of folic acid.
& Education of the obstetrician with review of the risks, one’s plans to mitigate
them, and a plan for the obstetrician to treat acute maternal seizures during
labor and delivery.
& Verification of the diagnosis of epilepsy. Not all seizures are epilepsy, and
one needs to be able to support the diagnosis.
& Considerations of alternative treatments from among the various
antiepileptic drugs (AEDs).
& Consideration of comorbidities (women with epilepsy have higher than
expected rates of depression, anxiety, and migraine, as well as eclampsia).
& Determination of the most effective AED and plasma concentration range
for an individual patient.
& Development of a plan for monitoring AED levels during pregnancy and the
postpartum period.
& Development of a plan for treating acute seizures.
& Development of a plan for postpartum management.
PROPER EVALUATION AND MANAGEMENT CODING
Epilepsy patients, particularly when pregnant, require a level of complexity and
extra time spent in their care that qualifies them for higher levels of medical
decision making and increased levels of service. This is also true for patients with
other neurologic diseases affecting or affected by the pregnancy. The documen-
tation for the level of service must meet Current Procedural Terminology (CPT)
requirements for the ‘‘bullet’’ method, be based on time, or make use of
prolonged service codes as discussed in previous issues of .3

186 www.ContinuumJournal.com February 2014

Copyright © American Academy of Neurology. Unauthorized reproduction of this article is prohibited.


DIAGNOSIS CODING
The Official Guidelines for Coding and Reporting for both International Classifica-
tion of Diseases, Ninth Revision, Clinical Modification (ICD-9-CM) and Interna-
tional Classification of Diseases, 10th Revision, Clinical Modification (ICD-10-CM)
have special sequencing instructions for coding of patient visits during pregnancy.4,5
A code designating the pregnancy is always listed first unless the condition for
which the patient is being evaluated has no effect on the pregnancy or is not
affected by the pregnancy in any way. Both classifications have codes specifically
created for use with neurologic conditions complicating or affected by pregnancy.

ICD-9-CM Codes Used in Visits for Patients With Neurologic


Conditions
(In the following table, x is a placeholder for the fifth digit, to be coded as
follows: 0 = unspecified as to episode of care or not applicable; 1 = delivered,
with or without mention of antepartum condition; 2 = delivered, with
mention of postpartum complication; 3 = antepartum condition or
complication; 4 = postpartum condition or complication.)
642.6x Eclampsia with convulsions
646.4x Peripheral neuritis in pregnancy
646.8x Other specified complications of pregnancy
Use for nonepilepsy conditions and add a code for the condition
649.4x Epilepsy complicating pregnancy, childbirth, or the puerperium
Use additional code to identify the specific type of epilepsy
Excludes: eclampsia (642.6x)
671.5x Other phlebitis or thrombosis, cerebral venous sinus thrombosis
674.0x Cerebrovascular disorders in the puerperium
Any condition classifiable to 430-434, 436-437 occurring during
pregnancy, childbirth, or the puerperium or specified as puerperal
V26.49 Other procreative management counseling and advice
V22.2 Pregnant state, incidental
Code this following the neurologic condition code when the condition
is not affected by or does not affect the pregnancy
ICD-10-CM will be used for diagnosis coding in the United States beginning
October 1, 2014. In ICD-10-CM, the trimester and delivery status is captured
in codes, but not always at the same character position.

ICD-10-CM Codes Used in Visits for Patients With Neurologic


Conditions
Note that the codes from the ICD-10-CM chapter on pregnancy begin with the
letter ‘‘O’’
O15.00 Eclampsia in pregnancy, unspecified trimester
O15.02 Eclampsia in pregnancy, second trimester
O15.03 Eclampsia in pregnancy, third trimester
O15.1 Eclampsia in labor
O15.2 Eclampsia in the puerperium
O15.9 Eclampsia, unspecified as to time period; Eclampsia, not otherwise
specified

Continuum (Minneap Minn) 2014;20(1):186–190 www.ContinuumJournal.com 187

Copyright © American Academy of Neurology. Unauthorized reproduction of this article is prohibited.


Coding in Pregnancy

O22.50 Cerebral venous thrombosis in pregnancy, unspecified trimester


O22.51 Cerebral venous thrombosis in pregnancy, first trimester
O22.52 Cerebral venous thrombosis in pregnancy, second trimester
O22.53 Cerebral venous thrombosis in pregnancy, third trimester
O26.821 Pregnancy related peripheral neuritis, first trimester
O26.822 Pregnancy related peripheral neuritis, second trimester
O26.823 Pregnancy related peripheral neuritis, third trimester
O26.829 Pregnancy related peripheral neuritis, unspecified trimester
O87.3 Cerebral venous thrombosis in the puerperium
O99.350 Diseases of the central nervous system complicating pregnancy,
unspecified trimester
O99.351 Diseases of the central nervous system complicating pregnancy, first
trimester
O99.352 Diseases of the central nervous system complicating pregnancy,
second trimester
O99.353 Diseases of the central nervous system complicating pregnancy, third
trimester
O99.354 Diseases of the central nervous system complicating childbirth
O99.355 Diseases of the central nervous system complicating the puerperium
O99.411 Diseases of the circulatory system complicating pregnancy, first
trimester
(These codes are used for cerebrovascular disease)
O99.412 Diseases of the circulatory system complicating pregnancy, second
trimester
O99.413 Diseases of the circulatory system complicating pregnancy, third
trimester
O99.419 Diseases of the circulatory system complicating pregnancy,
unspecified trimester
O99.42 Diseases of the circulatory system complicating childbirth
O99.43 Diseases of the circulatory system complicating the puerperium
Z31.69 Encounter for other general counseling and advice on procreation
Z33.1 Pregnant state, incidental

CASE EXAMPLES

Coding Case 1
A 24-year-old woman with a history of localization-related epilepsy and
rare secondary generalization controlled on levetiracetam, came for
consultation (requested by her internist) and advice on whether she should
remain on or change her medication, since she had recently married and
was planning to have children. She did not currently have an obstetrician.
After confirming her diagnosis, the neurologist reviewed the risks of
pregnancy and epilepsy in terms of both maternal seizures and antiepileptic
medicationYrelated risk to the fetus. The neurologist also discussed ways to
minimize risks and plans for monitoring her levetiracetam during pregnancy
and the postpartum period, as well as for acute seizure management, and

Continued on page 189

188 www.ContinuumJournal.com February 2014

Copyright © American Academy of Neurology. Unauthorized reproduction of this article is prohibited.


Continued from page 188
reviewed the information on breast-feeding. This visit took an hour. The
documentation met criteria for comprehensive history and physical exam.
The CPT Evaluation and Management code appropriate for this visit,
given the high level of medical complexity, would be 99245 if the patient’s
insurance or other payer allows use of consult codes. If not, then the
new-patient code 99205 is appropriate. Prolonged service codes are not
appropriate here because the ‘‘typical time’’ for 99245 is 80 minutes and
for 99205 is 60 minutes. Prolonged service must be at least 30 minutes
beyond the ‘‘typical time’’ for a visit code. The ICD-9-CM codes would
be 345.40 (Localization-related epilepsy and epileptic syndromes with
complex partial seizures, without mention of intractable epilepsy) and
V26.49 (Other procreative management counseling and advice). The
corresponding ICD-10-CM codes for use after October 1, 2014, are G40.209
(Localization-related epilepsy and epileptic syndromes with complex
partial seizures, not intractable, without status epilepticus) and Z31.69
(Encounter for other general counseling and advice on procreation).

Coding Case 2
A 28-year-old pregnant woman with epilepsy was admitted to the hospital
for non-neurologic complications of the second trimester of her pregnancy.
She had a seizure while hospitalized, and a neurologist who had not seen
her before was asked to consult. Her epilepsy had previously been well
controlled. After determining that she did not have eclampsia, the
neurologist established her epilepsy type as complex partial with secondary
generalization, evaluated her anticonvulsant medication, and made dose
adjustments. The neurologist then counseled the patient and her family
about seizures in pregnancy, effects of the seizure and medications on
her fetus, and developed a treatment plan for managing potential acute
seizures during the remainder of her pregnancy. The documentation met
criteria for a detailed history and physical examination. The visit took
85 minutes, and 55 minutes were spent in counseling and coordination
of care (times documented).
Although the history and physical examination did not fulfill the
requirements for a level five consultation (99245) or new patient (99205),
the time spent in counseling and coordination of care does. The ICD-9-CM
codes would be 649.43 (Epilepsy complicating pregnancy, childbirth, or
the puerperium) and 345.40 (Localization-related epilepsy and epileptic
syndromes with complex partial seizures, without mention of intractable
epilepsy). The ICD-10-CM codes will be O99.352 (Diseases of the central
nervous system complicating pregnancy, second trimester) and G40.209
(Localization-related epilepsy and epileptic syndromes with complex
partial seizures, not intractable, without status epilepticus).
Let us say, for example, that the neurologist in this case did determine
that the patient’s recent seizure was due to eclampsia. The first-listed
ICD-9-CM code would be 642.63 (Eclampsia, antepartum condition or
complication). In ICD-10-CM, the first-listed code would be O15.02
(Eclampsia in pregnancy, second trimester). The epilepsy code would be
listed secondarily, as this condition would also be necessarily addressed.

Continuum (Minneap Minn) 2014;20(1):186–190 www.ContinuumJournal.com 189

Copyright © American Academy of Neurology. Unauthorized reproduction of this article is prohibited.


Coding in Pregnancy

REFERENCES
1. Harden CL, Hopp J, Ting TY, et al. Practice parameter update: management issues for women with
epilepsyVfocus on pregnancy (an evidence-based review): obstetrical complications and change
in seizure frequency: report of the Quality Standards Subcommittee and Therapeutics and
Technology Assessment Subcommittee of the American Academy of Neurology and American
Epilepsy Society. Neurology 2009;73(2);126Y132.
2. Hernandez-Diaz S, Smith CR, Shen A, et al. Comparative safety of antiepileptic drugs during
pregnancy. Neurology 2012;78(21):1692Y1699.
3. Powers L. Coding issues: Current Procedural Terminology Evaluation and Management Coding for
Neurologic Consultations. Continuum (Minneap Minn) 2011;17(5):1129Y1134.
4. ICD-9-CM Official Guidelines for Coding and Reporting. www.cdc.gov/nchs/data/icd9/
icd9cm_guidelines_2011.pdf. Published October 2011. Accessed October 18, 2013.
5. ICD-10-CM Official Guidelines for Coding and Reporting. www.cdc.gov/nchs/data/icd10/
10cmguidelines_2013_final.pdf. Accessed October 18, 2013.

190 www.ContinuumJournal.com February 2014

Copyright © American Academy of Neurology. Unauthorized reproduction of this article is prohibited.

Potrebbero piacerti anche